Sunteți pe pagina 1din 322

TESTE GRILĂ

PENTRU EXAMENUL DE LICENȚĂ


MEDICINĂ

1
Testele grilă au fost concepute pentru a veni în sprijinul studenților și
absolvenților, în vederea susținerii examenului de licență și a concursului
național de rezidențiat.
Bibliografia și tematica care au stat la baza întocmirii testelor sunt cele
în vigoare pentru examenul de finalizare a studiilor la programul de studii
Medicină de la Facultatea de Medicină și Farmacie din cadrul Universității
„Dunărea de Jos” din Galați, dar și pentru concursul național de rezidențiat
aprobate de Ministerul Sănătății, care au fost modificate față de sesiunea
anterioară.
Testele grilă își propun fixarea cunoștințelor teoretice de către studenți
și însușirea deprinderii de a alege răspunsurile corecte din variantele de
răspuns enunțate.
Prezentele teste sunt orientative, Facultatea de Medicină și Farmacie,
rezervându-și dreptul de a formula teste noi în totalitate sau parțial.

SUCCES!

2
3
CUPRINS

MEDICINĂ CLINICĂ

Capitolul 1 - SEPSISUL ŞI TRATAMENTUL INFECŢIILOR


BACTERIENE ....................................................................... 7
Capitolul 2 - ECHILIBRUL HIDRO-ELECTROLITIC
ȘI ACIDO-BAZIC................................................................ 12
Capitolul 3 - TERAPIE INTENSIVĂ ........................................................ 17
Capitolul 4 - HEMATOLOGIE ................................................................. 20
Capitolul 5 - REUMATOLOGIE ............................................................... 24
Capitolul 6 - ENDOCRINOLOGIE ........................................................... 31
Capitolul 7 - DIABETUL ZAHARAT ...................................................... 34
Capitolul 8 - NEUROLOGIE ..................................................................... 46
Capitolul 9 - PNEUMOLOGIE .................................................................. 51
Capitolul 10 - BOALĂ VENOASĂ TROMBOEMBOLICĂ ...................... 58
Capitolul 11 - CARDIOLOGIE ................................................................... 63
Capitolul 12 - HIPERTENSIUNEA ARTERIALĂ ..................................... 75
Capitolul 13 - GASTROENTEROLOGIE ................................................... 78
Capitolul 14 - BOLILE HEPATICE ............................................................ 86
Capitolul 15 - TULBURĂRI RENALE ȘI ALE TRACTULUI URINAR . 94
Capitolul 16 - INFECŢII TRANSMISIBILE PE CALE SEXUALĂ ŞI
INFECŢIA CU VIRUSUL HIV ......................................... 102

CHIRURGIE GENERALĂ ȘI SPECIALITĂȚI CHIRURGICALE

Capitolul 17 - EVALUAREA SI MANAGEMENTUL


PERIOPERATOR AL PACIENTULUI
CHIRURGICAL ................................................................. 117
Capitolul 18 - SANGERARILE CHIRURGICALE:
TULBURARI ALE HEMOSTAZEI, STARI DE
HIPERCOAGULABILITATE SI TERAPIA CU
PRODUSE SANGUINE LA PACIENTUL
CHIRURGICAL ................................................................. 121

4
Capitolul 19 - INFECTIILE CHIRURGICALE ........................................ 123
Capitolul 20 - TRAUMATOLOGIE .......................................................... 126
Capitolul 21 - ARSURI .............................................................................. 128
Capitolul 22 - HERNIILE PERETELUI ABDOMINAL .......................... 132
Capitolul 23 - ESOFAGUL........................................................................ 136
Capitolul 24 - STOMACUL SI DUODENUL ........................................... 146
Capitolul 25 - INTESTIN SUBȚIRE ȘI APENDICE ............................... 158
Capitolul 26 - COLON, RECT, ANUS...................................................... 171
Capitolul 27 - CĂILE BILIARE ................................................................ 182
Capitolul 28 - PANCREASUL .................................................................. 185
Capitolul 29 - FICATUL ȘI SPLINA ....................................................... 188
Capitolul 30 - BOLILE SISTEMULUI VASCULAR ............................... 201
Capitolul 31 - OTORINOLARINGOLOGIA: BOLILE CAPULUI
ȘI GÂTULUI ...................................................................... 206
Capitolul 32 - CHIRURGIE ORTOPEDICĂ: BOLILE
SISTEMULUI MUSCULOSCHELETAL ......................... 219
Capitolul 33 - UROLOGIE: AFECȚIUNILE
APARATULUI URO – GENITAL .................................... 233

SINOPSIS DE MEDICINA

Capitolul 34 - DERMATOLOGIE ............................................................. 240


Capitolul 35 - PEDIATRIE ........................................................................ 245
Capitolul 36 - AFECȚIUNI GINECOLOGICE ȘI MAMARE ................. 258
Capitolul 37 - OBSTETRICĂ .................................................................... 268
Capitolul 38 - TULBURĂRI PSIHICE ..................................................... 279
Capitolul 39 - EPIDEMIOLOGIE ȘI ETICĂ ............................................ 303

TEMATICA ȘI BIBLIOGRAFIA PENTRU EXAMENUL DE


LICENȚĂ DOMENIUL MEDICINĂ ............................................................317

5
6
MEDICINĂ CLINICĂ

Capitolul 1
SEPSISUL ŞI TRATAMENTUL INFECŢIILOR
BACTERIENE

1. În sepsis, sediul infecţiei la nivelul tractului urinar este sugerat


de:
A. Durere în flancuri;
B. Durere în abdomenul superior;
C. Disurie;
D. Incontinenţă urinară;
E. Vărsături.
R: A, C, D (1, p. 152)

2. În sepsis, investigaţiile generale din urină, sunt:


A. Glucozurie;
B. Nitriţi;
C. Leucocite;
D. Urocultură;
E. Esteraza.
R: B, C, D, E (1, p. 152)

3. În insuficienţa terminală de organ, la nivel hemodinamic apar


următoarele modificări:
A. Tensiunea arterială sistolică este < 110 mmHg;
B. Tensiunea arterială sistolică scade cu > 40 mmHg sub valoarea de
bază;
C. Tensiunea arterială medie este < 65 mmHg;
D. Tensiunea arterială sistolică este < 90 mmHg;
E. Tensiunea arterială medie este < 90 mmHg.
R: B, C, D (1, p. 152)

4. În insuficienţa terminală de organ apar următoarele modificări:


A. Trombocite < 150 x 109 /L;
B. INR > 1;
C. Lactat seric > 2 mmol/l;
7
D. PaO2 / FiO2 < 300 mmHg;
E. Diureza < 0,5 ml/kg/h pentru 2 ore succesive.
R: C, D, E (1, p. 152)

5. Scorul quick SOFA este considerat pozitiv dacă se întrunesc


minim două din criteriile următoare:
A. Scor de comă Glasgow <15;
B. Trombocite < 100 x 109 /L;
C. Frecvenţa respiratorie ≥ 22 respiraţi/minut;
D. Temperatură > 38 gr celsius;
E. Presiune arterială sistolică ≤ 100 mmHg.
R: A, C, E (1, p. 154)

6. Grupele aflate la risc crescut de a dezvolta sepsis includ:


A. Vârstnicii (>55 de ani);
B. Pacienţii care au prezentat în antecedente TEP;
C. Pacienţii cu afecţiuni medicale imunosupresoare;
D. Pacienţi imunodeprimaţi iatrogen;
E. Gravidele.
R: C, D, E (1, p. 155)

7. Sepsisul asociat asistenţei medicale se defineşte ca sepsis instalat


la:
A. Pacienţi operaţi în urmă cu 3 luni;
B. Pacienţi externaţi în ultimele 30 de zile;
C. Pacienţi din unităţi de îngrijiri cronice;
D. Pacienţi care locuiesc în comunităţi închise;
E. Pacienţii care necesită hemodializă.
R: B, C, E (1, p. 155)

8. Cele mai frecvente infecții dobândite comunitar care pot evolua


către sepsis sunt:
A. Meningita;
B. Endocardita;
C. Infecţiile tractului urinar;
D. Infecţiile tractului biliar;
E. Infecţiile tractului respirator inferior.
R: C, D, E (1, p. 155)

9. Infecţiile nozocomiale:

8
A. Sunt mai virulente;
B. Adeseori sunt multirezistente;
C. Sunt mai dificil de tratat;
D. Deseori sunt asociate unor proceduri medicale;
E. Riscul poate fi redus prin profilaxie antibiotică adecvată.
R: B, C, D, E (1, p. 155)

10. Sepsis Six cuprinde:


A. Administrarea oxigen pentru a menţine SpO2 >90%;
B. Recoltarea de hemoculturi;
C. Începerea tratamentului cu antibiotic cu spectru restrâns;
D. Măsurarea nivelul lactatului;
E. Măsurarea repetată a hemoglobinei.
R: B, D (1, p. 157)

11. Resuscitarea iniţială în sepsis cuprinde:


A. Soluţii cristaloide 30 mL/kg în primele 3 ore;
B. Menţinerea tensiunii arteriale medii la 65 mmHg;
C. Soluţii cristaloide 30 mL/kg în primele 6 ore;
D. Menţinerea tensiunii arteriale sistolice la 65 mmHg;
E. Ghidarea resuscitării pentru normalizarea valorii lactatului
la pacienţi cu valori ridicate ale acestuia.
R: A, B, E (1, p. 157)

12. Terapia lichidiană în sepsis cuprinde:


A. Administrarea de lichide rapid când este nevoie;
B. Primă intenţie sunt soluţiile coloid osmotice;
C. Albumina umană când sunt necesare volume mari de lichid;
D. De evitat soluţiile de hidroxietil amidon;
E. Utilizarea de plasma proaspăt congelată pentru efectul coloid
osmotic.
R: A, C, D (1, p. 157)

13. Parametrii care trebuie evaluaţi la alegerea iniţială a


regimului antibiotic includ:
A. Calea de administrare;
B. Momentul iniţierii terapiei;
C. Frecvenţa administrării;
D. Durata tratamentului;
E. Monitorizarea posibilei toxicităţi.

9
R: A, C, D, E (1, p. 159)

14. Despre antibioterapia parenterală în ambulatoriu (OPAT,


outpatient parenteral antimicrobial treatment) sunt adevărate
următoarele afirmaţii:
A. Cele mai frevente afecţiuni medicale îndrumate către servicii
OPAT sunt meningitele si endocarditele, încă de la diagnosticare;
B. Nu sunt eligibili utilizatorii de droguri intravenoase;
C. Nu sunt eligibili pacienţii cu afecţiuni psihiatrice severe;
D. Se recomandă ca pacienţii să fie trataţi 3-5 zile cu antibiotice
intravenoase;
E. Infectile de tract urinar cu germeni multirezistenţi pot fi tratate
în comunitate cu antibiotice intravenoase.
R: B, C, D, E (1, p. 159-160)

15. Pacienţii care trebuie îndrumaţi către centrele de alergologie


pentru o testare structurată, sunt:
A. Cei cu istoric de reacţii plurialergice;
B. Cei alergici la antibiotice multiple;
C. Pacienţii care necesită un tratament antibiotic de scurtă durată
şi prezintă alergie la tratamentul „gold standard";
D. Pacientii care necesită cure repetate de antibiotic;
E. Pacienţii la care nu există o alternativă sigură de antibiotic.
R: A, B, D, E (1, p: 160)

16. Carbapenemele:
A. sunt antibiotice β-lactamice semi-sintetice;
B. Au cel mai larg spectru antibiotic;
C. Sunt active împotriva MRSA;
D. Ertapenemul nu este activ impotriva Pseudomonas;
E. Imipenem este parţial inactivat enzimatic în rinichi.
R: A, B, D, E (1, p: 163)
17. Aminoglicozidele:
A. Întrerup sinteza de proteine bacteriene prin inhibarea funcţiei
ribozomilor;
B. Sunt eficiente împotriva a numeroase organisme gram-pozitive;
C. Pot induce nefrotoxicitate dependentă de doză;
D. Ototoxicitatea este mai frecventă atunci când se asociază şi unele
tratamente diuretice;
E. Ototoxicitatea este mai frecventă la pacienţii tineri.

10
R: A, C, D (1, p. 164)

18. Cefalosporinele:
A. Sunt superioare penicilinelor prin faptul că nu sunt inactivate
de penicilinazele stafilococice;
B. Ceftazidima este activă împotriva p. Aeruginosa;
C. Ceftarolina este activă împotriva MRSA;
D. Cefalosporinele de generaţia a doua şi a treia sunt rezervate
pentru tratamentul anumitor infecţii severe;
E. Toxicitatea este similară cu cea a penicilinelor dar este mai rară.
R: A, B, D, E (1, p. 163)

19. Vancomicina:
A. Face parte din clasa glicopeptidelor;
B. Are acţiune rapid bactericidă;
C. Este administrată intravenos pentru MRSA;
D. Este utilizată pentru tratamentul şi profilaxia infecţiilor cu
bacterii Gram-pozitive la pacienţi cu alergie la penicilină;
E. Este administrată în meningita cu S. pneumoniae, în
combinaţie cu alte antibiotice eficiente, când boala este cauzată
de o tulpină cu rezistenţă la penicilină.
R: A, C, D, E (1, p. 164)

20. Chinolonele:
A. Inhibă sinteza de ADN bacterian prin inhibarea
topoizomerazei IV şi a ADN girazei;
B. Pot determina leziuni ale tendoanelor, inclusiv ruptură, în
primele 48h de utilizare;
C. Utilizarea lor trebuie evitată în sarcină şi la copii;
D. Pot induce scurtarea intervalului QT;
E. Pot fi utilizate şi în tratamentul tuberculozei.
R: A, B, C, E (1, p. 164)

11
CAPITOLUL 2
ECHILIBRUL HIDRO-ELECTROLITIC ȘI ACIDO-
BAZIC

1. Apa în organism:
A. Presiunea osmotică este factorul determinant principal al
distribuţiei apei între cele trei compartimente majore;
B. Concentraţiile electroliţilor majori din compartimente sunt
diferite;
C. Potasiul predomină în lichidul interstiţial;
D. Sodiul predomină în lichidul intracelular;
E. Majoritatea ionilor de magneziu din mediul intracelular sunt legaţi
şi osmotic inactivi.
R: A, B, E (1, p. 172)

2. În legătură cu reglarea volumului extracelular sunt adevărate


următoarele afirmaţii:
A. Volumul extracelular este determinat de concentraţia potasiului;
B. Principalul determinant al excreţiei renale de sodiu şi apă este
volumul circulant efectiv;
C. volumul arterial sanguin efectiv depinde de debitul cardiac şi
rezistenţa vasculară periferică;
D. Expansiunea volumul arterial sanguin efectiv determină scăderea
excreţiei urinare a Na+;
E. urina poate deveni liberă de Na + în mod virtual în cazul
depleţiei volumul arterial sanguin efectiv cu funcţie renală
normală.
R: B, C, E (1, p. 173)

3. Receptorii de presiune înaltă sunt localizaţi în:


A. Atriul stâng;
B. Venele toracice majore;
C. Carotidie;
D. Arcul aortic;
E. Aparatul juxtaglomerular.
R: C, D, E (1, p. 173)

12
4. Încărcarea cu sare, determină:
A. Creşterea volumului circulator efectiv;
B. Scăderea volumului extracelular;
C. Creşterea presiunii de perfuzie renală;
D. Scăderea presiunii de umplere atrială;
E. Creşterea presiunii de umplere arterială.
R: A, C, E (1, p. 173)

5. Aportul scăzut de sodiu:


A. Scade volumul circulator efectiv;
B. Potenţează sistemul renină-angiotensină-aldosteron;
C. Creşte secreţia de peptid natriuretic atrial;
D. Creşte reabsorbţia de sodiu în ducturile colectoare;
E. Scade excreţia de sodiu.
R: A, B, D, E (1, p. 173)

6. Formarea edemelor în insuficienţa cardiacă se datorează:


A. Activării sistemului renină-angiotensină-aldosteron;
B. Eliberării osmotice de ADH;
C. Scăderii activităţii simpatice secundar stimulării
baroreceptorilor;
D. Expansiunii volumului extracelular;
E. Creşterii presiunii venoase.
R: A, D, E (1, p. 177)

7. Reacţiile adverse ale diureticelor de ansă sunt reprezentate de:


A. Retenţie de uraţi;
B. Hipomagnezemie;
C. Scăderea toleranţei la glucoză;
D. Hiperpotasemie;
E. Hipocalciurie.
R: A, B, C (1, p. 178)

8. Diureticele cu efect de economisire a potasiului sunt:


A. Torasemid;
B. Spironolactona;
C. Eplerenona;
D. Bumetanida;
E. Acetazolamida.
R: B, C (1, p. 179)

13
9. Diureticele tiazidice:
A. Sunt mai puternice decât diureticele de ansă;
B. Reduc rezistenţa vasculară periferică;
C. Reduc excreţia renală a calciului;
D. Determină retenţie de uraţi;
E. Determină intoleranţă la glucoză.
R: B, C, D, E (1, p. 179)

10. Sunt aquaretice:


A. Amilorid;
B. Triamteren;
C. Lixivaptan;
D. Tolvaptan;
E. Satavaptan.
R: B, C, E (1, p. 179)

11. Cauze de hipotensiune posturală (ortostatică) prin insuficienţă


autonomă, sunt:
A. Medicamente care blochează α-adenoreceptorii;
B. Medicamente blocante ale canalelor de calciu;
C. Antedepresive triciclice;
D. Diabetul zaharat;
E. Boala Parkinson.
R: D, E (1, p. 181)

12. Sindromul Liddle se caracterizează prin:


A. Pierderi de potasiu;
B. Pierderi de sodiu;
C. Hipokaliemie;
D. Alcaloza;
E. Hipotensiune.
R: A, C, D (1, p.188)

13. Hiperkaliemia severă:


A. Este întotdeauna simptomatică;
B. Creşte excitabilitatea cardiacă;
C. Determină hipertensiune arterială;
D. Determină bradicardie;
E. Poate produce asistolă.
R: B, D, E (1, p.190)

14
14. Cauze de acidoză metabolică cu gaură anionică normală:
A. Boala renală cronică;
B. Acidoză lactică;
C. Hiperparatiroidism;
D. Acidoză tubulară renală;
E. Ileostomie.
R: C, D, E (1, p. 199)

15. Cauze de alcaloză metabolică cu depleţie de clor:


A. Pierderi gastrice;
B. Diuretice cloruretice;
C. Fibroza chistică;
D. Aldosteronismul primar;
E. Aldosteronismul secundar.
R: A, B, C (1, p. 202)

16. Referitor la tratamentul acidozei sunt adevărate următoarele


afirmaţii:
A. Corecţia rapidă a acidozei poate duce la tetanie;
B. Convulsiile pot apărea datorită corecţiei rapide a acidozei;
C. Administrarea de bicarbonat creşte producţia de CO2;
D. Se recomandă administrarea de bicarbonat de sodiu la un pH <
7,2;
E. Administrarea de bicarbonat poate induce expansiunea
volumului extracelular.
R: A, B, C, E (1, p. 201)

17. Acidoza lactică:


A. Este de 3 tipuri (A, B si C);
B. Poate apărea datorită vasoconstricţiei la nivel splahnic şi
periferic;
C. Cea mai frecventă formă este tipul B;
D. Metforminul poate fi cauză de acidoză lactică;
E. Lipsa de oxigen la nivel tisular induce acidoză lactică de tip B.
R: B, D (1, p.200)

15
18. Cauze de hiperpotasemie:
A. Spironolactonă;
B. Inhibitori ai enzimei de conversie ai angiotensinei;
C. Diaree;
D. Vărsături;
E. Acidoză.
R: A, B, E (1, p. 189)

19. Sindromul Gitelman se caracterizează prin:


A. Hipokamiemie;
B. Hipocalciurie;
C. Hipomagnezemie;
D. Hipertensiune arterială;
E. Creşterea reninei plasmatice.
R: A, B, C, E (1, p. 188)

20. Sindromul Bartter se caracterizează prin:


A. Acidoză metabolică;
B. Hipokaliemie;
C. Hipercalciurie;
D. Hipertenisune arterială;
E. Creşterea reninei plasmatice.
R: B, C, E (1, p. 187)

16
CAPITOLUL 3
TERAPIE INTENSIVĂ

1. Insuficienţa respiratorie, prin definiţie, este prezentă atunci


când:
A. PaO2 < 65 mmHg;
B. PaCO2 > 55 mmHg;
C. PaO2 < 60 mmHg;
D. PaCO2 > 60 mmHg;
E. PaCO2 > 65 mmHg.
R: B, C (1, p. 224)

2. Insuficienţa ventilatorie:
A. Reprezintă tipul I de insuficienţă respiratorie;
B. Apare atunci când ventilaţia alveolară este insuficientă pentru a
înlătura volumul de bioxid de carbon produs;
C. Cea mai frecventă cauză este BPOC;
D. Apare în sindromul Guillain-Barre;
E. Se datorează şuntului dreapta-stânga.
R: B, C, D (1, p. 224)

3. Tratamentul standard al pacienţilor cu insuficienţă


respiratorie include:
A. administrarea de oxigen suplimentar printr-o cale respiratorie
liberă;
B. tratament pentru obstrucţia căilor aeriene distale;
C. măsuri pentru limitarea edemului pulmonar;
D. profilaxia gastritei de stres;
E. tratamentul infecţiei pulmonare.
R: A, B, C, E (1, p. 225)

4. Toxicitatea oxigenului:
A. Determină leziuni pulmonare secundar creşterii radicalilori liberi
de oxigen;
B. Determină deteriorări oxidative ale ţesutului pulmonar;
C. Pentru prevenirea toxicităţii oxigenului, se consideră a fi
adecvate saturaţii de oxigen de 94-95%;

17
D. Chiar şi o scurtă perioadă de hiperoxie poate creşte
mortalitatea în comparaţie cu normoxia;
E. Toxicitatea oxigenului poate fi constatată şi după intubarea de
urgenţă.
R: A, B, D, E (1, p. 227)

5. Intubaţia pacientului cu insuficienţă respiratorie:


A. Ar trebui efectuată după administrarea de inotrope, atunci când
este necesar;
B. Necesită preoxigenare cu O2 100% înainte de intubaţie;
C. Volumul circulator se optimizează după intubaţie;
D. Poate precipita aritmii periculoase;
E. Poate induce stop cardiac.
R: A, B, D, E (1, p. 227)

6. Complicaţiile imediate ale intubaţiei traheale:


A. Leziuni ale traheei;
B. Edemul mucoasei şi ulceraţiile;
C. Hipoxie;
D. Leziune laringiană;
E. Leziuni ale buzelor.
R: A, C, E (1, p. 228)

7. Traheostomia:
A. Poate fi necesară pentru controlul pe termen lung a căi
aeriene la unii pacienţi cu stare de conştienţă persistent
redusă;
B. Permite gestionarea secreţiilor bronşice excesive;
C. Îngreunează sevrajul respirator;
D. Reduce necesarul de sedare;
E. Îmbunătăţeşte confortul pacientului.
R: A, B, D, E (1, p. 228)

8. Complicaţiile ale traheostomiei:


A. Incapacitatea de vindecare a stomei;
B. Granulomul traheal;
C. Eroziunea cartilajelor traheale;
D. Infecţii ale stomei;
E. Pneumonie.
R: C, D, E (1, p. 229)

18
9. Presiunea pozitivă la sfârşitul expirului:
A. Re-expansionează unităţile pulmonare insuficient ventilate;
B. Scade presiunea intratoracică medie;
C. Scade întoarcerea venoasă;
D. Scade rezistenţa vasculară pulmonară;
E. Reduce debitul cardiac.
R: A, C, E (1, p. 229)

10. Sindromul de detresă respiratorie acută:


A. Definiţia Berlin se aplică la PEEP < 5 cm H2O;
B. Se consideră sever la PaO2/FiO2 < 150 mmHg;
C. Se consideră moderat la PaO2/FiO2 de 100 – 200 mmHg;
D. Se consideră sever la PaO2/FiO2 < 100 mmHg;
E. Se consideră uşor la PaO2/FiO2 de 200 – 300 mmHg.
R: C, D, E (1, p. 232)

19
CAPITOLUL 4
HEMATOLOGIE

1. Semnele specifice observate în diferitele tipuri de anemie includ:


A. Koilonichia - apare în anemia megaloblastică;
B. Icter – apare în anemia feriprivă;
C. Deformări osoase – apar în talasemia majoră;
D. Ulcere la nivelul membrelor inferioare - apar în asociere cu
siclemia;
E. Koilonichia- apare în anemia feriprivă de lunga durată.
R: C, D, E (1, p. 327)

2. Anemia macrocitară normoblastică se poate întâlni în:


A. Deficitul de fier;
B. Deficitul de vitamina B12;
C. Hemoliză;
D. Boală hepatică;
E. Boli autoimune reumatice.
R: C, D (1, p. 328)

3. Următoarele afirmații legate de absorbția fierului sunt false:


A. Poteinele de transport, precum feroportina 1 și hefestină,
transferă fierul din enterocit în circulație în funcție de nivelul
feritinei;
B. Fierul dietetic este redus de la starea feroasă la cea ferică prin
reductaza ferică a marginii în perie a celulei apicale;
C. Proteinele de transport, precum feroportina 1 și hefestină,
transferă fierul din enterocit în circulație în funcție de nivelul
transferinei;
D. Absorbţia fierului poate fi modulată considerabil ca răspuns la
depozitele de fier din corp (reglatorul depozitelor);
E. Celulele de la nivelul criptelor duodenale percep cantitatea de
fier din organism prin legarea transferinei la complexul genic
HFE/B2M/TfR1.
R: A, B, C (1, p. 329)

20
4. Anemia sideroblastică presupune:
A. VEM scăzut;
B. Fier seric scăzut;
C. Capacitate totală de legare a fierului (CTLF) crescută;
D. Feritina crescută;
E. Receptori solubi ai transferinei scăzuti.
R: A, D (1, p. 331)

5. Următoarele investigații prezintă modificări în Anemia


pernicioasă:
A. Bilirubina serică şi LDH pot fi crescute ca urmare a eritropoiezei
ineficiente;
B. Acidul metilmalonic seric este scăzut în deficitul de vitamină
B12, dar testarea este recomandată doar în cazuri complexe ;
C. Homocisteina este crescută în deficitul de vitamină B12, dar
testarea este recomandată doar în cazuri complexe ;
D. Holotranscobalamina este fracţia „inactivă" a cobalaminei , iar
mă surarea acesteia poate fi un marker mai relevant pentru
deficitul de vitamină B12 decât dozarea serică a acesteia;
E. Valoarea serică a vitaminei B12 este de obicei mult scăzută, sub
100 ng/L, ceea ce reprezintă limita inferioară a normalului.
R: A, C (1, p. 334-335)

6. Acidul folic:
A. Este prezent în natură sub formă de monoglutamat;
B. Poliglutamații sunt degradați în monoglutamaţi la nivelul
tractului gastrointestinal superior;
C. Monoglutamatul metil THF este forma principală din ser;
D. Metilarea homocisteinei la metionină necesită doar
metilcobalamină;
E. Poliglutamaţii intracelulari sunt formele active ale folatului şi
acţionează ca şi coenzime în transferul moleculelor unice de
carbon în metabolismul aminoacizilor şi sinteza ADN-ului.
R: B, C, E (1, p. 335)

7. Deficitul de folați apare datorită utilizării excesive în:


A. Hemodializă/dializă peritoneală;
B. Consumul de alcool;
C. Hemoliză;

21
D. Malabsorbție – boala intestinului subțire;
E. Boală metabolică – homocistinuria.
R: A, C, D, E (1, p. 335)

8. Investigațiile în Anemiile hemolitice autoimune cu anticorpi la


„cald”:
A. Sferocitoza este prezentă ca urmare a deteriorării eritrocitare;
B. Testul antiglobulinic direct este pozitiv, doar cu Ig M (35%), cu
Ig M și complement ( 56%) sau doar complement (9%) prezente
pe suprafața eritrocitară;
C. Poate să apară trombocitopenia autoimună;
D. Poate să apară neutropenia;
E. Autoanticorpii nu pot avea afinitate pentru sistemul Rh al
grupelor sanguine.
R: A, C, D (1, p. 352)

9. Hemoglobinuria paroxistică la rece :


A. Este o afecţiune frecventă asociată cu infecţiile comune copiilor,
cum ar fi pojarul, oreionul şi varicela;
B. Hemoliza intravasculară este asociată cu anticorpi lgM cu
afinitate pentru complement;
C. Anticorpii sunt bifazici, reacţionând cu celulele roşii la rece în
circulaţia periferică, liza apărând datorită activării
complementului la reîntoarcerea celulelor în circulaţia centrală.;
D. Reacţia litică este demonstrată în vitro prin incubarea
eritrocitelor pacientului şi a serului la 4°C, iar apoi încălzirea
compoziţiei la 37° (testul Donath-Landsteiner);
E. Este o formă rară de anemie hemolitică care rezultă din expresia
clonală a celulelor stem hematopoetice care prezintă mutaţii în
genele PIG-A X-linkate.
R: C, D (1, p. 353)

10. Abordarea terapeutică postnatală în Anemia hemolitică a nou-


născutului:
A. În cazurile uşoare, fototerapia poate fi utilizată pentru
convertirea bilirubinei în biliverdină, care este hidrosolubilă, care
ulterior poate fi excretată renal;
B. Anti D ar trebui administrați la mamă , după naștere, atunci când
fătul este Rh D negativ;

22
C. Profilaxia poate fi necesară în cazul femeilor Rh D-negative când
sensibilizarea apare în urma unei sarcini ectopice, iminenţă de
avort sau avort;
D. Dacă sunt prezente în circulaţia maternală un număr crescut de
eritrocite fetale, nu este necesară o doză crescută sau adiţională
de anti-D;
E. Doza de anti-D este 250 UI anterior săptămânii 20 de sarcină şi
500 Ul după 20 de săptămâni.
R: A, C, E (1, p. 354)

23
CAPITOLUL 5
REUMATOLOGIE

1. Etiologia lupusului eritematos sistemic cuprinde:


A. Rudele de gradul I au şansa de 20% în dezvoltarea bolii;
B. Femeile în menopauză mai frecvent afectate;
C. Deficienţa homozigotă a genei complementului C2 şi C4 se
poate asocia cu risc mai mare de dezvoltare a bolii;
D. Aproximativ 20 de gene sunt asociate cu dezvoltarea lupusului;
E. Infecţia Herpes Zoster este considerată factor declanşator.
R: C, D (1, p. 457)

2. Patogenia lupusului eritematos sistemic cuprinde:


A. Prezentarea către Ly T a resturilor de apoptoză celulară ce conţin
ADN şi histone;
B. Menţinerea toleranţei imune este implicată în producerea de
autoanticorpi şi a complexelor imune prezente în lupus;
C. Celulele T natural killer invariabile sunt în număr crescut la
pacienţii cu lupus;
D. Alte celule implicate în lupus sunt: celulele dendritice, celulele T
natural killer invariabile;
E. În lupus este o producţie anormală de citokine: interferon
gamma, IL 6, IL 10,
R: A, D (1, p. 457)

3. Tabloul clinic al lupusului eritematos sistemic cuprinde:


A. Majoritatea pacienţilor prezintă febră;
B. Afectarea articulară este cea mai frecventă manifestare clinică;
C. Fatigabilitatea şi febra se corelează cu activitatea bolii şi
severitatea afectărilor de organ;
D. Artralgiile sunt simetrice şi afectează articulaţiile mici;
E. Artropatia Jaccoud reprezintă deformarea articulară şi eroziunea
apărute la articulaţiile mâinii.
R: B, D (1, p. 457)

24
4. Manifestările pulmonare în cadrul lupusului eritematos sistemic:
A. Fibroza pulmonară, hemoragia pulmonară sunt frecvente;
B. Disfuncţia pulmonară restrictivă cu ascensionarea de
hemidiafragme poate fi prezentă;
C. Peste 50% dintre pacienţii cu lupus sistemic pot prezenta
pleurezii recurente;
D. Afectarea pulmonară este mai rară în lupusul eritematos
sistemic;
E. Hemoragia intrapulomnară asociată vasculitei din lupus este o
complicaţie rară.
R: B, C, E (1, p. 458)

5. În lupusul eritematos sistemic afectarea organelor vitale


cuprinde:
A. Leziunile valvulare aortice sunt frecvent întâlnite;
B. Boala cardiacă ischemică şi accidentul vascular au o incidenţă
crescută;
C. Combaterea factorilor de risc cardiovasculari şi terapia cu statine
modifică riscul de dezvoltare a accidentului vascular cerebral şi a
bolii coronariene;
D. Pericardita şi miocardita sunt frecvente;
E. Endocardita noninfecţioasă cu interesare valvulară mitrală apare
frecvent.
R: B, D (1, p. 458)

6. Lupusul neurologic descrie:


A. Afectarea sistemului nervos apare în până la8= 60% din cazurile
de lupus eritematos sistemic;
B. Leziunile din lupusul neurologic sunt rezultatul depunerii locale
de complexe imune;
C. Leziunile din lupusul neurologic sunt rezultatul
microvasculopatiei noninflamatorii şi a trombozei;
D. În cazul apariţiei infecţiilor în lupusul neurologic se impune
oprirea terapiei imunosupresoare;
E. Tulburările psihice severe sunt frecvente în lupusul neurologic.
R: A, B, C (1, p. 458)

25
7. Afectarea renală în lupusul eritematos sistemic:
A. Are expresie clinică la 30 % din cazuri dar modificările
histopatologice sunt foarte frecvente;
B. Albumina serică scăzută şi raportul albumină/creatinină urinară
sunt indicatorii tardivi ai nefritei lupice;
C. Ureea şi creatinina pot creşte din faze incipiente ale bolii renale
în lupus;
D. Biopsia renală este cea care defineşte tipul şi severitatea
nefropatiei lupice;
E. Tromboza venei renale apare în sindromul nefrotic.
R: A, D, E (1, p. 458)

8. Diagnosticul paraclinic al lupusului eritematos sistemic cuprinde:


A. Proteina C reactivă crescută în caz de pleurezie lupică sau
peritonită sau artrită;
B. În 40% din cazurile de lupus anticorpii antifosfolipidici sunt
prezenţi dar nu toţi pacienţii dezvoltă şi sindrom antifosfolipidic;
C. Valori crescute ale C3 şi C4 în cazul bolii active;
D. Anticorpii anti-Ro şi anti-La sunt cei mai semnificativi
autoanticorpi în lupus;
E. IRM cerebral poate detecta leziuni cerebrale în lupus.
R: A, B, E (1, p. 459)

9. Evoluţia şi prognosticul lupusului eritematos sistemic:


A. Boala evoluează cu exacerbări şi remisiuni lungi;
B. Rata supravieţuirii la 10 ani este redusă;
C. Decesele timpurii apar în lupusul neurologic, cu afectare renală
sau cu complicaţii infecţioase;
D. Pacienţii cu lupus pot dezvoltă limfoame;
E. Pacientele cu lupus prezintă infertilitate.
R: A, C, D (1, p. 459)

10. Epidemiologia bolilor inflamatorii autoimune cuprinde:


A. Prevalenţa crescută a Artritei reumatoide la populaţia negrilor
din Africa şi a asiaticilor;
B. Lupusul este mai frecvent la femei, în raport de 9:1
(femei/bărbaţi);
C. Lupusul este mai frecvent la femeile africane;
D. Debutul Artritei reumatoide este, frecvent, sub 20 ani;

26
E. Populaţia din Africa şi Japonia au o incidenţă scăzută a HLA
B27 pozitiv şi a Spondilartritei axiale.
R: B, C, E (1, p. 437, 457, 448)

11. Etiopatogeneza Artritei reumatoide:


A. Asocierea cu halotipurile de HLA DR4, HLA DRB1 cresc
susceptibilitatea de dezvoltare a bolii;
B. Asocierea cu halotipul HLA DB1 nu predispune la dezvoltarea
APCA(anticorpii anti-peptide citrullinate);
C. Modificările microbiomului intestinal predispune la boala
paradontală nu şi la apariţia Artritei reumatoide;
D. Factorul reumatoid pozitiv apare la 80% dintre pacienţii cu
Artrită reumatoidă şi are valoare predictivă mare în populaţie;
E. APCA (anticorpii anti-peptide citrulinate) au susceptibilitate şi
sensibilitate mare în diagnosticul Artritei reumatoide.
R: A, E (1, p. 437-438)

12. Artrita reumatoida prezintă o sinovită proliferativă articulară:


A. Sinoviala hipertrofiată prezintă o proliferare vasculară marcată;
B. Mecanismul de afectare a cartilajului subiacent este exclusiv prin
blocarea căii normale de nutriţie a acestuia;
C. În timpul sinovitei active osteoporoza juxta-articulară este
consecinţa imobilizării articulare şi a citokinelor secretate local;
D. Leziunile distructive precoce justifică utilizarea medicamentelor
modificatoare de boală (DMARD) încă din primele 3 luni de la
debutul artritei;
E. În primele 3 – 6 luni de la debutul artritei sunt diagnosticate
radiologic eroziuni osoase.
R: A, C, D (1, p. 439)

13. Manifestările extra-spinale care sugerează diagnosticul de


Spondilită anchilozantă (SA):
A. Uveita anterioară acută – apare la aproximativ 30% dintre pacienţi
şi poate fi manifestarea de prezentare;
B. Inflamaţia joncţiunii costocondrale – provoacă dureri toracice
anterioare;
C. Afectarea articulară periferică – este simetrică, la nivelul
articulaţiilor interfalangiene proximale;

27
D. Afectarea sistemului nervos – apare la aproximativ 60% dintre
pacienţi şi se manifestă prin epilepsie, ataxie cerebeloasă, leziuni
ale nervilor cranieni sau polineuropatie;
E. Afectare cardiovasculară – insuficienţa aortică apare până la 1%
la pacienţii cu SA constituită.
R: A, B, E (1, p. 448-449)

14. Managementul terapeutic în Spondilita anchilozantă presupune:


A. Exerciţii matinale cu scopul de a menţine mobilitatea spinală,
postura şi expansiunea toracală;
B. Când inflamaţia este activă, durerea şi redoarea matinală sunt
severe, se recomandă exerciţii fizice intense;
C. AINS cu acţiune lungă sau cu eliberare lentă îmbunătăţesc somnul
şi controlul durerii;
D. Sulfasalazina poate fi utilă pentru afectarea spinală;
E. Când AINS au eşuat, sunt folosiţi blocanţii TNF alfa pentru a
reduce substanţial simptomele inflamaţiei articulaţiilor spinale şi
periferice.
R: A, C, E (1, p. 449)

15. Formele clinice ale Artritei Psoriazice (APs) includ:


A. Mono- sau oligo-artrita;
B. Artrita reactivă;
C. Spondilita;
D. Artrita mutilantă;
E. Artrita articulaţiilor interfalagiene distale.
R: A, C, D, E (1, p. 450)

16. Tabloul clinic din artrita reactiva prezintă:


A. Artrita acută, asimetrica, a membrelor inferioare care apare după
zile până la săptămâni de la infecţia declanşatoare;
B. Entezita este frecventă , provocând fasciita plantară sau entezita
tendonului lui Ahile;
C. Keratoderma blennorragica care provoacă ulceraţii superficiale
nedureroase ale glandului penisului;
D. Nu se poate dezvolta sacroileita sau spondilita;
E. Leziunile cutanate seamănă cu psoriazisul, apare şi distrofia
unghiilor.
R: A, B, E (1, p. 451)

28
17. Modalităţile prin care HLA-B27 creşte susceptibilitatea la Artrita
Reactivă nu includ:
A. Modul de prezentare a peptidelor derivate din bacterii către
limfocitele T;
B. Selectarea repertoriului receptorilor limfocitelor B;
C. Imitarea moleculară care cauzează autoimunitate împotriva HLA-
B27;
D. Producerea în exces a factorului de necroză tumorală alfa (TNF-
alfa), determinate de interacţiunea macrofagelor cu limfocitele B
şi T;
E. Selectarea repertoriului receptorilor limfocitelor T.
R: B, D (1, p. 451)

18. Cauze ale hiperuricemiei datorate afectării excreţiei acidului


uric:
A. Hiperactivitatea fosforibozil-pirofosfat sintazei;
B. Boala renală cronică;
C. Hiperparatiroidism, hipotiroidism;
D. Boli mieloproliferative: policitemia Vera;
E. Deficitul de glucozo-6-fosfataza.
R: B, C, E (1, p. 452)

19. Managementul terapeutic în Gută presupune:


A. Utilizarea agenţilor hipo-uricemianti cu scopul de a reduce şi de a
menţine nivelul acidului uric sub 5,04 mg/dl la toţi pacienţii;
B. Utilizarea coxibilor în doze mari, în primele 24-48 h, pentru a
reduce rapid durerea şi tumefacţia;
C. Utilizarea Colchicinei de 2-3 ori pe zi pentru terminarea atacului;
D. Utilizarea Alopurinolului atunci când atacurile sunt frecvente şi
severe sau asociate cu insuficienţa renală;
E. Utilizarea Losartanului – blocant de Calciu, uricozuric la pacienţii
hipertensivi.
R: B, C, D (1, p. 453)

20. Artropatia prin depunere de pirofosfat de calciu dihidrat descrie:


A. Depunerea cristalelor de pirofosfat de calciu dihidrat în cartilajul
hialin şi în fibrocartilaj;
B. Detectarea în lichidul articular a cristalelor în formă de ac cu
birefrigenţă negativă;

29
C. Depunerile de cristale de pirofosfat de calciu dihidrat într-o
articulatie precipită sinovita acută;
D. Lichidul articular pare purulent astfel încât ar trebui sa fie trimis
pentru cultură;
E. Nu asociază niciodată febra şi leucocitoza.
R: A, C, D (1, p. 454)

30
CAPITOLUL 6
ENDOCRINOLOGIE

1. Cauzele hipotiroidismului primar congenital sunt:


A. Tiroidita postpartum;
B. Tiroidita virală;
C. Agenezia tiroidiană;
D. Hipotiroidismul central;
E. Tiroida ectopică.
R: C, E (1, p. 611)

2. Tiroidita Hashimoto
A. Este o boală autoimună;
B. Mai frecventă la sexul masculin;
C. Anticorpii anti TPO în titruri crescute confirmă diagnosticul;
D. Are 2 forme clinice - forma atrofică și forma gușogenă;
E. Profilul hormonal îmbracă întotdeauna status normotiroidian
spontan.
R: A, C, D (1, p.612)

3. Boala Basedow Graves reprezintă:


A. Cea mai frecventă cauză de hipertiroidie autoimună;
B. Rata de concordanță a maladiei în rândul gemenilor monozigoți
este de 75%;
C. Există o asociere și o predispoziție a indivizilor ce prezintă HLA
B8, DR3 si DR2;
D. Singura opțiune terapeutică este tiroidectomia totală;
E. Anticorpii anti-receptor pentru TSH(Trab) reprezintă elementul
patognomonic pentru confirmarea maladiei.
R: A, C, E (1, p.614)

31
4. Tireotoxicoza amiodaron-indusă tip II:
A. Apare pe o tiroidă anterior indemnă/sănătoasă;
B. Apare pe o tiroidă ce asociază patologii preexistente - gușa
multinodulară/boala Basedow Graves;
C. Apare în urma efectului direct citotoxic al amiodaronei asupra
celulei foliculare tiroidiene cu eliberare în circulație a hormonilor
preformați T3,T4;
D. Amiodarona stimulează deiodinarea T4 în T3 asociind astfel
raporturi T4:T3 mai mare decât uzual;
E. Tiroidectomia totală este indicația terapeutică de urgență.
R: A, C (1, p. 615)

5. Din medicația anti-tiroidienelor de sinteză Carbimazolul are


următoarele efecte adverse:
A. Agranulocitoza;
B. Erupții cutanate;
C. Artralgii;
D. Micțiuni imperioase;
E. Cefalee violentă.
R: A, B, C (1, p.616)

6. Printre complicațiile tiroidectomiei totale se recunosc:


A. Sângerare precoce postoperator cauzând compresie și asfixie;
B. Hipercalcemie persistentă;
C. Hiperfosfatemie severă;
D. Pareza de nerv laringeu ( 1%);
E. Hipoparatiroidism tranzitoriu/permanent.
R: A, D, E (1, p.617)

7. Furtuna tiroidiană se caracterizează prin:


A. Mortalitate 100%;
B. Hipertiroidism extrem;
C. Hiperpirexie;
D. Bradicardie severă;
E. Lentoare psihomotorie.
R: B, C (1, p. 617)

32
8. Oftalmopatia Basedowiană:
A. Caracterizează un răspuns imun specific ce determină inflamația
retro-orbitară;
B. Protruzia GO este aproape întotdeauna unilaterală;
C. Niciodată nu se asociază atrofia optică;
D. Evoluează mai puțin sever la cei fumători;
E. Evoluția ei este independentă de profilul funcțional tiroidian
(hiper/hipo/normotiroidie).
R: A, E (1, p. 618)

9. Cancerul papilo-folicular :
A. Sunt cancere diferențiate derivate din epiteliul folicular tiroidian;
B. Tiroidectomia totală/sau cvasi totală este terapia de elecție;
C. Disecția limfoganglionară regională sau extinsă nu este necesară
în contextul implicării structurilor loco-regionale;
D. Tiroglobulina serică este markerul tumoral pentru monitorizarea
clinică și biochimică a pacienților tiroidectomizați total;
E. Plămânii și scheletul sunt sedii de afinitate pentru metastazare
secundară.
R: A, B, D, E(1, p. 621)

10. Cancerul medular tiroidian:


A. Tumora neuroendocrină ce derivă din celulele C parafoliculare
din tiroidă;
B. 75 % din pacienții diagnosticați cu CMT prezintă o mutație a
proto-oncogenei RET;
C. Calcitonina reprezintă markerul tumoral al CMT;
D. Se indică lobectomie fără evidare ganglionară;
E. Se indică screeningul familial și consiliere genetică a celor cu
mutații MEN 2 pozitive.
R: A, C, E (1, p. 622)

33
CAPITOLUL 7
DIABETUL ZAHARAT

1. Complicațiile microvasculare care apar pe termen lung în contextul


unui diabet dezechilibrat sunt:
A. Boala coronariană;
B. Retinopatie diabetică;
C. Nefropatie diabetică;
D. Accident vascular cerebral;
E. Boli vasculare periferice.
R: B, C (1, p. 702)

2. Insulina se caracterizează prin:


A. Hormon peptidic de 51 de aminoacizi care cuprinde 2 lanțuri
polipeptidice A și B;
B. Ca răspuns la ingestia de alimente se produce o descreștere a
concentrației de Insulină în circulație care revine la nivelul inițial în
termen de 2 ore;
C. Este sintetizat în celulele beta pancreatice ale insulelor Langerhans;
D. 100% degradată în ficat;
E. Insulina este secretată într-o rată bazală rapidă pe parcursul zilei
rezultând o concentrație ridicată de insulină între mese și peste
noapte.
R: A, C (1, p. 702)

3. Clasificarea diabetului mellitus cuprinde:


A. Diabet zaharat de tip 1 (autoimun);
B. Diabet zaharat de tip 2 (care rezultă dintr-o combinație de rezistență
la acțiunea insulinei și deficit de insulină mai puțin sever);
C. Diabet insipid;
D. Diabet secundar ( defectelor genetice, bolilor pancreasului exocrin,
bolilor endocrine, infecțiilor);
E. Toate răspunsurile de mai sus sunt adevărate.
R: A, B, D (1, p.704)

4. Diabetul zaharat autoimun se caracterizează prin:


34
A. Deficit sever de insulină;
B. Reprezintă 75% din toate cazurile de diabet;
C. Atinge o incidență maximă în perioada adultă;
D. Cuprinde și o variantă cu progresie mai lentă care apare târziu în
viață LADA (latent autoimmune diabetes in adults);
E. Este împărțit în tip 1A (mediat imun) și tip 1B (non- mediat imun).
R: A, D, E (1, p. 705)

5. În diabetul zaharat autoimun apar în circulație autoanticorpi


îndreptați împotriva antigenelor insulelor pancreatice:
A. Insulina;
B. Enzima decarboxilaza a acidului glutamic (GAD);
C. Tiroperoxidaza;
D. Proteina tirozin fosfatază (IA-2);
E. Tireoglobulina.
R: A, B, D (1, p. 706)

6. Susceptibilitatea genetică este incriminată în diabetul zaharat


autoimun cea mai mare contribuție o au polimorfismele din
regiunea HLA:
A. HLA-DR3-DQ2;
B. HLA-DQB1-0602;
C. HLA-DR4-DQ8;
D. HLA B27;
E. HLA C10.
R: A, C (1, p.707)

7. Care dintre următoarele afirmații despre Diabetul zaharat tip 2


sunt adevărate?
A. Susceptibilitatea genetică se confirmă și la tipul 2 de DZ , factorul
de transcriere 7-like 2 (TCF7-L2) este cea mai frecventă variantă
observată la europeni;
B. Este o tulburare poligenică, dar majoritatea markerilor genetici
explică mai puțin de 20% din ereditatea diabetului zaharat tip 2;
C. Funcția celulelor beta pancreatice crește cu vârsta și astfel incidența
DZ de tip 2 scade cu înaintarea în vârstă;
D. Diabetul tip 2 devine din ce în ce mai frecvent la copii și la adulții
tineri;
E. Greutatea scăzută la naștere predispune la intoleranța la glucoză mai
târziu.

35
R: A, B, D, E (1, p. 708)

8. Care sunt anomaliile de acțiune ale insulinei întâlnite în diabetul


zaharat tip 2?
A. Incapacitatea insulinei de a inhiba producția hepatică de glucoză;
B. Incapacitatea de a stimula absorbția de glucoză în mușchiul
scheletic;
C. Incapacitatea de stimula și accelera lipoliza în țesutul adipos;
D. Incapacitatea de a-și exercita efectele biologice la concentrații
fiziologice;
E. Mecanismul de bază din care rezultă Insulinorezistența nu implică
excesul de nutrienți.
R: A, B, D (1, p.708)

9. Diabetul zaharat monogenic se caracterizează prin:


A. Este determinat de o mutație genetică unică care afectează
prodominant funcția celulelor beta;
B. Dețin un procent de 1-3% dintre persoanele cu diabet diagnosticat
sub vârsta de 30 ani (MODY = maturity onset diabetes of the
young);
C. Poate debuta încă de la naștere, în perioada neonatală;
D. Cea mai frecventă cauză a DZ neonatal permanent sunt mutațiile din
gena KCNJ11;
E. Diabetul rezultă în urma distrugerii celulelor beta pancreatice, și nu
din anomaliile de eliberare a insulinei.
R: A, B, C, D (1, p.709)

10. Debutul acut al diabetului zaharat se manifestă prin:


A. Oligoanurie;
B. Polidipsie;
C. Scădere ponderală;;
D. Cetoacidoză;
E. Cetonurie.
R: B, C, D, E (1, p. 710)

11. Complicațiile ca modalitate de prezentare a Diabetului zaharat


includ:
A. Infecții tegumentare cu Pseudomonas aeruginosa;

36
B. Retinopatie remarcată la consultul oftalmologic;
C. Disfuncție erectilă;
D. Polineuropatie manifestată prin furnicături și amorțeală la nivelul
picioarelor;
E. Hipoglicemie.
R: B, C, D (1, p. 710)

12. Care dintre următoarele se regăsesc între criteriile de diagnostic al


diabetului zaharat:
A. Glicemia bazală a jeun > 7,0 mmol/L (126 mg/dL);
B. Glicemia în orice moment al zilei < 100 mg/dL;
C. HbA1c > 6.5%;
D. Glicemia în orice moment al zilei > 200mg/dL;
E. În cazul pacienților simptomatici sunt necesare 2 determinări
anormale.
R: A, C, D (1, p. 710)

13. Alterarea toleranței la glucoză conform testului oral de toleranță la


75 grame glucoză (după criteriile Organizației Mondiale a
Sănătății) identifică următoarele valori la 2 ore după ingestia
acesteia:
A. 7.8 - 11.0 mmol/L;
B. > 70 mmol/L;
C. 140 - 200 mg/dL;
D. < 140 mg/dL;
E. < 7.8 mmol/L;
R: A, C (1, p.710)

14. Controlul glicemic mai puțin strict se recomandă în următoarele


situații:
A. Istoric de hipoglicemie severă;
B. Durata lungă a bolii;
C. Durata scurtă a diabetului;
D. Istoric fără hipoglicemii;
E. Durata de viață limitată.
R: A, B, E (1, p.712)

15. Care dintre următoarele afirmații sunt corecte vizând educația


nutrițională și dieta recomandată pacienților diabetici:

37
A. Consumul crescut de carne roșie și procesată, hidrați de carbon
rafinați și băuturi îndulcite cu zahăr;
B. Înlocuirea grăsimilor nesaturate cu grăsimi saturate și creșterea
aportului de acizi grași trans;
C. Persoanele cu DZ tip 2, supraponderale trebuie să piardă cel puțin
5% din greutatea inițială;
D. Reducerea consumului de sare <6 g/zi;
E. Consumul a două porții de pește gras în fiecare săptămână.
R: C, D, E (1, p. 713)

16. Analogii de insulină cu acțiune scurtă (lispro, aspart):


A. Sunt concepuți să disocieze mai rapid după injectare fără a se
modifica efectul biologic;
B. Intră în circulație mai lent decât insulina solubilă umană;
C. Îmbunătățește controlul glicemic reducând total hipoglicemiile
nocturne la pacienții diagnosticați cu DZ tip 1;
D. Beneficiile la pacienții cu DZ tip 2 se cuantifică prin controlul mai
bun al Hb A1C și al glicemiilor post-prandiale;
E. Injectarea insulinei cu acțiune scurtă se realizează postprandial.
R: A, C, D (1, p. 714)

17. Analogii de insulină cu durată lungă de acțiune:


A. Au fost dezvoltați pentru a întârzia absorbția și a prelungi durata de
acțiune;
B. Au un risc mai mic de a produce hipoglicemii nocturne decât
insulina NPH
(neutral protamine Hagedorn);
C. Analogii de insulină sunt mai ieftini decât insulinele NPH;
D. Este terapie de primă intenție la pacienții diabetici atât de tip 1 cât și
de tip 2;
E. Sunt rezervate doar pacienților care nu răspund bine la tratamentul
cu insulină NPH.
R: A, B, E (1, p.714)

18. Regimul bazal - bolus:


A. Tratament de primă alegere pentru pacienții cu DZ tip 1 și cei tineri
cu DZ tip 2;
B. Presupune administrarea concomitentă de insulină cu durată scurtă
și lungă de acțiune;

38
C. Insulina cu acțiune scurtă se injectează imediat postprandial sub
formă de bolus;
D. Insulina cu durată lungă de acțiune se administrează injectabil cu
puțin timp înainte de mesele principale;
E. Regim terapeutic care imită cel mai bine fiziologia normală a
insulinei.
R: A, B, E (1, p. 714)

19. Hipoglicemia ca efect advers al insulinoterapiei:


A. ˝Hipoglicemia semnificativă˝ este definită ca o hipoglicemie situată
între 54 mg/dL si 70 mg/dL;
B. Persoanele tratate cu insulină experimentează mai multe episoade de
hipoglicemie simptomatică pe săptămână;
C. Este mai frecventă la persoanele cu DZ tip 1;
D. ˝Valoarea de alertă˝ se înregistrează doar la glicemii sub 40 mg/dL;
E. Apare mai frecvent la copii și la persoanele care încearcă un control
glicemic strict.
R: B, C, E (1, p. 716)

20. Hipoglicemia ca efect advers al insulinoterapiei este caracterizată


de :
A. Simptome autonome prin activarea adrenergică și colinergică a
sistemului nervos autonom;
B. Simptome neuroglicopenice prin furnizarea adecvată de glucoză
către creier;
C. Apare din cauza obiceiurilor alimentare neregulate;
D. Apare în urma unui exercițiu intens neprogramat;
E. Este o urgență medicală care necesită terapie imediată cu glucoză
orală sau administrare intravenoasă.
R: A, C, D, E (1, p. 716)

21. Metforminul:
A. Este o sulfoniluree care acționează asupra celulelor beta pancreatice
pentru a induce secreția de insulină;
B. Nu poate fi administrat în asociere cu agoniști ai receptorului
peptidei 1-glucagon like( GLP-1);
C. Nu trebuie inițiat la persoanele cu o rată de filtrare glomerulară
estimată mai mică de 45ml/min/1,73m2;

39
D. Interferă cu explorările imagistice care utilizează substanță iodată de
contrast din cauza riscului de insuficiență renală și acidoză lactică
ulterioară;
E. Ca efecte adverse asociază episoade hipoglicemice frecvente și
creștere în greutate.
R: C, D (1 p. 717)

22. Sulfonilureele:
A. Este indicație de primă linie la pacienții cu diabet zaharat tip 1;
B. Ca mecanism de acțiune se leagă de receptorul sulfonilureei de pe
membrana celulelor beta pancreatice blocând influxul de potasiu;
C. Hipoglicemia reprezintă cea mai frecventă reacție adversă;
D. Se administrează cu mare precauție la persoanele cu boală hepatică
și renală;
E. Aportul excesiv de alcool crește riscul de hipoglicemie.
R: C, D, E (1, p. 717, 718)

23. Meglitinidele:
A. Hipoglicemia și creșterea în greutate sunt cele mai frecvente efecte
adverse;
B. Sunt recomandate persoanelor cu hiperglicemie posprandială, ce
prezintă un nivel normal al glicemiei bazale;
C. Acționează prin închiderea canalelor de K-ATP în celulele beta
pancreatice;
D. Dețin o durată lungă de acțiune fiind contraindicate la persoanele în
vârstă, fragile unde este imperioasă evitarea hipoglicemiei;
E. Au fost concepute pentru a restabili eliberarea de insulină în faza
post-prandială.
R: A, B, C, E (1, p. 718)

24. Tiazolidindionele:
A. Cresc rezistența la insulină prin interacțiunea cu receptorul nuclear
PPAR-gamma;
B. Cel mai frecvent efect advers este creșterea în greutate;
C. Se recomandă utilizarea doar în asociere cu insulina NPH;
D. Mecanismul indirect de acțiune prin intermediul ciclului glucoză -
acizi-grași, scade nivelul de acizi grași , promovând astfel consumul
de glucoză în mușchi;
E. Potențează efectul insulinei endogene sau injectate.
R: B, D, E (1, p.718)

40
25. Inhibitorii de dipeptil-peptidază-4 (DPP-4):
A. Stimulează enzima DPP4 fapt care impiedică activarea rapidă a
GLP1;
B. Sunt indicate ca terapie de linia a doua în combinație cu mertformin
sau o sulfoniluree;
C. Dețin un risc scăzut de apariție ca efect advers al hipoglicemiei;
D. Pot fi folosite în siguranță la pacienții cu insuficiență renală;
E. Inhibitorii DPP-4 modifică incidența infarctului miocardic.
R: B, C, D (1, p.718,719)

26. Inhibitori ai transportorului de sodiu-glucoză(flozine):


A. Scad pragul renal pentru glucoză;
B. Facilitează pierderea în greutate;
C. Cresc riscul bolilor cardio-vasculare;
D. Produc vasoconstricție prin stimularea sistemului renină-
angiotensină;
E. Reduc natriureza determinând retenție hidro-salină și creșteri ale
TA.
R: A, B (1, p.719)

27. Efectele adverse ale inhibitorilor SGLT2 (Inhibitori ai


transportorului de sodiu-glucoză):
A. Candidoza genitală;
B. Deshidratarea;
C. Gangrena Fournier;
D. Cetoacidoza diabetică;
E. Alopecie.
R: A, B, C, D (1, p.719)

28. Inhibitorii de alfa-glucozidază:


A. Scad glicemia post-prandial prin reducerea absorbției
carbohidraților după ingestia de alimente;
B. Inhibă acțiunea alfa glucozidazei și mai precis descompunerea
dizaharidelor în monozaharide;
C. Au indicație limitată deoarece prezintă efecte majore neurologice;
D. Efectele adverse gastrointestinale apar din cauza carbohidraților
neabsorbiți și fermentați în intestin;
E. Toate afirmațiile de mai sus sunt false.
R: A, B, D (1, p. 719)

41
29. Cetoacidoza diabetică :
A. Reprezintă o complicație amenințătoare de viață a diabetului
zaharat;
B. Prezintă o rată de mortalitate de aproximativ 20%;
C. Apare cu o frecvență mai ridicată la vârstnici;
D. Este cauzată de un deficit marcat de insulină, și apare astfel mai
frecvent la persoanele cu DZ tip 1;
E. Într-un procent de 30-40% apare în condiții de stres sau patologii
intercurente/infecții.
R: A, D, E (1, p. 722)

30. Simptomele cetoacidozei diabetice:


A. Astenie;
B. Obnubilare;
C. Hiperhidratare;
D. Dureri abdominale;
E. Se prezintă în coma într-un procent de 80%.
R: A, B, D (1, p.723)

31. Diagnosticul cetoacidozei diabetice include:


A. Cetonemie >= 31mg/dl;
B. Cetonurie > = 2 pe stripurile urinare standard;
C. Glicemia < 200mg/dL;
D. Ph venos >7.3;
E. Bicarbonat < 15mmol/L;
R: A, B, E (1, p.723)

32. Starea hiperosmolară hiperglicemică:


A. Este caracteristică pacienților în etate cu DZ tip 2 necontrolat;
B. Nivelul endogen de insulină este suficient pentru a inhiba cetoza dar
insuficient pentru a inhiba producția hepatică de glucoză;
C. Deshidratarea ca semn caracteristic din tabloul clinic se instalează
mai repede la tineri;
D. Deprimarea stării de constiență este independentă de gradul de
osmolaritate;
E. Diureticele tiazidice se enumeră printre factorii precipitanți ai
acestei condiții medicale.
R: A, B, E (1, p.725)

42
33. Acidoza lactică se caracterizează prin:
A. Acidoza metabolică severă, cu o gaură anionică importantă;
B. Obligatoriu cu cetoză semnificativă;
C. Hipoglicemie severă;
D. Sunt predispuse persoanele care urmează tratament cu metformin și
asociază disfuncție hepatică sau renală severă;
E. Mortalitatea este mică< 2 %.
R: A, D (1, p.725)

34. Retinopatia diabetică neproliferativă:


A. Este complicația DZ diagnosticată cel mai rar;
B. Caracterizează cele mai precoce modificări ale retinei fiind
denumită și retinopatie incipientă;
C. Elementul patognomonic este reprezentat de ocluzia vaselor
sanguine ce conduce la arii neperfuzate către capilare;
D. Elementul patognomonic îl reprezintă lezarea pereților vaselor mici
ce cauzează microanevrisme;
E. Poate determina maculopatie dacă retinopatia neproliferativă este
situată în cadrul zonei maculare, responsabilă de vederea în detaliu.
R: B, D, E (1, p. 726)

35. Care din următoarele afirmații despre modificările morfologice


din cadrul retinopatiei diabetice sunt corecte?
A. Retinopatia neproliferativă caracterizează anomalii microvasculare
intraretinale și hemoragii rotunde, multiple și profunde;
B. Retinopatia avansată se caracterizează prin microanevrisme capilare
și hemoragii în pată;
C. Retinopatia preproliferativă se identifică prin prezența dezlipirii de
retină;
D. Maculopatia necesită o adresare promptă către oftlmolog;
E. Retinopatia proliferativă se caracterizează prin hemoragii în vitros și
neovascularizație.
R: D, E (1, p.728)

36. Alte complicații oculare consecutive diabetului enumeră:


A. Cataracta (,,cataracta în fulg de zăpadă,,);
B. Pareza de nerv cranian III și VI( pareze oculare externe);
C. Glaucom în unghi deschis;

43
D. Exoftalmie bilaterală;
E. Cecitate.
R: A, B, C, E (1, p. 729)

37. Care dintre următoarele afirmații despre nefropatia diabetică sunt


corecte?
A. Microalbuminuria este marca debutului nefropatiei diabetice;
B. Este rapid progresivă și se manifestă la 1-2 de ani după diagnosticul
DZ;
C. Afectează aprox 35% dintre pacienții diagnosticați cu DZ înainte de
30 ani;
D. Modificările microscopice din glomeruloscleroza difuză și nodulară
poartă numele de leziuni Kimmelstiel Wilson;
E. Proteinuria persistent marcată conduce către sindrom nefrotic.
R: A, C, D, E (1, p. 730)

38. Neuropatia diabetică acută dureroasă descrie :


A. Dureri de tip arsură cu topografie la nivel abdominal și față
anterioară a brațelor;
B. Severitate mai importantă pe timpul nopții;
C. Presiunea exercitată de așternuturi poate fi uneori intolerabilă;
D. Nu se remite nici în context de control glicemic strict mai bine de
12 luni;
E. Târziu în evoluția diabetului apare o formă cronică rebelă la aproape
toate formele de tratament.
R: B, C, E (1, p. 732)

39. Neuroartropatia Charcot:


A. Complicație a nefropatiei diabetice severe;
B. Se manifestă prin debut acut, cu distrucție osoasă, consolidare și
stabilizare;
C. Poate fi confundată cu inflamația țesutului celular subcutanat
(celulita) sau atacul de gută;
D. Piciorul se deformează foarte repede astfel că tratamentul nu trebuie
întârziat;
E. În urma procesului distructiv este necesară chirurgia reparatorie.
R: B, C, D, E (1, p.733)

44
40. Care sunt complicațiile macrovasculare ale DZ?
A. Infarct miocardic acut;
B. Accident vascular cerebral;
C. Boală arterială periferică;
D. Nefropatie diabetică;
E. Retinopatie diabetică.
R: A, B, C (1, p. 734)

41. Care sunt complicațiile microvasculare ale DZ?


A. Infarct miocardic acut;
B. Accident vascular cerebral;
C. Boală arterială periferică;
D. Nefropatie diabetică;
E. Retinopatie diabetică.
R: D, E (1, p. 734)

45
CAPITOLUL 8
NEUROLOGIE

1. Principalele mecanisme implicate în alterarea stării de


conștientă sunt:
A. Leziuni ale trunchiului cerebral;
B. Leziuni cerebrale difuze;
C. Compresiunea trunchiului cerebral;
D. Accident ischemic tranzitoriu;
E. Lacunarism cerebral.
R: A, B, C (1, p. 832)

2. Următoarele investigații sunt necesare în comă:


A. Imagistica cerebrală;
B. Examinarea LCR;
C. Ecografie Doppler carotidiană;
D. Electroencefalografie;
E. Sânge și urină.
R: A, B, D, E (1, p. 836)

3. Următoarele semne clinice sunt utile pentru diagnosticul


infarctului cerebral în teritoriul ACA:
A. Hemipareză predominant crurală;
B. Hemiplegie controlaterală;
C. Simptomatologie de lob frontal cu apatie și apraxie;
D. Sindrom de neglijare în afectarea emisferului non-
dominant;
E. Cecitate corticală.
R: A, C (1, p. 840)

4. Principalele tablouri semiologice care se întâlnesc în infarctul


lacunar sunt:
A. Hemipareză pur motorie;
B. Hemisindrom pur senzitiv;
C. Ataxie unilaterală brusc instalată;
D. Afazie predominant receptivă;
E. Oftalmoplegie internucleară.
46
R: A, B, C (1, p. 841)

5. Caracteristici clinice ale AIT în sistemul carotidian (circulația


anterioară) sunt:
A. Amauroză fugace;
B. Afazie;
C. Hemipareză;
D. Sindom senzitiv de hemicorp;
E. Ataxie.
R: A, B, C, D (1, p. 839)

6. Tromboliza prezintă următoarele criterii de eligibilitate:


A. Persistența deficitului neurologic;
B. Infarct cerebral mai vechi de 4,5 ore;
C. Imagistica evidențiază hemoragie cerebrală;
D. Momentul debutului este bine stabilit;
E. Simptome de AVC cu ameliorare rapidă.
R: A, D (1, p. 844)

7. Craniectomia decompresivă este indicată în următoarele


situații:
A. În primele 48 de ore reduce mortalitatea;
B. Infarct lacunar;
C. Infarct pontin însoțit de comă;
D. Infarct cerebral malign în teritoriul ACM;
E. Nu îmbunătățește dizabilitatea pe termen lung.
R: A, D (1, p. 840)

8. Diagnosticul de urgență la un pacient cu AVC presupune:


A. CT cerebral;
B. Screening pentru vasculită;
C. Hemogramă, glicemie și coagulogramă dacă pacientul este
anticoagulat;
D. Ultrasonografie Doppler carotidiană;
E. Screening pentru trombofilie.
R: A, C (1, p. 842)

47
9. Caracteristicile ce definesc un accident vascular cerebral
venos sunt următoarele:
A. Reprezintă doar 1% din totalitatea AVC;
B. Poate apare în sarcină, stări de hipercoagulabilitate, boli
trombotice;
C. Principalul factor de risc este ateroscleroza;
D. Prezintă infacte corticale, crize convulsive și
hipertensiune intracraniană;
E. Stenoza carotidiană este factor de risc.
R: A, B, D (1, p. 838)

10. Prevenția secundară după AVC ischemic presupune:


A. Tromboliză;
B. Terapie antihipertensivă;
C. Terapie hipolipemiantă;
D. Modificarea stilului de viață și a educației;
E. Craniectomie decompresivă.
R: B, C, D (1, p. 844)

11. Tratamentul accidentului vascular cerebral hemoragic:


A. Monitorizarea frecventă a scorului GCS și a semnelor
neurologice este esențială;
B. Este indicat tratamentul antiagregant plachetar;
C. Măsuri de reducere a presiunii intracraniene pot fi
necesare;
D. Anticoagularea trebuie rapid inițiată;
E. Controlul hipertensiunii arteriale este vital.
R: A, C, E (1, p. 846)

12. Hematoamele cerebeloase:


A. Nu pot cauza hidrocefalie obstructivă sau comă;
B. Evacuare neurochirurgicală de urgență la hematoame >
3cm;
C. Evacuare neurochirurgicală de urgență dacă pacientul este
somnolent;
D. Plasarea unui dren ventricular extern dacă hidrocefalia
obstructivă apare;
E. Evacuarea hematomului indiferent de dimensiunile
acestuia.
R: B, C, D (1, p.846)

48
13. Printre cauzele de hemoragie subarahnolidiană se regăsesc:
A. Angiopatia amiloidă;
B. Anevrismul sacular;
C. Malformațiile arteriovenoase;
D. Hipercolesterolemia;
E. Tulburări de sângerare.
R: B, C, E (1, p.846-847)

14. Hemoragia subarahnoidiană:


A. Reprezintă sângerarea arterială spontană intracerebrala
lobară;
B. Anevrismele saculare apar la nivelul poligonului Willis;
C. Hemangioamele cavernoase (cavernoame) sunt frecvente;
D. Apare de obicei ca o cefalee ușoară, cu debut brusc;
E. După o HSA majoră, apare rigiditatea cefei și semnul
Kernig pozitiv.
R: B, C, E (1, p.847)

15. Hemoragia extradurală:


A. Urmează de obicei o linie de fractură craniană;
B. Sângerarea extradurală se acumulează lent;
C. Examenul CT este cea mai utilizată investigație;
D. Nu necesită intervenție neurochirurgicală de urgență;
E. Reprezintă acumularea de sânge în spațial subdural.
R: A, C (1, p.848)

16. Sindromul meningian:


A. Este repezentat de triada: dureri de cap, rigiditatea cefei și
febră;
B. Fotofobia și vărsăturile sunt adesea absente;
C. Rigiditatea gâtului și semnul Kernig pozitiv apar de
obicei în câteva ore;
D. Pacientul este irascibil și preferă să stea nemișcat;
E. În meningita complicată se instalează somnolență și semne
de lateralizare.
R: A, C, D, E (1, p.870)

17. Meningita bacteriană acută:


A. Prezintă mortalitate și morbiditate scăzute;
B. Trebuie inițiată de urgență terapia antivirală;

49
C. Trebuie efectuată o evaluare CT înainte de puncție
lombară;
D. Aspectul LCR este tulbure/purulent;
E. Analiza biochimică a LCR arată monocitoză.
R: C, D (1, p.870)

18. Encefalita:
A. Semnifică inflamația acută a parenchimului cerebral;
B. Are de obicei etiologie bacteriană;
C. Convulsiile (focale și generalizate) sunt comune;
D. Schimbările de personalitate și de comportament sunt o
manifestare precoce;
E. Poate progresa spre un nivel redus al conștienței și comă.
R: A, C, D, E (1, p.871)

19. Encefalita virală:


A. Encefalita herpes simplex (HSV) afectează înțial, tipic,
lobii temporali;
B. EEG prezintă complexe de unde ascuțite și lente;
C. Biopsia cerebrală este adesea necesară;
D. tratament intravenos cu aciclovir inițiat imediat ce se
suspicionează infecția cu HSV sau VZV;
E. Tratamentul cu Aciclovir se temporizează până la primirea
rezultatelor investigațiilor.
R: A, B, D (1, p.871-872)

20. Herpes Zoster (Zona Zoster):


A. Este cauzată de primoimfectia cu virusul varicelo-
zosterian (VZV) ;
B. Este cauzată de reactivarea virusului VZV, de obicei în
interiorul ganglionilor rădăcinii dorsale;
C. Nevralgia postherpetică răspunde bine la analgezice;
D. Herpesul oftalmic se datorează implicării V1;
E. Poate indica o scădere a imunității mediate celular.
R: B, D, E (1, p.872)

50
CAPITOLUL 9
PNEUMOLOGIE

1. În etiologia astmului bronșic sunt incriminați următorii factori


nespecifici:
A. Dieta;
B. Medicamente;
C. Factorii de mediu;
D. Atopia și alergia;
E. Beta-blocantele.
R: A, B, E (1, p.950-951)

2. Tabloul clinic în astmul bronșic cuprinde:


A. Dispnee progresivă la efort;
B. Atacuri de wheezing;
C. Tuse cu predominență nocturnă;
D. Expectorație frecventă muco-purulentă;
E. Astenie fizică.
R: B, C (1, p. 951)

3. Obiectivele tratamentului în astmul bronșic sunt:


A. Restabilirea funcției pulmonare normale;
B. Eliminarea simptomelor;
C. Utilizarea celei mai crescute doze eficace din medicația
adecvată;
D. Permite o creștere normală a copiilor;
E. Reducerea riscului de exacerbări severe.
R: A, B, D, E (1, p.951)

4. Medicația de salvare cu durată lungă de acțiune folosită în


astmul bronșic cuprinde:
A. Corticosteroizi inhalatori;
B. Anticolinergice inhalatorii;
C. Cromoglicat disodic;
D. Antagoniști ai leucotrienelor;
E. Preparate de teofilină.
R: A, C, D (1, p.953)

51
5. Despre Omalizumab se poate afirma următoarele:
A. Leagă moleculele de IgE libere;
B. Scade numărul și activitatea mastocitelor și bazofilelor;
C. Se administrează în funcție de vârstă;
D. Se adminitrează lunar subcutanat;
E. Este eficient în cazul exacerbărilor frecvente ce necesită
corticosteroizi orali.
R: A, B, E (1, p.953)

6. Pacienții cu exacerbare astmatică severă au următoarele


caracteristici:
A. Tahicardie<110/min;
B. Un PEF de 33-50% din valoarea prezisă sau valoarea individuală
maximal cea mai bună;
C. Incapacitatea de a termina o propoziție în timpul unei singure
respirații;
D. Frecvența respiratorie<25 respirații/ min;
E. Cianoză.
R: B, C, D (1, p.954)

7. Bronșita acută:
A. Este rară la pacienții fumători cu BPOC;
B. Inițial tusea se asociază cu discomfort retrosternal;
C. Rar tusea devine productivă cu expectorație mucoasă;
D. Inițial tusea este iritativă, neproductivă;
E. Se deceleză raluri sibilante și rare raluri crepitante.
R: B, D, E (1, p.955)

8. Despre bronhopneumonia obstructivă cronică sunt adevărate


următoarele afirnmații:
A. Se datorează obstrucției difuze a căilor aeriene care nu este
ireversibilă;
B. Obstrucția este progresivă și se asociază cu un răspuns
inflamator anormal al plămânilor;
C. Se asociază frecvent cu multiple comorbidități: boală cardiacă
ischemică, diabet zaharat, HTA;
D. BPOC este un diagnostic care cuprinde un singur sindrom clinic;

E. Se asociază cu obstrucția difuză a căilor aeriene și distrugerea


parenchimului pulmonar.

52
R: B, C, E (1, p.955)

9. În patogenia BPOC-ului sunt incriminați următorii factori:


A. Sexul masculin;
B. Infecțiile;
C. Deficitul de alfa1- antitripsină;
D. Fumatul de țigarete;
E. Vârsta înaintată.
R: B, C, D (1, p.956)

10. Despre investigațiile paraclinice efectuate în BPOC sunt


adevărate următoarele afirmații:
A. Testele funcționale pulmonare nu arată obtrucția difuză a căilor
aeriene;
B. Radiografia toracică arată aspect de infiltrat inflamator difuz;
C. Hemoglobina și hematocritul sunt normale;
D. Tomografia computerizată toracică poate fi utilă când radiografia
pulmonară este normală;
E. Gazometria arterială poate fi utilă pentru identificarea
insuficienței respiratorii.
R: D, E (1, p.957)

11. Tratamentul exacerbarilor constă în:


A. Oxigenoterapie;
B. Reechilibrare hidroelectrolitică;
C. Corticosteroizi, antibiotice și bronhodilatatoare;
D. Evaluarea căii aeriene, a respirației și a circulației;
E. Aspirarea secrețiilor.
R: A, C, D (1, p.959-960)

12. Contraindicațiile relative ale ventilației non-invazive pe secție


sunt:
A. Arsuri faciale;
B. Stare confuzională sau agitație;
C. Tulburări cognitive;
D. Vărsături;
E. pH> 7,15.
R: B, C (1, p.961)

53
13. Despre apneea în somn sunt adevărate următoarele afirmații:
A. Prevalență crescută la menopauză, obezitate, hipotiroidism;
B. Este o formă de tulburare respiratorie legată de somn;
C. Simptomele cele mai frecvente(%) sunt: edeme gambiere, stare
de ebrietate matinală;
D. Incidența la copii nu are legătură cu obezitatea, trisomia 21 ;
E. Se datorează colabării căilor respiratorii superioare.
R: A, B, E (1, p.961)

14. Fumatul pasiv:


A. Crește frecvența și severitatea exacerbărilor;
B. Nu influențează incidența astmului;
C. Se asociază cu o creștere mică a riscului de cancer pulmonar;
D. Dezvoltă risc scăzut de comorbidități;
E. Accentuează tusea și dispneea la fumători și nefumători.
R: A, C, E (1, p.963)

15. Factorii de risc pentru pneumonia comunitară sunt:


A. Vârsta <16 ani sau >65 ani;
B. Corticoterapia prelungită;
C. Obezitatea;
D. Fumatul;
E. Diabetul zaharat.
R: A, B, D, E (1, p.964)

16. Pneumonia comunitară:


A. Infecția poate fi localizată sau difuză;
B. Apare frecvent la copii;
C. Cel mai frecvent se datorează infecției cu Streptococcus
pneumoniae;
D. În 50% din cazuri sunt prezente mai multe microorganisme;
E. Poate să apară și la pacienții cu neoplazie subiacentă.
R: A, C, E (1, p.964)

17. Cele mai frecvente manifestările extrapulmonare în pneumonia


comunitară sunt:
A. Cefaleea;
B. Miocardita și pericardita;
C. Diareea și vărsăturile;
D. Eritemul nodos;

54
E. Febra.
R: A, B, C, D (1, p.964)

18. Complicațiile pneumoniei comunitare:


A. Non-aderența;
B. Empiemul;
C. Abcesul pulmonar;
D. Revărsatul lichidian pleural parapneumonic;
E. Leziune obstructivă.
R: B, C, D (1, p.965)

19. Micro-organisme implicate în pneumonia nosocomială:


A. Bacterii gram pozitive;
B. Bacterii anaerobe;
C. Staphylococcus aureus;
D. Acinetobacter spp;
E. Bacterii gram negative.
R: B, C, D, E (1, p.967)

20. Pneumonia de aspirație:


A. Poate complica anestezia generală;
B. Poate fi fatală datorită puterii distructive a sucului gastric;
C. Spectrul terapeutic trebuie limitat în formele severe;
D. Poate progresa spre abces pulmonar;
E. Aspirația nu va recidiva în absența măsurilor corective adecvate.
R: A, B, D (1, p.967)

21. Factorii implicați în reactivarea tuberculozei latente:


A. Malnutriția;
B. Hepatitele virale;
C. Diabetul zaharat;
D. Neoplaziile;
E. Terapia imunosupresoare.
R: A, C, E (1, p.968)

22. Tuberculoza ganglionară:


A. Reprezintă o localizare rară;
B. Se manifestă prin adenopatii cervicale sau supraclaviculare
ferme, nedureroase;

55
C. Adenopatia poate fi prezentă cu câteva luni înaintea
diagnosticului;
D. La CT aria centrală are aspect normal;
E. Ganglionii extratoracici sunt mai frecvent interesați față de cei
mediastinali.
R: B, C, E (1, p.969)

23. Etambutolul:
A. Nu determină nevrită optică retrobulbară;
B. Leziunile oftalmologice sunt dependente de doză;
C. Înainte de începerea tratamentului este necesară evaluarea
oftalmologică;
D. Doza maximă este de 2g;
E. Leziunile oftalmologice sunt ireversibile.
R: B, C (1, p.971)

24. Cancerul bronhopulmonar:


A. Apare frecvent la pacienții expuși la radiații ionizante, azbest,
produse petroliere.
B. Fumatul este responsabil de 25% din cazuri;
C. Apare foarte rar la pacienții cu fibroză pulmonară;
D. Este cea mai frecventă cauză de deces;
E. Este cea mai frecventă tumoră malignă la nivel mondial.
R: A, D, E (1, p.975)

25. Cele mai frecvente sedii ale metastazării cancerului pulmonar


sunt:
A. Ficat;
B. Oase;
C. Sân;
D. Creier;
E. Glande suprarenale.
R: A, B, D, E (1, p.976)

26. Manifestări vasculare și hematologice non-metastatice ale


cancerului bronhopulmonar:
A. Anemie hipocromă microcitară;
B. Anemie hemolitică;
C. Purpura trombotică trombocitopenică;

56
D. Acanthosis nigricans;
E. Coagularea intravasculară diseminată.
R: B, C, E (1, p.978)

27. Tratamentul cancerului pulmonar cuprinde:


A. Tratament chirurgical;
B. Radioterapie cu intenție curativă;
C. Radioterapie paliativă;
D. Chimioterapie;
E. AINS.
R: A, B, C, D (1, p.978)

28. Stadializarea TNM a cancerului pulmonar cuprinde:


A. M1a – o singură metastază extratoracică într-un singur organ;
B. NX – fără metastaze în ganglionii locoregionali;
C. Tis – carcinom in situ;
D. T2b – tumoră mai mare de 4 cm, dar nu mai mare de 5 cm în cea
mai mare dimensiune;
E. N3 – metastaze în ganglionii mediastinali ipsilaterali și/sau
subcarianri.
R: C, D (1, p.979)

29. Diagnosticul diferențial al nodulului pulmonar solitar se face cu:


A. Metastaze pulmonare;
B. Chist hidatic;
C. Pleurezii închistate;
D. Granulom;
E. Tuberculoza pulmonară.
R: A, B, D (1, p.981)

30. Tumorile carcinoide bronșice:


A. Sunt frecvente și au un grad ridicat de malignitate;
B. Provin din tumori neuroendocrine;
C. Majoritatea sunt asimptomatice;
D. Tratamentul de elecție este reprezentat de radioterapie;
E. Pacienții pot prezenta hemoptizie și infecții.
R: B, C, E (1, p.982)

57
CAPITOLUL 10
BOALĂ VENOASĂ TROMBOEMBOLICĂ

1. Triada lui Virchow presupune:


A. Modificări ale fluxului sanguin;
B. Cianoză;
C. Disfuncția peretelui vascular;
D. Hipercoagulabilitate;
E. Trombocitopenie.
R: A,C,D (1, p.1002)

2. Factori de risc persistenți pentru tromembolismul venos sunt:


A. Trombofilia;
B. Istoric de trombembolism venos;
C. Cancer activ;
D. Sarcina/perioada postnatală;
E. Cateter venos central.
R: A, B (1, p.1002)

3. Factori de risc tranzitori pentru tromembolismul venos sunt:


A. Imobilizare/repaos la pat>3 zile;
B. Vârsta înaintată;
C. Sindrom nefrotic;
D. Intervenții chirurgicale majore, pe membrele inferioare;
E. Tromboza venoasă superficială.
R: A, D, E (1, p.1002)

4. Sunt adevărate urmatoarele afirmații despre tromboză:


A. Staza și hipercoagulabilitatea sunt factori ai trombozei arteriale;
B. Originea cea mai frecventă este în venele profunde ale
membrului inferior;
C. Tromboza venoasă proximală apare atunci când trombul ajunge
la vena poplitee sau deasupra ei;
D. Este nevoie de câteva minute pănă ce trombii produc manifestări
clinice;
E. Hipercoagulabilitatea poate fi produsă de cancer.
R: B, C, E (1, p.1002)
58
5. Manifestări clinice în TVP sunt:
A. Durerea unui membru inferior;
B. Tumefiere;
C. Parestezii;
D. Caldură locală;
E. Roșeață.
R: A, B, D, E (1, p. 1003)

6. Manifestări clinice în EP sunt:


A. Dispnee;
B. Durere toracică;
C. Raluri ronflante;
D. Frecatură pleurală;
E. Hemoptizie.
R: A, B, D, E (1, p.1003)

7. Caracteristicele EP sunt următoarele:


A. Aspect ECG clasic-S1Q3T3;
B. Tahicardie sinusală;
C. Scorul Wells care împarte EP în probabilă și improbabilă;
D. Aspectul ECG clasic este prezent la majoritatea pacienților;
E. Radiografia toracică poate fi normală, dar cel mai frecvent
prezintă manifestări nespecifice .
R: A, B, C, E (1, p.1003)

8. Examinarea imagistică a TVP/EP:


A. Ultrasonografia este mai puțin sensibilă decât venografia;
B. Angiografia pulmonară prin tomografia computerizată este
sensibilă și disponibilă pe scară largă;
C. Ultrasonografia este foarte sensibila pentru TVP proximale;
D. Un rezultat normal al scintigrafiei pulmonare ventilație-perfuzie
exclude diagnosticul;
E. Atunci când scorul Wells indică faptul ca EP este puțin
probabilă, D-dimerii negativi exclud diagnosticul fară a mai fi
nevoie de investigații suplimentare.
R: B, C, D, E (1, p.1004)

9. Scorul Wells pentru embolie pulmonară acordă 3 puncte pentru


următoarele, cu excepția:

59
A. Cancer;
B. Hemoptizie;
C. Edem al piciorului și durere la palparea venelor profunde;
D. Istoric de TVP/EP;
E. AV>100/minut.
R: A, B, D, E (1, p.1004)

10. Tratamentul anticoagulant pentru TVP se poate iniția cu:


A. Fondaparinux ;
B. Heparina cu greutate molecură mică;
C. Dabigatran;
D. Heparina nefractionată;
E. Apixaban.
R: A, B, D, E (1, p.1006)

11. Anticoagularea în sarcină:


A. Este sigur să se folosească DOAC;
B. Warfarina produce embriopatie și anomalii scheletice:
C. Warfarina nu traversează placenta;
D. Heparina cu greutate moleculară mică reprezintă tratamentul de
elecție;
E. Warfarina prezinta risc de deces intrauterin.
R: B, D, E (1, p.1007)

12. Durata tratamentului anticoagulant trebuie să fie:


A. Continuat pe termen lung, pentru prevenția secundară,
independent de riscul pacientului;
B. Minim 3 luni;
C. Minim 3 săptămâni;
D. Atunci când riscul de recurență este >5% în următorul an, se ia în
considerare continuarea tratamentului;
E. Dacă riscul de recurență este >5% în următorul an, se ia în
considerare oprirea tratamentului.
R: B, D (1, p.1008)

13. Sindromul post-trombotic se caracterizează prin:


A. Claudicație venoasă la efort;
B. Afectare permanentă a pielii cu roșeață, hiperpigmentare, ectazii
venoase;
C. Ulcerația membrului inferior implicat la 30-40% din pacienți;

60
D. Tumefiere;
E. Mâncărime și modificarea culorii pielii.
R: A, B, D, E (1, p.1010)
14. Diagnosticul de sindrom antifosfolipidic trebuie luat în
considerare atunci când pacienții prezintă:
A. TEV la vârstă tânară (<50 ani);
B. Tromboza venoasă cu localizare neobișnuită;
C. Pierderea a trei sau mai multor embrioni înainte de săptămâna a
5-a de sarcina;
D. APTT scăzut.
E. Naștere prematură a unui nou născut normal morfologic înainte
de săptamâna 34 de sarcină.
R: A, B, E (1, p.1012)

15. Următoarele aspecte prezintă un risc de sângerare legat de


pacient, mai puțin:
A. Intervenție neurochirurgicală sau oftalmologică;
B. Trombocitopenie;
C. Hipertensiune arterială sistolică necontrolată;
D. Hemofilie;
E. Puncție lombară în ultimele 4 ore.
R: A, E (1, p.1013)

16. Proprietățile farmacologice ale heparinei nefracționate sunt:


A. Are o greutate moleculară între 2000-9000 Da;
B. Timpul de înjumătățire este de 4 ore;
C. Este necesară monitorizarea APTT:
D. Antidotul său este protamina sulfat:
E. Este obținută din sursa biologică.
R: C, D, E (1, p.1014)

17. Antagoniștii vitaminei K- următoarele afirmații sunt adevărate:


A. Warfarina este antagonistul principal al vitaminei K;
B. Efectul anticoagulant se monitorizează prin determinarea
timpului de protrombină, ce este apoi convertit într-un raport
standardizat (INR);
C. Valorea țintă a INR-ului este cuprinsă între 2,0-3,0;
D. Warfarina nu are interacțiuni cu factorii alimentari;
E. Asocierea cu aspirina și clopidogrel sporesc riscul de sângerare.
R: A, B,C, E (1, p.1016)

61
18. Anticoagulantele orale directe-următoarele afirmații sunt false:
A. Toate se administrează subcutanat;
B. Prezintă interacțiuni cu factorii alimentari și alcoolul;
C. Antidotul pentru Warfarină este Vitamina K;
D. Antidotul pentru Dabigatran este Idarucizumab;
E. Cresc riscul de sangerare gastrointestinala.
R: A, B (1, p.1017)

19. Sunt anticoagulante orale directe:


A. Enoxaparinum;
B. Apixaban;
C. Edoxaban;
D. Rivaroxaban;
E. Dabigatran.
R: B, C, D, E (1, p.1017)

20. Contraindicațiile anticoagulantelor orale directe sunt:


A. Stenoza aortică;
B. Sarcină și alăptare;
C. Înlocuire mecanică a valvelor cardiace;
D. Stenoza mitrală moderată sau severă;
E. Boala vasculară periferică.
R: B, C, D (1, p.1017)

21. Managementul valorilor ridicate ale INR-ului și al sângerărilor


sub tratament cu warfarină, cuprinde:
A. Se va administra vitamina K 1-5 mg oral pentru orice sângerare;
B. Se va opri warfarina;
C. La INR> 8,0 poate exista sângerare;
D. La INR>5,0 fără sângerare se investighează cauza INR-ului
crescut;
E. În caz de sângerare majoră se va administra complex de
protrombină concentrată sau plasmă proaspăt congelată.
R: B, C, D, E (1, p. 1016)

62
CAPITOLUL 11
CARDIOLOGIE

1. Suportul vital de bază la adulți cuprinde:


A. 30 compresii toracice;
B. 4 respirații de salvare;
C. Se apelează 112 înaintea începerii manevrelor de salvare;
D. Se menține un raport de 30:2;
E. Sternul este comprimat ritmic o adâncime de 5-6 cm.
R: A, C, D, E (1, p.1045)

2. Suportul vital avansat cuprinde:


A. Monitorizare ECG;
B. Intubare orotraheală;
C. Se administrează un șoc bifazic 150-200 J;
D. Daca sunt ritmuri neșocabile de administrează 1mg
adrenalină;
E. Asistola este ritm șocabil.
R: A, B, C, D (1, p.1046)

3. Următoarele sunt cauze reversibile ale stopului cardio-


respirator:
A. Hipoxia;
B. Hipovolemia;
C. Tamponada cardiacă;
D. Hiponatremia;
E. Trombembolismul.
R: A, B, C, E (1, p.1047)

4. Cauze extrinseci de bradicardie sinusale sunt:


A. Ischemia acută și infarctul nodului sinusal;
B. Tratamentul cu betablocante, digitalice;
C. Hipotermia;
D. Hipertiroidismul;
E. Boala de nod sinusal;
R: B, C (1, p.1052)

63
5. Sunt sindroame mediate neural ce apar ca urmare a
reflexului Bezold Jarisch, cu excepția:
A. Sincopa vaso-vagală;
B. Sindromul vestibular;
C. Sindromul de tahicardie ortostatică posturală;
D. Sindrom de sinus carotidian;
E. Sindromul bradi/tahi.
R: B, E (1, p.1052)

6. Următoarele afirmații cu privire la blocurile


atrioventriculare sunt adevărate:
A. BAV gradul I se definește prin interval PR>0,20sec;
B. BAV gradul II Mobitz II reprezintă prelungirea progresivă a
intervalului PR până când o undă P este blocată;
C. BAV gradul II Mobitz I mai este denumit și Wenckebach;
D. Blocul atrioventricular gradul II poate complica infarctul
miocardic;
E. Indicația de cardiostimulare permanentă este pentru BAV
gradul I.
R: A, C, D (1, p.1053)

7. Cauzele blocului atrioventricular complet sunt:


A. Angina pectorală stabilă;
B. Infarct miocardic acut;
C. Stenoza aortică calcificată;
D. Post chirurgie cardiacă;
E. Lupus eritematos sistemic.
R: B, C, D, E (1, p.1055)

8. Blocul complet de ramură se caracterizează prin:


A. QRS larg peste 0,12 sec;
B. BRS are unde S adânci în V5;
C. BRD are unde S adânci în DI și V6;
D. BRS determină și apariția de unde Q anormale;
E. QRS îngust.
R: A, C, D (1, p.1056)

9. Sunt tahicardii supraventriculare:


A. Fibrilație ventriculară;
B. Fibrilație atrială;

64
C. Flutter atrial;
D. Tahicardie prin reintrare nodală;
E. Tahicardie joncțională.
R: B, C, D, E (1, p.1058)

10. Caracteristici ECG ale tahicardiilor supraventriculare:


A. Intervalele RR sunt neregulate în fibrilația atrială;
B. Undele P sunt absente in tahicardia sinusală;
C. In tahicardia atrială multifocala undele P sunt diferite;
D. Undele de flutter au o frecveță de până la 300/min;
E. Fibrilația atrială este cea mai frecentă tahicardie la pacienții
cu vârsta de peste 65 de ani.
R: A, C, E (1, p.1058)

11. Sunt patologii frecvente asociate cu blocul de ram drept:


A. Tetralogia Fallot;
B. Stenoza pulmonară;
C. Defect de sept atrial sau ventricular;
D. Stenoza aortică;
E. Infarct miocardic.
R: A, B, C, E (1, p.1057)

12. Tabloul clinic în tahicardia prin reintrare nodală


atriventriculară/prin reintrare atrio ventriculară cuprinde:
A. Palpitații cu ritm rapid;
B. Sincopa;
C. Hipertensiune arterială;
D. Terminarea palpitațiilor prin manevra Valsava;
E. Dureri precordiale cu caracter constrictiv.
R: A, B, D (1, p.1059)

13. Fibrilația atrială-caracteristici:


A. La pacienții cu hipotiroidism fibrilația atrială poate fi singura
manifestare a bolii;
B. Venele pulmonare reprezintă localizarea predominantă a
focarelor de depolarizare rapidă;
C. Se poate manifesta prin palpitații, dispnee sau durere
toracică;
D. Puls neregulat;

65
E. Pe ECG exista oscilații fine ale liniei de bază numite unde f
in dinți de fierăstrău.
R: B, C, D (1, p.1059)

14. Clasificare fibrilației atriale:


A. Persistentă-convertită spontan în cel mult 7 zile;
B. Permanentă-se abandonează încercările de convertire la ritm
sinusal;
C. Nou diagnosticată – indiferent de durata sau severitatea
simptomelor ;
D. Paroxistică-convertită spontan în cel mult 7 zile;
E. Persistentă-continuă, durată de peste 7 zile.
R: B, C, D, E (1, 1060)

15. Tratamentul fibrilației atriale în faza acută cuprinde:


A. Amiodarona;
B. Flecainidă;
C. Propafenona;
D. Adenozină;
E. Vernakalant.
R: A, B, C, E (1, p.1061)

16. Cardioversia electrică a fibrilației atriale se poate realiza:


A. Numai în faza acută, dacă pacientul este instabil
hemodinamic;
B. Prin șoc electric extern, de preferat bifazic;
C. Numai după tratament anticoagulant anterior de minim 3
săptămâni;
D. Pentru reducerea riscului tromboembolic anticoagularea se
face cu warfarină sau anticoagulante directe;
E. Nu este necesară anticoagularea post conversie.
R: B, D (1, p.1061)

17. Scorul CHA2DS2-VASC pentru fibrilație atrială non-


valvulară acordă câte 1 punct pentru:
A. Vârsta peste 75 ani;
B. Accident vascular cerebral;
C. Insuficientă cardiacă congestivă;
D. Hipertensiune arterială ;
E. Vârsta între 65-74 ani.

66
R: C, D, E (1, p.1062)

18. Scorul HAS BLED acordă câte 1 punct pentru:


A. Sexul feminin;
B. Hipertensiune arterială;
C. INR labil;
D. Consum de alcool;
E. Sexul masculin.
R: B, C, D (1, p.1062)

19. Anticoagularea în fibrilația atrială se inițiază:


A. Cu warfarină-fără monitorizarea INR-ului;
B. Cu anticoagulante directe cu menținerea INR-ului la valori
între 2-3;
C. La scor CHA2DS2-VASC=1 la femei;
D. La CHA2DS2-VASC=0 nu este necesară profilaxia
anticoagulantă;
E. Dabigatran-inhibitor direct al trombinei.
R: D, E (1, p.1062)

20. Aspectul ECG în flutterul tipic antiorar:


A. Unde F negative în derivațiile inferioare;
B. Unde F pozitive în derivațiile V1-V2;
C. Când unde F nu sunt vizibile se poate face masajul sinusului
carotidian;
D. Când unde F nu sunt vizibile se pot administra Verapamil sau
Adenozină;
E. Frecvență atrială de 100-150/minut.
R: A, B, C, D (1, p.1062)

21. Tahicardia atrială:


A. Este un tip rar de aritmie ce apare la cei cu boli cardiace
structurale;
B. Masajul sinusului carotidian o oprește;
C. Cel mai frecvent este cauzată de un focar localizat în crista
terminalis;
D. Poate degenera in fibrilație atrială;
E. Cardioversia și medicația antiaritmică nu sunt necesare
pentru menținerea ritmului sinusal .
R: A, C, D (1, p.1063)

67
22. La un pacient cu tahicardie ventriculară se poate observa pe
ECG:
A. Complexe QRS largi (peste 0,14 sec);
B. Disociație atrioventriculară;
C. Tahicardia supraventriculară cu bloc de ramură poate avea
aspect similar;
D. Undă S adâncă în V6;
E. Undă Q in teritoriul anterior.
R: A, B, C, D (1, p.1063)

23. Cauze de sindrom de QT lung pot fi, mai puțin următoarele:


A. Hipocalcemia;
B. Sotalol;
C. Metoprolol;
D. Amiodarona;
E. Antiepileptice.
R: C, E (1, p.1064)

24. Următoarele antiaritmice aparțin clasei III:


A. Sotalol;
B. Amiodaronă;
C. Dofetilidă;
D. Chinidina;
E. Flecainida.
R: A, B, C(1, p.1067)

25. Ablația cu cateter este eficientă în tratarea următoarelor


aritmii:
A. Tahicardie prin reintrare atrioventriculară cu cale accesorie;
B. Fibrilație atrială permanentă;
C. Flutter atrial;
D. Tahicardie prin reintrare nodală atrioventriculară;
E. Tahicardie ventriculară pe cord structural normal.
R: A, C, D, E (1, p.1068)

26. Diagnosticul de insuficiență cardiacă cu fracție de ejecție


redusă se bazează pe următoarele criterii:
A. Fracție de ejectie a VS redusă;
B. Semne de insuficiență cardiacă;

68
C. Modificări structurale cardiace relevante(HVS/dilatare de
atriu stâng);
D. Simptome de insuficiență cardiacă;
E. Disfuncție diastolică.
R: A, B, D (1, p.1072)

27. Tabloul clinic in insuficiența cardiacă cuprinde:


A. Raluri pulmonare subcrepitante;
B. Dispnee de efort;
C. Zgomote cardiace III și IV;
D. Presiune venoasă jugulară scăzută;
E. Ortopnee.
R: A, B, C, E (1, p.1072)

28. Tratamentul insuficienței cardiace prezintă următoarele


opțiuni:
A. Diuretice tiazidice;
B. Inhibitori ai enzimei de conversie a angiotensinei;
C. Valsartan în combinație cu sacubitril ca prima linie de
tratament;
D. Betablocante;
E. Antagonisti de aldosteron.
R: A, B, D, E (1, p.1073)

29. Dozele maxime de inhibitori ai enzimei de conversie a


angiotensinei care se pot administra în insuficiența cardiacă
sunt:
A. Lisinopril 50 mg/zi;
B. Ramipril 10mg/zi;
C. Enalapril 20 mg/zi;
D. Perindopril 16 mg/zi;
E. Captopril 15 mg/zi.
R: B, C, D (1, p.1074)

30. Contraindicațiile transplantului cardiac sunt:


A. Insuficiență renală/hepatică;
B. Boală psihică necontrolată;
C. Hipertensiune arterială;

69
D. Infecție necontrolată;
E. Trombembolism recent.
R:A, B, D, E (1, p.1076)

31. Sindroame clinice de insuficiență cardiacă acută:


A. Edem pulmonar acut;
B. Embolie pulmonară;
C. Șoc cardiogen;
D. Decompensare acută a insuficienței cardiace acute;
E. Insuficiență cardiacă acută hipertensivă.
R: A, C, D, E (1, p.1077)

32. În angina pectorală clasică durerea se caracterizează prin:


A. Toracică/retrosternală cu caracter de apăsare;
B. Apare la efort;
C. Poate iradia spre mandibulă/brațe;
D. Apare la repaus;
E. Dispare la repaus sau la nitroglicerină sublingual.
R: A, B, C, E (1, p.1081)

33. Sunt medicamente care reduc riscul de evenimente majore


cardiovasculare:
A. Atorvastatina;
B. Aspirina;
C. Clopidogrel;
D. Bisoprolol;
E. Amlodipina.
R: A, B, C (1, p.1083)

34. Modificările ECG apărute in infarctul miocardic acut:


A. Supradenivelare segment ST>0,5 mV la punctul J;
B. Supradenivelare ST in teritoriul inferior-II,III,aVF;
C. Modificări ale segmentului ST in teritoriul lateral- DI, aVL,
V5, V6;
D. Bloc major de ramură stângă;
E. Bloc minor de ram drept.
R: B, C, D (1, p.1088)

70
35. Reperfuzia în infarctul miocardic se poate realiza prin:
A. Angioplastie primară percutană în primele 60 minute;
B. Tromboliza atunci cand PCI nu poate fi efectuată în primele
120 minute de la diagnostic;
C. By-pass aorto-coronarian- scade mortalitatea atunci cand se
poate efectua în primele 72 ore;
D. PCI se însoțește de tratament anticoagulant intraprocedural:
warfarină sau anticoagulante directe;
E. Atunci când este disponibilă, tromboliza este intervenția
primară.
R: A, B (1, p.1089)

36. Complicațiile infarctului miocardic sunt:


A. Insuficiență cardiacă;
B. Ruptură miocardică;
C. Anevrism ventricular in regiunea infarctizată;
D. Sindrom Dressler;
E. Defect de sept atrial.
R: A, B, C, D (1, p.1090)

37. Medicamente ce pot reduce mortalitatea la pacienții post


infarct miocardic:
A. Betablocant pentru menținerea frecvenței cardiace>60/minut;
B. Aspirina;
C. Inhibitor de enzimă de conversie a angiotensinei;
D. Antagonist de aldosteron la pacienții cu hiperpotasemie;
E. Blocant al receptorilor de ADP.
R: B, C, E (1, p. 1091)

38. Tabloul clinic al pacientului cu stenoza mitrală cuprinde:


A. Dispnee importantă;
B. Suflu sistolic în cazul apariției hipertensiunii pulmonare;
C. Facies mitral;
D. Suflu diastolic de tonalitate joasă, uruitura diastolică;
E. Puls regulat la pacienții cu fibrilație atrială.
R: A, C, D (1, p.1092)

39. Auscultația în stenoza aortică:


A. Se ascultă în spațiul II intercostal parasternal stâng;

71
B. Suflu sistolic de ejecție;
C. Iradiază pe arterele carotide;
D. Zgomotul 2 dedublat;
E. Uruitura Austin Flint.
R: B, C (1, p.1098)

40. Pacienții care beneficiază de înlocuire valvulară aortică sunt:


A. Indicație pentru intervenție chirurgicală pentru aorta
ascendentă;
B. Velocitate maximă transaortică >5,5m/s;
C. Presiune sistolică în artera pulmonară >80 mmHg;
D. Scăderea tensiunii arteriale la testul de efort;
E. Stenoză aortică critică la copii și adolescenți.
R: A, B, D (1, p.1099)

41. Care dintre următoarele nu aparțin criteriilor majore Duke


modificate:
A. Două hemoculturi pozitive;
B. Masă intracardiacă cu mobilitate proprie pe valve sau
structurile vecine;
C. Regurgitare valvulară nou documentată;
D. Febră peste 38°C;
E. Noduli Osler.
R: D, E (1, p.1105)

42. Antibioterapia recomandată în endocardita infecțioasă:


A. Endocardita streptococică-penicilină/gentamicină;
B. Endocardita enterococică-ampicilina/amoxicilină;
C. Se administrează pe cale orală;
D. Se administrează timp de 4-6 săptămâni;
E. În caz de alergie la penicilină se poate administra
vancomicină.
R: A, B, D, E (1, p.1105)

43. Cauze ale miocarditei pot fi:


A. Autoimună;
B. Medicamentoasă-antiaritmice;
C. Streptococ-cel mai frecvent cardită reumatismală;
D. Boala Lyme;
E. Virus gripal.

72
R: A, C, D, E (1, p.1118)

44. Cardiomiopatia hipertrofică-generalități:


A. Afectează frecvent septul interventricular;
B. Cea mai frecventă cauză de moarte subită la tineri;
C. 75% prezintă obstrucție dinamică în calea de ejecție a VS;
D. Cauzată de mutații ale genelor care codifică proteine
sarcomerice;
E. Cele mai multe cazuri sunt autozomal recesive.
R: A, B, D (1, p.1119)

45. Cardiomiopatia dilatativă:


A. Poate fi familială sau sporadică;
B. Dilatare VS cu funcție contractilă globală redusă;
C. Biopsia este gold standard;
D. Se poate manifesta prin insuficiență cardiacă,aritmii,
tulburări de conducere;
E. Terapia de resincronizare cardiacă reprezintă o opțiune de
tratament la pacienții în clasa funcțională NYHA II.
R: A, B, D (1, p.1122)

46. Cardiomiopatia restrictivă:


A. Puls venos jugular accentuat;
B. Colaps diastolic (semnul lui Kussmaul);
C. Se asociază cel mai frecvent cu amiloidoza;
D. ECG-ul evidențiazăpoate evidenția QRS microvoltat;
E. Aspect caracteristic de ,,șine de tramvai” la ecografia
cardiacă în modul M.
R: A, C, D, E (1, p.1123)

47. Caracteristicile pericarditei acute:


A. Este denumită sindrom Dressler când apare post infarct
miocardic;
B. Probele inflamatorii sanguine sunt crescute;
C. Supradenivelare de segment ST pe ECG;
D. Durerea toracică este ameliorată de poziția aplecată în față;
E. Se poate administra aspirină 75-100 mg/zi.
R: B, C, D (1, p.1125)

73
48. Semnele tamponadei cardiace sunt:
A. Semnul lui Kussmaul;
B. Puls paradoxal;
C. Scăderea debitului cardiac;
D. Creșterea presiunii venoase jugulare;
E. Semnul lui Ewart-matitate la percuția bazei plămânului drept.
R: A, B, C, D (1, p.1127)

49. Pericardita constrictivă-tratament:


A. Pericardectomia precoce este recomandată în prezența
calcificărilor pericardice;
B. Chirurgical-pericardectomie completă;
C. Este recomandat înainte de a dezvolta constricție severă și
atrofie miocardică;
D. Pericardectomia se realizează la 4-6 săptămâni de terapie
antibiotică;
E. Pericardectomia este urmată de tratament antiinflamator.
R: B, C (1, p.1127)

50. Clasificarea Fontaine:


A. Stadiul I: asimptomatic;
B. Stadiul III: durere de repaus/nocturnă;
C. Stadiul II: claudicație intermitentă;
D. Stadiul III: durere de efort;
E. Stadiul IV: necroză/ gangrenă.
R: A, B, C, E (1, p.1128)

74
CAPITOLUL 12
HIPERTENSIUNEA ARTERIALĂ

1. Care afirmații referitoare la HTA sunt adevărate?


A. Este o boală asociată îmbătrânirii;
B. Este mai frecventă la tineri;
C. Este un predictor pentru alte evenimente cardiovasculare;
D. Nu este o patologie comună;
E. Majoritatea pacienților își ating valorile țintă ale TA.
R: A, C (1, p.1133)

2. Între examinările efectuate pentru afectarea de organ țintă în


HTA se efectuează:
A. Arteriografie;
B. CT cerebral;
C. Examenul fundului de ochi;
D. Ascultație cardiacă;
E. Ecografie cardiacă.
R: C, D (1, p.1135)

3. Investigațiile efectuate pentru afectarea de organ țintă în HTA


cuprind:
A. Coronarografie;
B. RFGe;
C. ECG;
D. EEG;
E. HBa1c.
R: B, C, E (1, p.1135)

4. Problemele măsurării TA la domiciliu includ:


A. Efectul de “halat alb”;
B. Imposibilitatea măsurării exacte a TA;
C. Costurile ridicate ale aparaturii;
D. Necesitatea unei instruiri privind masurarea TA;
E. Lipsa citirii obiective a valorilor TA.
R: D, E (1, p.1136)

75
5. Care afirmații sunt adevărate referitoare la HTA secundară de
cauza renală?
A. Se poate manifesta clinic printr-un suflu paravertebral;
B. Reprezinta o cauză frecventă a HTA;
C. Stenoza de arteră renala se poate diagnostica angiografie CT;
D. Poate ncesita biopsie renala pentru diagnostic;
E. Necesita angioplasie cu balon pentru stenozele de artera renală.
R: C, D, E (1, p.1139)

6. Reprezintă valori prag pentru diagnosticul hipertensiunii:


A. “Normal-înaltă”: TA sistolică 140-159 mmHg;
B. HTA sistolică izolată : TA sistolică >140 mmHg;
C. HTA gradul 1: TA diastolică 140-159 mmHg;
D. HTA gradul 2: TA sistolică 160-179 mmHg;
E. TA normală: TA sistolică <80 mmHg.
R: B, D (1, p.1140)

7. Terapia inițială a HTA conform ghidurilor actuale cuprinde


următoarele clase de medicamente:
A. Antagoniști ai receptorilor de renina;
B. Blocante ale canelelor de calciu;
C. Diuretice tiazid-like;
D. Betablocante;
E. Diuretice de ansă.
R: A, B, C (1, p.1142)

8. Care dintre următoarele măsuri de reducere a riscului


cardiovascular total este corectă?
A. Tratament cu statine la toți pacienții cu un risc >1% pe an;
B. Tratament cu aspirina în doza mică la toți pacienții;
C. Tratament cu aspirina doar la pacienții diabetici;
D. Nu se recomanda administrarea de statina la tineri;
E. Nu exista un beneficiu demonstrat al tratamentului cu aspirina în
prevenția primară.
R: A, E (1, p.1142)

76
9. Care dintre următoarele relații între clasele de medicamente și
reacțiile adverse sunt corecte?
A. Inhibitori de enzimă de conversie – angioedem;
B. Diuretice tiazid-like – hiperpotasemie;
C. Beta-blocante – constipație;
D. Diuretice economisitoare de K – ginecomastie;
E. Beta-blocante – bronhospasm.
R: A, D, E (1, p.1143)

10. În tratamentul de urgență al HTA maligne se utilizează:


A. Metildopa;
B. Labetolol;
C. Nitroglicerina;
D. Nicardipina;
E. Morfina.
R: B, D (1, p.1145)

77
CAPITOLUL 13
GASTROENTEROLOGIE

1. Selectați enunțurile adevărate referitoare la boala de reflux gastro-


esofagian (BRGE):
A. Este mai des întâlnită în Asia comparativ cu lumea occidentală;
B. Relaxarea tranzitorie a sfincterului esofagian inferior apare mai
frecvent la pacienții cu BRGE;
C. Presiunea intraabdominală scăzută predispune la apariția BRGE;
D. Simptomul principal este pirozisul;
E. Corelația dintre pirozis și esofagită este slabă.
R: B, D, E (1, p. 1162)

2. Caracterele clasice ale durerii din refluxul gastroesofagian sunt:


A. Are caracter de arsură, accentuată la aplecare, înclinare sau decubit;
B. Iradiază la baza gâtului sau în membrul superior stâng;
C. Se agravează la efort;
D. Este însoțită de dispnee;
E. Se ameliorează la ingestia de antiacide.
R: A, E (1, p. 1163)

3. Selectați enunțurile corecte referitoare la monitorizarea pH-ului


intraluminal esofagian:
A. Se poate efectua prin introducerea unui tub nazogastric cu un senzor
de pH la vârf, care este plasat la nivelul sfincterului esofagian
superior;
B. Permite măsurarea expunerii la acid a esofagului pe 24 ore;
C. Este indicată la pacienții cu BRGE care nu răspunde la administrarea
de inhibitori de pompă de protoni;
D. Se realizează cu ajutorul unui cateter care măsoară rezistența la
trecerea ”curentului alternativ” prin conținutul esofagului;
E. Poate fi utilă pentru evaluarea dismotilității esofagiene ca și cauză a
simptomelor BRGE.
R: B, C (1, p. 1163)

4. Despre tratamentul cu inhibitori de pompă de protoni în boala de


reflux gastro-esofagian se poate afirma că:
78
A. Inhibă adenozin trifosfataza sodiu/potasiu gastrică;
B. Reduc secreţia de acid gastric cu până la 90%;
C. În BRGE cu simptome severe se administrează o dată pe zi timp de 4
săptămâni;
D. Reprezintă medicamentele de elecţie pentru toate cazurile de BRGE
în afara celor uşoare;
E. Tratamentul pe termen lung poate avea reacții adverse (osteoporoză).
R: B, D, E (1, p. 1163, 1164)

5. Conform clasificării Los Angeles esofagitele pot fi de:


A. Grad A = discontinuităţi (eroziuni) ale mucoasei limitate la pliuri,
dar care nu depăşesc 5 mm;
B. Grad A = o discontinuitate (eroziune) a mucoasei mai lungă de 5
mm, limitată la nivelul pliurilor;
C. Grad B = discontinuităţi (eroziuni) ale mucoasei care se continuă
între vârfurile pliurilor, dar nu sunt circumferenţiale;
D. Grad C = cel puţin o discontinuitate (eroziune) a mucoasei mai lungă
de 5 mm, limitată la nivelul pliurilor, fără continuitate între vârfurile
a două pliuri;
E. Grad D = discontinuităţi (eroziuni) confluente ale mucoasei
confluente, interesând peste 75% din circumferinţa esofagului.
R: A, E (1, p. 1164)

6. Sunt reprezentanți ai antagoniștilor receptorilor histaminici H2, cu


excepția:
A. Famotidina;
B. Metoclopramida;
C. Dexlansoprazolul;
D. Nizatidina;
E. Domperidona.
R: B, C, E (1, p. 1164)

7. Selectați enunțurile adevărate referitoare la esofagul Barrett:


A. Diagnosticul se stabilește prin endoscopie digestivă superioară;
B. Se caracterizează prin înlocuirea unei părți din mucoasa columnară
esofagiană normală cu epiteliu scuamos metaplazic;
C. Se asociază rar cu hernia hiatală;
D. Obezitatea abdominală crește riscul de a dezvolta esofag Barrett;
E. Se poate complica cu dezvoltarea adenocarcinomului esofagian.
R: A, D, E (1, p. 1165)

79
8. Despre supravegherea endoscopică a esofagului Barrett se pot face
următoarele afirmații corecte:
A. Cromoendoscopia și endoscopia în bandă îngustă pot facilita
diagnosticarea displaziei și a carcinomului esofagian;
B. Displazia de grad înalt într-o leziune vizibilă endoscopic nodulară
poate fi îndepărtată prin rezecție endoscopică mucosală;
C. În cazul detecției displaziei de grad înalt fără leziune vizibilă
endoscopic se recomandă monitorizare endoscopică la 6 luni;
D. Dacă se detectează displazie de grad scăzut se recomandă repetarea
biopsiilor la 3 luni;
E. Ecoendoscopia este frecvent utilizată pentru stadializarea pacienților
cu esofag Barrett și displazie de grad înalt.
R: A, B, E (1, p. 1166)

9. Selectați enunțurile false referitoare la infecția cu Helicobacter


pylori (H. pylori):
A. H. pylori este o bacterie cu dezvoltare rapidă, Gram-pozitivă;
B. H. pylori colonizează stratul de mucus de la nivelul fornixului
gastric;
C. Prevalența infecției cu H. pylori este ridicată (80-90%) în țările
dezvoltate;
D. Infecția cu H. pylori este dobândită de obicei în perioada copilăriei;
E. Calea de transmitere a H. pylori poate fi fecal-orală sau oral-orală.
R: A, B, C (1, p. 1172)

10. Despre infecția cu Helicobacter pylori (H. pylori) se poate afirma


că:
A. Producția de urează permite conversia ureei în amoniu și clorură, cu
efect citoprotector;
B. Ulcerele apar cel mai frecvent atunci când tulpina H. pylori exprimă
genele CagA și VacA;
C. Gastrita cronică antrală cu H. pylori determină hipergastrinemie;
D. Prevalența infecției cu H. pylori la pacienții cu ulcere duodenale este
în creștere;
E. Fumatul și susceptibilitatea genetică sunt implicați în apariția
ulcerelor duodenale la pacienții infectați cu H. pylori.
R: B, C, E (1, p. 1173)

80
11. Noțiunea de ulcer peptic se caracterizează prin următoarele leziuni:
A. Pierdere de substanţă de la nivelul mucoasei gastrice sau duodenale
care depăşeşte în profunzime muscularis mucosae;
B. Bază fibroasă;
C. Creşterea numărului celulelor non-inflamatorii;
D. Pierdere de substanţă limitată la mucoasă;
E. Mucoasa din jurul ulcerelor duodenale apare congestionată,
edemaţiată, sângerândă sau friabilă.
R: A, B, E (1, p. 1173)

12. Tabloul clinic al bolii ulceroase peptice cuprinde:


A. Durere retrosternală cu iradiere posterioară;
B. Greață;
C. Vărsături frecvente;
D. Durere epigastrică recurentă, având caracter de arsură;
E. Sensibilitate epigastrică frecventă la palpare.
R: B, D, E (1, p. 1173)

13. Metodele non-invazive de diagnostic ale infecției cu Helicobacter


pylori sunt:
A. Testele serologice ce detectează anticorpii de tip IgG;
B. Testul rapid pentru activitatea ureazică din proba bioptică de
mucoasă gastrică;
C. Dozarea antigenului fecal Helicobacter pylori;
D. Realizarea de culturi din biopsiile prelevate din mucoasa gastrică;
E. Testul respirator cu uree marcată 13C.
R: A, C, E (1, p. 1174)

14. Semnele și simptomele ”de alarmă” care necesită evaluarea


endoscopică în cadrul bolii ulceroase sunt:
A. Anemia feriprivă;
B. Pierderea involuntară din greutate;
C. Pirozisul;
D. Hematemeza sau melena;
E. Vărsăturile persistente.
R: A, B, D, E (1, p. 1174)

81
15. Regimurile terapeutice de eradicare a infecției cu Helicobacter
pylori includ:
A. IPP (Omeprazol 20 mg) x 2/zi + Claritromicină 500 mg x 2/zi +
Amoxicilină 1g x 2/zi, 7 sau 14 zile;
B. IPP (Omeprazol 20 mg) x 2/zi + Claritromicină 500 mg x 2/zi +
Metronidazol 400 mg x 2/zi, 7 sau 14 zile;
C. IPP (Omeprazol 20 mg) x 2/zi + Claritromicină 500 mg x 2/zi +
Metronidazol 400 mg x 2/zi + Amoxicilină 1g x 2/zi, 10 zile;
D. IPP (20-40 mg) x 2/zi + Subcitrat de bismut 120 mg x 4/zi +
Tetraciclină 500 mg x 4/zi + Metronidazol 400 mg x 3/zi, 14 zile;
E. Terapie secvențială IPP x 2/zi + Amoxicilină 1g x 2/zi, 7 zile, apoi
IPP x 2/zi + Claritromicină 500 mg x 2 zi + Metronidazol 500 mg x
3/zi, 7 zile.
R: A, B, D (1, p. 1175)

16. Selectați enunțurile adevărate referitoare la rolul


antiinflamatoarelor nesteroidiene (AINS) în boala ulceroasă:
A. AINS stimulează producția de prostaglandine la nivel local și
sistemic prin blocarea căii ciclooxigenazei;
B. Inhibitorii specifici COX-2 au un efect mai mic asupra enzimei
COX-1 din mucoasa gastrică;
C. Utilizarea inhibitorilor specifici COX-2 este limitată de riscul de
apariţie a reacţiilor adverse cardiovasculare;
D. Ulcerele induse de AINS reprezintă o problemă majoră, în special la
tineri;
E. La pacienții cu risc ridicat de a dezvolta boală ulceroasă indusă de
AINS, se pot utiliza inhibitori de pompă de protoni ca terapia
citoprotectoare profilactică.
R: B, C, E (1, p. 1176)

17. Enunțurile adevărate referitoare la factorii de risc implicați în


apariția bolilor inflamatorii intestinale (BII) sunt:
A. Fumatul favorizează exacerbarea colitei ulcerative;
B. Consumul de antiinflamatoare nesteroidiene este asociat cu
perioadele de acutizare la pacienții diagnosticați anterior cu BII;
C. Igiena deficitară se asociază cu un risc mai scăzut de apariție a bolii
Crohn;
D. Alăptarea poate oferi protecție împotriva dezvoltării BII la copii;
E. Apendicectomia poate creşte riscul dezvoltării colitei ulcerative.
R: B, C, D (1, p. 1198)

82
18. Selectați enunțurile adevărate referitoare la rolul microbiotei
intestinale în etiopatogeneza bolilor inflamatorii intestinale:
A. Microbiota intestinală joacă un rol crucial în perpetuarea inflamației
intestinale în BII;
B. Numărul bacteriilor aderente de mucoasă este crescut la pacienții cu
colită ulcerativă comparativ cu indivizii sănătoși;
C. S-au raportat concentrații mai mari de Bacteroides și Escherichia
coli în probele de mucoasă și materii fecale la pacienții cu boală
Crohn;
D. Există o aderență crescută a Escherichiei coli la celulele ileale în
boala Crohn;
E. α defensina-1 umană (HD-1) este crescută în mucoasa intestinală în
BII.
R: A, C, D (1, p.1199)

19. Boala Crohn se caracterizează prin:


A. Poate afecta oricare segment al tractului gastrointestinal, de la
cavitatea bucală până la anus;
B. Afectarea poate fi discontinuă, în arii multiple ale intestinului
separate de porţiuni intestinale cu structură normală;
C. Inflamaţia este superficială limitată la mucoasă;
D. Mucoasa este eritematoasă, cu aspect inflamator, sângerează uşor și
este friabilă;
E. Este asociată cu apariţia fisurilor și fistulelor perianale.
R: A, B, E (1, p.1200)

20. Modificările microscopice întâlnite în colita ulcerativă sunt:


A. Inflamaţia se extinde la toate straturile (transmural) intestinului;
B. Pot fi prezente granuloame;
C. Sunt prezente infiltrate de celule inflamatorii cronice în lamina
propria;
D. Pot fi prezente abcesele criptice;
E. Poate fi prezentă depleţia celulelor caliciforme.
R: C, D, E (1, p. 1200, 1201)

21. Manifestările clinice majore ale bolii Crohn sunt:


A. Rectoragiile;
B. Diareea;
C. Durerea abdominală;

83
D. Pierderea în greutate;
E. Febra înaltă.
R: B, C, D (1, p. 1202)

22. Tratamentul de menținere a remisiunii în boala Crohn se poate face


cu:
A. Glucocorticosteroizi;
B. Aminosalicilați;
C. Antibiotice;
D. Imunosupresoare (azatioprină);
E. Agenți (anticorpi) anti-TNF.
R: D, E (1, p. 1203)

23. Factorii de prognostic favorabil în boala Crohn sunt:


A. Vârsta înaintată la diagnostic;
B. Afectarea extensivă a intestinului subțire;
C. Prezența afectării perianale/rectale;
D. Ulcerația limitată;
E. Statusul non-fumător.
R: A, D, E (1, p. 1203)

24. Terapiile biologice noi pentru tratamentul bolii Crohn includ


următoarele medicamente:
A. Infliximab;
B. Vedolizumab;
C. Adalimumab;
D. Ustekinumab;
E. Risankizumab.
R: B, D, E (1, p. 1204)

25. Colita ulcerativă se caracterizează prin:


A. Simptomul clinic major este diareea cu mucus și sânge;
B. Pot fi prezente ulcerații aftoase la nivel oral;
C. Manifestările generale sunt mai severe decât în boala Crohn;
D. Proctita se caracterizează prin tenesme, emisii frecvente de mucus și
sânge;
E. În atacul acut de colită ulcerativă stângă sau extensivă apare diareea
cu sânge (până la 10-20 scaune lichide pe zi) cu orar strict diurn.
R: A, B, D (1, p. 1205)

84
26. Episodul sever de colită ulcerativă se caracterizează prin:
A. Tahicardie (> 90 bătăi/minut);
B. Viteza de sedimentare a hematiilor < 30 mm/oră;
C. Prezența a peste 6 scaune/zi cu sânge +++;
D. Necesită confirmarea diagnosticului printr-o colonoscopie completă;
E. Anemie: hemoglobină < 100 g/L.
R: A, C, E (1, p. 1206)

27. Indicațiile tratamentului chirurgical în episodul acut fulminant de


colită ulcerativă sunt:
A. Răspunsul incomplet la tratamentul medical;
B. Identificarea displaziei la controlul colonoscopic;
C. Megacolonul toxic;
D. Hemoragia;
E. Riscul de perforaţie iminentă.
R: C, D, E (1, p. 1207)

28. Colita colagenoasă microscopică se caracterizează prin:


A. Existența unui strat de colagen subepitelial îngroşat (>10 µm)
adiacent membranei bazale;
B. Prezența unui important infiltrat limfocitar în epiteliul de suprafaţă şi
numeroase celule mononucleare în lamina proprie;
C. Afectează predominant bărbații tineri;
D. Este asociată cu o varietate de afecţiuni autoimune;
E. Tratamentul constă în administrarea de budesonidă.
R: A, D, E (1, p. 1208)

85
CAPITOLUL 14
BOLILE HEPATICE

1. Reprezintă cauze ereditare de hepatită cronică următoarele:


A. Bolile infiltrative (amiloidoza);
B. Steatoza hepatică non-alcoolică;
C. Boala Wilson;
D. Hepatita autoimună;
E. Hemocromatoza.
R: C, E (1, p. 1275)

2. Selectați enunțurile adevărate despre hepatita virală A:


A. Se întâlnește de obicei primăvara;
B. Afectează copiii și adulții tineri;
C. Transmiterea se face în principal pe cale fecal-orală;
D. Boala prezintă infecțiozitate maximă după remiterea icterului;
E. Infecția anicterică este comună la copii și conferă imunitate pe viață.
R: B, C, E (1, p. 1276)

3. Virusul hepatitei B se poate transmite:


A. Pe cale fecal-orală;
B. Pe cale oral-orală;
C. Pe cale verticală (de la mamă la copil);
D. Pe cale intravenoasă;
E. Pe cale sexuală.
R: C, D, E (1, p. 1277, 1278)

4. Selectați enunțurile adevărate referitoare la virusul hepatitei B:


A. Prin secvențiere genetică s-au identificat mai multe genotipuri VHB
(A-H);
B. Genotipurile VHB pot influenţa şansa de răspuns la tratamentul cu
analogi nucleozidici;
C. Miezul sau nucleocapsida este format din proteine core (AgHBc);
D. Proteinele core conţin ADN circular complet dublu catenar şi ADN
polimeraza/revers transcriptaza;
E. AgHBe este un marker util al replicării active și cu rol în inducerea
imunotoleranței.
R: A, C, E (1, p. 1278)
86
5. Selectați enunțurile adevărate referitoare la modificările ce apar în
evoluția infecției cronice VHB:
A. În faza 1 (precoce) există un nivel redus de replicare virală;
B. În faza 1 se recomandă administrarea tratamentului de la începutul
acesteia;
C. Faza 2 se manifestă frecvent prin distrucţie hepatică, cu
transaminaze (ALT) fluctuant crescute;
D. Faza 3 se caracterizează prin niveluri crescute de ADN-VHB,
prezența AgHBe şi valori normale ale ALT;
E. Faza 4 este caracterizată prin niveluri crescute de replicare virală,
AgHBe negativ şi ALT crescut.
R: C, E (1, p. 1279)

6. În hepatita acută virală B se întâlnesc următorii markeri serologici:


A. Ag HBs pozitiv;
B. Ag HBe pozitiv;
C. Ac anti-HBs pozitiv;
D. Ac anti-HBc de tip IgM pozitivi;
E. Ac anti-HBc de tip IgG pozitivi.
R: A, B, D (1, p. 1280)

7. Factorii predictivi pentru un răspuns susținut la tratament în


hepatita cronică virală B sunt:
A. Transaminazele serice scăzute;
B. Boală hepatică activă (uşoară spre moderată);
C. Niveluri scăzute de ADN VHB;
D. Sexul masculin;
E. Virusul Delta pozitiv.
R: B, C (1, p. 1281)

8. În prezent, cele mai utilizate medicamente în tratamentul hepatitei


cronice virale B sunt:
A. Interferonul pegylat alfa-2a;
B. Lamivudina;
C. Entecavirul;
D. Tenofovirul;
E. Sofosbuvirul.
R: A, C, D (1, p. 1281)

87
9. Selectați enunțurile adevărate referitoare la hepatita virală D:
A. Infecția cronică VHB + VHD determină o boală hepatică cu evoluție
lentă;
B. Virusul hepatitei D nu se poate replica singur, dar este activat de
prezenţa VHB;
C. Insuficiența hepatică acută este mai comună după suprainfecția VHD
la un pacient cu hepatită cronică virală B;
D. Tratamentul hepatitei cronice virale D se face cu interferon pegylat
alfa-2a;
E. Coinfecția VHB + VHD se caracterizează prin Ac anti VHD IgM
serici pozitivi și Ac anti HBc IgG pozitivi.
R: B, D (1, p. 1282)

10. Factorii asociaţi cu o progresie rapidă a fibrozei în infecția cu


virusul hepatitei C sunt:
A. Consumul de alcool;
B. Co-infecţia cu HIV;
C. Statusul ponderal normal;
D. Diabetul zaharat;
E. Infecţia cu genotipul 6.
R: A, B, D (1, p. 1283)

11. Selectați enunțurile adevărate referitoare la hepatita virală E:


A. Virusul hepatitei E se transmite pe cale parenterală;
B. Poate evolua cu insuficienţă hepatică fulminantă, cu un risc mai
mare la femeile însărcinate;
C. Evoluează frecvent către boală hepatică cronică;
D. Diagnosticul se stabilește prin detectarea anticorpilor anti-VHE de
tip IgG sau IgM;
E. ARN VHE poate fi pus în evidenţă în ser sau scaun prin PCR.
R: B, D, E (1, p. 1284)

12. Fac parte din cauzele comune ale cirozei hepatice următoarele:
A. Hepatita B ± D;
B. Alcoolul;
C. Hepatita C;
D. Hemocromatoza ereditară;
E. Ficatul gras non-alcoolic (NAFLD).
R: A, B, C, E (1, p. 1289)

88
13. Cei mai buni indicatori ai funcţiei hepatice sunt:
A. Albumina serică;
B. Sodiul seric;
C. Transaminazele;
D. Creatinina serică;
E. Timpul de protrombină.
R: A, E (1, p. 1289)

14. Pentru diagnosticarea hepatocarcinomului dezvoltat pe fondul


cirozei hepatice se pot utiliza următoarele investigații imagistice:
A. Ultrasonografia abdominală;
B. Computer tomografia abdominală;
C. Endoscopia digestivă superioară;
D. Imagistica RMN;
E. Elastografia tranzitorie unidimensională.
R: A, B, D (1, p. 1290)

15. Biopsia hepatică permite:


A. Confirmarea tipului și gravității bolii hepatice;
B. Cuantificarea colagenului prin analiza imaginii digitale a coloraţiei
cu roşu picrosirius;
C. Identificarea virusurilor, structurilor angiogenice şi markerilor
oncogeni prin teste imunohistochimice;
D. Determinarea cantitativă a fierului în boala Wilson;
E. Determinarea cantitativă a cuprului în hemocromatoză.
R: A, B, C (1, p. 1290)

16. Sunt indicatori de prognostic nefavorabil în ciroza hepatică


următorii:
A. Sodiu seric scăzut (<125 mmol/L);
B. Timp de protrombină prelungit >6 secunde peste valoarea normală;
C. Hepatomegalia;
D. Hipertensiunea arterială persistentă;
E. Icterul persistent.
R: A, B, E (1, p. 1291)

17. Un pacient este diagnosticat cu ciroză hepatică și prezintă


parametrii: bilirubină serică 63 µmol/L, albuminemie 27 g/L, ascită
mică, encefalopatie absentă, timp de protrombină 5 secunde peste
normal; conform clasificării Child-Pugh modificate, pacientul prezintă:

89
A. Scorul 9;
B. Scorul 10;
C. Scorul 11;
D. Se încadrează în clasa Child B;
E. Se încadrează în clasa Child C.
R: C, E (1, p. 1291)

18. Reprezintă contraindicații absolute pentru transplantul hepatic


următoarele:
A. Tromboza venoasă splanhnică extinsă;
B. Vârsta de peste 70 de ani;
C. Sepsis-ul activ în alt teritoriu decât arborele hepatobiliar;
D. Metastazele hepatice (cu excepţia celor neuroendocrine);
E. Tumorile maligne extrahepatice.
R: C, D, E (1, p. 1292)

19. Cauzele intrahepatice pre-sinusoidale de hipertensiune portală


includ următoarele etiologii:
A. Ciroza;
B. Colangita biliară primitivă;
C. Sarcoidoza;
D. Fibroza hepatică congenitală;
E. Tromboza venei porte.
R: B, C (1, p. 1293)

20. Riscul sângerării variceale în ciroza hepatică este corelat cu:


A. Dimensiunile mari ale varicelor;
B. Prezența semnelor roşii la endoscopie;
C. Prezența herniei hiatale;
D. Prezența bolilor hepatice severe;
E. Vârsta tânără.
R: A, B, D (1, p. 1293)

21. Pentru tratamentul hemoragiilor digestive superioare variceale se


pot utiliza:
A. Scleroterapia varicelor esofagiene prin injectare considerată a fi cel
mai bun tratament;
B. Somatostatină;

90
C. Terlipresină;
D. Injectarea endoscopică de cianoacrilat în cazul sângerărilor provenite
din varice esofagiene;
E. Simvastatină.
R: B, C (1, p. 1294)

22. Pentru prevenirea sângerării variceale recurente se pot utiliza:


A. Propranolol;
B. Nifedipin;
C. Carvedilol;
D. Tehnici endoscopice (injectarea de adrenalină);
E. Metode chirurgicale (transplantul hepatic).
R: A, C, E (1, p. 1295)

23. Factorii precipitanţi care intervin în apariția ascitei în ciroza


hepatică sunt:
A. Anemia;
B. Consumul excesiv de alcool;
C. Infecţia/sepsisul;
D. Dezvoltarea hepatocarcinomului;
E. Hipertensiunea arterială.
R: B, C, D (1, p. 1296)

24. Gradientul albumină ser-ascită crescut (>11 g/L) poate avea


următoarele cauze:
A. Ciroza hepatică;
B. Tuberculoza peritoneală;
C. Carcinomatoza peritoneală;
D. Sindromul Budd-Chiari;
E. Pancreatita.
R: A, D (1, p. 1296)

25. Factorii precipitanţi ai encefalopatiei portosistemice sunt:


A. Hemoragia gastrointestinală;
B. Constipaţia;
C. Regimul alimentar bogat în lipide;
D. Peritonita bacteriană spontană;
E. Poliglobulia.
R: A, B, D (1, p. 1297)

91
26. În patogeneza encefalopatiei hepatice sunt incriminate următoarele
mecanisme:
A. Acumularea de falși neurotransmiţători (octopamină);
B. Alterarea echilibrului neurotransmiţătorilor cerebrali indusă de
amoniac;
C. Activarea sistemului neurotransmiţător inhibitor al acidului γ-
aminobutiric (GABA);
D. Nivelurile scăzute de aminoacizi aromatici (tirozină şi fenilalanină);
E. Creșterea nivelurilor aminoacizilor cu lanţ ramificat (valină, leucină
şi izoleucină).
R: A, B, C (1, p. 1297)

27. Selectați enunțurile false referitoare la sindromul hepatorenal:


A. Apare de obicei la pacienţii cu ciroză avansată, hipertensiune
portală, icter şi ascită;
B. Diureza este scăzută cu o concentraţie crescută a sodiului urinar;
C. Capacitatea de concentrare a urinei este redusă;
D. Apariția sa poate fi precipitată de administrarea de antiinflamatoare
nesteroidiene;
E. Tratamentul include terlipresină sau noradrenalină asociată cu
albumină.
R: B, C (1, p. 1298)

28. Selectați enunțurile adevărate referitoare la colangita biliară


primitivă:
A. Este o afecţiune hepatică cronică caracterizată prin distrugerea
progresivă a micilor canale biliare interlobulare;
B. Afectează îndeosebi bărbații;
C. Evoluează în etapele târzii către ciroză;
D. Anticorpii anti-mitocondriali serici (AMA) se regăsesc la aproape
toţi pacienţii;
E. Afectează pacienții cu vârste cuprinse între 60 și 80 ani.
R: A, C, D (1, p. 1298)

29. Selectați enunțurile adevărate referitoare la colangita sclerozantă


primitivă:
A. Singurul tratament dovedit a fi eficient este transplantul hepatic;
B. Este asociată cu o boală inflamatorie intestinală (de obicei, colită
ulcerativă);

92
C. Anticorpii Escherichia coli şi Novosphingobium aromaticivorans
sunt prezenţi în titru ridicat;
D. Afectează îndeosebi femeile;
E. Manifestările clinice includ prurit supărător fluctuant, icter şi
colangită.
R: A, B, E (1, p. 1299, 1300)

30. Din tabloul clinic al hemocromatozei ereditare fac parte:


A. Manifestările neurologice (tremor, disartrie, mişcări involuntare,
demenţă);
B. Pigmentarea bronzantă a pielii (prin depunere de melanină);
C. Hepatomegalia;
D. Diabetul zaharat;
E. Inelul Kayser-Fleischer (semn specific).
R: B, C, D (1, p. 1301)

31. În tratamentul bolii Wilson se pot utiliza:


A. Penicilamina;
B. Trientina;
C. Desferioxamina;
D. Flebotomia cu emisie de sânge;
E. Acetatul de zinc.
R: A, B, E (1, p. 1302)

32. Selectați enunțurile adevărate despre deficitul de alpha-1


antitripsină:
A. Manifestările clinice apar doar la adulți;
B. Este uneori asociat cu boală hepatică şi emfizem pulmonar;
C. Majoritatea pacienţilor cu boală clinică sunt homozigoţi cu un
fenotip PiZZ;
D. α-1 antritripsina serică este scăzută la 10% din valoarea normală în
fenotipurile PiZZ;
E. Majoritatea pacienților decedează din cauza bolii hepatice.
R: B, C, D (1, p. 1302, 1303)

93
CAPITOLUL 15
TULBURĂRI RENALE ȘI ALE TRACTULUI URINAR

1. Biopsia renală se folosește în investigarea:


A. Disfuncției grefei renale;
B. Afecțiunilor renale acute și cronice;
C. Sindroamelor nefritice și nefrotice;
D. Infecțiilor tractului urinar;
E. Hematuria după investigații urologice.
R: A, B, C, E (1, p. 1351)

2. Materialul obținut prin biopsie trebuie examinat prin:


A. Colorare histochimică convențională;
B. Colorare cu hematoxilin-eozină;
C. Hibridizare in situ;
D. Imunoperoxidază;
E. Imunofluorescență.
R: A, C, D, E (1, p. 1351)

3. Complicațiile biopsiei renale transcutanate sunt reprezentate


de:
A. Infecții;
B. Rata de mortalitate mai mare de 1%;
C. Hematurie microscopică pană la 10%;
D. Hematurie abundentă necesitând transfuzii de sange 1-3%;
E. Hematom perirenal.
R: A, D, E (1, p. 1352)

4. Sindromul nefrotic se caracterizează prin:


A. Proteinurie <3,5 g/zi;
B. Proteinurie >3,5 g/zi;
C. Lipsa edemelor;
D. Hiperlipidemie;
E. Hipoalbuminemie.
R: B, D, E (1, p. 1353)

5. Sepsisul:
94
A. Se datorează parțial pierderii imunoglobulinelor prin urină;
B. Este o cauză minoră de deces;
C. Vaccinarea anti-pneumococică este opțională;
D. Depistarea precoce și tratamentul agresiv al infecțiilor este
cea mai bună abordare;
E. Profilaxia cu antibiotice pe termen lung este foarte
importantă.
R: A, D (1, p. 1354)

6. Despre inhibitorii ACE și/sau antagoniștii receptorilor


angiotensinei II se poate afirma:
A. Reduc proteinuria prin scăderea presiunii de filtrare capilară
glomerulară;
B. Sunt indicați pentru proprietățile lor antiproteinurice în
sindromul nefrotic;
C. Nu este obligatorie monitorizarea funcției renale;
D. Sunt indicați pentru proprietățile lor antiproteinurice în toate
tipurile de glomerulonefropatii;
E. Trebuie monitorizată regulat tensiunea arterială.
R: A, B, D, E (1, p. 1354)

7. Terapia cu corticosteroizi în doze mari:


A. Răspunsul la adulți este semnificativ mai mic și poate apărea
după mai multe luni;
B. Pot avea loc și remisiuni spontane, iar tratamentul se
temporizează dacă proteinuria nu provoacă edeme sau
hipoalbuminemie;
C. Se administrează prednisolon 60 mg/m2 pentru 4-6
săptămâni;
D. Dacă remisiunea se menține timp de 4 ani, recidivele sunt
foarte frecvente;
E. Scăderea dozei de prednisolon se face cu 40
mg/m2/săptămână.
R: A, B, C (1, p. 1354)

8. Ciclosporina:
A. Are potențial nefrotoxic;
B. Efectul antiproteinuric este atribuit acțiunii imunosupresoare;
C. Efectul antiproteinuric nu se datorează stabilizării
citoscheletului de actină din podocite;

95
D. Inhibă defosforilarea mediată de calcineurină a
sinaptopodinei;
E. Nu o protejează de degradarea mediată de catepsina L.
R: A, B, D (1, p. 1355)

9. Ciclofosfamida:
A. Nu crește probabilitatea de remisiune pe termen lung;
B. Se administrează în doză de 1,5-2,0 mg / kg / zi timp de 8-12
săptămâni;
C. Nu se asociază cu Prednisolon;
D. Pacienții non-responsivi la steroizi nu răspund nici la
Ciclofosfamidă;
E. Efectele secundare cele mai frecvente includ infertilitatea.
R: B, E (1, p. 1355)

10. GSFS secundară:


A. Reprezintă leziunea glomerulară finală;
B. Se datorează suprasolicitării hemodinamicii glomerulare;
C. Nu este considerată fenomen secundar atunci când numărul
nefronilor funcționali este redus;
D. Nu este considerată nefropatie reziduală;
E. Când nefronii devin nefuncționali se poate produce
hipertrofie, hiperfiltrare glomerulară și hiperpresiune.
R: A, B, E (1, p. 1356)

11. Cauzele glomerulonefritei membranoase secundare includ:


A. Neoplazii;
B. Infecții;
C. Medicamente;
D. Modificări genetice;
E. Alte boli autoimune.
R: A, B, C, E (1, p. 1357)

12. Despre amiloidoză sunt adevărate următoarele afirmații:


A. Boala poate fi dobândită sau moștenită;
B. Formele cele mai rare sunt reprezentate de amiloidoza AL și
amiloidoza AA;
C. Reprezintă o tulburare sistemică a plierii proteinelor;
D. Distrucția progresivă a organelor si decesul sunt rare;

96
E. Clasificarea bolii se bazează pe natura proteinelor plasmatice
precursoare care formează depozite fibrilare.
R: A, C, E (1, p. 1357)

13. Amiloidozele familiale:


A. Sunt boli cu transmitere autosomal recesivă în care proteina
mutantă formează fibrile amiloide;
B. Afecțiunile cardiace sunt foarte rar reprezentate de tulburări
de conducere;
C. Depozitele de amiloid intracerebral și cerebrovascular sunt
observate în boala Alzheimer;
D. Neuropatia senzitivă motorie periferică și autonomă este
frecventă;
E. Biopsia renală nu este necesară în cazurile cu suspiciune de
afectare renală.
R: C, D (1, p. 1358)

14. Clasificarea Societății de Patologie Renală a nefropatiei


diabetice conține:
A. Clasa IIb: expansiune mezangială ușoară;
B. Clasa I: nu se evidențiază expansiune mezangială;
C. Clasa IIa: expansiune mezangială ușoară;
D. Clasa IV: glomeruloscleroză diabetică avansată și <50%
glomeruloscleroză globală;
E. Clasa III: glomeruloscleroză nodulară intercapilară.
R: B, C, E (1, p. 1359)

15. Nefrita acută se prezintă clasic:


A. Proteinurie;
B. Oligurie temporară și uremie;
C. Hipotensiune;
D. Hematurie macroscopică și microscopică fără cilindrii
eritrocitari;
E. Hipertensiune.
R: A, B, E (1, p. 1360)

16. Clasificarea histologică Oxford a nefropatiei cu IgA:


A. Hipercelularitate mezangială medie > 0.5 – clasă M0;
B. Fără scleroză segmentală – clasă S0;
C. Atrofie tubulară/fibroză interstițială 26-50%– clasă T2;

97
D. Hipercelularitate care produce îngustarea lumenului capilar –
clasă E1;
E. Atrofie tubulară/fibroză interstițială >50%– clasă T0.
R: B, D (1, p. 1362)

17. Tabloul clinic in nefropatia cu IgA cuprinde:


A. Hematurie microscopică asimptomatică;
B. Proteinurie nefrotică;
C. Hematurie macroscopică recurentă;
D. Nu are legătură cu infecțiile de etiologie virală;
E. Prognosticul e rezervat la cei cu tensiune arterială normală.
R: A, B, C (1, p. 1362)

18. Vasculitele de vase mici ce interesează rinichii sunt:


A. Vasculita limitată renal;
B. Granulomatoza cu poliangeită;
C. Vasculite pauciimune;
D. Poliangeita microscopică;
E. Granulomatoza eozinofilică.
R: A, B, D, E (1, p. 1363)

19. Glomerulonefrita mezangiocapilară tip 2:


A. Evidențiază proliferare celulară mezangială cu depozite
intramembranoase liniare;
B. Se asociază doar cu deficit de factor H;
C. Prezintă niveluri scăzute de C3;
D. Nu sunt prezenți autoanticorpi îndreptați către enzima
convertaza a C3;
E. Boala cu depozite dense poate fi idiopatică.
R: A, C, E (1, p. 1365)

20. Clasificarea nefritei lupice:


A. Clasa IV: nefrită lupică sclerozantă avansată;
B. Clasa II : nefrită lupică proliferativ mezangială;
C. Clasa III : nefrită lupică focală;
D. Clasa V: nefrită lupică membranoasă care afectează 10-20%
din pacienți;
E. Clasa VI: nefrită lupică difuză.
R: B, C, D (1, p. 1366)

98
21. Tabloul clinic în boala Fabry cuprinde:
A. Neuropatie;
B. Mialgii;
C. Angiokeratoame cutanate;
D. Boala cronică de rinichi fără proteinurie;
E. Hipotensiune arterială.
R: A, C, (1, p. 1367)

22. Tuberculoza tractului urinar se poate manifesta cu:


A. Piurie;
B. Hematurie;
C. Oliogurie;
D. Poliurie;
E. Disurie.
R: B, D, E (1, p. 1385)

23. Nefrita tubulointerstițială indusă medicamentos:


A. Pacienții pot fi complet asimptomatici;
B. Pacienții pot rămâne cu fibroză interstițială și boală cronică
de rinichi;
C. Pacienții pot prezenta insuficiență renală care poate fi
oligurică sau cu diureză păstrată;
D. Histologic nu apar modificări de necroză tubulară;
E. Se poate continua tratamentul cu AINS.
R: A, B, C (1, p. 1385)

24. Nefropatia analgetică:


A. Apare frecvent la persoanele vârstnice;
B. Consumul cronic nu predispune la apariția de tumori
uroepiteliale;
C. Pot apărea pierderi de lichide și sare;
D. Paraclinic se evidențiază rinichi cu dimensiuni normale;
E. Se poate asocia cu anemie, boală cronică de rinichi, infecții
de tract urinar.
R: C, E (1, p. 1386)

25. Cauzele tubulointerstițiale ale bolii cronice de rinichi sunt:


A. Nefropatia de reflux;
B. Mielomul multiplu;
C. Nefroscleroza hipertensivă;

99
D. Boala renovasculară;
E. Fibroza retroperitoneală.
R: A, B, (1, p. 1392)

26. Despre anemia din boala cronică de rinichi sunt adevărate


următoarele afirmații:
A. Este de tip hipocrom microcitar;
B. Nu afectează starea generală a pacientului și calitatea vieții;
C. Se datorează deficitului de eritropoietină;
D. Se poate asocia cu deficit de fier, vitamina B12 sau folați;
E. Hemodializa nu cauzează un grad de hemoliză.
R: C, D (1, p. 1394)

27. Boala cardiovasculară – factori de risc:


A. Sexul feminin – mai frecvent în rândul pacienților cu BCR;
B. Hipertensiunea arterială – foarte frecventă în BCR;
C. Dislipidemia – rară la pacienții uremici;
D. Diabetul zaharat – cea mai frecventă cauză de BCR;
E. Fumatul – la fel de frecvent ca și în populația generală.
R: B, D, E (1, p. 1396)

28. Despre insulină în BCR se poate afirma că:


A. Necesarul de insulină crește pe măsură ce BCR progresează;
B. Insulina este catabolizată și într-o oarecare măsură excretată
de rinichi;
C. În BCR avansată scade rezistența periferică la insulină;
D. Producția renală de glucoză este diminuată pe măsură ce
BCR progresează;
E. Rezistența la insulină nu contribuie la hipertensiunea
arterială.
R: B, D (1, p. 1397)

29. Tratamentul acidozei în BCR :


A. Corecția acidozei metabolice face cu bicarbonat de sodiu 1,8
g/zi pentru a reduce rata de declin a filtrării glomerulare;
B. Poate scădea catabolismul muscular;
C. Ajută la tratamentul hiperpotasemiei;
D. Suplimentele cu bicarbonat de sodiu prezintă risc
semnificativ de edeme și hipertensiune arterială;
E. Carbonatul de calciu nu are efect benefic în acidoză.

100
R: A,B, C (1, p. 1398)

30. Despre complicațiile specifice hemodializei sunt adevărate


următoarele afirmații:
A. Hipertensiune intradialitică;
B. Malfuncție, tromboză sau sângerare la nivelul abordului
vascular;
C. Bacteriemie;
D. Dezechilibre de dializă;
E. Amiloidoza asociată dializei.
R: B, C, D, E (1, p. 1400)

101
CAPITOLUL 16
INFECŢII TRANSMISIBILE PE CALE SEXUALĂ ŞI
INFECŢIA CU VIRUSUL HIV

1. Consecințele utilizării terapiei antiretrovirale înalt active sunt:


A. A redus drastic mortalitatea pentru cei care au acces la îngrijiri
medicale;
B. A transformat boala HIV dintr-o infecţie fatală, într-o boală cronică;
C. A contribuit la scăderea prevalenței globale;
D. Permite săderea ratei de transmitere și a numărului de cazuri noi;
E. A contribuit la creșterea prevalenței globale;

R: A, B, D, E (1, p. 1426)

2. Caracteristicile actuale ale epidemiei globale HIV/SIDA sunt:


A. Scaderea numarului de cazuri la nivel global;
B. Constituie principala cauza de deces la femei cu varsta intre 15 si 49
ani;
C. Constituie principlala cauza de invaliditate pentru persoane cu varsta
intre 30 si 45 de ani;
D. Prezinta diversitate geografica considerabila;
E. Numărul cazurilor continua sa crească în Europa de Vest si SUA.
R: A, B, C, D (1, p. 1426)

3. Tintele Programului HIV/SIDA al Naţiunilor Unite (UNAIDS) sunt:


A. Încheierea epidemiei de SIDA până în anul 2030;
B. 90% din toate persoanele care trăiesc cu HIV să fie diagnosticate şi
să-şi cunoască statusul serologic;
C. 90% dintre persoanele diagnosticate cu infecţie HIV să primească
terapie ARV;
D. 90% dintre persoanele care primesc ARV sa aiba LCD4>500/mm3;
E. 90% din totalul persoanelor care primesc ARV să aibă supresie
virală.
R: A, B, C, E (1, p. 1426)

102
4. Particularitățile regionale ale epidemiei HIV sunt:
A. Africa subsahariană rămâne regiunea cea mai grav afectată de
epidemia de HIV;
B. Țara cu cea mai mare prevalenţă de HIV este Swaziland;
C. Infecţia HIV în Africa subsahariană este mai frecventă la bărbaţi
decât la femei;
D. Persoanele cu varsta peste 50 de ani reprezinta o tremie din populația
infectată cu HIV în Marea Britanie;
E. Cea mai frecventa cauza de morbiditate si mortalitate asociată HIV
în Marea Britanie este diagnosticul tardiv.
R: A, B, D, E (1, p. 1426)

5. Factorii care contribuie la scăderea ratei de noi infectări cu HIV în


țările cu nivel ridicat de trai sunt:
A. Utilizarea mai frecventă a profilaxiei post-expunere;
B. Scăderea numărului de profilaxii pre-expunere;
C. Intensificarea și repetarea testării;
D. Administrarea tratamentului ARV la persoanele cu T CD4+ <
350/mL;
E. Inițierea rapidă a terapiei ARV eficiente pentru pacienții depistați cu
infecție HIV
F. R: A, C, E (1, p. 1426)

6. Modificări actuale în epidemiologia infecţiei HIV, în condițiile


accesului la administrarea terapiei ARV eficiente, sunt:
A. Supresia virală durabilă pe termen lung;
B. Creșterea frecvenței bolilor definitorii SIDA;
C. Predominența afecţiunilor netransmisibile între problemele de
sănătate pentru persoanele cu HIV;
D. Apariția comorbidităților cronice multiple la o vârstă mai tânără la
persoanele cu infecție HIV decât la cele neinfectate;
E. Problemele de sănătate mentală și dizabilități associate fragilității
fizice nu apar la persoanele cu infecție HIV.
R: A, C, D (1, p. 1426).

7. Testarea HIV nu este recomandată la:


A. Toţi pacienţii diagnosticaţi cu o infecţie cu transmitere sexuală;
B. Persoanele care vin din ţări cu prevalenţă HIV scăzută (<1%);
C. Toate persoanele care au călătorit în străinătate;
D. Persoanele care nu prezintă istoric de consum de droguri.

103
E. Persoanele care raportează contacte sexuale cu indivizi din ţări cu
prevalenţă HIV crescută.
R: B, D (1, p. 1427).

8. Transmiterea sexuală a infecției cu HIV:


A. Este favorizată de ulcerațiile genitale și de alte boli cu transmitere
sexuală;
B. Este mai importantă de la femeie la bărbat;
C. Este mai scăzută la partenerul receptiv în relațiile sexuale anale;
D. Circumcizia crește atât dobândirea, cât și transmiterea HIV;
E. Tratamentul ARV reduce transmiterea sexuală dacă încărcătura
virală scade sub limita de detecție.
R: A, E (1, p. 1426)

9. Transmiterea verticală a infecției HIV:


A. Este cea mai frecventă cale de transmitere la copii;
B. Majoritatea infecțiilor se produc intrauterin;
C. Fără nicio intervenție profilactică între 15-40% din nou-născuții din
mame HIV pozitive, pot dobândi HIV;
D. Poate fi prevenită prin alăptarea nou-născutului;
E. Este crescută în cazul mamelor cu infecție avansată, încărcătură
virală crescută, corioamniotită.
R: A, C, E (1, p. 1427)

10. Transmiterea HIV:


A. Are risc de 0,3% la cadrele medicale, după o singură înțepătură cu ac
contaminat cu sângele unei persoane infectate;
B. Se poate transmite iatrogen prin reutilizarea acelor si seringilor;
C. Utilizarea în comun de ace și seringi pentru consumul de droguri nu
este o cale principală de transmitere a HIV;
D. Se poate face prin utilizarea factorilor de coagulare
E. Transmiterea HIV prin transfuzii a dispărut în toate țările lumii,
datorită testării sângelui.
R: A, B, D (1, p. 1427)

11. Virusului imunodeficienței umane este caracterizat de:


A. Apartenența la familia lentivirusuri și prezența enzimei revers-
transcriptaza;
B. Multe dintre medicamentele antiretrovirale care sunt utilizate în
HIV-1 sunt eficiente și în infecția cu HIV-2;

104
C. Existența a două tipuri, HIV-1 și HIV-2, cu distribuție globală
similară;
D. Variabilitatea genetică scăzută;
E. Împărțirea în patru grupuri, reprezentând căile de transfer între
specii.
R: A, E (1, p. 1428)

12. Interrelația HIV cu sistemul imunitar al gazdei


A. Începe prin transportul virusului de către celulele dendritice de pe
suprafața mucoaselor expuse către ganglionii limfatici regionali;
B. Molecula CD4 definește populațiile de celule susceptibile la infecție;
C. Glicoproteina virală de suprafață gp120 este responsabilă pentru
intrarea HIV în celule;
D. HIV nu se poate produce și independent de receptorii CD4 la nivelul
astrocitelor și în celulele epiteliale renale;
E. Infectarea astrocitelor si celulelor epiteliale renale explică
deteriorarea rapidă neurologică și renală în evoluția HIV.
R: A, B, C, E (1, p. 1429)

13. Deficitul imun determinat de HIV este explicat de:


A. Diversitatea de subtipuri și clade virale generate de erorile de
transcriere inversă și turnoverul viral crescut;
B. Moartea celulară prin efectele directe ale HIV;
C. Distrugerea LT CD4+ de către LT CD8+;
D. Compromiterea efectului antiviral al celulelor NK;
E. Apariția precoce a anticorpilor neutralizanți, activi pentru toate
subtipurile virale.
R: A, B, C, D (1, p. 1429)

14. Inflamația cronică persistentă din cursul infecției cu HIV


A. Este impiedicată de terapia ARV;
B. Este favorizată de fenomenul de activare imună;
C. Este asociată cu translocația bacteriană, mai ales a
lipopolizaharidelor;
D. Este asociată cu HIV în sine și cu alte co-infecții;
E. Explică nivelurile crescute de citokine inflamatorii și activarea
procesului de coagulare.
R: B, C, D, E (1, p. 1429)

15. Tulburările clinice asociate cu infecția HIV sunt consecințele:

105
A. Efectelor directe ale HIV;
B. Disfuncției imune asociate cu HIV;
C. Comorbidităților și/sau a co-infecțiilor pacientului cu HIV;
D. Efectelor medicației ARV;
E. Categoriei socio-demografice.
R: A, B, C, D (1, p. 1429)

16. Primoinfecția HIV:


A. Se referă la perioada de 3 luni de la contactarea HIV;
B. Se caracterizează prin replicare virală rapidă și infecțiozitate crescută
C. Este silențioasă serologic în primele 2-4 săptămâni;
D. Manifestările clinic pot să apară după 3-6 săptămâni de la expunere,
mai frecvent ca sindrom mononucleozic;
E. Limfocitele CD4 cresc semnificativ în această etapă.
R: B, C, D (1, p. 1430)

17. Perioada de latență clinică a infecției HIV:


A. Este asimptomatică în majoritatea cazurilor;
B. Are durata variabilă;
C. Pacienții vârstnici au progresie mai lentă;
D. Femeile au o evoluție mai puțin severă;
E. Sarcina accelerează rata de progresie.
R: A, B (1, p. 1430)

18. Limfadenopatia generalizată persistentă (LGP)


A. Este definită de limfadenopatii > 1 cm în două sau mai multe zone
extra-inghinale cu durata peste 3 luni;
B. Este determinată de asocierea co-infecției cu virus cytomegalic;
C. Ganglionii sunt simetrici, fermi, mobili;
D. Ganglionii sunt dureroși;
E. Are prognostic nefavorabil.
R: A, C (1, p. 1430)

19. Demența asociată HIV (ADC- AIDS dementia complex):


A. Asociază frecvent tulburări de comportament, depresie sau
fenomene psihotice
B. Modificările computer-tomografice sunt atrofia cerebrală;
C. Rezonanța magnetică arată leziuni ale substanței albe, hiperintense
pe secvențele ponderate T2;
D. LCR indică celularitate crescută;

106
E. Măduva spinării are aspect histologic de mielopatie vacuolară.
R: A, B, C, E (1, p. 1431)

20. Manifestările neurologice asociate HIV


A. Sunt consecința infectării sistemului nervos din stadii incipiente ale
bolii;
B. Se agravează pe măsura progresiei infecției HIV;
C. Frecvența a crescut dupa introducerea terapiei ARV;
D. Neuropatia autonomă se poate manifesta prin hipotensiune posturală
și diaree;
E. Polineuropatia senzitivă apare instadii incipiente ale infecției HIV.
R: A, B, D (1, p. 1431)

21. Următoarele infecții pot determina manifestări retiniene ca o primă


formă de manifestare a infecției HIV:
A. Pneumocistoza;
B. Tuberculoza;
C. Sifilisul;
D. Toxoplasmoza;
E. Limfomul.
R: A, C, D, E (1, p. 1431)

22. Manifestările cutaneo-mucoase legate de infecția HIV sunt


caracterizate de:
A. Alterarea funcțională a celulelor dendritice și Langerhans de către
HIV
B. Creșterea hipersensibilității de tip întârziat;
C. Frecvența crescută a pruritului numai în stadiile avansate
D. Diagnosticul obligatoriu prin biopsie a tuturor leziunilor;
E. Apariția în cadrul unor infecții oportuniste.
R: A, E (1, p. 1431)

23. Complicatiile hematologice ale infecției HIV


A. Apar in stadii precoce dupa primoinfectie;
B. Neutropenia este frecventă și severă;
C. Pancitopenia este cauzată de infecții oportuniste, mai ales M. avium-
intracellulare , infecția diseminată cu virus citomegalic și limfoame;
D. Agranulocitoza poate fi produsă de cotrimoxazol;
E. Medicamentele ARV pot avea efect medulotoxic.
R: C, D, E (1, p.1431)

107
24. Cașexia (sindromul “wasting”) asociată infecției HIV:
A. Apare în stadiile tardive ale infecției;
B. Este consecința anorexiei;
C. Este consecința enteropatiei cronice HIV cu hipertrofia vilozităților
intestinale;
D. Greutatea si masa musculară rămân de obicei normale pe durata
perioadelor de latență clinică;
E. Hiperclorhidria este consecința diareei și favorizează dezvoltarea
excesivă a bacteriilor intestinale.
R: A, B, D (1, p.1431-1432)

25. Nefropatia asociată infecției HIV (HIVAN):


A. Poate determina insuficiență renală în stadiile avansate;
B. Este mai frecventă la femei, rasa caucaziană;
C. Tenofovirul este asociat cu sindromul Fanconi;
D. Leziunile frecvente sunt glomeruloscleroza focală segmentară
asociată cu tubulopatie;
E. Nefropatia cu IgA și microangiopatia trombotică sunt forme de
glomerulopatii prin complexe imune asociate infecției HIV.
R: A, C, D, E (1, p. 1432)

26. Pneumonia interstițială limfoidă:


A. A fost descrisă frecvent la copii cu infecție HIV;
B. Este specifică adulților cu infecție HIV tardivă;
C. Clinic, se manifestă cu tuse și dispnee, putând fi confundată cu
Pneumocystis;
D. Aspectul radiologic este de tip reticulo-nodular;
E. Corticoterapia agravează evoluția clinică șî histologică.
R: A, C, D (1, p. 1432)

27. Afecțiunile cardio-vasculare la pacienții cu infecție HIV:


A. apar de 2 ori mai frecvent decât în populația generală;
B. Ischemia cardiac a fost mai frecventă la pacientii cu terapie
discontinua față de cei care au luat constant terapia si au avut încărcătură
virală nedetectabilă;
C. Cardiomiopatia asociată HIV poate duce la insuficiență cardiacă
congestivă
D. Terapia ARV a scăzut frecvența bolii coronariene ischemice;

108
E. Dislipidemia nu a fost asociată cu terapia ARV.
R: A, B, C (1, p. 1432)

28. Infecțiile oportuniste:


A. Sunt cauzate de microorganisme cu virulență crescută;
B. Sunt cauzate de microorganisme ubicuitare, de obicei nepatogene,
sau de virusuri oncogene;
C. Markerul pentru riscul acestor infecții este numărul limfocitelor T
CD4+;
D. Manifestările clinice sunt neobișnuite;
E. Diagnosticul este dificil din cauza alterării răspunsului inflamator.
R: B, C, D, E (1, p. 1432)

29. Consecințele clinice ale afectării sistemului imun prin HIV depind
de:
A. Expunerea pacientului la microorganism de-a lungul vieții și
reactivarea infecțiilor latente;
B. Gradul de virulență al genților patogeni;
C. Gradul de imunodepresie al gazdei;
D. Cryptosoridium se dezvoltă la pacienți cu L CD4> 100/mm3;
E. Candida și virusurile herpetice nu determină manifestări clinice
relevante și la pacienții cu imunodepresie ușoară;
R: A, B, C (1, p. 1433)

30. Detecția anticorpilor anti-HIV:


A. Este markerul infecției HIV utilizat cel mai frecvent;
B. Perioada de “fereastră serologică” medie este de 6 săptămâni;
C. Au rol protector;
D. Dispar după 6 luni de la data infecției;
E. Tehnicile ELISA sunt utilizate pentru confirmarea diagnosticului.
R: A, B (1, p. 1433)

31. Diagnosticul nou-născuților din mame HIV-pozitive:


A. Anticorpii HIV nu sunt markeri de infecție activă neonatală,
deoarece traversează bariera placentară;
B. Toți nou-născuții din mame HIV pozitive au la naștere anticorpi
HIV;
C. Anticorpii HIV de la mamă scad treptat, dispărând la nou-născut în
primele 18 luni de viață;

109
D. Identificarea acidului nucleic viral este standardul de diagnostic la
nou-nascut;
E. Izolarea virusului în culturi este cea mai folosită metodă de
diagnostic.
R: A, B, C, D (1, p. 1433)

32. Evaluarea inițială a persoanelor cu infecție HIV necesită:


A. Anamneza completă
B. Biopsia ganglionară;
C. Istoricul de vaccinare;
D. Evaluarea co-infecțiilor și co-morbidităților;
E. Genotipul viral.
R: A, C, D, E (1, p. 1434)

33. Detectarea viremiei ARN- HIV


A. Se poate determina prin tehnici de AND-branched -chain (bDNA) și
reacții de polimerizare în lanț (RT-PCR)
B. Se poate determina prin amplificarea bazată pe secvențe de acid
nucleic (NASBA);
C. Nivelul viremiei reflectă rata replicării virale;
D. Între nivelul ARN-HIV și prognosticul pe termen lung există o
corelație, dependentă de numărul limfocite CD4;
E. Este markerul pentru evaluarea eficienței tratamentului ARV.
R: A, B, C, E (1, p. 1434)

34. Nivelul ARN-HIV:


A. Se exprimă în copii/ml sau valori logaritmice;
B. Cea mai sensibila limită de detecție este 200 copii/ml;
C. Valori peste 100000 copii/ml indică risc de progresie către SIDA de
10 ori mai mare în următorii 5 ani, fată de pacienții cu valori sub
10000 copii/ml;
D. Scăderea maximă a ARN-HIV dupa inițierea/schimbarea
tratamentului ARV apare dupa 10-12 săptămâni;
E. Creșteri tranzitorii pot să apară după imunizări sau în timpul
infecțiilor intercurente.
R: A, C, D, E (1, p. 1434)

35. Evoluția medicației antiretrovirale pentru HIV are următoarele


deziderate:
A. Interacțiuni medicamentoase crescute;

110
B. Eficacitatea crescută;
C. Toxicitatea redusă;
D. Administrarea convenabilă;
E. Raportul cost-eficacitate favorabil.
R: B, C, D, E (1, p. 1434)

36. Inițierea terapiei antiretrovirale:


A. Trebuie administrată la toți pacienții cu infecție HIV;
B. Este indicate când L CD4 scade sub 350/mm3;
C. Se face imediat după diagnosticul primoinfecției HIV;
D. La pacienții cu infecții oportuniste se așteaptă 2 săptămâni după
începerea tratamentului acestor infecții;
E. Se face concomitent cu inițierea tratamentului infecțiilor oportuniste.
R: A, C, D (1, p. 1435)

37. Dificultățile administrării terapiei ARV în infecția HIV sunt:


A. Interacțiunile medicamentoase;
B. Riscul selecției tulpinilor virale rezistente;
C. Riscul efectelor secundare;
D. Menținerea aderenței terapeutice pe termen lung;
E. Utilizarea combinațiilor fixe medicamentoase.
R: A, B, C, D (1, p. 1437)

38. Inhibitorii nucleozidici/nucleotidici de revers-transcriptază (INRT):


A. Inhibă replicarea virală în etapa de revers-transcriere;
B. Forma activă a INRT necesita fosforilarea intracelulară
C. Sunt asociaţi cu toxicitate mitocondrială;
D. Kivexa este indicată doar la pacienții cu test HLA-B*5701 pozitiv;
E. Tenofovir-alafenamida (TAF) crește riscul de toxicitate renală și
osoasă comparative cu Tenofovir disoproxil (TDF).
R: A, B, C (1, p. 1437)

39. Inhibitorii non-nucleozidici de revers-transcriptază (INNRT):


A. Interferă cu revers-transcriptaza prin legarea directă de enizmă;
B. Acționează asupra citocromului P450, având numeroase interacțiuni
medicamentoase;
C. Cele mai frecvente efecte adverse sunt erupțiile cutanate și creșterea
enzimelor hepatice;
D. Sunt eficienți impotriva HIV-2;

111
E. Cele mai puține efectele secundare asupra sistemului nervos sunt
cunoscute pentru Efavirenz.
R: A, B, C (1, p. 1437)

40. Inhibitorii de protează (IP):


A. Acționează asupra enzimei aspartil protează care produce proteine
virale funcționale;
B. au barieră genetică înaltă;
C. Efectele secundare ale IP sunt tulburarea metabolismului lipidic și
glucidic;
D. Nu interacționează cu citocromul P450;
E. Efectul de “boosting” înseamnă blocarea metabolismului unui IP cu
doze mici de ritonavir sau cobicistat, obținând concentrații plasmatice
mai mari și risc de rezistență mai scăzut.
R: A, B, C (1, p. 1438)

41. Identificati clasele terapeutice HIV corespondente următoarelor


medicamente ARV:
A. Inhibitori nucleozidici de revers-transcriptază: Tenofovir
alafenamida;
B. Inhibitori non-nucleozidici de revers-transcriptază: Darunavir;
C. Inhibitori de integrază: Raltegravir;
D. Inhibitori de protează: Elvitegravir;
E. Blocanți de co-receptori: Maraviroc.
R: A, C, E (1, p. 1437-1438)

42. Aderența la tratamentul ARV al pacienților cu HIV este influențată


de:
A. Gradul de complexitate al schemei;
B. Tolerabilitatea medicației;
C. Gradul de motivație, convingerile referitoare la sănătate, starea
psihologică a pacientului;
D. Suportul familial, social și medical pentru aderență;
E. Nivelul optim al aderenței trebuie să depășească 90%.
R: A, B, C, D (1, p. 1439)

43. Eșecul terapeutic al medicației ARV la pacientul cu HIV:


A. Este definit de valori peste 400 copii/ml la două determinări
successive ale ARN-HIV la pacient cu supresie virală anterioară;
B. Se traduce prin persistența replicării virale;

112
C. Este favorizat de aderența scăzută și interacțiunile medicamentoase;
D. Simplificarea regimurilor terapeutice complexe contribuie la eșecul
terapeutic;
E. Impune un test de genotipare înaintea schimbării cu un nou regim
terapeutic.
R: A, B, C, E (1, p. 1440)

44. Sindromul lipodistrofic este caracterizat de:


A. Lipoatrofie la nivelul membrelor și feței;
B. Acumulare de țesut adipos abdominal, mamar, visceral;
C. Scăderea colesterolului total;
D. Rezistență la insulină;
E. Scăderea morbidității cardio-vasculare.
R: A, B, D (1, p. 1440)

45. Sindromul inflamator de reconstrucție imună (IRIS):


A. Constă în reacții inflamatorii paradoxale după inițierea ARV;
B. Este mai frecvent la persoane cu imunosupresie severă;
C. Clinic, se exacerbează semne clinic preexistente sau apar manifestări
noi;
D. Sindromul lipodistrofic poate fi o manifestare a IRIS;
E. Deteriorarea funcției hepatice poate indica IRIS la pacienți cu co-
infecții VHB.
R: A, B, C, E (1, p. 1441)

46. Infecția CMV la pacientul cu HIV:


A. Se manifestă în stadia tardive, când CD4 < 100cel/mL;
B. Retinita CMV debutează unilateral, se manifestă cu pierderea
acuități vizuale, durere orbitară, cefalee;
C. Incluziunile citoplasmatice în “ochi de bufnită” sunt caracteristice
leziunilor histologice de colită CMV;
D. Poliradiculonevrita CMV afectează rădăcinile lombo-sacrate,
manifestată cu slăbiciune musculară și tulburări sfincteriene;
E. Tratamentul de elecție este Aciclovirul.
R: A, B, C, D (1, p. 1446)

47. Infecțiile virale la pacientul cu infecție HIV:


A. Infecția cu Poliomavirusul JC determimă condiloame genitale și se
asociază cu neoplazii anale și genitale care pot progresa către
carcinom de col uterin și rect;

113
B. Papilomavirusul uman HPV determină leucoencefalopatia
multifocală progresivă cu demielinizare la nivelul substantei albe și
evoluție imprevizibilă;
C. Herpesul zoster cuprinde mai multe dermatoame și are o evoluție
mai severă;
D. Virusul Epstein Varr se asociază cu limfomul primar cerebral,
limfoamele non-Hodgkin și leucoplazia păroasă a limbii;
E. Virusul herpetic uman 8 (HHV-8) este agentul cauzal al sarcomului
Kaposi.
R: C, D, E (1, p. 1447)

48. Sarcomul Kaposi în asociere cu infecția HIV:


A. Leziunile cutanate sunt violacee, bine circumcise și apar în mai
multe regiuni;
B. Poate afecta ganglionii limfatici, plămânii și tractul gastro-intestinal;
C. Formele viscerale au un prognostic mai bun;
D. Chimiterapia sistemică este obligatorie în toate formele de boală;
E. Evoluția este similar la persoanele HIV și la cele neinfectate.
R: A, B (1, p. 1449)

49. Infecția cu Mycobacterium avium- intracellulare (MAI):


A. Răspunde la tratamentul standard cu anti-tuberculoase;
B. Diagnosticul poate fi clarificat de culturile din sânge, ganglioni
limfatici, măduvă osoasă sau ficat;
C. Macrofagele infectate contribuie la diseminarea infecției de la
nivelul plămânului sau tractului gastro-intestinal;
D. Se manifestă clinic cu febră, diaree, scădere ponderală, malabsorbție;
E. Apare din stadiile de imunodepresie moderată.
R: B, C, D (1, p. 1448)

50. Infecția cu Mycobacterium tuberculosis (MTB)


A. Tuberculoza la pacienții HIV reprezintă frecvent o reactivare a unei
infecții tuberculoase latente;
B. La pacienții aflați în stadii avansate de boală, manifestările clinice
sunt atipice, frecvent cu localizare extrapulmonară
C. Răspunde la terapia standard, dar durata este mai lungă;
D. Interacțiunile medicamentoase ale medicației anti-TB și anti-HIV
necesită ajustarea dozelor;
E. Intradermoreacția la tuberculină este relevant pentru diagnostic.
R: A, B, C, D (1, p. 1447-1448).

114
51. Criptosporidioza la pacientul cu infecție HIV:
A. Nu este influențată de terapia ARV;
B. Poate determina diaree apoasă severă și persistentă;
C. Se poate asocia cu colangita sclerozantă;
D. Chisturile se atașează la nivelul epiteliului peretelui intestinului
subțire;
E. Chisturile se vizualizează microscopic cu colorația Kinyon.
R: B, C, D, E (1, p. 1445)

52. Toxoplasmoza in context HIV:


A. Determină mai frecvent encefalită și abcese cerebrale;
B. Manifestările clinice cuprind leziuni neurologice focale, convulsii,
febră, cefalee;
C. Aspectul tipic CT cu substanță de contrast identifică multiple leziuni
cu priză de contrast periferică, inelară, cu edem perilezional şi efect
de masă;
D. În toate cazurile de infecție cu Toxoplasma, serologia este pozitivă;
E. Terapia empirică cu clindamicină și atovaquone este contraindicată.
R: A, B, C (1, p. 1444-1445)

53. Criptococoza la pacienții cu HIV:


A. Debutează deseori brusc, cu febră, cefalee, greață;
B. Evoluează frecvent ca meningită, dar redoarea de ceafă și fotofobia
pot lipsi;
C. Criptococii pot fi identificați în culturile obținute din LCR și sânge;
D. Tratamentul initial se efectuează cu Fluconazol;
E. Profilaxia secundară se efectuează cu Flucitozină.
R: B, C (p. 1444)

54. Infecția cu Pneumocystis jirovecii:


A. Determină cel mai frecvent pneumonie, cu debut insidios, tuse, febră
și limitarea respirației;
B. Examenul CT pulmonar cu rezoluţie înaltă evidenţiază imagini în
„sticlă mată';
C. Cultura din lavajul bronhoalveolar și din sânge confirmă
diagnosticul;
D. Corticoterapia crește riscul de deces;
E. Profilaxia secundară este indicate la pacienți cu limfocite CD4<
200/mm3.
R: A, B, E (1, p. 1443)

115
55. Profilaxia post-expunere HIV :
A. PEP poate fi benefică dacă este administrată în primele 5 zile după
expunere;
B. Fiecare situaţie trebuie evaluată individual pentru a estima riscul
potenţial de infectare şi beneficiul potenţial al tratamentului;
C. Utilizează tripla terapie;
D. PEP nu este recomandată când sursa este un pacient cunoscut cu
încărcătură virală nedetectabilă (<200 copii ARN HIV /ml);
E. PEP administrată în urma expunerii pe cale sexuală nu trebuie să
înlocuiască celelalte metode de protecţie.
R: B, C, D, E (1, p. 1441)

116
CHIRURGIE GENERALĂ ȘI SPECIALITĂȚI CHIRURGICALE

CAPITOLUL 17
EVALUAREA SI MANAGEMENTUL PERIOPERATOR
AL PACIENTULUI CHIRURGICAL

1. Testele de screening preoperator:


A. Pot substitui anamneza și examenul clinic axate pe identificarea
comorbidităților;
B. Scopul consultului de specialitate preoperator este de a
contraindica intervenția chirurgicală;
C. Electrocardiograma este indicata la pacienți selectați cu
comorbidități semnificative;
D. Principala țintă în managementul perioperator al pacienților cu
BCR sau IRA este menținerea euvolemiei și a perfuziei renale;
E. Screeningul de rutină al concentrației de hemoglobină se
realizează doar la pacienții supuși procedurilor asociate cu
pierderi sangvine mari ( peste 500 ml) sau la pacienții cu anemie
de cauză necunoscută.
R: C, D, E (2, p. 1 )

2. Evaluarea pulmonară perioperatorie:


A. Evaluarea pulmonară preoperatorie are scopul de a identifica
pacienții la risc pentru complicații perioperatorii;
B. Efectuarea de rutină a radiografiei toracice și a testelor
functionale pulmonare este necesară;
C. Testele specializate sunt rezervate pacienților cu risc semnificativ
sau intervenții cu risc relativ mare de complicații pulmonare;
D. Radiografia toracica se indică pacienților care vor efectua
intervenții intratoracice sau pacienților ce prezintă semne și
simptome de boală pulmonară activă;
E. Factori de risc ce țin de pacient: vârsta, boala pulmonară cronică,
fumatul, insuficiența cardiacă congestivă;
R: A, C, D, E (2, p. 2)

3. Măsuri preoperatorii, care reduc riscul de complicații


pulmonare postoperatorii:
117
A. încetarea fumatului cu minim 6 săptămâni înaintea intervenției
chirurgicale;
B. antrenarea musculaturii expiratorii;
C. terapia bronhodilatatoare;
D. antibioticoterapia preventivă;
E. administrarea de corticosteroizi la pacienții astmatici.
R: A, C, E (2, p. 2)

4. Contraindicații pentru chirurgia electivă la pacienții cu afecțiuni


hepatice:
A. Insuficiența hepatică cronică;
B. Injuria renală acută;
C. Hepatita virală acută;
D. Cardiomiopatia;
E. Coagulopatia ușoară.
R: B, C, D (2, p. 3)

5. Evaluarea preoperatorie a pacientului diabetic include:


A. Aprecierea controlului metabolic și a complicațiilor asociate
diabetului;
B. Riscul infecțios al pacientului diabetic este minim;
C. Fluxul sanguin redus al pacienților cu patologie vasculară
încetinește vindecarea plăgii;
D. La pacienți cu diabet insulino-necesitant nu este indicată ajustarea
dozelor de insulină perioperator;
E. Preparatele orale cu durată lungă de acțiune sunt oprite cu 48-72
ore înaintea intervenției.
R: A, C, E (2, p. 4)

6. Managementul operator:
A. Pacientul și chirurgul trebuie să decidă împreună care este cel
mai bun parcurs diagnostic și terapeutic;
B. Consimțământul informat și formularul de consimțământ sunt
același lucru;
C. Recomandările medicale cuprind: diagnosticul, dieta, nivelul de
activitate, instrucțiuni speciale de îngrijire, îngrijirea plăgilor;
D. Vindecarea per secundam a plăgilor chirurgicale implică faptul că
marginile plăgilor au fost lăsate neapoziționate;
E. Pacienții cu risc mare de tromboembolism venos postoperator:
pacienții imobili, cu insuficiență cardiacă congestivă, boli

118
maligne, după operații pelvine sau protezare articulară, fracturi
de coloană, bazin sau oase lungi.
R; A, C, D, E (2, p. 6-9)

7. Complicații postoperatorii:
A. Atelectazia apare în cazul rahianesteziei;
B. Definiția atelectaziei variază de la o simplă colabare alveolară
până la stări cu semne clinice de colaps sau consolidare.
C. Managmentul atelectaziei postoperatorii începe postoperator;
D. Eviscerația este rezultatul desfacerii suturii fasciale;
E. Semnele de infecție a plăgii chirugicale sunt: eritem, edem,
căldură locală și durere locală.
R: B, D, E (2, p. 9-10)

8. Strategiile de practică dovedite a crește siguranta pacientului:


A. Echipa de răspuns rapid care intervine la primele semne de
degradare a pacientului;
B. Prevenția efectelor adverse medicamentoase;
C. Prevenția infecțiilor de plagă chirurgicală prin administrarea de
antibiotic tuturor pacienților;
D. Prevenția pneumoniei asociate ventilației mecanice;
E. Tratament bazat pe dovezi pentru infarctul miocardic acut pentru
a preveni decesul.
R: A, B, D, E (2, p. 12)

9. Febra ca și complicație postoperatorie:


A. Creșterea temperaturii centrale postoperator este o stare
postoperatorie normală;
B. La creșterea temperaturii mai mult de 38.3 grade Celsius, trebuie
inițiate investigații privind cauza;
C. Se instituie tratament empiric initial;
D. Procesul de evaluare începe cu analiza circumstanțelor: locația
pacientului, durata spitalizării, instrumentarea, medicația, plăgile
chirurgicale;
E. Solicitarea de teste diagnostice se face ca prima intenție.
R: B, D (2, p. 10)

10. Terapia antialgică:

119
A. Interogarea pacientului asupra intensității durerii trebuie să
devină parte integrată din evaluarea sistematică zilnică;
B. Natura patologiei pacientului, precum și comorbiditățile acestuia
, nu determină strategia de management a durerii;
C. Tranziția către analgezice cu administrare per os este facilă când
pacientul este capabil să primească medicația pe cale orală;
D. În unitatea de terapie intensivă este recomandat administrarea de
analgezie controlată;
E. Pacienții cu incizii toracice sau abdominale beneficiază de
analgezie epidurală.
R: A, C, E (2, p. 9)

120
CAPITOLUL 18
SÂNGERĂRILE CHIRURGICALE: TULBURĂRI ALE
HEMOSTAZEI, STĂRI DE HIPERCOAGULABILITATE
ȘI TERAPIA CU PRODUSE SANGUINE LA PACIENTUL
CHIRURGICAL

1. Care din următoarele operaţii sunt frecvent asociate cu pierderi


mari de sânge:
A. Chirurgia glandei tiroide
B. Chirurgia piciorului diabetic
C. Chirurgia transplantului hepatic
D. Chirurgia prostatei
E. Chirurgia mâinii
R: C, D (2, p. 39)

2. Cauze ale tulburărilor dobândite ale hemostazei sunt:


A. Uremie
B. Terapia cu anticoagulante
C. Hipertensiunea arterială
D. Medicaţie fără prescripţie
E. CID
R: A, B, D, E (2, p. 40)

3. Hemoragia masivă ca urmare a plăgilor reprezintă sângerarea:


A. Necesită transfuzie de cel puţin 10 unităţi de MER în 24 ore
B. Ȋnlocuire a întregului volum circulator al pacientului în 48 ore sau
a > 50 % în 6 ore
C. Rată a sângerării de > 150 ml/minut cu instabilitate
hemodinamică
D. Frecvenţa cardiacă < 100 bătăi/minut
E. Presiunea sistolică <= 100 mmHg
R: A, C (2, p.40)

4. Cele mai utilizate medicamente care inhibă trombocitele sunt:


121
A. Warfarina
B. Aspirina
C. Bivalirudina
D. Heparina
E. Clopidogrelul
R: B, E (2, p. 40)

5. Reacţii transfuzionale majore sunt:


A. Hipotensiune
B. Creşterea diurezei
C. Hemoglobinurie
D. Oligurie
E. Febra/frison
R: A, C,E (2, p. 45)

122
CAPITOLUL 19
INFECȚIILE CHIRURGICALE

1. Dintre factorii de risc sistemici care cresc incidenţa infecţiilor


chirurgicale se numără:
A. Obezitate
B. Diabet zaharat
C. Terapie cu corticosteroizi
D. Ţesut necrotic
E. Chimoterapia cancerului
R: B, C, E (2, p. 75)

2. Factorii de risc locali care cresc incidenţa infecţiilor chirurgicale


sunt:
A. Ţesut necrotic
B. Şocul
C. Corp străin
D. Alcoolismul
E. Obezitate
R: A, C, E (2, p. 75)

3. Plăgi chirurgicale curate sunt după următoarele operaţii:


A. Tiroidectomie
B. Mastectomie
C. Rezecţie gastrică
D. Explorarea căii biliare principale
E. Rezecţie electivă a colonului
R: A, B (2, p. 77)

4. Ȋn abcesele abdominale, controlul sursei prin reintervenţie este


necesar atunci când:
A. Nu este obligatoriu controlul sursei primare
B. Ţesut devitalizat care necesită debridare
C. Eşuarea drenării percutanate
D. Progresie spre peritonită generalizată
E. Există o sursă de contaminare continuă
R: B, C, D, E (2, p. 78)

123
5. Principiile de bază pentru prevenirea PAV (pneumonia asociată
ventilatorului) sunt:
A. Reducerea sedării şi întreruperea zilnică a acesteia
B. Evaluarea disponibilităţii pentru detubare zilnic cu încercări de
respiraţie spontană
C. Reducerea la minimum a acumulării de secreţii deasupra manşetei
tubului endotraheal folosind un tub endotraheal cu un dispozitiv
de aspiraţie subglotic
D. Coborârea toracelui prin poziţionarea patului la 30-45 grade
E. Menţinerea circuitului ventilatorului
R: A, B, C, E (2, p. 79)

6. Mionecroza clostridiană este caracterizată de:


A. Eritem care nu dispare la digitopresiune
B. Necroză tisulară şi crepitaţii care avansează rapid
C. Rata scăzută a mortalităţii
D. Necesită o debridare chirurgicală radicală imediată
E. Nu necesită tratament antibiotic
R: B, D (2, p. 80)

7. Riscul de tetanos există în următoarele tipuri de plăgi:


A. Zdrobire
B. Contuzie
C. Arsuri
D. Chirurgicale
E. Avulsie
R: A, C, E (2, p. 81)

8. Factorii de risc în dezvoltarea abcesului mamar includ:


A. Alăptarea
B. Vârsta maternă < 30 ani
C. Vârsta de gestaţie > 41 săptămâni
D. Alcoolul
E. Fumatul
R: A, C, E (2, p. 81)

9. Bacteriile implicate cel mai frecvent în infecţiile tractului biliar


sunt:

124
A. Pseudomonas aeruginosa
B. Escherichia coli
C. Klebsiella spp
D. Enterococcus spp
E. Streptococcus
R: B, C, D (2, p. 82)

10. Ȋn peritonitele acute secundare perforaţiei colonice, intervenţia


chirurgicală este necesară pentru:
A. Gestionarea perforaţiei
B. Drenajul colecţiilor purulente
C. Debridarea ţesuturilor devitalizate
D. Ȋndepărtarea materiilor fecale
E. Nu este necesară intervenţia chirurgicală
R: A, B, C, D (2, p. 83)

125
CAPITOLUL 20
TRAUMATOLOGIE

1. Suportul vital avansat se bazează pe următoarele cu excepția:


A. Evaluare primară;
B. Resuscitarea;
C. Îngrijirea provizorie;
D. Evaluarea secundară;
E. Evaluarea definitivă.
R: C, E (2, p. 87)

2. Evaluarea primară se bazează pe:


A. Tratarea imediată a leziunilor amenințătoare de viață;
B. Identificarea leziunilor amenințătoare de viață;
C. Identificarea, dar nu și tratarea imediată a leziunilor
amenințătoare de viață;
D. Tratarea imediată a leziunilor amenințătoare de viață simultană
cu inițierea resuscitării;
E. Primordial este inițierea resuscitării și tratarea ulterioară a
leziunilor amenințătoare de viață.
R: A, B, D (2, p. 87)

3. Statusul neurologic se realizează prin utilizarea scorului


Glasgow; următoarea afirmație este adevărată cu excepția:
A. Răspunsul ocular la stimuli verbali – 4 puncte;
B. Răspunsul motor cu localizarea durerii – 5 puncte;
C. Răspunsul motor cu retragere la durere – 6 puncte;
D. Răspuns verbal confuz- 4 puncte;
E. Răspunsul ocular la stimuli dureros – 2 puncte.
R: A, C, (2, p. 89)

126
4. Următoarele afirmații despre clasificarea hemoragiei sunt
adevărate:
A. Pierderea de sânge la o persoană de 70 de kg < 750ml – clasa I;
B. Volumul de sânge pierdut între 15 și 30 % - clasa II;
C. Tensiunea arterială scăzută- clasa II;
D. Diureza neglijabilă – clasa IV;
E. Frecvența respiratorie 30-40 resp/min – clasa III.
R: A, B, D, E (2, p. 88)

5. Examinarea secundară cuprinde:


A. Examen fizic incomplet;
B. Istoricul medical al pacientului;
C. Identificarea eventualelor alergii;
D. Momentul primei mese;
E. Statusul de imunizare împotriva tetanosului.
R: B, C, E (2, p. 89)

6. Despre traumatismele cranio-cerebrale următoarele afirmații


sunt adevărate cu excepția:
A. Traumatismele cranio-cerebale reprezintă cauza principală de
mortalitate prin traumă;
B. Acestea reprezintă cauza principală de dizabilitate produsă prin
traumă;
C. Hipotensiunea nu este cea mai frecventă cauză de leziune
cerebrală secundară;
D. O leziune cerebrală primară suferită la momentul impactului nu
este dificil de tratat;
E. Leziune cerebrală primară suferită la momentul impactului este
dificil de tratat.
R: C, D (2, p. 89)

127
CAPITOLUL 21
ARSURI

1. In cazul arsurii de grad II, este afectat :


A. Osul
B. Epidermul
C. Tesutul celular subcutanat
D. Tesutul muscular
E. Dermul
R: B, E. (2, p. 109)

2. Tratamentul leziunilor de arsura grad I consta in:


A. Controlul durerii(analgezice orale)
B. Reechilibrare hidro-electrolitica I.V.
C. Hidratare orala
D. Aplicare compusi topici calmanți locali
E. Regim alimentar bogat in continut proteic
R: A, C, D (2, p. 109)

3. Arsurile de profunzime partiala/predominant superficiala(Grad


II), prezinta
A. Necroza tegumentara
B. Piele eritematoasa
C. Flictene cu continut lichidian proteic
D. Durere la digitopresie
E. Flictene cu continut sangvin
R: B, C, D (2, p. 109)

4. Arsurile de profunzime partiala profundă(Grad II) sunt


caracterizate de:
A. Textura uscata si ingrosata a tegumentului
B. Culoare alb-cerata
C. Aparitia flictenelor cu continut sangvin
D. Durere
E. Leziunea nervilor cutanati determină o durere mai mare decât în
cazul arsurilor superficiale
R: A, B, D (2, p. 109)

128
5. Tratamentul leziunilor de arsura Grad II este:
A. Tratament conservator 3-5 zile
B. Amputatie
C. Excizia zonelor de arsura
D. Fasciotomii
E. Grefarea leziunii postexcizionale
R: C, E (2, p.109)

6. Arsura de gradul IV descrie leziunea:


A. Globului ocular
B. Tuturor structurilor tegumentului
C. Muschilor
D. Osului
E. Cailor aeriene
R: B, C, D (2, p.110)

7. In stadiile avansate ale intoxicatiei cu CO2, apar:


A. Deces
B. Ameteala
C. Hiperventilatie
D. Coma
E. Convulsii
R: A, D, E (2, p. 110)

8. Intubatia oro-traheala, in cazul arsurilor de cai aeriene, este


necesara pentru:
A. Combaterea edemului
B. Protejarea cailor aeriene
C. Aport adecvat de oxigen
D. Scaderea oxigenarii tesuturilor
E. Evaluarea cailor aeriene
R: B, C (2, p. 110)

129
9. In leziunile prin inhalare de fum, celulele deteriorate ale
parenchimului pulmonar:
A. Colmateaza
B. Produc obstructii
C. Favorizeaza aparitia infarctului pulmonar
D. Produc bronsiectazii
E. Pneumonia poate apărea la acești pacienți
R: A, B, D, E (2, p. 112)

10. Criteriile de transfer catre centrul de arsuri pot fi urmatoarele:


A. Arsuri gradul I cu <20% SC
B. Leziuni inhalatorii
C. Arsuri de gradul III la orice grupa de varsta
D. Arsuri la pacienti cu afectiuni medicale ce nu pot complica
managementul arsurii
E. Arsuri chimie
R: B, C, E (2, p. 114)

11. Diagnosticul ischemiei progressive datorata cresterii presiunii


hidrostatice de la nivel tisular se poate face prin evaluarea
frecventa a:
A. Gazelor sangvine
B. Pulsului extremitatilor
C. Functiei motorii
D. Functiei senzitive
E. Durerii
R: B, C, D, E (2, p. 115-116)

12. Prin efectuarea escarotomilor se obtine:


A. Eliberarea lichidului de edem
B. Oprirea fenomenelor de compresie
C. Favorizarea vindecarii spontane
D. Pregatirea leziunii pentru grefare
E. Restabilirea circulatiei distale
R: A, B, E (2, p. 116)

130
13. Escara din arsura, ca arie cu integritate tegumentara disparuta:
A. Se poate suprainfecta
B. Se vindeca primar
C. Duce la creșterea pierderii de fluide prin evaporare
D. Provoaca o reactie inflamatorie intensa
E. Favorizeaza aparitia durerii severe
R: A, C, D, E (2, p. 116)

14. Excizia fasciala a leziunii de arsura:


A. Nu este folosita ca si tehnica de tratament al arsurilor
B. Este greu de realizat
C. Asigura o rata buna de prindere a grefelor
D. Poate produce rigidizarea articulatiilor
E. Presupune indepartarea cu bisturiul a tegumentului sanatos
R: C, D (2, p. 117)

131
CAPITOLUL 22
HERNIILE PERETELUI ABDOMINAL

1. Herniile peretelui abdominal:


A. Reprezintă un defect la nivelul peritoneului parietal;
B. Pot duce la blocarea conținutului (strangulare), ocluzie
intestinală sau ischemie (încarcerare);
C. Se grupează în 2 categorii principale: hernii ale peretelui
abdominal și hernii inghinale;
D. Hernia obturatorie este un tip particular de hernie;
E. Hernia ventrală se dezvoltă pe locul unei incizii anterioare.
R: C, D (2, p. 125)

2. Herniile peretelui abdominal:


A. Reprezintă un defect parietal bine definit, la nivelul structurilor
fasciale sau musculare, prin care se exteriorizează conținutul
cavității abdominale;
B. Principalele tipuri de hernii: ventrale, ombilicale și incizionale;
C. Hernia Spiegel implică exteriorizarea elementelor anterioare ale
peretelui abdominal;
D. Diastazisul de mușchi drepti abdominali este un tip de hernie
ventrală;
E. Hernia obturatorie este un tip particular de hernie.
R: A, B, E (2, p. 125-127)

3. Tabloul clinic în herniile peretelui abdominal:


A. Pot fi de 4 tipuri: asimptomatice, simptomatice, subacute și
acute;
B. Herniile reductibile sunt, întotdeauna, asimptomatice;
C. Durerea din herniile subacute este exacerbată de efortul de
ridicare sau de activitatea fizică uzuală;
D. Herniile simptomatice au mereu indicație chirurgicală de
urgență;
E. Hernia încarcerată nedureroasă nu necesită intervenție
chirurgicală de urgență.
R: A, C, E (2, p. 127)

132
4. Încarcerarea herniei:
A. Este definită ca o “blocare” a conținutului, astfel încât organele
din sacul herniar nu mai pot fi reduse în cavitatea peritoneală;
B. Nu poate evolua spre strangulare;
C. Pentru a evita complicațiile, herniile recent încarcerate necesită
intervenție chirurgicală în 4-6 ore de la prezentare;
D. Hernia cronică încarcerată are indicație de intervenție
chirurgicală în regim de urgentă;
E. Hernia strangulată este asimptomatică.
R: A, C (2, p. 127)

5. Tratamentul chirurgical al herniilor peretelui abdominal:


A. Tehnica anatomică este rezervată herniilor cu risc crescut de
recidivă;
B. Tehnicile alloplastice presupun plasarea unei proteze pentru a
consolida peretele abdominal și a reduce tensiunea în țesuturi;
C. Tehnica de separare a componetelor este rezervată herniilor mari,
complexe, ale liniei mediene a peretelui abdominal și în cazul
recidivelor herniare;
D. Abordul minim invaziv are avantajul unui timp operator mai
redus;
E. Tehnica alloplastica este utilizată în special în herniile cu risc
crescut de recurență.
R: B, C, E (2, p. 128-129)

6. Herniile inghinale:
A. Există 3 tipuri de hernii ale orificiului miopectineal Fruchaud:
femurale, inghinale indirecte și inghinale directe;
B. Spațiul orificiului miopectineal superior ligamentului ilioinghinal
poartă denumirea de “canal inghinal” și este locul unde cordonul
spermatic sau ligamentul rotund, părăsește cavitatea abdominală;
C. Herniile indirecte și directe sunt cauzate de pătrunderea
conținutului abdominal în canalul inghinal printr-un defect la
nivelul aponevrozei mușchiului oblic extern;
D. Hernia inghinală indirectă apare atunci când conținutul
abdominal herniază medial de vasele epigastrice;
E. La femei, cea mai frecventă este hernia inghinală directă.
R: A, B (2, p. 131-133)

133
7. Tabloul clinic în herniile orificiului miopectineal:
A. În cazul herniilor femurale asimptomatice, supravegherea este
suficientă;
B. Se prezintă ca o proeminență în regiunea inghinală care se poate
mări odată cu orice creștere a presiunii intraabdominale;
C. Hernia femurală se palpează deasupra ligamentului ilioinghinal;
D. Noțiunile de încarcerare și strangulare se aplică în aceeași
măsură ca la herniile peretelui abdominal;
E. IRM-ul pune diagnosticul de certitudine.
R: B, D (2, p. 133)

8. Tehnici chirurgicale în herniile orificiului miopectineal:


A. Are ca principii: cura herniei trebuie efectuată în tensiune și
plasa poate fi folosită și în cazul plăgilor contaminate;
B. Procedeul alloplastic Lichtenstein este o operație pe cale
anterioară, efectuată cu plasă, care acoperă suprafața anterioară a
peretelui posterior al canalului inghinal;
C. Standardul astazi sunt procedeele tisulare;
D. Procedeul tisular este util în situația unui câmp operator
contaminat;
E. Abordarea posterioară este efectuată laparoscopic sau robotic,
prin abordare extraperitoneală totală sau transabdominală
preperitoneală.
R: B, D, E (2, p. 133-134)

9. Complicațiile postoperatorii ale herniilor orificiului


miopectineal:
A. Simptomele neuropate pot include: hipoestezie la nivelul
inciziei tegumentare, parestezii sau arsuri;
B. Lezarea nervului ilioinghinal va produce durere care iradiază
spre scrot sau labii;
C. Atrofia testiculară reprezintă o complicație imediată
postoperatorie;
D. Orhita este în general limitată și se tratează cu AINS;
E. Pacientul poate resimți inflamația postoperatorie sau apariția
unor echimoze pe scrot, labii sau penis.
R: A, B, D, E (2, p. 134-136)

10. Hernia obturatorie:

134
A. Se încadrează în categoria herniilor peretelui abdominal;
B. Este rezultatul unui defect al planșeului pelvin la nivelul
canalului obturator;
C. Este numită “mica hernie a femeii tinere”;
D. Este un diagnostic dificil, fiind rară iar prezentarea sa este
intermitentă;
E. La examenul clinic, pacientul poate avea parestezii sau durere
intensă pe fața anteromedială a coapsei.
R: B, D, E (2, p. 137)

135
CAPITOLUL 23
ESOFAGUL

1. Cele patru segmente ale esofagului sunt:


A. Intramural
B. Cervical
C. Toracic proximal
D. Toracic mijlociu
E. Abdominal distal
R: B, C, D, E (2, p. 140)

2. Care sunt zonele anatomice de îngustare esofagiană cu


importanţă clinică:
A. Esofagul proximal la nivelul muşchiului cricofaringian
B. Esofagul proximal la nivelul arcului aortic
C. Esofagul mijlociu la nivelul arcului aortic
D. Esofagul distal la nivelul diafragmului
E. Esofagul mijlociu la nivelul diafragmului
R: A, C, D (2, p. 140)

3. Esofagul toracic este alimentat arterial de:


A. Artera tiroidiană inferioară
B. Artera gastrică stângă
C. Arterele bronşice
D. Arterele esofagiene mici
E. Artera gastrică dreaptă
R: C, D (2, p. 140)

4. Esofagul distal este drenat prin:


A. Vena coronariană gastrică
B. Vena tiroidiană inferioară
C. Vena gastrică stângă
D. Vena gastrică dreaptă
E. Vena azygos
R: A, C (2, p. 140)

5. Esofagul toracic este drenat prin:


136
A. Vena tiroidiană inferioară
B. Vena azygos
C. Vena gastrică stângă
D. Vena coronariană gastrică
E. Vena hemiazygos
R: B, E (2, p. 140)

6. Drenajul limfatic al esofagului toracic este reprezentat de:


A. Ganglionii limfatici de la nivelul mediastinului posterior
B. Ganglionii limfatici paratraheali
C. Ganglionii limfatici jugulari
D. Ganglionii limfatici hilari pulmonari
E. Ganglionii limfatici celiac
R: A, B, D (2, p. 140)

7. Peretele esofagian este format din următoarele straturi:


A. Submucoasa
B. Mucoasa
C. Muscularis propria
D. Adventice
E. Seroasa
R: A, B, C, D (2, p.140)

8. La ecografia endoscopică se evidenţiază următoarele zone la


nivelul esofagului:
A. Mucoasa superficială
B. Mucoasa profundă
C. Submucoasa
D. Adventice
E. Seroasa
R: A, B, C, D (2, p. 141)

9. Tulburările motilității esofagiene cuprind:


A. Achalazia, tulburări ale peristaltismului, sdr. Mallory-Weiss,
herniile hiatale;
B. Achalazia se traduce prin “eșecul relaxării” sfincterului
esofagian superior la deglutiție;
C. În achalazie, anomalia primară este o boală degenerativă a
plexului nervos mienteric (Auerbach);

137
D. Simptomatologia principală a pacienților cu achalazie constă în
disfagie mai întâi pentru lichide;
E. În achalazie, pacienții acuză inițial simptome de regurgitare,
fiind adesea diagnosticați greșit cu BRGE.
R: C, E (2, p. 158-159)

10. Achalazia:
A. Achalazia este adesea diagnosticată prin tranzit baritat, care
arată clasic “semnul ciocului de pasăre”;
B. Substanța de contrast întâlnește o obstrucție conică progresivă la
nivelul sfincterului esofagian superior;
C. Studiile manometrice indică eșecul relaxării SEI odată cu
deglutiția, combinat cu absența contracțiilor peristaltice normale
ale esofagului;
D. Endoscopia nu este indicată;
E. Tratamentul este limitat la intervenții paliative.
R: A, C, E (2, p. 158-159)

11. Diverticulii esofagieni:


A. Reprezintă o ectazie parietală;
B. Diverticulul Zenker este un diverticul de tracțiune;
C. Diverticulii de pulsiune sunt mai frecvenți și sunt aproape
întotdeauna asociați cu disfuncția motilității esofagiene;
D. Diverticulul Zenker apare la nivelul esofagului distal;
E. Simptomele includ: disfagie, regurgitarea alimentelor nedigerate
și aspirație ocultă.
R: A, C, E (2, p. 161)

12. În legătură cu diverticulul Zenker sunt adevărate următoarele


afirmații :
A. sunt diverticuli de tracțiune
B. sunt localizați la nivelul esofagului cervical
C. anatomic, aceștia apar posterior, într-o zonă de slăbiciune la
tranziția dintre hipofaringe și esofag
D. anatomic, aceștia sunt situați imediat sub mușchiul cricofaringian
E. pacienții sunt de obicei tineri
R : B, C (2, p. 161)

13. Următoarele afirmații despre diverticulii medioesofagieni sunt


false :

138
A. sunt de cele mai multe ori diverticuli adevărați
B. se pot prezenta și ca diverticuli de pulsiune
C. Nu sunt asociați cu inflamația ganglionului paratraheal din
patologii precum tuberculoza, pneumoconioza sau cancerul
pulmonar
D. pacientul prezintă adesea halitoză
E. poate fistuliza în trahee sau vasele de sânge adiacente, ceea ce va
duce la simptome respiratorii și hemoragice
R : C, D (2, p. 161)

14. Următoarele tehnici imagistice sunt indicate în cercetarea


ingestiei de corp străin :
A. radiografia simplă cervicală
B. radiografia cu incidențe laterale cervicale
C. tranzitul baritat
D. computer tomografia cervicală și toracică
E. endoscopia digestivă superioară
R : A, B, D (2, p. 162)

15. Următoarele leziuni sunt tumori esofagiene benigne :


A. leiomiomul
B. rabdomiomul
C. chistul esofagian
D. tumora cu celule granulare
E. polipul fibrovascular
R: A, C, D, E (2, p. 162)

16 .În legătura cu ingestia de corpi stăini, sunt adevărate


următoarele enunțuri :
A. la copii, majoritatea cazurilor se produc prin impactarea
alimentelor
B. este frecventă în cazul bolnavilor psihic
C. o mare parte din impactări apar la nivelul îngustării aortice a
esofagului
D. o mare parte din impactări apar la nivelul stricturilor peptice din
esofagul distal
E. pacientii prezintă sialoree
R : B, D, E (2, p. 162)

139
17. Următoarele tehnici diagnostice sunt indicate în cercetarea
leziunilor esofagiene benigne, cu excepția :
A. tranzitul baritat
B. vizualizarea endoscopică directă
C. ecoendoscopia
D. biopsia
E. computer tomografia
R : D, E (2, p. 162)

18. În legătura cu epidemiologia ingestiei de substanțe caustice se


pot afirma următoarele :
A. este intenționată la copii
B. poate fi intenționată, ca în cazul tentativei de suicid
C. reprezintă un accident, mai ales în cazul adulților
D. este mai frecventă la sexul feminin
E. reprezintă o urgență medicală
R : B, E (2, p. 162-163)

19. În legătura cu ingestia de substanțe caustice sunt false


următoarele afirmații :
A. produsele acide sunt cele mai vătămătoare pentru mucoasa
esofagiană
B. ingestia de produse alcaline conduce la o vătămare esofagiană în
toată grosimea peretelui
C. complicația pe termen scurt a ingestiei de substanțe caustice este
formarea stenozelor
D. identificarea agentului etiologic nu reprezintă un aspect esențial al
tratamentului
E. cel mai important detaliu anamnetic este cantitatea de substanță
ingerată
R: A, C, D, E (2, p. 163)

20. Următoarele afirmații sunt adevărate în ceea ce privește tipul de


substanță ingerată în leziunile caustice esofagiene :
A. ingestia de substanțe acide este de regulă mai gravă

140
B. ingestia de acide produce leziuni superficiale
C. ingestia de acide va produce o senzație extremă de arsură în
cavitatea bucală, limitând expunerea la substanță
D.ingestia de baze este mai puțin gravă
E. ingestia de baze produce leziuni mai profunde
R : B, C, E (2, p. 163)

21. Următoarele afirmații sunt adevărate în legătură cu evaluarea


ingestiei de substanțe caustice:
A. Gravitatea lezională este clasificată în funcție de gradul de
profunzime
B. identificarea naturii agentului utilizat nu reprezintă o prioritate
imediată
C. abordarea terapeutică nu diferă în funcție de natura agentului
ingerat
D. pentru estimarea severității leziunilor este necesar un examen
atent al cavității orofaringiene
E. cel mai important pas initial este corectarea rapidă a acidozei
sistemice
R : A, D (2, p. 163)

22. În legătură cu leiomiomul esofagian, următoarele afirmații sunt


adevărate :
A. se evidențiază ușor prin radiografia toracică din profil
B. tehnicile diagnostice preferate sunt tranzitul baritat și eco-
endoscopia
C. poate prezenta indicație chirurgicală
D. tehnica chirurgicală preferată este enucleerea
E. tehnica chirurgicală preferată este rezecția esofagului implicat cu
anastomoza segmentelor
R: B, C, D (2, p. 162-163)

23. Următoarele afirmații sunt adevărate în legătura cu evaluarea


ingestiei de substanțe caustice :
A. medicul trebuie să fie pregătit pentru intubația oro-traheală de
urgență, cu fibroscopul, pecienții prezentând risc crescut de a
dezvolta edem glotic
B. bronhoscopia reprezintă un examen esențial, care va fi realizat de
urgență

141
C. se realizează computer tomografia de urgență pentru evaluarea
severității lezionale
D. endoscopia flexibilă se realizează rapid, până în 24 de ore, pentru
a reduce la minimum riscul de perforație
E. pentru a evalua profunzimea lezională, se recomandă rezonanța
magnetică transmurală
R : A, D (2, p. 163)

24. Următoarele afirmații sunt false în legătură cu ingestia de


substanțe caustice, cu excepția:
A. gravitatea lezională este clasificată în funcție de gradul de
suprafață esofagiană afectată
B. gravitatea lezională este clasificată în funcție de gradul de
profunzime
C. ingestia de substanțe alcaline tinde să afecteze peretele esofagian
mai profund decât substanțele acide
D. riscul de stenozare, o complicație acută, crește odată cu
profunzimea leziunii
E. riscul de perforație, o complicație cronică, crește odată cu
profunzimea leziunii
R : B, C (2, p. 163)

25. Reprezintă complicații pe termen lung la nivel esofagian ale


ingestiei de substanțe caustice:
A. perforația esofagiană
B. stenozele esofagiene
C. adenocarcinomul
D. carcinomul scuamocelular
E. ulcerul gastric
R: B, D (2, p. 164)

26. Se realizează o endoscopie unui pacient care a ingerat


accidental o substanță acidă. Se constată leziuni esofagiene
caustice de gradul II. Îi recomandați acestuia:
A. tratament antibiotic intravenos, pentru prevenirea unei infecții
B. tratament cu corticosteroizi obligatoriu, pentru prevenirea
formării unei stenoze
C. tranzit baritat la 4-6 luni
D. nimic per oral timp de o săptămână

142
E. monitorizarea atentă a unei eventuale perforații: dureri toracice,
pneumomediastin
R: A, D (2, p. 164)

27. Se realizează o endoscopie unui pacient care a ingerat accidental


o substanță acidă. Se constată leziuni esofagiene caustice de grad
I. Îi recomandați acestuia:
A. tratament antibiotic, pentru prevenirea unei infecții
B. tratament cu corticosteroizi, pentru prevenirea formării unei
stenoze
C. tranzit baritat la 4-6 săptămâni
D. nil per oral timp de o săptămână
E. începerea progresivă a dietei
R: C, E (2, p. 164)

28. Următoarele indicații fac parte din managementul inițial al


ingestiei de substanțe caustice :
A. asigurarea căilor aeriene
B. asigurarea a cel puțin o linie venoasă
C. nimic per os
D. montarea unei sonde nazogastrice
E. realizarea unei radiografii toracice
R : A, B, C, E (2, p. 164)

29. În legătura cu managementul ingestiei de substanțe caustice,


următoarele afirmații sunt adevărate :
A. provocarea vărsăturilor poate fi dăunătoare, re-expunând esofagul
la contactul cu substanța caustică
B. se recomandă neutralizarea substanțelor identificate, cu ajutorul
unei sonde nazogastrice
C. menținerea permeabilității esofagiene reprezintă prima prioritate
D. managementul inițial presupune și menținerea căilor aeriene
E. medicația poate fi administrată per oral în cazul în care stomacul
nu este afectat într-o măsura extinsă
R : A, D (2, p. 164)

30. Următoarele afirmații sunt adevărate în legătura cu gradarea


leziunilor cauzate de ingestia de substanțe caustice esofagiene :

143
A. gradarea este realizată în urma vizualizării endoscopice a
leziunilor
B. ulcerațiile mucoase superficiale corespund gradului I
C. eroziunile mucoase corespund gradului II
D. benzile albe corespund gradului II
E. ulcerațiile adânci corespund gradului IV
R : A, D (2, p. 164)

31. Se realizează o endoscopie unui pacient care a ingerat


accidental o substanță puternic alcalină. Se constată leziuni
esofagiene caustice de grad III. Îi recomandați acestuia :
A. tratament antibiotic per os, pentru prevenirea unei infecții
B. tratament cu corticosteroizi, pentru prevenirea formării unei
stenoze
C. tranzit baritat de monitorizare la 4-6 luni
D. monitorizarea stenozei sau a carcinomului printr-o
esofagoscopie la distanță
E. monitorizarea atentă a unei eventuale perforații: dureri,
toracice, pneumomediastin
R: D, E (2, p. 164 )

32. Următoarele afirmații sunt false în legătură cu managementul


ingestiei de substanțe caustice :
A. antibioticele sunt recomandate standard ca terapie adjuvantă
B. steroizii s-au dovedit eficienți în prevenirea formării stenozei
C. antiinflamatoarele steroidiene sunt eficiente în prevenirea
formării stenozei
D. în cazul leziunilor de până în gradul II, medicația se poate
administra per oral din prima zi
E. dieta și administrarea de substanțe per os trebuie începută
progresiv
R: A, B, C, D (2, p. 164)

33. În legătură cu rezecția chirurgicală din urma ingestiei de


substanțe caustice, se pot afirma următoarele :
A. este indicată în caz de perforații
B. este indicată în caz de stenoze refractare
C. este o tehnică dificilă, datorită fibrozelor extinse din țesuturile
peri-esofagiene

144
D. necesită frecvent interpoziția de stomac, anastomozele
esofagiene fiind dificil de realizat în cazul unei leziuni extinse
E. dacă rezecția este imposibilă datorită fibrozei extinse, se poate
realiza interpunerea de colon pe cale retroesofagiană
R : A, B, C (2, p. 164)

34. Despre realizarea unei gastrostome în cazul ingestiei de


substanțe caustice sunt adevărate următoarele afirmați :
A. va fi necesară în cazul leziunilor mai grave
B. este recomandată în cazul leziunilor extinse ale esofagului și
stomacului
C. se realizează, de obicei, printr-o tehnică minim invazivă, în
timpul evaluării endoscopice inițiale
D. este utilă pentru alimentație sau administrarea medicamentelor
E. este utilă pentru accesul viitor în vederea realizării dilatării
anterograde ghidate cu bujii, pentru prevenirea stenozelor
R : A, C, D (2, p. 164)

145
Capitolul 24
STOMACUL ȘI DUODENUL

1. Stomacul are urmatoarele raporturi anatomice:


A. Lateral dreapta cu rinichiul drept
B. Superior cu diafragmul
C. Lateral cu splina
D. Posterior cu pancreasul
E. Inferior cu marele epiploon
R: B, C, D, E (2, p. 167)

2. Jonctiunea gastroduodenala:
A. Mai poarta denumirea de pilor
B. Este un muschi striat
C. Previne refluxul continutului duodenal in stomac
D. Controleaza evacuarea gastrica alaturi de pompa ventrala
E. Controleaza evacuarea gastrica alaturi de pompa antrala
R: A, C, E (2, p. 167)

3. Corpul gastric:
A. La acest nivel se secreta HCl
B. Are rol in depozitarea alimentelor
C. Este cea mai distala regiune
D. Contine cea mai mare parte a celulelor parietale
E. La acest nivel are loc maruntirea alimentelor prin miscari
peristaltice
R: A, B, D, E (2, p. 167)

4. Peretele stomacului este reprezentat de urmatoarele straturi:


A. Mucoasa
B. Aponevrotic
C. Submucoasa
D. Musculara
E. Fascial
R: A, C, D (2, p. 167)

5. Vascularizatia arteriala a stomacului cuprinde arterele:


146
A. Gastrice lungi
B. Gastrica dreapta si stanga
C. Gastrice scurte
D. Gastroepiploica dreapta
E. Gastroepiploica stanga
R: B, C, D, E (2, p.167)

6. Partile in care este impartit duodenul sunt urmatoarele


A. Bulbul duodenal
B. Duodenul mijlociu
C. Duodenul ascendent
D. Duodenul descendent
E. Duodenul transversal
R: A, C, D, E (2, p. 167)

7. Duodenul, ca organ endocrine, secreta:


A. Bicarbonat
B. HCl
C. Secretina
D. Colecistokinina
E. Pepsinogen
R: C, D (2, p. 167)

8. Vascularizatia arteriala a duodenului provine din:


A. Artera mezenterica superioara
B. Artera mezenterica inferioara
C. Artera pancreatico-duodenala inferioara
D. Artera gastro-duodenala
E. Artera pancreatico-duodenala superioara
R:A, D (2, p. 167-168)

9. Acidul clorhidric:
A. Este secretat in conditii bazale
B. Este secretat in conditii de stimulare
C. Secretia HCl are un ritm circadian, cu nivel maxim in timpul
diminetii
D. Gastrina favorizeaza secretia de HCl
E. Toate peptidele inhiba secretia de HCl
R: A, B, D (2, p. 168)

147
10. Sistemul de bariere protectoare ale mucoasei gastrice sunt:
A. Stratul de mucus-bicarbonat
B. Prostaglandine
C. Glandele Brunner
D. Inhibitorii pompei de protoni
E. Gastrina
R: A, B (2, p. 169)

11. Clasificarea Ulcerelor Gastrice:


A. Ulcerele gastrice de tip I sunt cele mai frecvente si apar in
curbura mica a stomacului
B. Ulcerele gastrice de tip II apar in combinatie cu ulcerele
duodenale
C. Ulcerele gastrice de tip III se dezvolta in regiunea prepilorica
D. Ulcerele gastrice de tip IV sunt cel mai frecvente si apar pe
curbura mare
E. Ulcerele gastrice de tip V sunt cel mai rar intalnite
R: A, B, C (2, p.170)

12. Helicobacter Pylori:


A. Bacterii mici, curbate
B. Gram-pozitive
C. Microaerofile
D. Transmitere gastro-orala
E. Transmitere fecal-orala
R: A, C, D, E (2, p. 170)

13. In ulcerele gastrice necomplicate pacientul acuza, de obicei:


A. Durere gastrica cu caracter de arsura si iradiaza in hipocondrul
drept
B. Durere epigastrica cu caracter de arsura ce iradiaza posterior
C. Durerea este asociata cu ingestia de alimente
D. Anorexie
E. Scadere ponderala
R: B, C, D, E (2, p. 170)

14. Esofagogastroduodenoscopia (EGD):


A. Poate confirma prezenta unui ulcer
B. Permite examinarea mucoasei in detaliu
C. Permite documentarea si compararea in evolutie a vindecarii

148
D. Se pot efectua biopsii multiple in timpul endoscopiei
E. Nu exista caracteristici endoscopice ce pot sugera malignitate
R: A, B, C, D (2, p. 170)

15. Tratamentul de prima linie pentru ulcerul gastric necomplicat


este:
A. Tratamentul medical
B. Tratamentul chirurgical
C. Intreruperea fumatului
D. Medicatie cu AINS
E. Intreruperea ingestiei de alcool
R: A, C, E (2, p. 171)

16. In cazul tratamentului chirurgical al ulcerului gastric care cu se


vindeca prin tratament medical correct:
A. Se practica excizie complete datorita posibilitatii de malignitate
B. Pentru ulcerele de tip I,II si III se realizeaza o antrectomie
generoasa
C. Pacientii cu ulcere de tip II si III nu beneficiaza de vagotomie
D. Ulcerele de tip IV pot necesita gastrectomie totala sau subtotala
E. In cazul stenozei gastrice nu este necesara intervetia chirurgicala
de urgenta
R: A, B, D (2, p. 171)

17. Gastrita acuta prezinta urmatoarele simptome:


A. Cresterea poftei de mancare
B. Varsaturi
C. Reflux gastroesofagian
D. Hematochezie
E. Hematemeza
R: B, D, E (2, p. 171)

18. Etiologia gastritei acute este reprezentata de:


A. Infectia cu H. Pylori
B. Utilizarea AINS
C. Refluxul gastroesofagian
D. Iradiere
E. Utilizarea AIS
R: A, B, D (2, p. 171)

149
19. Gastrita de stress:
A. Eroziunile de la nivelul mucoasei progreseaza lent si pot
cuprinde intreg stomacul
B. Apare la pacientii cu arsuri grave
C. Apare la pacientii cu insuficienta multipla de organe
D. Profilaxia consta in administrarea de AINS
E. Apare la pacientii cu leziuni ale SNC (ulcerul Cushing)
R: B, C, E (2, p. 171)

20. Tratamentul gastritei de stress consta in:


A. Suprimarea agresiva a aciditatii gastrice
B. Scaderea PH’ului intraluminal sub 4,0
C. Administrarea i.v. a IPP
D. Administrarea I.v. a blocantilor de H2
E. Administrarea de Sucralfat si Misoprostul nu reprezinta o
optiune
R: A, C, D (2, p. 171)

21. Tratamentul hemoragiei, ca si complicatie a gastritei de stress,


consta in:
A. Stabilizare si resuscitare volemica
B. Electrocauterizare
C. Injectare de agenti vasoconstrictori
D. Sutura in situ a eroziunilor este o procedura de prima intentie in
controlul hemoragiei
E. Gastrectomie totala
R: A, B, C, E (2, p. 172)

22. Cauzele aparitiei sindromului Mallory-Weiss pot fi urmatoarele:


A. Sincopa
B. AVC
C. Peritonita
D. Ridicarea de greutati mari
E. Convulsii
R: D, E (2, p. 172)

23. In sindromul Mallory-Weiss, pacientii prezinta In mod obisnuit:


A. Eroziuni ale mucoasei gastrice
B. Hematemeza
C. Durere ‘’in bara’’

150
D. Hematochezie
E. Melena
R: B, D, E (2, p. 172)

24. Tratamentul in Sindromul Mallory-Weiss este reprezentat de:


A. Resuscitare hidroelectrolitica
B. Administrare AINS
C. Terapie pentru scaderea aciditatii
D. Sutura chirurgicala a rupturilor mucoasei
E. Interventia chirurgicala este de prima intentie
R: A, C (2, p. 172)

25. Polipii gastrici:


A. Sunt rar intalniti
B. Pot fi hiperplastici sau adenomatosi
C. Polipii hiperplastici sunt, de obicei, maligni
D. Polipii adenomatosi au un risc mai mic de degenerare maligna
E. Se gasesc doar la nivel gastric
R: A, B (2, p. 171)

26. Sindromul Peutz-Jeghers, este caracterizat prin:


A. Prezenta unor polipi maligni la nivelul intestinului gros
B. Prezenta unor polipi benigni la nivelul intestinului subtire
C. Prezenta petelor de melanina pe buze si mucoasa bucala
D. Este o boala autosomal dominanta cu un grad ridicat de
penetrabilitate
E. La acesti pacienti, tumorile sunt hamartoame si rareori benigne
R: B, C, D (2, p. 172)

27. Factorii de risc importanti pentru adenocarcinomul gastric:


A. Infectia cu H.Pylori
B. Anemia pernicioasa
C. Achlorhidia
D. Hipertensiune portala
E. Gastrita de stress
R: A, B, C (2, p. 172)

151
28. Cancerele gastrice sunt adesea clasificate in functie de aspectul
endoscopice, pot fi:
A. Conopidiforme
B. Rugoase
C. Ulcerate
D. Polipoide
E. Extinse superficial
R: C, D, E (2, p. 172)

29. Linita plastica:


A. Descrie cancerele ce infiltreaza local portiuni din peretele gastric
B. Descrie cancerele ce infiltreaza difuz portiuni din peretele gastric
sau stomacul in intregime
C. Stomacul are aspect de tub rigid
D. Prognosticul este favorabil
E. Prognosticul este deosebit de rezervat
R: B, C, E (2, p. 172)

30. Manifestarile clinice ale adenocarcinomului gastric avansat,


sunt:
A. Durere vaga epigastrica
B. Pierdere inexplicabila a greutatii
C. Melena
D. Durere in hipocondrul drept
E. Varsaturi alimentare
R: A, B (2, p. 173)

31. In cazul adenocarcinomului gastric, semnele suggestive pentru


boala extensa pot fi urmatoarele:
A. Nodul limfatic supraclavicular stang (nodulul lui Virchow)
B. Nodul ombilical palpabil (semnul Sister Mary Joseph)
C. O formatiune rectala palpabila la tuseul rectal (semnul Blumer)
D. Prezenta ascitei
E. Anemie cu deficit de fier
R: A, B, C, D (2, p. 173)

32. Investigatiile paraclinice necesare diagnosticului si stadializarii


cancerului gastric pot fi urmatoarele:
A. Ecografia abdominala
B. Endoscopia digestive superioara

152
C. Ecografia endoscopica
D. CT abdomen si pelvis
E. PET CT
R: B, C, D, E (2, p. 173)

33. In cazul unei laparoscopii pentru stadializarea cancerului


gastric:
A. Ar trebui sa identifice leziuni ale afectarii peritoneale, hepatice
sau omentale
B. Leziunile nu pot fi biopsiate
C. Se practica lavaj peritoneal si se determina extensia locala a
tumorii
D. Prezenta bolii metastatice indica rezectia curative a formatiunii
E. Laparoscopia de stadializare este practicata in mod selectiv
R: A, C, E (2, p. 173)

34. In cazul unui limfom gastric:


A. Pacientii prezinta aceleasi simptome observate in
adenocarcinomul gastric
B. De obicei, nu este necesara stadializarea limfomului
C. Endoscopia cu biopsie tisulara stabileste diagnosticul
D. Este preferat tratamentul prin chimioterapie
E. Cand leziunea este limitata la stomac, supravietuirea la 5 ani
dupa rezectie se poate apropie de 75%
R: A, C, E (2, p. 174)

35. Tabloul clinic al ulcerului duodenal necomplicat este reprezentat


de:
A. Epigastralgie cu caracter de arsura
B. Durere ce apare inainte de masa si nu este calmata de ingestia
alimentara
C. Durere ce apare la 1-3 ore postprandial si poate trezi pacientul
din somn
D. Crestere recenta in greutate
E. Scadere ponderala
R: A, C, D (2, p. 174)

153
36. Factorii de risc in ulcerul duodenal sunt reprezentati de:
A. Antecedente de infectie cu H. pylori
B. Tabagism cronic
C. Consum de AINS
D. Istoric de BUP
E. Utilizarea prelungita de IPP
R: A, B, C, D (2, p. 173)

37. Semnele pierderii oculte de sange sunt:


A. Rectoragia
B. Paloarea
C. Hipotensiunea ortostatica
D. Melena
E. Scaunele cu test guaiac pozitiv
R: B, C, E (2, p. 174)

38. Investigatia de electie in ulcerul duodenal nu este reprezentata


de:
A. Endoscopia digestive superioara
B. Determinarea antigenului H. pylori din fecale
C. Testul la ureaza
D. Tomografia computerizata abdominala
E. Ecografia abdominala
R: B, C, D, E (2, p. 175)

39. Tratamentul ulcerului duodenal necomplicat se face prin:


A. Intreruperea agentilor ulcerogenic precum fumatul si AINS
B. Eradicarea infectiei cu H. pylori
C. Consum de alimente acide
D. Administrarea de IPP sau antagonisti ai receptorului H2 pentru
suprimarea aciditatii gastrice
E. Administrarea de AINS
R: A, B, D (2, p. 175)

40. Urmatoarele afirmatii despre eradicarea infectiei cu H. pylori


sunt adevarate:
A. Tratamentul antibiotic dureaza 14 zile
B. Tratamentul antibiotic dureaza 10 zile
C. Tratamentul de prima linie include tripla terapie pe baza de
levofloxacina

154
D. Tratamentul de linia a doua include tripla terapie de baza de
levofloxacina
E. Tratamentul de prima linie include tratamentul cu bismut
R: A, D (2, p.175)

41. Manifestarile principale in boala ulcerative peptica sunt:


A. Perforatia
B. Hemoragia
C. Stenoza gastrica
D. Disfagia
E. Malignizarea
R: A, B, C, E (2, p. 175)

42. Tabloul clinic al ulcerului perforat nu este reprezentat de:


A. durere epigastrica severa cu debut acut
B. durere epigastrica severa cu debut lent
C. durere severa in hipocondrul drept cu iradiere in umar
D. durere epigastrica de intensitate moderata
E. durere severa in hipocondrul drept de tip colicativ
R: B, C, D, E (2, p. 175)

43. Ulcerul hemoragic este caracterizat de urmatoarele simptome:


A. Diaree
B. Hematemeza
C. Melena
D. Rectoragii
E. Durere in flancul stang
R: B, C, D (2, p. 175-176)

44. La examenul fizic, in cazul unui ulcer hemoragic, pacientul


prezinta:
A. Hipotensiune arteriala
B. Hipertensiune arteriala
C. Tahicardie
D. Bradicardie
E. Sangerare active
R: A, C, E (2, p. 175)

155
45. Sindromul Zollinger-Ellison:
A. Este o afectiune tumorala exocrine
B. Este foarte rar
C. Este consecinta directa a unui neoplasm(gastrinom)
D. Aproximativ 60% din toate gastrinoamele sunt benigne
E. Gastrinoamele pot aparea sporadic sau ca parte a unui sindrom
familial mostenit
R: B, C, E (2, p. 179)

46. Diagnosticul sindromului Zollinger-Ellison:


A. Se bazeaza pe stabilirea prezentei hipergastrinemiei cu
hipersecretie acida gastrica
B. Pacientul intrerupe orice tratament cu IPP cu o zi inainte de
recoltare
C. Investigatia de electie este testul de stimulare la secretina
D. Pacientii cu sindromul Zollinger-Ellison prezinta o crestere a
valorii initiale a gastrinei >200pg/mL
E. Prezenta nivelurilor crescute de gastrina sub 1000 pg/mL este
diagnostica
R: A, C, D (2, p. 179)

47. Sindroamele postgastrectomie sunt:


A. Sindromul de dumping precoce
B. Sindromul Mallory-Weiss
C. Sindromul de dumping tardive
D. Diareea post-vagotomie
E. Constipatia post-vagotomie
R: A, C, D (2, p. 180)

48. Pacientul cu dumping precoce prezinta:


A. Evacuarea necontrolata a lichidului hipertonic gastric in
intestinul subtire
B. Debut la 15-30 minute dupa o masa hiperosmolara
C. Senzatie de foame
D. Debut 1-3 ore dupa o masa hiperosmolara
E. Dificultati de concentrare
R: A, B (2, p. 180)

156
49. Pacientul cu dumping tardiv, prezinta:
A. Debut la 15-30 minute dupa o masa hiperosmolara
B. Debut la 1-3 ore dupa o masa hiperosmolara
C. Fatigabilitate
D. Senzatie de foame
E. Hipoglicemie
R: B, C (2, p. 180)

50. Despre boala ulcerului marginal si recurent putem afirma


urmatoarele:
A. Se dezvolta pe partea jejunala a unei anastomose gastrojejunale
B. Pacientii pot prezenta dureri abdominale in timpul alimentatiei,
greata si varsaturi
C. Managementul conservator presupune intreruperea consumului
de tutun si initierea terapiei cu IPP
D. Tratamentul de prima intentie este cel chirurgical
E. Boala ulceroasa recurenta dupa interventia chirurgicala in BUP
benigna este cauzată cel mai frecvent de o vagotomie completă
R: A, B, C (2, p. 182)

157
CAPITOLUL 25
INTESTIN SUBȚIRE ȘI APENDICE

1. Ocluziile intestinului subțire sunt clasificate în:


A. Complete
B. Ascendente
C. Parțiale
D. Deschise
E. Particulare
R : A, C (2, p. 193)

2. Este adevărat despre ocluzia intestinului subțire:


A. Aderențele postoperatorii sunt cele mai frecvente cauze ale
ocluziei intestinului subțire în țările industrializate
B. Aderențele sunt cel mai frecvent cauza ocluziei în timpul sau în
urma unor boli inflamatorii acute ale organelor peritoneale
C. Aderențele postoperatorii sunt prezente la mai mult de 90% dintre
pacienții cu două sau mai multe intervenții chirurgicale
abdominale în antecedente
D. Tumorile sunt cele mai frecvente cauze ale ocluziilor intestinului
subțire în țările industrializate
E. Procedeele laparoscopice produc cu 80% mai puține aderențe
decât procedeele deschise
R: A, C, E (2, p. 193)

3. Este adevărat despre ocluzia intestinului subțire, cu excepția:


A. Cele mai frecvente cauze sunt reprezentate de factori intrinseci
B. Aderențele postoperatorii sunt prezente la mai puțin de o treime
din pacienții cu una sau mai multe intervenții chirurgicale
abdominale
C. Cele mai frecvente cauze sunt reprezentate de factorii extrinseci
D. Aderențele postoperatorii sunt prezente la două treimi dintre
pacienții care au suferit o intervenție chirurgicală abdominală
E. Ocluzia intestinului subțire secundară sindromului aderențial
constituie o problemă de sănătate publică în țările industrializate
R: A, B (2, p. 193)

158
4. Volvulusul:
A. Reprezintă telescoparea unei porțiuni a intestinului în el însuși
B. Reprezintă răsucirea intestinului subțire în jurul axului mezenteric
C. Poate apărea spontan la copiii mici
D. Poate apărea din cauza fixării aderențiale postchirurgicale a
intestinului
E. Poate a părea ca urmare a bridelor sau malrotațiilor congenitale
R : B, D, E (2, p. 194)

5 . Obstrucțiile intestinale prin stricturi:


A. Sunt adesea incomplete
B. Sunt adesea complete
C. Au frecvent debut insidios
D. Deoarece mezenterul nu este compromis, strangularea este
improbabilă
E. Cele mai multe stricturi benigne necesită intervenție chirugicală
R: A, C, D (2, p. 194)

6. Sunt afirmații corecte despre invaginație:


A. O porțiune a intestinului se telescopează în el însuși
B. Reprezintă răsucirea intestinului subțire în jurul axului mezenteric
C. Poate apărea spontan la vârstnici
D. Necesită atenție imediată
E. Patologia este autolimitantă
R : A, E (2, p. 194-195)

7. Alegeți afirmațiile adevărate despre oculuziile intestinului


subțire:
A. Debutul tipic al OIS este marcat de dureri abdominale colicative
B. Durerea poate deveni constantă, pe măsură ce activitatea
peristaltică scade
C. Durerea se localizează epigastric, cu iradiere în bară
D. Greața și vărsăturile sunt acuze frecvente la mulți pacienți
E. Privind greața și vărsăturile, debutul acestora poate fi întârziat în
obstrucții mai distale
R: A, B, D, E (2, p. 195)

8. În obstrucția intestinului subțire, deshidratarea este o consecință


a:

159
A. Diareei
B. Vărsăturilor
C. Scăderii absorbției și a secreției stimulată hormonal de distensie
lumenului
D. Creșterii absorbției
E. Constipației
R: B, C (2, p. 195)

9. În ocluziile intestinului subțire, sunt indicatori ai unui risc mare


de strangulare:
A. Tahicardie
B. Sensibilitate abdominală localizată
C. Leucocitoză
D. Trombocitoză
E. Leucopenie
R: A, B, C (2, p.195)

10. Sunt semne și simptome frecvente de prezentare a ocluziei


intestinului subțire, cu excepția:
A. Leucocitoză
B. Durere
C. Prezența tranzitului intestinal pentru gaze
D. Vărsături
E. Sensibilitate la palpare
R: A, C (2, p. 195)

11. Sunt manifestări ce impun intervenția chirurgicală de urgență


în ocluziile intestinului subțire :
A. Durerea persistentă
B. Durerea intermitentă
C. Sensibilitatea la percuție
D. Teama de mișcare
E. Durerea care se remite spontan

R : A, C, D (2, p. 196)

160
12. În ceea ce privește ocluzia intestinului subțire, la examenul
obiectiv:
A. Pacienții au în general o stare generală bună în fazele avansate
B. Pacienții se pot prezenta cu stare generală acut alterată în fazele
precoce
C. În cazuri mai avansate, apar tahicardia și hipotensiune relativă
D. La pacienții obezi, sensibilitatea în apropierea unei cicatrici
chirurgicale poate fi singurul indiciu pentru încarcerarea
intestinală
E. O accelerare a activității intestinale este caracteristică
R: B, C, D (2, p. 196)

13. În ocluzia intestinului subțire, în absența altor patologii


existente, descoperirile tipice pe radiografie includ:
A. Distensie intestinală distală
B. Distensie intestinală proximală de punctul de ocluzie
C. Aspectul de cotor de măr
D. Semnul vârtejului
E. Colabarea intestinului distal
R: B, E (2, p. 196)

14. Despre examinările de laborator în ocluzia intestinului subțire


putem afirma:
A. Nu sunt specifice
B. Pot avea utilitate în excluderea altor patologii
C. Leucopenia este semnul sugestiv
D. Electroliții trebuie monitorizați atent
E. Include examenul ECG
R: A, B, D (2, p. 197)

15. Este adevărat despre examinările de laborator în ocluziile


intestinului subțire:
A. Leucocitoza care persistă în ciuda decompresiei pe cale
nazogastrică poate fi semn deprogresie spre ischemie
B. Monitorizarea electroliților nu este utilă
C. Alcaloza hipokalemică de contracție este frecventă la pacienții
cu deshidratare avansată

D. Hipoamilazemia ridică suspiciunea de pancreatită cronică

161
E. Examenul sumar de urină este util pentru excluderea infecției
urinare
R : A, C , E (2, p. 197)

16. Investigațiile cu substanță de contrast ale intestinului subțire


sunt utile pentru:
A. Pacientul cu simptome ce se remit spontan
B. Pacienții cu simptome obstructive totale
C. Când obstrucția este sigură
D. Pacientul cu simptome obstructive parțiale, persistente
E. Când este dificilă diferențierea ileusului paralitic de obstrucția
mecanică
R: D, E (2, p. 197)

17. Tratamentul obstrucției intestinului subțire cuprinde:


A. Reechilibrare hidroelectrolitică
B. Administrarea de noradrenalină
C. Dietă hiposodată
D. Administrarea de soluții hipertone
E. Administrarea intravenoasă de soluții izotone
R: A, E (2, p. 198)

18. În cadrul ocluziei intestinului subțire, montarea sondei


nazogastrice:
A. Crește cantitatea de lichide care pot fi aspirate
B. O cantitate de urină de cel puțin 0,5 mL/kg/h indică resuscitare
volemică adecvată
C. Poate fi suplimentată cu monitorizarea invazivă a presiunii
venoase centrale
D. Reduce cantiatea de lichide care pot fi aspirate
E. Crește riscul de microaspirație
R: D, E (2, p. 198)

19. Referitor la tratamentul ocluziilor intestinului subțire, sunt


adevărate următoarele:
A. Statusul normovolemic trebuie restabilit înainte de înlocuirea
agresivă a electroliților
B. Înlocuirea agresivă a electroliților trebuie realizată înaintea
restabilirii statusului normovolemic

162
C. Monitorizarea diurezei prin cateterul Foley este eficientă pentru
evaluarea volumului înlocuit
D. La adulți, o cantitate de urină de cel puțin 0,3mL/kg/oră indică
resuscitare volemică adecvată
E. La adulți, o cantiate de urină de peste 0,5mL/kg/oră indică
resuscitare volemică adecvată
R : A, C, E (2, p. 198)

20. În ceea ce privește tratamentul ocluziei intestinului subțire,


eșuarea reducerii unei hernii se poate manifesta prin:
A. Lipsa durerii
B. Durere persistenă
C. Sensibilitate cu modificări inflamatorii sau de laborator
D. Sensibilitate fără modificări inflamatorii sau de laborator
E. Leucopenie
R: B, C, D (2, p. 199)

21. În tratamentul chirurgical al ocluziei intestinului subțire este


importantă evitarea rezecțiilor extinse, mai ales în cazul:
A. Pacienților colecistectomizați
B. Pacienților cu colită ulcero-hemoragică
C. Când lungimea intestinului subțire este de 150cm
D. Pacienților cu boala Crohn
E. Când lungimea intestinului subțire restant se apropie de 100cm
R: D, E (2, p. 199)

22. În ocluziile intestinului subțire, pacienții la care terapia non-


operatorie este justificată inițial, ar trebui propuși pentru
intervenția chirurgicală dacă apar:
A. modificări la examenul clinic
B. necesar hidric crescut
C. parametri de laborator alterați
D. absența ameliorării clinico-radiologice
E. necesar hidric extrem de scăzut
R: A, B, C, D (2, p. 199)

23. Sunt complicații ale intervenției chirurgicale în ocluzia


intestinului subțire :
A. Infecția plăgii
B. Scurgeri anastomotice

163
C. Abces
D. Gangrenă
E. Peritonită
R: A, B, C, E (2, p. 199)

24.Complicațiile intervenției chirurgicale în ocluzia intestinului


subțire sunt:
A. Abces
B. Durere persistentă
C. Peritonită
D. Formarea de fistule
E. Sensibilitate la palpare
R: A, C, D (2, p. 199)

25. Alegeți afirmațiile false despre ischemia mezenterică acută :


A. Diagnosticul se face prin vizualizarea directă a arborelui
vascular
B. Standardul de aur este examenul CT
C. Arteriografia mezenterică nu permite vizualizarea ocluziei AMS
D. Arteriografia mezenterică permite identificarea altor semne de
infarct intestinal, ca îngroșarea peretelui intestinal
E. În IMNO se vor observa spasmul și îngustarea arborelui vascular
R: B, C, D (2, pag. 205)

26. Despre ischemia mezenterică acută este adevărat:


A. În mod clasic, durerea are debut lent, insidios
B. Examenul obiectiv decelează abdomen mărit de volum, tare
C. Durerea este severă și are debut brusc
D. Examinarea precoce a abdomenului este adesea neconcludentă
E. Dacă pacientul prezintă semne clinice de peritonită, infarctul
intestinului subțire este deja instalat
R: C, D, E (2, p. 205)
27. În tratamentul ischemiei mezenterice acute putem afirma:
A. Odată ce fluxul sanguin este restabilit, intestinul necrozat
trebuie rezecat
B. Frecvent este necesară o a treia intervenție chirurgicală
C. Intestinul cu aspect ischemic nu ar trebui excizat
D. O anastomoză primară este adesea evitată în vederea unei
intervenții secundare în 12- 24h
E. Intestinul necrozat nu necesită rezecție

164
R: A, C, D (2, p. 205-206)
28. Complicațiile ischemiei mezenterice acute includ:
A. Insuficiență respiratorie
B. Insuficiență hepatică
C. Infecții intraabdominale
D. Ulcerul duodenal
E. Dehiscența anastomotică
R: A, C, E (2, p. 206)

29. Despre tumorile benigne ale intestinului subțire putem afirma:


A. Tumorile maligne sunt mult mai frecvente decât cele benigne
B. Majoritatea tumorilor prezintă simptomatologie specifică
C. Ambele sexe sunt afectate în mod egal
D. Sexul feminin este mai afectat
E. Cele mai frecvente leziuni benigne au fost denumite leiomioame
R: C, E (2, p. 206)

30. Alegeți afirmațiile corecte despre adenocarcinoamele


intestinului subțire:
A. Sunt aproximativ jumătate din tumorile maligne ale intestinului
subțire
B. Sunt mai frecvente în jejun
C. Ocluzia constituie cea mai frecventă formă de prezentare
D. Sângerarea masivă constituie cea mai frecventă formă de
prezentare
E. Tumorile din regiunea periampulară se pot prezenta cu icter
nedureros
R: A, C, E (2, p. 207)

31. Carcinomul duodenal al regiunii periampulare:


A. Este o tumoră benignă non-epitelială
B. Apare de obicei dintr-un adenom benign preexistent
C. Este cea mai frecventă neoplazie ce afectează indivizii cu
polipoză familială post-proctocolectomie
D. Atât duodenul cât și papila duodenală trebuie monitorizate
endoscopic periodic
E. Monitorizarea endoscopică periodică este de utilitate redusă
R: B, C, D (2, p. 207)

32. Adenoamele glandelor Brunner:

165
A. Sunt mai predispuse la transformare malignă
B. Ar trebui să fie excizate
C. Apar de obicei dintr-un adenom preexistent
D. Apar în principal în duoden
E. Sunt de obicei asimptomatice
R : D, E (2, p. 207)

33. Sunt tumori non-epiteliale ale intestinului subțire:


A. Lipoamele
B. Adenoamele
C. Hemangioamele
D. Hamartoamele
E. Limfangioamele
R: A, C, D, E (2, p. 207)

34. Sindromul carcinoid include:


A. Eritem cutanat episodic
B. Bronhospasm
C. Crampe și constipație
D. Instabilitate hemodinamică
E. Valvulopatie cardiacă stângă
R: A, B (2, p. 207)

35. Despre tumorile non-epiteliale ale intestinului subțire, este


adevărat :
A. Lipoamele sunt mai frecvente la bărbați
B. Hemangioamele reprezintă o cauză importantă de hemoragie
ocultă
C. Enteroscopia cu capsulă oferă posibilitate delimitării
hamartoamelor
D. Hamartoamele sunt de obicei izolate și asimptomatice
E. Mai multe hamartoame pot fi identificate în sindromul Osler-
Rendu-Weber
R: A, B, D (2, p. 207)

36. Pentru ca sindromul carcinoid să apară:


A. Descărcarea peptidelor vasoactive în circulația portală
B. Leziunea intestinală trebuie să metastazeze în plămâni
C. Leziunea intestinală trebuie să metastazeze în ficat
D. Tumora trebuie să se afle într-o zonă drenată de vena portă

166
E. Tumora primară trebuie să se afle într-o altă zonă care nu
drenează prin sistemul port
R: C, E (2, p. 208)

37. Referitor la limfoamele intestinului subțire, este adevărat:


A. Intestinul subțire este cea mai comună localizare
extraganglionară a limfoamelor
B. Doar 5% dintre limfoame se regăsesc la nivelul intestinului
subțire
C. Ileonul este cel mai frecvent implicat
D. Duodenul este cel mai frecvent implicat
E. Limfoamele sunt adesea descrise la nivelul stomacului, în
prezența gastritei asociate cu Helicobacter Pylori
R: A, B, C, E (2, p. 208)
38. În diverticulul Meckel, hemoragia:
A. Se prezintă ca melenă
B. Se prezintă ca rectoragie
C. Este cu sânge roșu aprins
D. Este de obicei nedureroasă
E. Este întâlnită cel mai frecvent la vârstnici
R : B, C, D (2, p. 209-210)

39. În cadrul diverticulului Meckel, simptomele se pot manifesta la:


A. 5% dintre sugarii cu diverticul
B. 20% dintre sugarii cu diverticul
C. 5% dintre indivizii de 40 ani
D. 1,5% dintre indivizii de 40 ani
E. Extrem de rar la vârstnici
R: A, D, E (2, p. 209)

40. Sunt cele mai frecvente manifestări asociate diverticulului


Meckel :
A. Febra
B. Ocluzia
C. Inflamația
D. Mialgiile
E. Fistula ombilicală
R: B, C, E (2, p. 209)

167
41. Persistența canalului vitelin poate provoca o varietate de
probleme :
A. Dacă există o persistență completă, poate rezulta o comunicare
între ombilic și ileon
B. Dacă peristența este completă, apare o comunicare între duoden
și jejun
C. Dacă doar capătul ombilical al ductului rămâne patent, rezultă
fistulă ombilicală
D. În cazul persistenței complete, poate rezulta o fistulă ombilicală
E. Dacă ductul se obliterează, dar rămâne sub forma unui cordon
fibros, acest vestigiu acționează ca punct de fixare a intestinului
la peretele abdominal
R: A, C, E (2, p. 209)

42. Sunt manifestări asociate diverticulului Meckel cu excepția:


A. Febra
B. Ocluzia
C. Inflamația
D. Hemoragia
E. Fistula mezenterică
R: A, E (2, p. 209)

43. Referitor la diverticulul Meckel, rezecția incidentală poate fi


considerată adecvată la:
A. Pacienții cu abdomene ostile
B. Pacienții adulți cu comorbidități
C. Pacienții vârstinic
D. La pacineții care prezintă acuze obstructive
E. Pacienții care urmează să efectueze radioterapie
R: A, E (2, p. 210)

44. În malrotația intestinală, este adevărat:


A. Rotația inițială de 90 ° în sensul acelor de ceasornic a
mezenterului, plătește cecul în retroperitoneu
B. Rotația inițială de 90 ° în sens contrar acelor de ceasornic a
mezenteronului, plasează duodenul în retroperitoneu
C. Rotația ulterioară de 180 ° în sens contrar acelor de ceasornic a
mezenterului, plasează cecul în cadranul inferior drept
D. Rotația ulterioară de 180º în sensul acelor de casornic a
mezenterului, plasează cecul în cadranul inferior drept

168
E. Rotațiile incomplete sau absente se traduc printr-o varietate de
anomalii anatomice
R: B, C, E (2, p. 210)

45. În apendicita acută, durerea inițială poate fi asociată cu:


A. Constipație
B. Anorexie
C. Fatigabilitate
D. Greață
E. Vărsături
R: B, D, E (2, p. 211)

46. Este fals cu privire la intestinul scurt:


A. Ca răspuns la pierderea în lungime, intestinul uman răspunde
prin dilatare
B. Efectul rezecției nu depinde de gradul de absorbție al
intestinului rămas
C. În unele cazuri, aportul crescut poate fi suficient pentru
menținerea statusului nutrițional
D. Pierderea ileonului proximal duce la o pierdere permanentă a
absorbției vitaminelor D
E. Ca răspuns la pierderea în lungime, intestinul uman răspunde
printr-o creștere a înălțimii vilozităților intestinale
R: B, D (2, p. 211)

47. În apendicită, trebuie să excludem prin diagnostic diferențial


patologiile care sunt gestionate non-operator:
A. Infarctul mezenteric
B. Pancreatita cronică
C. Boala inflamatorie pelvină
D. Durerea ovulatorie
E. Durerea menstruală
R: C, D, E (2, p. 212)

48. În apendicita acută, descoperirile scanării CT sunt, cu excepția:


A. Evidențierea de niveluri hidroaerice
B. Umplerea apendicelui cu substanță de contrast
C. Prezența lichidului liber

169
D. Congestia apendicelui
E. Identificare fecaliților
R: A, B, D (2, p. 212)

49. Despre apendicita acută sunt adevărate sunt afirmații:


A. La gravide, apendicele este adesea deplasat într-o poziție mai
cranială
B. Progresia spre perforație și peritonită este asociată cu o rată a
decesului de doar 1% la gravide
C. Leucopenia din sarcină îngreunează mult diagnostic
D. Progresia spre perforație și peritonită este asociată cu o rată a
decesului fetal de 35% sau mai mult
E. Leucocitoza din sarcină poate îngreuna și mai mult diagnosticul
R : A, D, E (2, p. 212)

50. Tumorile carcinoide ale apendicelui:


A. Reprezintă aproximativ jumătate din carcinoidele gastro-
intestinale
B. Majoritatea sunt maligne
C. Leziunile sub 2 cm sunt de obicei tratate prin apendicectomie
simplă
D. Creșterea dimensiunilor carcinoidelor nu se corelează cu riscul
de malignizare
E. În cazul leziunilor cu diametrul peste 2 cm este recomandat
hemicolectomie dreaptă
R : A, C, E (2, p. 213-214)

170
CAPITOLUL 26
COLON, RECT, ANUS

1. Ramuri ale arterei mezenterice inferioare asigura


vascularizatia:
A. Colon descendent
B. Treimea proximala a colonului transvers
C. Colon ascendent
D. Colon sigmoid
E. Intestin subtire
R: A, D (2, p. 217)

2. Vascularizatia rectului este asigurata de:


A. Artera rectala superioara
B. Artera hemoroidala inferioara
C. Artera rectala mijlocie
D. Artera rusinoasa interna
E. Artera iliaca interna
R: A, C (2, p. 218)

3. Functiile principale ale colonului:


A. Defecatie
B. Rezervor pentru fecale
C. Absorbtia apei din scaunul lichid
D. Fermentatie
E. Absorbtia electrolitilor din scaunul lichid
R: B, C, D, E (2, p. 219)

4. Urmatoarele investigatii despre cea mai precisa metoda de


diagnostic in patologia tractului digestiv distal este adevarata:
A. Anoscopie
B. Rectoscopie
C. Colonoscopie
D. Sigmoidoscopoie
E. Ecografie
R: A, B, D (2, p. 220)

171
5. Indicatiile ileostomiei sau colostomiei pentru devierea
exteriorizarii fiziologice a continutului digestive sunt:
A. Pentru cicatrizarea unai anastomoza proximale
B. Lipsa unui segment intestinal necesar pentru efectuarea unei
anastomose
C. Conditiile locale nu permit anastomoza
D. Capetele intestinale nu permit efectuarea unei anastomose
E. Excizia abdomino-perineala de rect
R: B, C, D, E (2, p. 221)

6. Manifestari clinice in diverticulita necomplicata sunt:


A. Febra
B. Durere in cadranul inferior drept cu instalare subacuta
C. Diaree
D. Constipatie
E. RectoragiI
R: A, C, D (2, p. 223)

7. Indicatiile tratamentului chirurgical in boala diverticulara:


A. Durerere in flancul drept
B. Perforatie liberă
C. Obstructie
D. Hematemeza
E. Fistule
R: B, C, E (2, p. 224)

8. Simptomatologia in colita ulcerative este reprezentata de:


A. Constipatie
B. Greata si varsaturi
C. Diaree sanguinolenta
D. Tenesme rectale
E. Senzatie imperioasa de defecatie
R: C,D,E (2, p. 226)

9. In Boala Crohn aspectul microscopic se caracterizaza prin:


A. Inflamatia submucoasei
B. Granuloame
C. Rar fibroza

172
D. Inflamatie transmurala
E. Abcese criptice
R: B, D (2, p. 228)

10. In screeningul cancerului colorectal persoanele cu risc mediu ar


trebui sa opteze pentru:
A. RMN la fiecare 3 ani
B. Colonoscopoie la fiecare 5 ani
C. Sigmoidoscopie flexibila la fiecare 5 ani
D. Colangiografie CT la fiecare 5 ani
E. Colonoscopie la fiecare 10 ani
R: C, D, E (2, p. 232)

11. Cancerele rectale se pot manifesta prin urmatoarele simptome:


A. Meteorism abdominal
B. Sangerare rectala
C. Alternanta diaree-constipatie
D. Scaune creionate
E. Dificultate la defecatie
R: B, C (2, p. 233)

12. In cadrul sistemului de stadializare TNM, tumora primara (T)


reprezinta:
A. T3 tumora invadeaza submucoasa
B. T1 carcinom in situ
C. T2 tumora invadeaza musculara proprie
D. T4 tumora invadeaza indirect alte organe
E. T3 tumora invadeaza prin musculara proprie in subseroasa
R: C, E (2, p. 234)

13. Ganglionii limfatici regionali (N) reprezinta:


A. N0 ganglionii limfatici regionali nu pot fi evaluati
B. N1 metastaze in 1-3 ganglioni pericolici sau perirectali
C. N2 metastaze in >= 4 ganglioni pericolici sau perirectali
D. N3 metastaze in oricare dintre ganglionii situati de-a lungul
trunchiurilor vasculare
E. N0 fara metastaze in ganglionii regionali
R: B, C, D, E (2, p. 234)

173
14. In cancerul colorectal colonoscopia postoperatorie se efectueaza:
A. La 3 ani
B. La 1 an
C. La 4 ani
D. La 7 ani
E. La fiecare 5 ani
R: B, C, E (2, p. 235)

15. Simptomatologia prolapsului rectal include:


A. Durerea rectala
B. Eliminare de mucus sau anus umed
C. Diaree
D. Constipatie
E. Sangerare usoara
R: A, B, E (2, p. 236)

16. Localizarea pozitiilor constante a hemoroizilor:


A. Medial stanga
B. Anterior dreapta
C. Posterior stanga
D. Lateral stanga
E. Posterior dreapta
R: B, D, E (2, p. 236)

17. Clasificarea hemoroizilor interni:


A. Grad I nu prolabeaza
B. Grad II prolabeaza in timpul defectiei si nu revin la normal
C. Grad III prolabeaza in timpul defecatiei si revin la normal
D. Grad IV nu sunt reductibili
E. Grad II prolabeaza in timul defecatiei si necesita reducere manuala
R: A, D (2, p. 236)

18. Hemoroidectomia este indicate in cazul:


A. Hemoroizilor de grad II
B. Hemoroizii micsti de gradul III cu o componenta externa
voluminoasa
C. Hemoroizilor de grad IV
D. Puseu hemoroidal cu gangrene
E. Hemoroizilor de grad I
R: B, C, D (2, p. 236)

174
19. Abcesul anorectal se caracterizeaza prin:
A. Durere continua
B. Evidentierea unei tumefactii calde la nivel perianal
C. Nedureros la palpare
D. Zona eritematoasa
E. Febra si frison
R: A, B, D, E (2, p. 237)

20. Procedurile pentru drenajul complex al abcesului este indicat in:


A. Abces perianal
B. Abces in potcoava
C. Abces inschiorectal
D. Abces intersfincterian
E. Abcesul profund al spatiului perineal posterior
R: B, D, E (2, p. 237)

21. Principalele caracteristici in stabilirea conduitei chirurgicale


adecvate a fistulei perianale:
A. Traiectul
B. Profunzimea
C. Gradul de afectare musculara
D. Localizarea exacta
E. Prezenta abceselui perianal
R: A, B, C, D (2, p. 238)

22. Factorii care duc la esecul tratamentului cu lambouri de


avansare endoanale sunt:
A. Antecedente de chimioterapie
B. Boala Chron
C. Fistula recto-vaginala
D. Neoplazii
E. Antecedente de radioterapie
R: B, C, D, E (2, p. 238)

23. Masuri conservatoare de tratament in cazul fisurilor anale:


A. Consum de fibre adecvat
B. Aport hidric
C. Administrarea de antibiotice

175
D. Bai calde de sezut
E. Aplicarea locala deanestezice
R:A, B, D, E (2, p. 238)

24. Simptomatologia infectiilor bacteriene anorectale sunt:


A. Febra si frison
B. Rectoragii
C. Durere
D. Tenesme rectale
E. Scurgeri muco-purulente
R: B, C, D, E (2, p. 239)

25. Simptomele sistemice din stadiului secundar al sifilisului


anorectal este:
A. Cefalee
B. Dureri in gat
C. Atralgii
D. Rectoragii
E. Tenesme rectale
R: A, B, C (2, p. 239)

26. Tratamentul de prima intentie in cancerul cu celule scuamoase


la nivelul canalului anal este:
A. Chirurgical
B. Medicamentos
C. Chimioterapie
D. Radioterapie
E. Imunoterapie
R: C, D (2, p. 240)

27. Simptomele sistemice al virusului herpetic rectal sunt:


A. Tenesme rectale
B. Prurit
C. Scurgeri rectale
D. Febra
E. Mialgii
R: D, E (2, p. 240)

176
28. Simptomele locale al virusului herpetic rectal sunt:
A. Durere
B. Scurgeri rectale si prurit
C. Tenesme
D. Cefalee
E. Febra
R: A, B, C (2, p. 239)

29. Simptomele care insotesc leziunea parianala sau anala de tipul


condiloame acuminate sunt:
A. Tenesme rectale
B. Febra si frison
C. Prurit
D. Sangerari
E. Scurgeri
R: C, D, E (2, p. 239)

30. Diagnosticul leziunilor primare sau secundare in sifilisul


anorectal se realizeaza prin:
A. Vizualizarea spirochetelor cu ajutorul microscopiei in camp
intunecat
B. Teste serologice
C. Imunohistochimie
D. Examinare histologica
E. Prelevare de frotiu Papanicolau
R: A, B (2, p. 239)

31. Nu exista apendici epiploici la nivelul:


A. Cecului
B. Colonului
C. Rectului
D. Apendicelui cecal
E. Anusului
R: A, C, D, E (2, p. 216)

32. Defecatia implica o interactiune complexa intre:


A. Colon proximal
B. Colon distal
C. Rect

177
D. Anus
E. Muschii planseului pelvin
R: B, C, E (2, p. 219)

33. Care portiune a colonului regleaza cea mai mare parte a


absorbtiei de sodiu siapa:
A. Ascendent
B. Descendent
C. Rect
D. Cec
E. Transvers
R: A, D, E (2, p. 219)

34. Fiziologia continentei fecale este rezultatul interactiunilor


complexe intre urmatoarel functiile ale rectului, muschilor
pelvini si ale sfincterului anal:
A. Sensitive
B. Motorii
C. Senzoriale
D. Involuntare
E. Voluntare
R: B, C, D, E (2, p. 219)

35. Cea mai importanta evaluare initiala a colonului,rectului si


anusului sunt:
A. Examen rectal
B. Anamneza completă
C. CT abdominal
D. Ecografie abdominala
E. Examen clinic abdominal
R: A, B, E (2, p. 220)

36. Sigmoidoscopia flexibilia poate identifica:


A. Stenoza de la nivelul colonului transver
B. Tumori de unghi hepatic colonic
C. Localizarea hemoragiei de la nivelul rectului
D. Localizarea obstructiei de la nivelul colonului sigmoid
E. Prezenta polipilor
R: C, D, E (2, p. 220)

178
37. Indicatiile CT abdomino-pelvin include evaluare:
A. Bolii metastatice
B. Gravitatea colitelor
C. Determinarea nivelului obstructiei intestinale
D. Fistulelor
E. Abceselor
R: A, B, C (2, p. 220-221)

38. Manifestarile clinic in diverticulita acuta complicate include:


A. Abcese
B. Fistule
C. Rectoragii
D. Diaree
E. Perfoatia libera in peritoneu
R: A, B, E (2, p. 223)

39. Fistulele se manifesta prin:


A. Constipatie
B. Diaree
C. Rectoragii
D. Eliminarea de materii fecale prin vagin
E. Infectii urinare repetate
R: B, D, E (2, p. 223)

40. Abcesele de mari dimensiuni (>5cm) sunt tratate prin:


A. Antibioterapie IV
B. Drenaj percutan
C. Radiologie interventionala
D. Rezectie sigmoidiana cu anastomoza
E. Chirurgical
R: A, B, C (2, p. 224)

41. Polipii rectali de tip adenoma vilos prezita:


A. Potential malign scazut
B. Localzare intestin subtire
C. Tratament prin excizie endoscopica
D. Frecvent 25% dintre adulti>50ani vor avea un adenoma
E. Potential de malignizare ridicat
R: C, D, E (2, p. 231)

179
42. Semnele tipice pentru pseudo-ocluzia acuta a colonului sunt:
A. Dilatarea colonului ascendent si transvers oe radiografie
B. Distensia abdominala
C. Diaree
D. Constipatie
E. Greata si varsaturi
R: A, B, E (2, p. 230)

43. Ocluzia colonului este determinate cel mai frecvent de:


A. Cancerul de colon
B. Cancerul de rect
C. Boala Chron
D. Volvulus de colon
E. Colita ulcerative
R: A, B, D (2, p. 228)

44. Semnele si simptomele ocluziei colonului sunt:


A. Constipatie severa
B. Diaree
C. Absenta tranzitului pentru gaze
D. Debut lent
E. Ocazional greata si varsaturi
R: A, C, E (2, p. 228)

45. Simptomele comune ale volvulusului de colon sigmoid si cec


include:
A. Crampe abdominale
B. Constipatie severa
C. Greata si varsaturi
D. Rectoragii
E. Diaree
R: A, B, C (2, p. 229)

46. Tratamentul de prima linie pentru pseudo-ocluzia acuta a


colonului include:
A. Laxativele orale
B. Stimulante ale peristalticii intestinale
C. Montarea sondei nazo-gastrice

180
D. Repausul digestive
E. Corectarea dezechilibrelor electrolitice
R: C, D, E (2, p. 230)

47. Hamartoamele prezinta:


A. Potential malign scazut
B. Regreseaza spontan
C. Se autoamputeaza
D. Potential malign crescut
E. Monitorizarea fiind o masura eficienta si sigura
R: A, B, C, E (2, p. 231)

48. Simptomele in cancerul de colon drept sunt:


A. Obstructii
B. Rectoragii
C. Scadere ponderala
D. Tumorala palpabila la examenul abdominal
E. Diaree
R: C, D (2, p. 233)

49. Cancerele rectale T3-T4 stadializate prin IRM sunt tratate


initial prin:
A. Interventie chirurgicala
B. Chimioterapie
C. Radioterapie
D. Imunoterapie
E. Tratament medicamentos
R:B,C (2, p. 234)

50. Recomandarile in hemoroizii asimptomatici:


A. Agentii de formare a bolului fecal
B. Hemoroidectomia
C. Scleroterapia
D. Evitarea constipatiei
E. Limitarea timpului petrecut pe toaleta
R: A, D, E (2, p. 236)

181
CAPITOLUL 27
CĂILE BILIARE

1. Care dintre următorii factori predispun la formarea calculilor


biliari?
A. Insuficienţa cardiacă
B. Multiparitatea
C. Doze crescute de contraceptive orale pe bază de estrogen
D. Diabetul zaharat tip 2
E. Nutriţia parenterală totală prelungită
R: B, C, E (2, p. 242)

2. Ȋn colica biliară durerea se caracterizează prin:


A. Descrisă ca o durere surdă, continuă, supărătoare, cu durată de 1-4 ore
B. Localizată în flancul drept şi uneori în epigastru
C. Este de tip visceral
D. Se însoţeşte de absenţa tranzitului intestinal
E. Poate determina pacientul să se prezinte de urgenţă la spital
R: A, C, E (2, p. 244)

3. Diagnosticul diferenţial în colecistita acută include următoarele


afecţiuni:
A. Pancreatita acută
B. Ulcerul perforat
C. Apendicita acută
D. Diverticulita acută
E. Rectocolita ulcero-hemoragică
R: A, B, C (2, p.246)

4. Ȋn colecistita acută sunt specifice următoarele semne ecografice:


A. Vezicula biliară fără distensie
B. Ȋngroşarea peretelui colecistului
C. Prezenţa semnului Murphy ecografic
D. Ecografic nu se vizualizează calculi
E. Lichid pericolecistic
R: B, C, E (2, p. 246)

182
5. Diagnosticul diferenţial în colecistita cronică include:
A. Refluxul gastro-esofagian
B. Ulcerul peptic
C. Infarct miocardic acut
D. Pelvi-peritonita
E. Obstrucţie ureterală
R: A, B, E (2, p. 249)

6. Triada Charcot este reprezentată de:


A. Dureri abdominale în hipocondrul drept
B. Icter
C. Febra şi frison
D. Hipotensiune arterială
E. Stare de confuzie psihică
R: A, B, C (2, p. 249)

7. Pentada Reynold cuprinde:


A. Dureri abdominale în flancul drept şi epigastru
B. Icter
C. Febra şi frison
D. Hipertensiune arterială
E. Stare de confuzie psihică
R: B, C, E (2, p. 249)

8. Cele mai frecvent întâlnite sindroame clinice consecutive litiazei


veziculare şi complicaţiile litiazei veziculare sunt:
A. Colecistita cronică
B. Neoplasmul veziculei biliare
C. Colica biliară
D. Colangita acută
E. Pancreatita biliară
R: C, D, E (2, p. 252)

9. Dintre factorii de risc pentru cancerul veziculei biliare se enumeră:


A. Litiaza biliară
B. Colangita sclerozantă primară
C. Colecistita cronică

D. Steatoza hepatică

183
E. Hepatita cronică
R: A, B (2, p. 251)

10. Avantajele abordului laparoscopic sunt:


A. Creşterea numărului zilelor de spitalizare
B. Reducerea durerilor postoperatorii
C. Reducerea complicaţiilor de plagă
D. Recuperarea rapidă după intervenţie
E. Reducerea complicaţiilor pulmonare
R: B, C, D, E (2, p. 254)

184
CAPITOLUL 28
PANCREASUL

1. Despre vascularizaţia arterială a pancreasului, sunt false


următoarele afirmaţii:
A. Trunchiul celiac irigă în totalitate mezenteronul
B. Există numeroase variaţii ale anatomiei clasice a trunchiului celiac
C. Artera pancreaticoduodenală inferioară are o ramură anterioară şi una
posterioară
D. Artera mezenterică superioară vascularizează intestinul subţire, colonul
drept şi colonul stâng
E. Artera pancreatică dorsală interconectează trunchiul celiac şi artera
mezenterică superioară în mai multe locuri
R: A, D (pag. 258-260)

2. În etiologia pancreatitei acute sunt implicaţi următorii factori:


A. Disfuncţia sfincterului Oddi
B. Medicamente: furosemidul, estrogenii, corticosteroizii, azatioprina
C. Consumul de alcool
D. Virusul Epstein-Barr
E. Sepsisul
R: A, B, C, D (pag. 262, 263)

3. Explorările imagistice indicate într-o pancreatită acută sunt:


A. Ecografia abdominală
B. Radiografia toracică
C. Scintigrafia
D. Radiografia abdominală simplă
E. Computer tomografia
R: A, B, D, E (pag. 263)

4. Complicaţii sistemice ale unei pancreatite severe sunt:


A. Insuficienţa pulmonară, definită ca PaO2/FiO2 <100
B. Coagularea intravasculară diseminată
C. Insuficienţa renală (creatinina > 177 μmol/L
D. Abcesul pancreatic
E. Hemoragie digestivă
R: B, C (pag. 265)
185
5. Despre pancreatita cronică sunt adevărate următoarele afirmaţii:
A. Este cauzată cel mai adesea de fibroza chistică
B. Cel mai frecvent se manifestă clinic prin durere cu localizare epigastrică
şi iradiere posterioară
C. Consumul de alimente ameliorează durerea
D. Explorările de laborator pot să fie normale
E. CT arată, de obicei, hipertrofie şi calcificări
R: B, D (pag. 267)

6. Următoarele tumori pancreatice sunt premaligne:


A. Pseudochistul
B. Adenocarcinomul
C. Tumora Hamoudi
D. Limfomul
E. Adenomul mucinos
R: C, E (pag. 268)

7. Insulinoamele pancreatice:
A. Sunt benigne de cele mai multe ori
B. Sunt maligne în 20-30 % din cazuri
C. Sunt, în general, solitare
D. Cele mai multe sunt localizate la nivelul corpului şi cozii pancreasului
E. Se manifestă prin hiperglicemie
R: A, C, D (pag. 271-272)

8. Tumorile Zollinger-Ellison:
A. Constituie majoritatea tumorilor pancreatice neuroendocrine
B. Mai mult de 50 % dintre ele sunt localizate la nivelul pancreasului
C. Sunt multicentrice în 50 % din cazuri
D. Se manifestă prin triada: diaree apoasă, hipopotasemie, hipoclorhidrie
E. La pacienţii cu sindrom MEN 1, sunt cele mai frecvente tumori
funcţionale ale celulelor insulare
R: C, E (pag. 272-273)

9. Următoarele asocieri între tipul de leziune chistică pancreatică şi analiza


conţinutului chistic sunt corecte:

186
A. Chistadenom seros- nivel crescut de ACE
B. Chistadenom mucinos- nivel scăzut de ACE
C. Chistadenom mucinos- nivel nivel crescut de ACE şi 19-9
D. Chistadenocarcinom mucinos- nivel scăzut de ACE şi 19-9
E. IPMN- nivel crescut de ACE
R: C, E (pag. 274)

10. Despre neoplasmele chistice mucinoase ale pancreasului sunt false


următoarele afirmaţii:
A. Apar aproape întotdeauna la femei de vârstă mijlocie
B. Sunt localizate cel mai frecvent la nivelul capului pancreasului
C. Examenul histologic relevă stromă ovariană
D. Se tratează conservator
E. Sunt frecvent asimptomatice
R: B, D (pag. 274)

187
CAPITOLUL 29
FICATUL ȘI SPLINA

1. Anatomia ficatului:
A. Ficatul este cea mai mare glandă unică din organism;
B. Este localizat subdiafragmatic, cu cea mai mare parte din masa
hepatică situată la stânga liniei mediene;
C. Cranio-caudal, se întinde de la spațiul intercostal IV-V pe partea
dreaptă, coborând până sub nivelul rebordului costal;
D. Ligamentul falciform, coronar și triunghiular atașează ficatul de
diafragm și peretele abdominal anterior;
E. Venele suprahepatice drenează sângele de la nivelul parenchimului
hepatic direct în vena cavă superioară.
R: A, C, D (2, p. 317)

2. Fiziologia ficatului:
A. unitatea funcțională a ficatului este lobulul hepatic;
B. la periferia fiecărui lobul există câte o arteră hepatică și un ram
vascular portal și central se află câte o venă hepatică de drenaj;
C. funcțiile hepatice majore: sinteza proteică, metabolismul energetic,
procese de detoxifiere, producția de bilă și functia imună
reticuloendotelială;
D. hepatocitul se divide continuu si poate regenera întreaga masă
celulară a ficatului la fiecare 100 zile;
E. hepatocitul monitorizează și reglează nivelele plasmatice ale lipidelor.
R: A, B, C (2, p. 317)

3. Funcțiile ficatului:
A. sinteza proteică;
B. sinteza lipidelor;
C. metabolismul energetic;
D. funcția imunologică;
E. producția de bilă.
R: A, C, D, E (2, p. 317)

4. Leziunile hepatice traumatice:


188
A. se pot împărți în traumatisme penetrante sau contuzii;
B. modalitatea imagistică de elecție utilizată în diagnostic este
radiografia abdominală;
C. instabilitatea hemodinamică este contraindicată pentru intervenția
chirurgicală;
D. ficatul dispune de o protecție substanțială oferită de cutia toracică și
musculatura abdominală;
E. ultrasonografia și computer tomografia sunt investigațiile imagistice
de elecție în diagnostic.
R: A, D, E (2, p. 317-318)

5. Vascularizația ficatului:
A. Este triplă;
B. Este realizată prin intermediul arterei hepatice și a venei porte;
C. Un sfert din fluxul sanguin hepatic este asigurat de vena portă;
D. Sistemul arterial hepatic poate avea o distribuție variabilă;
E. Ficatul prezintă 2 vene suprahepatice.
R: B, D (2, p. 317)

6. Vena portă:
A. Un sfert din fluxul sanguin hepatic este asigurat de vena portă;
B. Reprezintă confluența sistemului de drenaj venos al intestinului și al
colonului;
C. Reprezintă confluența venei mezenterice superioare și a venei
splenice;
D. La periferia fiecărui lobul hepatic există câte o arteră hepatică și un
ram vascular portal.
E. Vena portă se împarte în vena suprahepatică dreaptă, mijlocie și
stângă.
R: C, D (2, p. 317)

7. Fiziologia ficatului:
A. Unitatea funcțională a ficatului este lobul hepatic;
B. Funcțiile hepatice: sinteza proteică, metabolismul energetic, procese
de detoxifiere, producția de bilă;
C. Hepatocitul reprezintă celula principală a ficatului;
D. Hepatocitul nu se divide;
E. Funcția de sinteză se realizează prin celulele Kupffer.
R: B, C (2, p. 317)

189
8. Tumorile hepatice benigne:
A. Chistele hepatice benigne sunt rare în populație;
B. Sunt identificate tot mai des prin creșterea disponibilității tehnicilor
imagistice moderne;
C. Necesită obligatoriu tratament chirurgical;
D. Biopsia percutanată este necesară în cazul unor tumori hepatice
nerezecabile de natură necunoscută;
E. Anumite leziuni benigne hepatice trebuie rezecate chiar și la pacienții
asimptomatici.
R: B, D, E (2, p. 319)

9. Biopsia percutanată hepatică:


A. Este necesară în cazul unor tumori hepatice nerezecabile de natură
necunoscută;
B. Poate aduce informații eronate și este asociată cu anumite riscuri;
C. Riscurile pot fi: hemoragia și însămânțarea tumorală;
D. Biopsia percutanată nu prezintă riscuri procedurale;
E. Biopsia percutanată este obligatorie în stadiile incipiente la un pacient
asimptomatic;
R: A, B, C, (2, p. 319)

10. Tumorile hepatice:


A. Hemangiomul este o tumoră malignă;
B. Hiperplazia nodulară focală este o leziune benignă;
C. Adenomul hepatic este o tumoră malignă;
D. Carcinomul hepatocelular este o tumoră malignă;
E. Tumora Klatskin este o leziune benignă a arborelui biliar.
R: B, D (2, p. 319-322)

11. Hemangiomul hepatic:


A. Este cea mai frecventă tumoră hepatică malignă;
B. Este o leziune dobândită;
C. Este un hamarton embriologic;
D. Evaluarea microscopică arată spații endoteliale vasculare separate de
septuri fibroase;
E. Nu cresc în dimensiuni în timpul vieții.
R: C, D (2, p. 319)

12. Hemangiomul hepatic:

190
A. Este cea mai frecventă tumoră hepatică benignă;
B. Este, probabil, o leziune congenitală;
C. Aceste leziuni nu cresc în dimensiuni în timpul vieții;
D. Au o sensibiliate hormonală, fiind de 5 ori mai frecvente la femei;
E. Nu cresc în dimensiuni în timpul sarcinii.
R: A, B, D (2, p. 319)

13. Hemangiomul hepatic:


A. Este cea mai frecventă tumoră malignă hepatică;
B. Este o leziune congenitală;
C. Nu necesită terapie specifică, funcția hepatică păstrându-se normală;
D. Majoritatea bolnavilor sunt simptomatici;
E. Sunt prezente frecvent hemoragii spontane sau rupturi.
R: B, C (2, p. 319)

14. Hemangiomul hepatic:


A. Este cea mai frecventă tumoră hepatică malignă;
B. Sunt descoperite deseori incidental și nu necesită terapie specifică;
C. Majoritatea bolnavilor sunt asimptomatici;
D. Ocazional, pacienții cu hemangioame voluminoase pot prezenta
durere abdominală;
E. Tratamentul chirurgical este întotdeauna necesar.
R: B, C, D (2, p. 319)

15. Hiperplazia nodulară focală hepatică:


A. Este o leziune malignă;
B. Funcția hepatică este normală;
C. Sunt asociate frcvent cu ruptură sau hemoragie;
D. Provocarea constă în stabilirea diagnosticului diferențial cu adenomul
hepatic și cu carcinomul hepatocelular;
E. Imagistica prin rezonantă magnetică este cea mai eficientă în
descrierea leziunii.
R: B, D, E (2, p. 320)

16. Hiperplazia nodulară focală hepatică:


A. Este o leziune benignă;
B. Este sinonimă cu adenomul hepatic;
C. Ar trebui tratată conservator când diagnosticul a fost stabilit imagistic;

D. Ultrasonografia abdominală simplă este investigația de elecție;

191
E. Funcția hepatică este normală.
R: A, C, E (2, p. 320)

17. Adenomul hepatic:


A. Este o tumoră malignă;
B. Majoritatea pacientelor au istoric de tratament de lungă durată cu
contraceptive orale;
C. Este o leziune solitară, încapsulată;
D. Este format dintr-o plajă de hepatocite fără triadă portală sau duct
biliar;
E. Întreruperea contraceptivelor orale poate duce la o regresie
dimensională.
R: B, D, E (2, p. 320-321)

18. Adenomul hepatic:


A. Este o tumoră malignă;
B. Este o leziune solitară, neîncapsulată;
C. Elementul patognomonic în stabilirea rezultatului este aspectul
heterogen al acestuia la examenul IRM;
D. Are indicație chirugicală absolută;
E. Complicații: poate crește dimensional sau poate sângera;
R: B, C, E (2, p. 320-321)

19. Adenomul hepatic:


A. Este o tumoră malignă;
B. Pot fi clasificate în adenoame cu risc scăzut sau înalt;
C. Întreruperea contraceptivelor orale reprezintă prima măsură
terapeutică atunci când leziunea este asimptomatică;
D. Adenoamele cu risc înalt de a dezvolta complicații au indicație clară
chirurgicală;
E. Este o leziune solitară, neîncapsulată.
R: B, C, D, E (2, p. 320-321)

20. Carcinomul hepatocelular:


A. Este o tumoră benignă;
B. Apare în mod uzual la pacienții cu boală hepatică cronică;
C. Factori de risc: pacienții infectați cu virus hepatic B sau C, ciroză
hepatică, consumul de alcool și steatoza hepatică;
D. Orice formațiune hepatică mai mare de 5 cm la un pacient cu ciroză ar
trebui investigată pentru excluderea unui posibil hepatocarcinom;

192
E. Toți pacienții cirotici trebuie sa fie evaluați imagistic bianual prin
ultrasonografie abdominală datorită riscului crescut de CHC.
R: B, C, E (2, p. 321)

21. Carcinomul hepatocelular:


A. Apare în mod uzual la pacienții cu boală hepatică cronică;
B. Factori de risc: pacienții infectați cu virus hepatic B sau C, ciroză
hepatică, consumul de alcool și steatoza hepatică;
C. Orice formațiune hepatică mai mare de 5 cm la un pacient cu ciroză ar
trebui investigată pentru excluderea unui posibil hepatocarcinom;
D. Este suspectat la orice pacient cu ciroză cunoscută și decompensare
clinică bruscă;
E. Trebuie inclus în diagnosticul diferential al oricărei tumori hepatice
lichide.
R: A, B, D (2, p. 321)

22. Carcinomul hepatocelular:


A. Orice formațiune hepatică mai mare de 1 cm la un pacient cu ciroză ar
trebui investigată pentru excluerea unui posibil hepatocarcinom;
B. Pentru diagnostic, pot fi utilizate: ultrasonografia, CT sau IRM;
C. Biopsia este recomandată întodeauna;
D. Numărul de noduli, dimensiunea și prezența invaziei vasculare sunt
factori importanți în determinarea stadiului bolii și al prognosticului
oncologic;
E. Tratamentul depinde de dimensiunea și de extensia bolii hepatice
cronice.
R: A, B, D, E (2, p. 321)

23. Opțiuni de tratament ale carcinomul hepatocelular:


A. Rezecția;
B. Transplant;
C. Întreruperea contraceptivelor orale reprezintă prima măsură
terapeutică atunci când leziunea este asimptomatică;
D. Chemoembolizare transarterială;
E. Radioterapia.
R: A, B, D (2, p. 322)

24. Opțiuni de tratament ale carcinomul hepatocelular:

193
A. Pentru pacienții cu ciroză hepatică, chimioterapia este tratamentul
standard;
B. Transplantul hepatic poate fi o opțiune pentru pacienții cu CHC cu
rezerve hepatice restrânse;
C. Strategiile curative sunt: chemoembolizarea și radioembolizarea
tumorală;
D. Criteriile Milano ( tumoră unică mai mică de 5 cm sau până la 3
tumori mai mici de 3 cm) au fost dezvoltate pentru a putea ghida
indicațiile transplantului hepatic;
E. Nu există risc de recidivă după rezecția tumorală la pacienții cu CHC
și ciroză hepatică.
R: B, D (2, p. 322)

25. Colangiocarcinomul:
A. Este o tumoră benignă;
B. Localizarea determină natura simptomelor;
C. Tumora Klatskin este o leziune periferică;
D. Diagnosticul se pune pe baza colangio-IRM și ERCP;
E. Icterul asociat cu dilatarea ductelor biliare intrahepatice și cu vezicula
biliară de dimenisuni mici, este înalt sugestiv pentru colangiocarcinom
periferic.
R: B, D (2, p. 322)

26. Metastazele hepatice:


A. În majoritatea cazurilor sunt de origine gastrointestinale;
B. În cancerul colorectal, tratamentul leziunilor hepatice nu
îmbunătățește supraviețuirea;
C. În cazul metastazelor de cancer colorectal, cel mai mult beneficiază de
rezecția hepatică pacienții cu leziuni de dimenisuni mici, nivel scăzut
al antigenului carcinoembrionar în sânge și o perioadă mai mare de 1
an de lipsă de progresie a bolii;
D. Tratamentele ablative nu se folosesc pentru tratamentul metastazelor;
E. Tratamentele chimioterapice moderne oferă rezultate bune pe termen
scurt si lung.
R: A, C, E (2, p. 322)

27. Chisturile hepatice:


A. Pot fi congenitale sau dobândite;
B. Chisturile congenitale pot fi neoplazice sau infecțioase;

194
C. Majoritatea sunt de mici dimensiuni, asimptomatice, cu conținut lichid
seros și nu comunică cu arborele biliar;
D. Tratamentul standard pentru chistul simptomatic este decorticarea
chirurgicală;
E. Transplantul hepatic poate fi o opțiune terapeutică în boala hepatică
polichistică.
R: A, D, E (2, p. 323)

28. Abcesele hepatice:


A. Simptome: durere în hipocondrul drept, febră si leucocitoză;
B. Fosfataza alcalină este scazută la majoritatea bolnavilor;
C. Ultrasonografia evidențiază o masă hipoecogenă, deseori asociată cu
un perete ecogen;
D. Aspirația percutanată și plasarea unui tub de dren ajută atât la
diagnostic, cât și în remiterea procesului infecțios;
E. Stentarea biliară este obligatorie.
R: A, C, D (2, p. 323)

29. Chistul hidatic:


A. Omul este gazda finală;
B. Reprezintă infecția bacteriană a omului cu Echinococcus granulosus;
C. Omul se infectează intrând în contact cu fecalele câinelui;
D. În 2/3 dintre cazuri apare la nivelul parenchimului hepatic;
E. Poate crește până la 10-20 cm.
R: C, D, E (2, p. 324)

30. Complicațiile chistul hidatic pot fi:


A. Infecția secundară piogenică,
B. Fistula biliară,
C. Extensia la nivelul toracelui,
D. Eroziunea și ruptura la nivelul arborelui bronșic sau în cavitatea
peritoneală,
E. Malignizarea.
R: A, B, C, D (2, p. 324)

31. Chistul hidatic:


A. Confirmarea prin teste serologice;
B. Dacă boala este suspicionată, aspirația percutanată sau biopsia în scop
diagnostic, sunt obligatorii;
C. Terapia antiparazitară cu albendazol este indicată în chisturile mari;

195
D. Tratamentul chirurgical conservativ se realizează prin îndepărtarea
conținutului chistului și inactivarea protoscolecșilor;
E. Timpul cel mai important al intervenției chirurgicale este prevenirea
însămânțării intraoperatorii.
R: A, D, E (2, p. 324)

32. Anatomia splinei:


A. Este situată în cadranul superior stâng al abdomenului;
B. Este în raport intim, prin intermediul unor ligamente suspensoare, cu
capul pancreasului;
C. Vascularizația arterială este dublă;
D. Artera splenică este o ramură a arterei mezenterice inferioare;
E. Vena splenică se unește cu vena mezenterică inferioară și formează
vena porta;
R: A, C (2, p. 333)

33. Funcțiile splinei:


A. Are un rol cheie în menținerea integrității sistemului imun;
B. Hematopoieza;
C. Filtrarea sângelui;
D. Procese de detoxifiere;
E. Sinteza proteică.
R: A, B, C (2, p. 333)

34. Fiziologia splinei:


A. Prezintă circulația deschisă și circulația închisă;
B. Rolul splinei în hematopoieza extramedulară este definită la adult;
C. Are rol de a filtra sângele, acumulând eritrocite anormale sau
îmbătrânite, granulocite și plachetele;
D. O jumătate din trombocite este depozitată în splină;
E. Splenectomia este urmată de trombocitoză pasageră.
R: A, C, E (2, p. 333-335)

35. Considerații diagnostice generale în anomaliile splinei:


A. Medicul trebuie să întrebe pacientul despre istoricul sângerărilor, dacă
există;
B. Istoricul personal și familial de afecțiuni hematologice, cum ar fi
limfomul sau leucemia, nu au relevanță;
C. O splină normală este ușor de palpat;

196
D. Palparea se realizează bimanual, cu pacientul în decubit dorsal;
E. Discomfortul la palpare alertează clinicianul către o afecțiune
hematologică.
R: A, D (2, p. 335)

36. Explorări imagistice în anomaliile splinei:


A. Aspectul unei spline normale este mereu evidențiat prin radiografia
abdominală pe gol;
B. Ecografia abdominală este o investigație extrem de utilă;
C. CT-ul cu substanță de contrast intravenoasă si per os este cea mai utilă
tehnică radioimagistică pentru determinarea dimensiunilor și a
leziunilor splinei;
D. Scintigrafia ofera informații despre dimensiunea și funcția splinei;
E. AngioCT-ul este util în stabilirea diagnosticului de tromboză de venă
splenică.
R: B, C, D, E (2, p. 336-337)

37. Explorări imagistice în anomaliile splinei:


A. Radiografia abdominală pe gol este cea mai bună metodă de a evalua
rapid pacienții cu traumatisme abdominale pentru prezența sângelui în
cavitatea peritoneală;
B. Ecografia abdominală oferă informații despre dimensiunea și funcția
splinei;
C. CT-ul cu substantă de contrast intravenoasă si per os este cea mai utilă
tehnică radioimagistică pentru determinarea dimensiunilor și a
leziunilor splinei;
D. Scintigrafia este de ajutor pentru a diferentia o splină accesorie de o
tumoră a cozii pancreasului;
E. angioCT-ul se indică pentru evaluarea lacerațiilor splenice la pacienții
cu traumatism abdominal, instabili hemo-dinamic.
R: C, D (2, p. 336-337)

38. Indicațiile splenectomiei:


A. Afecțiunile hematologice reprezintă cea mai frecventă cauză pentru
indicația de splenectomie;
B. Trauma;
C. Tumori primare și metastatice;
D. Anevrism de arteră splenică;
E. Abces splenic;
R: B, C, D, E (2, p. 337)

197
39. Traumatismele splenice:
A. Trauma reprezintă cea mai frecventă cauză pentru indicația de
splenectomie;
B. Splina este cel mai frecvent organ afectat în urma contuziilor
abdominale;
C. La pacientul instabil hemo-dinamic cu lacerație splenică, splenectomia de
urgență este tratamentul standard;
D. AngioCT-ul se indică pentru evaluarea lacerațiilor splenice la pacienții cu
traumatism abdominal, instabili hemo-dinamic;
E. Ecografia abdominală este una dintre cele mai bune metode de a evalua
rapid pacienții cu traumatism abdominal pentru prezența sângelui în
cavitatea peritoneală.
R: A, B, C, E (2, p. 336-338)

40. Anomaliile funcției splenice:


A. Mărimei splinei este corelată cu funcția sa hematologică;
B. Splenectomia este cea mai comună cauză pentru asplenie;
C. Sunt clasificate ca anatomice si funcționale;
D. Splenectomia este recomandată în sferocitoza ereditară;
E. Splenomegalia este sinonimă cu hipersplenismul.
R: B, C, D (2, p. 338)

41. Clasificarea splenomegaliei:


A. Mică în congestia pasivă cronică;
B. Mare în infecție acută și subacută;
C. Moderată în ciroza hepatică;
D. Mare în leucemia mielocitară cronică;
E. Mare în tromboza de venă splenică.
R: A, C, D, E (2, p. 338)

42. Patologii asociate cu hipersplenism:


A. Sarcoidoza;
B. Tromboza de arteră splenică;
C. Hipertensiunea portală;
D. Boli infiltrative ale splinei;
E. Sindromul Felty.
R: A, C, D, E (2, p. 340)

198
43. În hipersplenism, splenectomia este indicată:
A. La valoarea trombocitelor mai mică de 50.000;
B. Întotdeauna;
C. Prezența hemoragiei;
D. În cazul neutropeniei mai mică de 2.000, cu sau fără infecții intercurente;
E. Anemie care să necesite transfuzii.
R: A, C, D, E (2, p. 341)

44. În hipersplenismul secundar, splenectomia:


A. Necesită o monitorizare atentă a numărului trombocitelor în perioada
postoperatorie;
B. Necesită inițierea unei terapii antiagregante plachetare, atunci când este
necesar;
C. Rezolvă complet citopenia;
D. Postoperator, numărul de trombocite scade dramatic;
E. Se poate complica cu tromboze și trombembolism.
R: A, B, E (2, p. 341)

45. Complicațiile splenectomiei:


A. Creșterea numărului de leucocite cu 50% peste limita superioară a
normalului;
B. Trombocitoza revine de obicei la normal în aproximativ 2 săptămâni;
C. Apariția infecției severe post splenectomie;
D. Riscul cel mai ridicat pentru sindromul OPSI este în cazul pacienților cu
splenectomie efectuată pentru patologie traumatică;
E. Rata mortalității și a morbidității după splenectomie este mare.
R: A, B, C (2, p. 341-342)

46. Consecințele imunologice ale splenectomiei:


A. Riscul dezvoltării infecției severe post splenectomie nu depinde de
vârsta pacientului și motivul splenectomiei;
B. Riscul cel mai ridicat este intâlnit la pacienții cu patologii hematologice;
C. Sindromul OPSI apare în perioada imediat postoperatorie;
D. Streptococcus pneumoniae este cel mai frecvent agent incriminat;
E. Vaccinarea antipneumococică nu este obligatorie dupa splenectomiile
partiale posttraumatice.
R: B, D (2, p. 342)

199
47. Complicațiile postsplenectomie:
A. Trombocitoza;
B. Scăderea numărului leucocitelor;
C. Infecțiile severe postsplenectomie;
D. Leziunile pancreatice;
E. Abcesele subfrenice.
R: A, C, D, E (2, p. 342)

48. Trombocitopenia:
A. Este definită ca prezența unui număr crescut al trombocitelor;
B. Splenectomia este cel mai frecvent efectuată în cazul trombocitopeniilor
idiopatice, mediate imunologic, a căror cauză este necunoscută;
C. Pacienții prezintă multiple peteșii sau purpură;
D. Anticorpii antiplachete sanguine sunt identificați la 85% dintre pacienții
cu trombocitopenie imună;
E. Corticosteroizii reprezintă prima linie în tratamentul trombocitopeniei
imune.
R: B, C, D, E (2, p. 339-340)

49. Complicațiile după splenectomie:


A. Modificări hematologice;
B. Consecințe imunologice;
C. Vaccinarea antipneumococică nu este obligatorie după splenectomiile
parțiale posttraumatice;
D. Scăderea numărului leucocitelor;
E. Pancreatita acută.
R: A, B, E (2, p. 341-342)

50. Complicațiile după splenectomie:


A. Sindromul OPSI apare în perioada imediat postoperatorie;
B. Vaccinarea antipneumococică este obligatorie după splenectomiile
totale și parțiale posttraumatice;
C. Pacienții cu traumatism splenic manageriați non-operator trebuie
vaccinați;
D. Pacienții cunoscuți cu afecțiuni mieloproliferative au indicație de
tratament postoperator cu medicație antiagregantă plachetară;
E. Semnele clinice ale abceselor subfrenice: febră, durere în hipocondrul
stang.
R: B, C, D, E (2, p. 341-342)

200
CAPITOLUL 30
BOLILE SISTEMULUI VASCULAR

1. Predilecţia către boala aorto-iliacă este favorizată de:


A. Diabet zaharat
B. Dislipidemie
C. Fumat
D. Varice hidrostatice
E. Insuficienţa cardiacă
R: B, C (2, p. 520)

2. Sindromul Leriche este definit prin:


A. Impotenţă
B. Venectazii
C. Atrofia muşchilor fesieri
D. Claudicaţia extremitaţilor inferioare
E. Insuficienţa venoasă cronică
R: A, C, D (2, p. 521)

3. Examinarea fizică a pacientului cu boala arterială periferică trebuie


să includă:
A. Palparea prezenţei şi caracterului pulsului arterial inghinal (artera
femurală)
B. Ȋn fosa poplitee (artera poplitee)
C. La nivelul feţei dorsale a piciorului (artera pedioasă)
D. Posterior, la nivelul maleolei mediale (artera tibială posterioară)
E. Medial, traiectul venei safene
R: A, B, C, D (2, p. 522)

4. Principalele complicaţii ale angiografiei includ:


A. Sângerare sau tromboză la locul puncţiei
B. Formarea unui pseudoanevrism la locul puncţiei
C. Crearea iatrogenă a unei fistule arterio-venoase

D. Purpura cutanată
201
E. Flegmon la locul puncţiei
R: A, B, C (2, p. 523)

5. Triada factorilor de risc pentru tromboza venoasă profundă


cuprinde:
A. Prezenţa venectaziilor
B. Staza
C. Leziuni endoteliale venoase
D. Edemele
E. Stările de hipercoagulabilitate
R: B, C, E (2, p. 535)

6. American College of Chest Physicians recomandă tromboliză:


A. La pacienţii cu TVP iliofemurală
B. Simptome timp de> 14 zile
C. Stare clinică bună
D. Speranţă de viaţă < I an
E. Risc scăzut de sângerare
R: A, C, E (2, p. 538)

7. Indicaţiile clare pentru terapia trombolitică includ:


A. Tromboza venei femurale
B. Tromboza venei subclavii
C. Tromboza venei renale
D. Ocluzia cronică a venei mezenterice superioare prin mecanism
trombotic
E. Ocluzia acută a venei mezenterice inferioare
R: B, C (2, p. 538)

8. Contraindicaţiile pentru terapia anticoagulantă includ:


A. Diatezele hemoragice
B. Ulcerele tegumentare
C. Accidentul vascular cerebral vechi
D. Intervenţiile chirurgicale recente
E. Tulburările hematologice
R: A, D, E (2, p. 538)

9. Din tabloul clinic al emboliei pulmonare fac parte:

202
A. Insuficienţa cardiacă stângă
B. Hemoptizie
C. Dispnee
D. Tahipnee
E. Tahicardie
R: B, C, D, E (2, p. 538)

10. Simptomele cauzate de venele varicoase superficiale sunt


reprezentate de:
A. Senzaţie de greutate şi oboseală după ortostatism prelungit
B. Crampe diurne
C. Ocazional edem perimaleolar
D. Tromboflebită profundă
E. Sângerare de la nivelul venelor superficiale
R: A, C, E (2, p. 539)

11. Cauzele hipertensiunii venoase sunt:


A. Incompetenţa valvulară profundă
B. Obstrucţia venoasă
C. Refluxul din venele perforante
D. Ateroscleroza
E. Embolii
R: A, B, C (2, p. 540)

12. Manifestările clinice ale insuficienţei venoase sunt :


A. Edemul acut al membrelor inferioare
B. Edemul cronic al membrelor inferioare
C. Hiperpigmentarea
D.Ulceraţiile venoase
E. Purpura
R: B, C, D (2, p. 540)

13. Examinarea clinică evidenţiază în insuficienţa venoasă cronică:


A. Tegument portocaliu-maroniu la nivelul gleznei cu depozite de
hemosiderină
B. Edem al membrului inferior
C. Telangiectazii

D. Ulceraţii

203
E. Cianoză
R: A, B, C, D (2, p. 540)

14. Boala ocluzivă tibială este tipică la :


A. Pacienţii cu intervenţii chirurgicale recente
B. Pacienţii cu insuficienţă venoasă cronică
C. Pacienţii cu diabet
D. Pacientii cu insuficienţă renală în stadiu final
E. Vârstă înaintată
R: C, D, E (2, p. 520)

15. Alte cauze mai puţin frecvente ale bolii arteriale periferice sunt :
A. Boala Buerger (trombarterită obliterantă)
B. Fibroza pulmonară
C. Endofibroza iliacă
D. Compresia arterelor prin benzi musculare fibroase
E. Insuficienţa renală cronică
R: A, C, D (2, p. 520)

16. Ischemia de la nivelul extremităţilor inferioare poate provoca


progresiv :
A. Claudicaţie permanentă
B. Dureri de repaus de tip ischemic
C. Ulceraţii ale pielii
D. Gangrenă
E. Claudicaţie intermitentă
R: B, C, D, E (2, p. 521)

17. Terapia medicală în boala arterială periferică include:


A. Modificarea dietei
B. Renunţarea la fumat
C. Terapie antiplachetară
D. Purtare ciorapi compresivi
E. Purtare încălţăminte ortopedică
R: A, B, C, E (2, p. 523)

18. Complicaţiile imediate ale interventiei de bypass arterial includ :


A. Nu prezintă complicaţii imediate
B. Sângerarea postoperatorie de la nivelul anastomozelor
C. Tromboza grefonului

204
D. Infecţia plăgilor şi limforagia
E. Necroza
R: B, C, D (2, p. 526)

19. În plus faţă de tromboza venoasă profundă, diagnosticul diferenţial


al edemului acut şi al durerii de membre inferioare include
A. Traumatismele
B. Ruptura musculară
C. Infecţia
D. Limfangita
E. Chistul Baker rupt
R: A, C, D, E (2, p. 537)

20. Ȋn tromboza venoasă profundă idiopatică este necesar:


A. Măsurarea nivelurilor de proteină C
B. Măsurarea nivelurilor de proteină S
C. Măsurarea nivelurilor de antitrombină III
D. Măsurarea nivelurilor factor VIII
E. Măsurarea nivelurilor anticorpilor anticardiolipină
R: A, B, C, E (2, p. 537)

205
CAPITOLUL 31
OTORINOLARINGOLOGIA: BOLILE CAPULUI ȘI
GÂTULUI

1. Faringele este situat posterior de:


A. Esofag;
B. Cavitatea nazala;
C. Laringe;
D. Trahee;
E. Cavitatea bucala.
R: B, C, E (2, p. 566)

2. Etapele deglutiei sunt, cu exceptia:


A. Preparare;
B. Voluntara bucală;
C. Faringiana;
D. Larigiana;
E. Esofagiana.
R: A, B, C, E (2, p. 567)

3. Ingestia acuta de substante caustice poate provoca la nivelul


mucoaselor faringo-esofagiene următoarele tipuri de leziuni:
A. Echimoze;
B. Necroză de lichefiere;
C. Necroză de coagulare;
D. Plăgi secționate;
E. Stenoze.
R: B, C (2, p. 568)

4. Stomatita cu Candida Albicans apare cel mai frecvent după:


A. Probiotice;
B. Antibiotice;
C. Criocauterizare;
D. Corticoterapie;
E. Citostatice.
R: B, D (2, p. 568)

206
5. Abcesul periamigdalian este caracterizat de:
A. Edem laterocervical;
B. Fistula faringiană;
C. Asimetria amigdalelor;
D. Tumefacția palatului moale;
E. Deviația uvulei.
R: C, D, E (2, p. 568)

6. Cele mai frecvente tulburări sistemice în apneea obstructivă în


somn sunt:
A. Insuficiența cardiacă dreaptă;
B. Hipertensiunea pulmonară;
C. Retard de dezvoltare la copii;
D. Tulburări de limbaj;
E. Tulburări de deglutitie.
R: A, B, C (2, p. 569)

7. Carcinoamele cavității orale apar de obicei:


A. Pe vălul palatin;
B. Pe planșeul bucal;
C. Intraamigdalian;
D. Pe porțiunea mobilă a limbii;
E. Faringian.
R: B, D (2, p. 569)

8. Laringele are conexiuni musculare cu:


A. Limba;
B. Mandibula;
C. Endobaza;
D. Sternul;
E. Clavicula.
R: A, B, D, E (2, p. 570)

9. Cartilajul aritenoid este:


A. Nepereche;
B. Cu forma piramidala;
C. Cu trei fete;
D. Articulat cu hioidul;
E. Articulat cu cricoidul.
R: B, C, E (2, p. 570)

207
10. Muschii extrinseci laringieni:
A. Ascensionează laringele;
B. Mobilizează corzile vocale;
C. Tensionează laringele;
D. Rotesc aritenoidul către lateral;
E. Determină adducția corzilor vocale.
R: A, C (2, p. 570)

11. Prin laringoscopie indirecta se vizualizeaza:


A. Baza de limba;
B. Epiglota;
C. Sinusurile paranazale;
D. Laringele;
E. Corzile vocale.
R: A, B, D, E (2, p. 571)

12. Stridorul expirator este determinat de:


A. Stenoza corzilor vocale;
B. Stenoza supraglotică;
C. Laringomalacie;
D. Crupul recurent;
E. Stenozele subglotice.
R: C, D, E (2, p. 572)

13. Laringotraheobronșita acută:


A. Afectează de regula adulții;
B. Are etiologie virală;
C. Afectează regiunea subglotică;
D. Afectează epiglota;
E. Afectează de regulă copiii cu vârsta până în 2 ani.
R: B, C, E (2, p. 574)

14. Paralizia unilaterală de coardă vocală este cauzată de:


A. Afectarea nervului facial;
B. Infarctul cerebral la nivel bulbar;
C. O tumoră localizată la baza craniului;
D. O tumoră esofagiană;
E. O tumoră tiroidiană;
R: B, C, D, E (2, p. 575)

208
15. Granulomul laringian:
A. Este în treimea anterioară a corzii vocale;
B. Se mai numește nodul vocal;
C. Este la unirea a două treimi anterioare cu treimea posterioară a
corzii vocale lângă apofiza vocală;
D. Poate fi determinat de refluxul gastroesofagian;
E. Frecvent este secundar intubației traheale.
R: D, E (2, p. 575)

16. Cancerul laringian:


A. Este mai frecvent în decadele 6 și 7 de viață;
B. Este mai frecvent la bărbați decât la femei;
C. Fumatul și alcoolul sunt factori de risc importanți;
D. Nu necesită biopsie;
E. Nu determina adenopatie.
R: A, B, C (2, p. 576)

17. La nivelul triunghiului cervical anterior se întâlnesc următoarele


structuri anatomice:
A. Muschiul sternocleidomastoidian;
B. Osul hioid;
C. Faringele;
D. Traheea;
E. Glanda tiroidă.
R: A, B, D, E (2, p. 576)

18. Glanda parotidă se află:


A. Anterior de pavilionul auricular;
B. Inferior de mandibulă;
C. Între pântecele anterior și posterior al mușchiului digastric;
D. Deasupra mușchiului maseter;
E. Sub vârful apofizei mastoide.
R: A, D, E (2, p. 577)

19. Cea mai frecventa tumora maligna la nivelul glandelor salivare


principale este:
A. Carcinomul mucoepidermoid;
B. Carcinomul chistic adenoid;
C. Carcinomul cu celule acinare;

209
D. Adenocarcinomul;
E. Carcinomul cu celule scuamoase.
R: A, B, C (2, p. 579)

20. Chistul de tract tireoglos:


A. Aproape totdeauna este pe linia mediană;
B. Se află lateral de-a lungul mușchiului sternocleidomastoidian;
C. este tapetat de epiteliu scuamos sau columnar;
D. Are potențial de degenerare malignă a tesutului tiroidian
disgenetic;
E. Este mai frecvent la copii sau adulții tineri.
R: A, D, E (2, p. 580-581)

21. Privind nervul facial, putem afirma urmatoarele:


A. În cazul în care există o disfuncție a nervului facial, apare
paralizia mușchilor pe care îi inervează;
B. Nu inervează mușchii mimicii;
C. În funcție de localizarea leziunii la nivelul nervului, pot fi
prezente și alte manifestări clinice precum pierderea secreției
lacrimale;
D. Dacă lezarea nervului facial este severă, nu necesită regenerare;
E. Inervează mușchii mimicii.
R: A, C, E (2, p. 551)

22. Urmatoarele afirmații privind examinarea urechii sunt


adevărate:
A. Conductul auditiv extern este examinat cu ajutorul otoscopului;
B. Se tracționeaza pavilionul auricular spre superior si anterior
pentru o examinare facila;
C. Examinarea conductului auditiv extern nu va evalua prezenta
obstructiilor sau a unor patologii tegumentare;
D. Evaluarea membranei timpanice trebuie facuta tinand cont de
orice pierdere a transparentei;
E. Prin transparenta membranei timpanice se pot vizualiza portiuni
ale lantului osicular.
R: A, D, E (2, p. 551)

210
23. Urmatoarele afirmatii sunt adevarate:
A. Perforațiile membranei timpanice >1 mm pot fi ușor de
vizualizat;
B. Perforațiile membranei timpanice nu pot fi eronat diagnosticate
ca retracții ale timpanului;
C. Otoscopul pneumatic poate fi folosit pentru evaluarea mișcării
membranei timpanice fără aplicare de presiune asupra acesteia;
D. Pacientul va fi evaluat dacă aude vocea conversațională, de
regulă la aproximativ 55 dB, cu frecvențe între 500-3000 Hz;
E. În cazul afectării vestibulare, nu trebuie efectuată și investigare
neurologică suplimentară.
R: A, D (2, p. 551)

24. Etiologii ale hipoacuziei neurosenzoriale sunt următoarele:


A. Meningită;
B. Ototoxicitate;
C. Otoscleroză;
D. Traumatism os temporal;
E. Neurinom de acustic.
R: A, B, D, E (2, p. 553)

25. Hipoacuzia de conducere:


A. Presupune transmiterea inadecvată a sunetului către urechea
internă;
B. Apare ca urmare a unor patologii la nivelul conductului auditiv
extern;
C. Este caracterizată de o accentuare a amplificării sunetului;
D. Nu apare ca urmare a unei patologii la nivelul membranei
timpanice;
E. Este frecvent descrisă de pacienți ca o “ureche înfundată”.
R: A, B, E (2, p. 553)

26. Privind testele cu diapazonul putem afirma următoarele:


A. Ajuta la diferențierea celor două tipuri de hipoacuzie;
B. Testul Weber este efectuat cu ajutorul unui diapazon plasat pe
frunte;
C. La testul Rinne pacientul este rugat să evalueze dacă aude mai
bine cu diapazonul poziționat pe frunte sau la 2 cm de conductul
auditiv extern;

211
D. Aceste teste au valoare diagnostic mai mare dacă hipoacuzia este
mixtă și unilaterală;
E. La testul Weber dacă pacientul aude sunetul într-o singură
ureche, poate exista o hipoacuzie de conducere în urechea
contralaterală.
R: A, B (2, p. 553)

27. Audiometria tonala:


A. Clasifica hipoacuzia în funcție de frecvențe și măsoara gradul
acesteia în funcție de componentele neurosenzoriale si de
conducere;
B. Evalueaza recunoasterea vorbirii sau discriminarea;
C. Este efectuata prin producerea unei serii de tonuri de diferite
intensitati prin intermediul unor casti pe cel putin 8 frecvente;
D. Pragul reprezinta stimulul cu cea mai mare intensitate care poate
fi perceput de catre pacient;
E. Nivelul de auz al pacientilor este consemnat ca dB peste sau sub
nivelul zero-db pe fiecare dintre frecventele utilizate.
R: A, E (2, p. 553)

28. Dintre următoarele afirmații putem afirma ca sunt adevărate:


A. Un prag auditiv între 0-20 dB constituie hipoacuzie ușoară;
B. Un prag auditiv între 40-70 dB constituie hipoacuzie severă;
C. Hipoacuzia neurosenzorială este determinată de pragurile
conducerii osoase generate de căști prin mastoidă, fără a trece
prin urechea externă și medie;
D. Hipoacuzia de conducere este determinată de pragurile
conducerii aeriene;
E. În cazul conducerii aeriene, sunetul nu trebuie să treacă prin
urechea externă și medie pentru a putea fi auzit.
R: C, D (2, p. 553)

29. Privind otalgia, putem afirma urmatoarele:


A. Prezintă cauze în general ușor de identificat;
B. Durerea din alte surse nu poate fi reflectată la ureche și
interpretată ca patologie otică;
C. Poate fi utilă investigarea factorilor agravanți;
D. Otalgia agravată de masticație poate indica o afectare a
structurilor miofasciale ale articulației temporo-mandibulare;

212
E. Nu necesită examinarea întregului tract aerodigestiv superior în
cazul otalgiei reflectate.
R: A, C, D (2, p. 555)

30. Privind tinitusul, putem afirma urmatoarele:


A. Tinitusul bilateral nu poate fi cauzat de toxicitate sistemică;
B. Există tratamente care și-au dovedit eficiența în tratamentul
tinitusului;
C. Reprezintă țiuitul sau percepția altui sunet în urechi;
D. Tinitusul pulsatil este de obicei de natură vasculară;
E. Tinitusul unilateral implică patologie nervoasă sau la nivelul
cohleei.
R: C, D, E (2, p. 555)

31. Privind otoreea, putem afirma urmatoarele:


A. Scurgerile de la nivelul urechii nu pot avea consistențe diferite;
B. Dacă în urma examinării sunt evidențiate structuri otice normale,
scurgerile subțiri, apoase de culoare galben-transparent sunt
considerate a fi patologice;
C. În urma unui traumatism, când există scurgeri apoase, de culoare
galben transparent, poate fi suspicionată o fistula de lichid
cefalo-rahidian;
D. O secreție mucoidă și purulentă de la nivelul urechii semnifică
infecție;
E. Curățarea atentă a urechii nu poate evidenția o perforație veche a
membranei timpanice.
R: C, D (2, p. 555)

32. Privind fractura de bază de craniu, putem afirma următoarele:


A. Fractura de bază de craniu poate interesa osul temporal cu
lezarea sistemului vestibular, auditiv sau ambele;
B. Fracturile longitudinale sunt mai rar întâlnite;
C. Semnele clinice nu pot include hemotimpanul și otoreea;
D. În funcție de localizarea traiectului de fractură în raport cu
porțiunea pietroasă a osului temporal, sunt descrise două tipuri
de fracturi: longitudinale și transversale;
E. Dezintegrarea lanțului osicular nu necesită intervenție
chirurgicală.
R: A, D (2, p. 556)

213
33. Urmatoarele afirmatii privind otita externa nu sunt adevarate:
A. tegumentul pavilionului auricular si al conductului auditiv extern
reprezinta locul de electie de aparitie;
B. otita externa poate fi localizata sau difuza;
C. tratamentul implica antibiotice;
D. la examenul obiectiv nu putem observa modificari precum un
edem fluctuant;
E. otita externa localizata nu poate fi cauzata de trauma.
R: D, E (2, p. 556-557)

34. Urmatoarele afirmații privind otita externă sunt adevărate:


A. Reprezinta un proces difuz care afectează întregul conduct
auditiv extern;
B. Cel mai frecvent agent patogen implicat este Klebsiella;
C. Cel mai frecvent agent patogen implicat este Pseudomonas;
D. Mai este denumită “urechea înotatorului”;
E. La examenul clinic se observă doar o congestie a tegumentului.
R: A, C, D (2, p. 557)

35. Despre otita externă necrotizantă, putem afirma următoarele:


A. Reprezintă osteomielita osului temporal și a bazei craniului;
B. Pacienții cu risc sunt cei cu otită externă care suferă de diabet
zaharat sau sunt imuncompromiși;
C. Mortalitatea nu este ridicată;
D. A mai fost denumită și “otita externă malignă”;
E. Răspunde de obicei la tratament topic.
R: A, B, D (2, p. 557)

36. Despre otita medie acută, putem afirma următoarele:


A. Este cea mai frecventă infecție pentru care se prescriu antibiotice
copiilor;
B. Prezintă un debut acut, cu prezența lichidului și a semnelor de
inflamație la nivelul urechii medii;
C. Până la vârsta de 4 ani, 45% dintre copii suferă de cel putin 3
episoade de otită medie acută;
D. Frecvent pot fi prezente semne de iritație meningeala;
E. Cei mai frecvenți agenți patogeni implicați sunt Streptococcus
pneumonia, Moraxella catarrhalis și Haemophilus influenzae.
R: A, B, E (2, p. 558)

214
37. Tratamentul otitei medii acute presupune:
A. Inițial se recomandă doar monitorizare, cu reevaluare la 48-72
ore;
B. Doze mari de amoxicilină (90mg/kgc/zi) sunt recomandate de
prima linie;
C. În cazul pacienților alergici la peniciline, pot fi utilizate
azitromicina, claritromicina sau eritromicina;
D. După inițierea tratamentului, durerea cedează în câteva ore, iar
funcția trompei lui Eustachio revine la normal în câteva zile;
E. Daca infecția persistă după 3 zile de tratament antibiotic, agentul
antibacterian nu trebuie schimbat.
R: A, B, C (2, p. 558)

38. Otita medie acută netratată poate progresa către:


A. Mastoidită;
B. Meningită;
C. Sinuzită;
D. Laringită;
E. Labirintită.
R: A, B, E (2, p. 558)

39. Otita medie seroasă:


A. Este a doua cea mai frecventă patologie a urechii medii;
B. Constă în prezența de lichid în cavitatea urechii medii;
C. Prezintă semne și simptome de infecție;
D. Timpanometria este utilă la diagnosticul acestei patologii;
E. Nu poate apărea spontan din cauza disfuncției tubare.
R: A, B, D (2, p. 558)

40. Privind colesteatomul, urmatoarele afirmatii sunt adevarate:


A. Reprezintă un chist al urechii medii acoperit de tegument;
B. Are origine posibilă la nivelul unei membrane timpanice
patologice;
C. Nu poate determina paralizia de nerv facial;
D. Tratamentul este medicamentos;
E. Enzimele osteolitice din baza membranei colesteatomului produc
osteonecroză.
R: A, B, E (2, p. 559)

215
41. Privind otoscleroza, urmatoarele afirmatii sunt false:
A. Reprezintă o anomalie spontană a urechii externe și, în cazurile
foarte avansate, a urechii medii;
B. Afectează cel mai frecvent adulții tineri;
C. Determină hipoacuzie de transmisie;
D. Nu are predispoziție genetică;
E. Afectează doar femeile.
R: A, D, E (2, p. 559)

42. Despre hipoacuzia neurosenzoriala putem afirma:


A. Presbiacuzia rezultă în urma degenerării cohleare;
B. Poate avea printre cauze expunerea cronică la zgomot sau o
medicație ototoxică;
C. Eritromicina și aspirina nu pot determina o hipoacuzie
neurosenzorială reversibilă;
D. Cisplatina și vancomicina nu pot determina disfuncție cohleară;
E. Protezele auditive sunt de obicei utile.
R: A, B, E (2, p. 559)

43. Neuronita vestibulara:


A. Implică inflamația nervului vestibular;
B. Apare cel mai frecvent ca urmare a unei infecții virale;
C. Prezintă debut acut cu vertij sever ce durează de la câteva zile
până la câteva săptămâni;
D. Tratamentul nu cuprinde reabilitare vestibulară;
E. Antibioticele pot fi utile în tratament.
R: A, B, C (2, p. 559)

44. Vertijul pozitional paroxistic benign:


A. Este un vertij acut și sever, cu durată scurtă;
B. Este un vertij cronic, cu durată lungă;
C. Poate dura de la cateva secunde până la câteva minute;
D. Poate dura de la câteva ore până la câteva zile;
E. Este diagnosticat de elecție prin manevra Dix-Hallpike.
R: A, C, E (2, p. 559)

45. Boala Meniere:


A. Se mai numește și hidrops endolimfatic;
B. Determină hipoacuzie, vertij, tinnitus și senzație de plenitudine
auriculară;

216
C. De obicei este bilaterală și este asociată cu ruperea membranei
Reissner;
D. Tratamentul medical cuprinde diuretice și regim hiposodat;
E. Nu prezintă opțiuni de tratament chirurgical.
R: A, B, D (2, p. 560)

46. Privind nervul facial, putem afirma urmatoarele:


A. Poate fi afectat atât intracranian, cât și extracranian;
B. Foarte rar, paralizia de nerv facial este idiopatică;
C. Prezintă un prognostic rezervat pentru recuperarea funcției
nervoase, dacă apare în contextul unei tumori maligne;
D. Prognosticul recuperarii functiei nervoase este bun dacă leziunea
este completă;
E. Paralizia facială se instaleaza în câteva ore.
R: A, C, E (2, p. 560)

47. Privind paralizia de nerv facial, putem afirma următoarele:


A. Majoritatea cazurilor nu se vindecă spontan;
B. Paralizia facială persistentă necesita investigatii imagistice
pentru a exclude o formatiune tumorală;
C. Tratamentul este în general medical, orientat către cauza de bază;
D. Paralizia facială în contextul unei otite medii acute se tratează cu
antibioterapie topică și sistemică;
E. Infecția HIV, granulomatoza Wegener și boala Lyme se pot
asocia cu paralizie facială.
R: B, C, D, E (2, p. 560)

48. Urmatoarele afirmatii nu sunt adevarate:


A. Keratoza actinică este cel mai frecvent observată la nivelul
tegumentului pavilionului auricular;
B. Tumorile maligne specifice conductului auditiv extern sunt
osteoamele și exostozele;
C. Exostozele conductului auditiv extern au forma unor noduli
multipli,de obicei bilaterali;
D. Osteoamele conductului auditiv extern sunt bilaterale și multiple;
E. În situatia în care osteoamele sau exostozele de conduct auditiv
extern produc obstrucție importantă, se recomanda excizie
chirurgicală.
R:A, B, C, E (2, p. 560)

217
49. Privind neoplaziile urechii medii si ale mastoidei, putem afirma
urmatoarele:
A. Sunt extrem de rare;
B. Glomus tympanicum este un paragangliom vascular al urechii
medii;
C. Glomus tympanicum nu este histologic similar cu tumorile
glomice ale corpului carotidian;
D. Pacientii cu glomus tympanicum acuză tinitus pulsatil unilateral
și hipoacuzie;
E. Excizia chirurgicala este de elecție în cazul unui glomus
tympanicum.
R: A, B, D, E (2, p. 560)

50. Privind neoplaziile urechii interne, putem afirma urmatoarele:


A. Determină hipoacuzie neurosenzorială;
B. Cea mai frecventă neoplazie a urechii interne este schwanomul
vestibular;
C. Schwanomul vestibular prezintă ca semn patognomonic vertijul
sever;
D. Ecografia stabilește diagnosticul;
E. Tratamentul este reprezentat de excizia chirurgicala.
R: A, B, E (2, p. 560)

218
CAPITOLUL 32
CHIRURGIE ORTOPEDICĂ: BOLILE SISTEMULUI
MUSCULOSCHELETAL

1. Fractura următoarele afirmații sunt adevărate cu excepția:


A. Reprezintă o întrerupere a continuității structurale a unui os;
B. Reprezintă o pierdere a continuității structurale a unui os;
C. Reprezintă o întrerupere, dar nu o pierdere a continuității
structurale a unui os;
D. Reprezintă o pierdere a contactului dintre suprafețele articulare;
E. Reprezintă o pierdere incompletă a contactului dintre suprafețele
articulare.
R: C, D, E (2, p. 585)

2. Următoarele afirmații sunt adevărate:


A. În subluxație superfețele articulare sunt parțial lipsite de contact
și nu poate fi un fenomen tranzitoriu;
B. Subluxația articulară poate fi un fenomen tranzitoriu;
C. Luxația este caracterizată prin pierderea totală a contactului
dintre suprafețele articulare;
D. O manevră de reducere este adesea necesară pentru a restabili
alinierea articulară;
E. Subluxația nu se poate reduce spontan.
R: B, C, D (2, p. 585)

3. Tipul unei fracturi poate fi:


A. Deschisă;
B. Închisă;
C. De stres;
D. Pe os patologic;
E. Pe os patologic cu excepția osteoporozei.
R: A, B, C, D (2, p. 586)

4. Traiectul fracturii – următoarele afirmații sunt adevărate cu


excepția:
219
A. Fractura transversală apare printr-un traumatism de energie
înaltă;
B. O fractura transversă este rezultatul unui traumatism direct
asupra unui os lung;
C. Fracturile spiroide sunt rezultatul unui traumatism de rotație;
D. Fracturile oblice rezultă printr-un traumatism de torsiune;
E. O fractură transversă a ulnei apare atunci când antebrațului nu i
se aplică o lovitură directă.
R: A, E (2, p. 586)

5. O fractură impactată:
A. Este frecvent întâlnită în osul diafizar;
B. Este frecvent întâlnită în fracturile de col femural;
C. Este frecvent produsă la nivelul platoului tibial;
D. Este frecvent întâlnită la nivelul extremității distale a radiusului;
E. Este rezultatul unui traumatism de energie înaltă.
R: B, C, D (2, p. 587)

6. Deplasarea fragmentelor osoase – următoarele afirmații sunt


adevărate:
A. Este măsurată în plan medio-lateral;
B. Este măsurată în plan sagital;
C. Poate fi măsurată în plan antero-posterior;
D. Poate fi măsurată în plan cranio-coronal;
E. Poate fi măsurată în plan coronal.
R: A, B, C, E (2, p. 587)

7. Următoarele afirmații sunt adevărate cu excepția:


A. Genu varum este asociat cu picioarele în X;
B. Genu valgum este asociat cu picioarele in paranteză;
C. Dacă vârful angularii este îndreptat către linia mediană se
numește varus;
D. Dacă vârful angulăriii este îndreptat în direcția opusă față de
linia mediană se numește valgus;
E. termenii de varus-valgus sunt utilizați în descrierea facturilor.
R: A, B, C, D (2, p. 588)

8. Clasificarea Salter- Halles:


A. Tipul I – presupune o separare a epifizei de metafiză;

220
B. Tipul II – fractura trece prin placa de creștere și nu traversează
metafiză;
C. Tipul III – fractura nu se extinde de la cartilajul de creștere prin
epifiză;
D. Tipul IV – fractura se extinde din metafiză prin cartilajul de
creștere în epifiză;
E. Tipul III – nu este o fractură intra-articulară.
R: A, D (2, p. 588)

9. Evaluarea pacienților cu traumatism musculo-scheletal


presupune:
A. Inspectarea circumferențiară;
B. Integritatea vasculară;
C. Integritatea neurologică;
D. Integritatea țesuturilor moi;
E. Nu este necesară evaluarea integrității neurologice.
R: A, B, C, D (2, p. 588)

10. Evaluarea radiologică completă include:


A. Realizarea unei incidențe anteroposterioară;
B. Trebuie vizualizat locul traumei;
C. Trebuie vizualizat articulația proximală și distală față de locul
traumatizat;
D. Realizarea unei incidențe laterale;
E. Asocierile cunoscute ale leziunilor nu determină examinare
radiologică specială.
R: A, C, D (2, p. 588-590)

11. Principii de tratament în fracturi:


A. Nu toate traumatismele musculo-scheletale trebuie imobilizate în
atele;
B. Un pacient cu fractură trebuie tratat inițial ca un pacient
traumatizat;
C. Montarea corectă a unei atele nu presupune imobilizarea
articulațiilor supra- și sub- iacente fracturii;
D. Montarea unei atele previne mișcarea în focarul de fractură;

E. Repetarea examenului clinic nu trebuie efectuat după orice


reducere sau manipulare a extremității fracturate.
R: B, D (2, p. 590)

221
12. Etapele vindecării fracturii:
A. Presupune inflamație și proliferare celulară;
B. Proliferarea condroblastelor și osteoclastelor;
C. Formarea hematomului;
D. Transformarea osului fibros în os lamelar;
E. Remodelarea și resorbția calusului fără reconstruirea cavității
medulare.
R: A, C, D (2, p. 591)

13. Tracțiunea continuă:


A. Nu poate fi tegumentară;
B. Poate fi trans-osoasă;
C. Nu poate fi gravitațională;
D. Nu poate fi folosită decât la adulți;
E. Poate fi folosită la copii mici.
R: B, E (2, p. 591)

14. Complicațiile asociate imobilizării ghipsate și tracțiunii sunt


următoarele cu excepția:
A. În ghips prea strâns trebuie parțial înlăturat;
B. Tracțiunea excesivă trans-scheletică poate duce la defecte de
consolidare;
C. Tracțiunea excesivă trans-scheletică nu conduce la leziuni de
nervi periferici;
D. Un ghips adecvat aplicat poate determina o leziune ulcerativă de
presiune;
E. Redoarea articulară apare cel mai frecvent după o imobilizare de
scurtă durată.
R: A, C, D, E (2, p. 591)

15. Ortezele funcționale:


A. utilizate frecvent pentru fracturile umărului;
B. nu permit mișcări precoce ale articulației;
C. mențin reducerea fracturii printr-un efect compresiv asupra
țesuturilor moi;

D. nu grăbesc vindecarea;
E. grăbesc recuperarea.
R: A, C, E (2, p. 591)

222
16. Indicații pentru fixare internă a unei fracturi:
A. Reducerea anatomică a fracturilor extra-articulare;
B. Eșecul metodelor nonoperatorii de reducere;
C. Fracturi pe os patologic;
D. Fracturi multiple pe aceeași extremitate;
E. Fracturi la care se poate aplica tracțiunea.
R: B, C, D (2, p. 594)

17. Indicații pentru fixarea externă a unei fracturi:


A. Fracturi deschise stabile;
B. Fracturi infectate;
C. Fracturi cominutive severe la care se poate practica și fixarea
internă;
D. Fracturi pelvine instabile;
E. Fracturi care presupun pierdere osoasă.
R: B, D, E (2, p. 594)

18. Complicații locale în procesul de vindecare al fracturii:


A. Necroza avasculară;
B. Pseudoartroza;
C. Infecția;
D. Sepsisul;
E. Gangrena gazoasă.
R: A, B, C (2, p. 594)

19. Complicații locale în fracturi:


A. Pseudoartroza este caracterizată de o întârziere în consolidare;
B. Necroza avasculară apare când vascularizația osului nu este
afectată de traumatism;
C. Artroza apare direct în urma unei diformități angulare severe;
D. Întârzierea în consolidare se consideră atunci când perioada de
vindecare a fracturii este mai lungă decât cea fiziologică;
E. Pesudoartoza numită și falsa articulație.
R: D, E (2, p. 594)

20. Fractura de olecran – următoarele afirmații sunt adevărate cu


excepția:
A. Este în general rezultatul unui traumatism prin cădere pe cot;
B. Fractura este deplasată prin contracția mușchiului triceps sural;

223
C. Produce pierderea extensiei pasive a cotului;
D. Fractura implică și suprafețele articulației cotului;
E. Nu orice deplasare a fragmentelor necesită intervenție
chirurgicala pentru refacerea suprafețelor articulare.
R: B, C, E (2, p. 596)

21. Luxația umărului:


A. Este printre cele mai puțin întâlnite din luxațiile traumatice;
B. Luxația anterioară a umărului se produce prin rotația externă
forțată a brațului în adducție;
C. Risc de lezare a arterei și nervului axilar;
D. O radiografie de incidență AP surprinde deplasarea posterioară a
capului humeral;
E. Sedarea și relaxarea musculară facilitează reducerea.
R: C, E (2, p. 596)

22. Fracturile diafizei femurale – următoarele afirmații sunt


adevarate:
A. Fractura diafizei femurale nu necesită un traumatism de energie
mare;
B. Nu necesită radiografie de bazin;
C. Într-o fractură închisă se pot pierde 1-3 unități de sânge;
D. Tratamentul preferat constă în osteosinteză cu focar închis cu tijă
intramedulară;
E. Tratamentul preferat constă în osteosinteza cu focar deschis cu
placă și șuruburi.
R: C, D (2, p. 598)

23. Fracturile de coloană:


A. Rezultă la tineri ca urmare a unui traumatism de energie mică;
B. Se pot asocia cu leziuni intra-abdominale;
C. La vârstnici pot apărea ca urmare a unui traumatism minor pe un
os osteoporotic;

D. Nu pot fi asociate cu fracturi ale extremităților;


E. La vârstnici pot apărea ca urmare a unui traumatism minor pe un
os tumoral dar nu și osteoporotic.
R: B, C (2, p. 599)

224
24. Amputația traumatică și reimplantarea:
A. Copiii suportă bine regenerarea nervoasă în reimplantare;
B. Cu cât masa musculară atașată părții amputate este mai mare,cu
atât prognosticul funcțional după reimplantare este mai favorabil;
C. Nivelul cel mai favorabil de amputație pentru reimplantare la
adult este halucele;
D. Contraindicațiile reimplantării include amputația prin secționare;
E. Un segment amputat rămâne viabil pentru aproximativ șase ore
în condiții de ischemie caldă.
R: A, E (2, p. 600)

25. Sindromul de compartiment-următoarele afirmații sunt


adevărate:
A. Poate fi consecința fracturilor,a contuziilor musculare severe,a
sindromului de strivire, a leziunilor vasculare cronice;
B. Semnele clasice ale sindromului de compartiment de
compartiment produs prin ischemie tisulară sunt: durere,
parestezie, paloare, paralizie;
C. Poate aparea și în situația unui puls și aunei sensibilități normale;
D. Decompresia se face prin fasciotomie deschisă;
E. Decompresia este indicată la o presiune compartimentală mai
mare de 30-40 mmHg la un pacient inconștient.
R: B, C, D, E (2, p. 600-601)

26. Fracturile de stres:


A. Sunt leziuni clasice de suprasolicitare;
B. Au fost descrise prima dată la nivelul primului metacarp;
C. Tratamentul tradițional constă in repaus și protecție față de stres;
D. Nu sunt întâlnite la majoritatea oaselor;
E. Pot fi indentificate precoce pe baza radioizotopilor de pirofosfat
de technețiu.
R: A, C, E (2, p. 601)

27. Tendinita coafei rotatorilor-următoarele afirmații sunt


adevarate:
A. Cei patru mușchi ai coafei rotatorilor sunt – subscapularul,
suprasinos, infraspinos, rotundul mic;

225
B. Cei patru mușchi ai coafei rotatorilor sunt – subscapularul,
rotundul mare, suprascapularul, infrasinosul;
C. Aceasta este frecvent întâlnită la vârstnici;
D. Factorii ce contribuie la durerea coafei rotatorilor –
suprasolicitarea, antrenament intens, aruncarea incorectă;
E. Pot aparea modificări patologice succesive ale tendonului, de la
inflamație pâna al calcificări degenerative, subțiere și chiar
rupere.
R: A, D, E (2, p. 602)

28. Entorsa – urmatoarele afirmații sunt false, cu excepția:


A. Este o leziune capsulară;
B. Reprezintă întinderea excesivă a articulațiilor;
C. Este clasificată in funcție de 3 grade;
D. Gradul I prezintă sensibilitate ligamentară;
E. Gradul III prezintă pierderea stabilității articulare.
R: B, C, D, E (2, p. 604)

29. Ruptura tendonului Achillean:


A. Apare la sportivi de vârstă adolescentină;
B. Injecțiile locale cu corticosteroizi slăbesc țesutul tendinos și
predispun la ruperea tendonului;
C. Flexia plantară activă a gleznei este prezentă, dar slabă, deoarece
flexorul tibial posterior și flexorul lung al degetelor sunt încă
funcționale;
D. Tratamentul ortopedic presupune imobilizarea piciorului in
flexie dorsală;
E. Testul Thompson verifică dacă complexul format din
gastrocnemian și plantar este intact.
R: B, C (2, p. 608-609)

30. Miozita osificantă:


A. Depunere de os la nivelul unui mușchi ca urmare a unei
inflamații;
B. O metaplazie a celulelor muscular sau o eliberare de material
osteogen din osul subiacent determină formare de os în mușchiul
afectat;
C. Intr-o leziune mare sau ce cauzează afectare mecanică se indică
excizie chirurgicală;

226
D. O formă sistemică de miozită osificantă apare la pacienții cu
arsuri extinse;
E. Sunt prezente sensibilitate musculară superficială și pierderea
mișcării în articulația adiacentă.
R: B, C, D (2, p. 610)

31. Diformități angulare ale membrelor următoarele afirmații sunt


false cu excepția:
A. Genu varum este o diformitate angulară ce nu necesită consult
ortopedic;
B. Genu varum este o diformitate angulară ce necesită consult
ortopedic;
C. Dacă genu varum persistă peste vârsta de 3 ani justifică
evaluarea radiologică;
D. După vârsta de 5 ani membrele ating aliniamentul adult normal
de 7 grade valgus scheletal;
E. Sugarii au genu varum fiziologic cu un unghi tibio-femural ≥ de
20 grd.
R: B, E (2, p. 611)

32. Piciorul plat:


A. Cel mai rar întâlnit tip este piciorul plat flexibil;
B. Arcul longitudinal al piciorului este absent sau plat în ortostatism
și reapare atunci cînd piciorul nu sprijină greutatea corpului în
piciorul plat rigid;
C. Piciorul plat rigit prezintă o mișcare pasivă limitată;
D. Picior plat asociat cu equin poate fi cauzat de o distrofie
musculară;
E. Arcul accentuat al piciorului cu degetele în gheară reprezintă o
conseciță a neuropatiilor periferice.
R: C, D, E (2, p. 611-613)

33. Boala Legg-Calve-Perthes:


A. Reprezintă osteonecroza capului femural la copii cu vâste între 5
și 8 ani;
B. Raportul dintre sexul masculin și cel feminim este de 1\8;
C. Clinic limitare ușoară a mișcărilor de abducție și rotație internă;
D. Mers Trendelenburg prezent;
E. Tartamentul chirurgical reprezentat de osteotomie femurală sau
pelvină.

227
R: C, D, E (2, p. 614)

34. Epifizioliza femurală proximală – următoarele afirmații sunt


false cu excepția:
A. Este reprezentată de fractura cartilajului de creștere distal;
B. Mai frecventă la sexul feminin;
C. Bilaterală în aproximativ 33% din cazuri;
D. Prezentă frecvent între 11 și 14 ani;
E. Membrul inferior afectat este în rotație externă.
R: C, E (2, p. 614)

35. Scolioza – următoarele afirmații sunt false:


A. Scolioza decompensată paote fi asociată cu un grad mare de
apariție a durerilor de membre;
B. Scolioza congenitală frecvent asociată cu malformații la tubului
neural, aparat cardio-vascular și a aparatului cardio-repirator;
C. Curbura scoliotică neuro-musculară implică de obicei întreaca
lungime a coloanei vertebrale;
D. În neurofibromatoză scolioza se acracterizează printr-o curbare
severă, dar pe o porțiune limitată;
E. Corpurile vertebrale iau formă de pană iar coastele proemină pe
partea concavă a curburii.
R: A, B, E (2, p. 615)

36. Osteomielita hematogenă acută:


A. Clinic – durere în apropierea diafizei osului lung;
B. În caz de septicemie apare febră iritabilitate crescută fără
alterarea stării generale;
C. Tumefierea țesuturilor moi se produce mai târziu;
D. Numărul de leucocite și VSH-ul – de regulă crescute;
E. Radiologic modificările apar imediat.
R: C, D (2, p. 616)

37. Tenosinovita septică a flexorilor mâinii:


A. Cel mai frecvent etiologia acesteia este streptococul;
B. Este cauzată adesea de o plagă palmară nepenetrantă;
C. Infecția piogenică a mâinii poate duce la pierderea membrului;

228
D. Absența semnelor și simptomelor de amelioare în primele 24-48
de ore impune tratament chirurgical;
E. Kinetoterapia de recuperare începută precoce poate agrava
funcția normală a mâinii.
R: C, D, E (2, p. 618)

38. Osteoartroza radiologic prezintă următoarele semne:


A. Pensarea spațiului articular;
B. Scleroza osului subcondral;
C. Osteofite;
D. Chiste în osul condral;
E. Absența chistelor în osul subcondral.
R: A, B, C (2, p. 619)

39. Tratamentul chirurgical în osteoartroză presupune:


A. Artroplastie totală;
B. Artrodeză;
C. Osteotomii de realiniere a articulației;
D. Artoplastii parțiale;
E. Foraje decompresive.
R: A, B, C, D (2, p. 619)

40. Artrita reumatoidă:


A. Este o boală inflamatorie de etiologie cunoscută;
B. Este o poliartrită cronică simetrică ce prezintă recidive care duc
frecvent la distrucția articulară progresivă, diformități și deficit
funcțional;
C. Apare mai frecvent la femei în raport de 4/1;
D. Nu are bază genetică;
E. Prezența factorului reumatoid nu este diagnostică pentru artrita
reumatoidă.
R: B, E (2, p. 620)

41. Hernia de disc:


A. Este rezultatul extruziei nucleului fibros prin inelul pulpos;
B. Apare la adulți între 30 și 50 de ani;
C. Cele mai frecvent afectate discuri sunt L3-L5, L5-S1;
D. Durerea este accentuată de aplecare, șezut și tuse;
E. Când sunt implicate mai multe rădăcini tabloul clinic este clar.
R: B, D (2, p. 623)

229
42. Osteoporoza:
A. Rezistența osoasă depinde de cantitatea de componentă minerală
pe unitatea de volum;
B. Cel mai frecvent tip de osteoporoza este cel involutiv senil;
C. Bifosfonații și calcitonina pot fi utilizați in tratatmentul acesteia;
D. Exercițiile fizice, calciu din alimenatație și suplimentele de
vitamina D nu oferă oprofilaxie eficientă împotriva pierderii
osoase în osteoporoză;
E. Cauzele osteoporozei pot fi – mielom multiplu, leucemie,
corticosteroizi, hipogonadism, hipertiroidism.
R: A, B,C, E (2, p. 627)

43. Hiperparatiroidismul:
A. Acesta provoacă osteopenie osoasă difuză;
B. Hiperparatiroidismul primar se datorează unui adenom sau
hiperplaziei glandei paratiroide;
C. Hiperparatiroidismul secundar se datorează insuficineței renale
acute, cu scăderea excreției de fosfați;
D. Radiografic se observă rarefierea osoasă difuză fără leziuni
osteolitice focale;
E. Radiologic prezență de osteofite la nivelul articulațiilor.
R: A, B (2, p. 628)

44. Neoplasmele osoase:


A. Tumorile cu punct de plecare în țesuturile musculoscheletale
sunt frecvente;
B. Implicarea osoasă datorita tumorilor metastatice sunt frecvente la
pacienții cu vârstă peste 60 de ani;
C. Tumorile primare de sân, prostată, tiroidă, metastazează deseori
la nivelul osului;
D. Metastazele cancerului de sân și de prostată pot fi osteoblastice
sau osteolitice;
E. Cea mai frecventă tumoră primară osoasă este mielomul
multiplu.
R: C, D, E (2, p. 629-630)
45. Tumorile osoase primare în funcție de țesutul lor de origine:
A. țesut de origine osos pot fi -osteom osetoid, osteosarcom,
osteocondrom;
B. țesut de origine cartilaginos – encondrom, condrosarcom;

230
C. țesut de origine fibros – fibrom, fibrosarcom, osteofibrom;
D. țesut de origine măduvă osoasă – granulom eozinofil, sarcom
Ewing;
E. țesut de origine incert – tumoră cu celule gigante, tumoră
agresivă cu celule gigante.
R: B, D, E (2, p. 630)

46. Caracteristicile principale ale celor mai frecvente tumori osoase:


A. osteosarcomul – localizat metafizar, interesează femurul distal și
tibia proximal și distal;
B. condrosarcomul – întâlnit între 40 și 60 de ani;
C. Fibrosarcomul radiologic se prezintă ca o leziune slab delimitată,
distructivă și radiotransparentă;
D. Mielomul – este una dintre cele mai frecvente tumori maligne
primare osoase cu origine celulară plasmatică;
E. Sarcomul Ewing – localizare frecventă la nivelul diafizei
femulare, osul iliac, humerus și tibie.
R: B, C, D, E (2, p. 631)

47. Tratamentul tumorilor osoase:


A. Depinde de diagnosticul histologic, gradul de anaplazie celulară,
extindere tumorală și starea generală a pacientului;
B. Leziunile metastatice pot fi tratate paleativ fără iradiere locală;
C. În general tumorile osoase maligne sunt îndepărtate prin rezecție
chirurgicală marginală;
D. Chimioterapia adjuvantă și radioterapia sunt folosite pentru a
eradica micrometastazele tumorale care se presupun a fi
prezente;
E. Tehnicile de salvare a membrelor trebuie să compromită
eradicarea fomațiunii tumorale.
R: A, D (2, p. 632)

48. Caracteristicile pricipale ale celor mai frecvente tumori osoase:


A. Osteomul osteoid – întalnită ≥ 30 de ani;
B. Osteocondromul – tumefacție palpabilă, poate interfera cu
funcția tendonului;
C. Encondromul – radiologic zona de radiotrasparență bine
demarcată care poate conține arii de calcificări;
D. Fibromul neosifiant – se osifică odată cu maturarea scheletală;

231
E. Tumora cu celule gigante localizată frecent a nievlul difizei
oaselor lungi.
R: B, C, D (2, p. 630)

49. Biopsia tumorilor osoase:


A. Biopsia chirurgicală se practică cu scopul de a obține țesut
tumoral pentru diagnosticul serologic;
B. Biopsia nu poate fi efectuată pe cale deschisă sau inchisă;
C. Trebuie efectuată de echipa chirurgicală responsabilă de
tratamentul parțial al tumorii;
D. Incizia pentru biopsie trebuie plasată încât să nu compromită
procedura finală și tumora să poată fi rezecată în totalitate;
E. Când este posibil se recurge la efectuarea unor secționări la
gheață pentru a ne asigura că proba de țesut osos este adecvată.
R: D, E (2, p. 632)

50. Diagnostice diferențiale în hernia de disc:


A. Infecția la nivelul coloanei vertebrale;
B. Boli vasculare (anevrismul de arteră femurală);
C. Neoplasme primare sau metastatice;
D. Durere psihogenă sau simulată;
E. Boli reumatismale (artrita reumatoidă).
R: A, C, D, E (2, p. 625)

232
CAPITOLUL 33
UROLOGIE: AFECȚIUNILE APARATULUI URO –
GENITAL

1. Epidemiologia hiperplaziei benigne de prostată cuprinde:


A. este un proces degenerativ de hiperplazie
B. este un proces degenerativ de hipertrofie
C. asociază procese obstructive
D. procesul patologic apare în zona tranzițională
E. poate fi demonstrată histologic de la vârsta de 20 de ani
R: A, B, C, D (2, p. 636)

2. Simptomele obstructive clasice din hiperplazia benignă de prostată


includ:
A. inițerea ușoară a micțiunii
B. disuria
C. driblingul terminal
D. jet urinar bine proiectat
E. senzația de golire incompletă a vezicii urinare
R: B, C, E (2, p. 637)

3. Examinarea prostatei prin tușeu rectal, în cazul hiperplaziei benigne


de prostată relevă următoarele:
A. creșterea palpabilă a acesteia
B. prostata poate fi asimetrică
C. conturul este neted și consistența este moale
D. conturul este neregulat
E. consistența este fermă la palpare
Răspuns corect:A,B,C (2, p. 637)

4. Hipetrofia benignă de prostată cu evoluție îndelungată poate duce la


complicații cum ar fi:
A. ureterohidronefroză
B. hidronefroză
C. insuficiență renală
D. absența retenției de urină
E. hematurie microscopică
R: A, C (2, p. 637)

233
5. Terapia medicamentoasă modernă utilizată în hiperplazia benignă de
prostată:
A. utilizează antibioticele cu spectru larg de primă linie
B. blocanții beta1 adrenergici sunt utilizați de primă intenție
C. blocanții alfa1 adrenergici sunt utilizați de primă linie
D. doza terapeutică include o singură administrare pe zi
E. administrarea medicamentului se realizează per oral
R: C, D, E (2, p. 637)

6. Finasterida și dutasterida utilizate în tratamentul medicamentos al


hiperplaziei benigne de prostată:
A. inhibă 5alfa reductaza
B. se utilizează pentru formele simptomatice
C. blochează activitatea androgenică de la nivelul celulei prostatice
D. determină o scădere a dimensiunii prostatei
E. relaxează musculatura netedă de la nivelul capsulei prostatice
ameliorând astfel simptomatologia pacienților
R: A, B, C, D (2, p. 637)

7. Tratamentul chirurgical în hipertrofia benignă de prostată:


A. pentru prostate de peste 100g greutate, se optează pentru abordarea
deschisă sau robotică
B. se realizează prin extragerea capsulei prostatice care este obstructivă
C. poate fi realizată prin procedee deschise sau transuretrale
D. continuitatea uretrală nu este afectată
E. adenomectomia transvezicală se realizează prin abord deschis
suprapubian
R: A, C, D, E (2, p. 637)

8. Diagnosticul în cancerul de prostată:


A. antigenul specific prostatic nu este utilizat pe scară largă
B. antigenul specific prostatic este considerat ca fiind piatra de temelie
pentru diagnosticul carcinomului de prostată
C. la pacienții cu istoric familial de cancer de prostată screeningul ar
trebui să se facă după vârsta de 40 de ani
D. afroamericanii sunt la fel de afectați comparativ cu americanii
caucazieni de cancerul de prostată
E. implementarea screeningului pentru cancerul de prostată a redus rata
deceselor cauzate de această boală cu 40%
R: B, C, E (2, p. 638)

234
9. Tratamentul non medicamentos în cancerul de prostată include:
A. prostatectomia radicală
B. radioterapia
C. brahiterapia nu este o soluție viabilă pentru tratament
D. rezecția transuretrală a prostatei (TUR-P) este considerat tratamentul
de elecție
E. supraviețuirea la 10 ani după prostatectomia radicală sau radioterapia
este similară
R: A, B, E (2, p.640)

10. Pielonefrita xantogranulomatoasă se caracterizează prin


următoarele:
A. apare preponderent la copii
B. este o afecțiune inflamatorie a rinichiului
C. afectează cu precădere sexul feminin
D. tratamentul este chirurgical și constă în nefrectomie
E. nu asociază infecții unrinare cronice
R: B, C, D (2, p. 645)

11. Patologia tumorală a rinichiului include:


A. tumorile renale sunt doar de natură benignă
B. tumorile maligne sunt doar secundare altor tumori
C. tumorile maligne pot fi primare sau metastatice
D. carcinomul cu celule clare este cel mai frecvent întâlnit
E. chistul simplu este o leziune tisulară renală
R: C, D, E (2, p. 645)

12. Tratamentul litiazei urinare depinde de următoarele:


A. mărimea calculului
B. numărul calculilor
C. localizarea calculilor
D. compoziția chimică a calculilor
E. valoarea creatininei
R: A, C, D (2, p. 647)

13. Pielonefrita cronică obstructivă se caracterizează prin:


A. asociază un calcul obstructiv
B. apare pe un rinichi fără calculi

235
C. este prezentă infecția tractului urinar superior
D. evoluează cu dureri în flanc și febră
E. nu este o urgență urologică
R: A, C, D(2, p. 647)

14. Tratamentul chirurgical de urgență în pielonefrita obstructivă


include:
A. pielolitotomie
B. nefrolitotomie percutanată
C. montare nefrostomie
D. montare stent intern
E. uretero-pielolitotomie de urgență
R: C, D

15. Factorii de risc pentru dezvoltarea calculilor urinari sunt:


A. tulburări metabolice
B. deshidratarea
C. imobilizarea prelungită
D. istoric familial de litiază salivară
E. infecții urinare cronice pentru calculii infecțioși
R: A, B, C, E (2, p. 647)

16. Obstrucția ureterală este secundară următoarelor afecțiuni:


A. calculii ureterali
B. fibroza peritoneală
C. peristalticii normale
D. calculi vezicali
E. anevrisme vasculare
R: A, E (2, p.648) ?

17. Sistemul limfatic al vezicii urinare drenează în:


A. ganglionii iliaci comuni
B. ganglionii iliaci interni
C. ganglionii iliaci externi
D. ganglionii hipogastrici
E. ganglionii sacrali

R: A, C, D, E (2, p. 649)

236
18. Extrofia vezicală este rezultatul:
A. dezvoltării incomplete a peretelui abdominal anterior
B. dezvoltării incomplete a peretelui abdominal posterior
C. dezvoltării incomplete a centurii pelvine
D. dezvoltării incomplete a oaselor bazinului
E. dezvoltării incomplete a peretelui anterior al vezicii urinare
R: A, C, E (2, p. 651)

19. Cele mai frecvente bacterii patogene implicate în cistite sunt:


A. bacilii Gram-negativi
B. germeni din familia Enterobacteriaceae
C. streptococul și stafilococul
D. E-Coli implicat în peste 80% din cazuri
E. bacilii Gram-pozitivi
R: A, B, D (2, p.654)

20. Cauzele urologice care trebuie tratate în infecțiile urinare recurente


sunt:
A. duplicația renală bilaterală
B. hipertrofia prostatei
C. litiaza
D. abcesul perivezical
E. refluxul uretro-vezical
R: B, C, D (2, p.655)

21. Incontinența urinară se clasifică în:


A. de stres
B. prin imperiozitate
C. prin prea plin
D. prin retenție cronică fără distensie vezicală
E. secundară anuriei
R: B, C (2, p.655)

22. Incontinența urinară de efort survine:


A. la tuse
B. postoperator TUR-P
C. la râs
D. la strănut
E. variantele c și d nu sunt adevărate
R: A, C, D (2, p.655)

237
23. Cistectomia radicală pentru tumorile vezicale infiltrative la bărbat
presupune:
A. excizia vezicii urinare
B. excizia prostatei
C. înlăturarea doar a uretrei anterioare
D. înlăturarea grăsimii perivezicale
E. înlăturarea ganglionilor limfatici pelvini
R: A, B, D, E (2, p.659)

24. Cistectomia radicală pentru tumorile vezicale infiltrative la femeie


presupune:
A. excizia vezicii urinare
B. excizia doar a vezicii urinare
C. excizia peretelui anterior al vaginului
D. excizia uterului și a ganglionilor limfatici pelvini
E. nu se excizează și ganglionii limfatici pelvini
R: A, B, D (2, p.659)

25. Care din următoarele leziuni peniene sunt considerate a fi


premaligne:
A. leukoplakia
B. balanita cronică
C. balanita xerotică obliterantă
D. eritroplazia Queyrat
E. fimoza congenitală
R: A, C, D (2, p.660)

26. Priapismul poate fi:


A. cu flux constant
B. cu flux redus
C. cu flux crescut
D. cu flux fluctuant
E. nu depinde de fluxul sangvin
R: B, C (2, p.662)

27. Fimoza se caracterizează prin:


A. decalotarea se efectuează în mod fiziologic
B. este contractura fibrotică a tegumentului prepuțial
C. nu permite retracția prepuțiului peste glandul penian

238
D. apare frecvent la pacienții cu diabet zaharat
E. tratamentul electiv este circumcizia
R: B, C, D, E (2, p.662 – 663))

28.Uretra masculină include:


A. segmentul posterior cu uretra prostatică și membranoasă
B. segmentul posterior include uretra colului vezical și a trigonului
C. segmentul anterior conține meatul uretral, fosa naviculară
D. segmentul anterior este un segment fix
E. segmentul anterior conține uretra peniană, uretra bulbară
R: A, C, E (2, p.664)

29. Uretritele non-gonococice se caracterizează prin:


A. disurie
B. poliurie
C. prurit uretral cu secreție uretrală clară
D. polakiurie
E. micțiuni nocturne
R: A, C, D (2, p.668)

30. Următorii markeri tumorali sunt folosiți în diagnosticul tumorilor


testiculare:
A. alfa feto-proteina
B. PSA-ul
C. Free PSA-ul
D. beta HCG
E. antigenul testicular fetal
R: A, D (2, p.670)

239
SINOPSIS DE MEDICINA

CAPITOLUL 34
DERMATOLOGIE

1. In celulită sunt adevarăte următoarele afirmații cu două excepții


A.Este o dermoepidermită
B.Are ca factor de risc diabetul zaharat
C. Culturile bacteriene cutanate sunt frecvent utile pentru diagnostic
D.Se tratează cu cefalosporine
E. In hemoleucogramă se observă leucocitoză
R: A, C (3, p 196)

2. Care sunt cele trei afirmații false privind Impetigo?


A. Nu se transmite in colectivități
B. Apare mai frecvent la adulți
C. Este determinat mai frecvent de germenii Gram Negativi
D.Formele ușoare se pot trata topic cu mupirocin
E.Clinic sunt prezente cruste melicerice
R: A, B, C (3, p 197)

3. Dintre următoarele,două afirmații sunt false în privința acneei


A. Retinoizii topici sunt recomandați ca primă linie de tratament
B. Retinoizii în combinație cu antibioticele topice pot inhiba
multiplicarea bacteriană
C. Retinoizii de sinteză administrați pe cale internă se pot utiliza la femei
fără contraceptive
D. Alimentația în acnee are un rol decisiv
E. Corticosteroizii pot induce acnee la adulți
R: C, D (3, p 198)

4. Care dintre următoarele sunt cele trei afirmații false privind infecția
cu Herpes Virusuri?
A. Se transmit prin contactul cu secrețiile orale sau genitale
B. HSV -1 determină în special afectare genitală
C. HSV-2 determină în special afectare orală
D. La pacienții imunosupresați pot determina encefalită
240
E. Famciclovirul nu poate scădea frecvența recurentelor eruptive
R: B, C, E (3, p 198)

5. Printre caracteristicile herpesului zoster trei sunt false


A. Leziunile sunt infecțioase până formează crustă
B. Nevralgia postherpetică are o durată scurtă
C. Afectarea mai multor dermatoame semnifică boală diseminată
D. Pacienții nu au în istoric varicelă sau herpes zoster
E. Retinita nu este dezvoltată de pacienții imunocompromiși cu afectare
oftalmică
R: B, D, E (3, p 199)

6. Care dintre următoarele enunțuri sunt cele două adevărate


A. În contextul măsurilor de tratare a scabiei, arderea lenjeriei este
esențială
B. Maculele descuamative din Pityriasisul versicolor se pigmentează la
soare
C. În diagnosticul Tinea ,examenul microscopic pe lamă al scuamelor
tratate cu KOH arată prezența de hife
D. Clinic Tinea se prezintă sub forma unor plăci eritematoscuamoase cu
margine inactivă și accentuare centrală
E. Intertrigo candidozic se exprimă clinic prin afectarea eritematoasă cu
pustule periferice,la nivelul pliurilor
R: C, E (3, p 199-200)

7. Care sunt cele trei afirmații false privind reacțiile de


hipersensibilitate la nivelul pielii?
A. Mecanismul de tip IV se realizează prin degranulare mastocitară
B. În dermatita de contact apare o erupție eritematoasă,pruriginoasă cu
dispoziție distinctă
C. Mecanismul de tip I se realizează prin activitatea limfocitelor
D. Epinefrina nu este utilă în anafilaxie
E. Mânușile de latex pot produce dermatita alergică de contact
R: A, C, D (3, p 201,202)

8. In dermatita atopică trei enunțuri sunt adevărate:


A. Erupția este transmisibilă prin contact direct
B. Între factorii de risc, există istoricul familial
C. Tegumentele sunt seboreice
D. La nou-nascuți leziunile apar mai frecvent pe față

241
E. În tratament sunt utile cremele emoliente și dermatocorticoizi topici
R: B, D, E (3, p 203)

9. Referitor la psoriasis, următoarele afirmații sunt adevărate cu două


excepții:
A. Este o boală contagioasă
B. Plăcile eritematoase sunt bine delimitate
C. Poate fi prezent pitting unghial
D. La examenul histopatologic cutanat este prezentă atrofia epidermică
E. Tratamentul cu Methotrexat se utilizează în formele severe
R: A, D (3, p 203)

10. În eritemul nodos 3 enunțuri sunt adevărate


A. Este localizat tibial anterior
B. Este o inflamație epidermică
C. Durerea este un simptom rar
D. Este cauzat de o reacție imunologică de tip întârziat
E. Titrul ASLO poate fi de ajutor diagnostic
R: A, D, E (3, p 204)

11. Dintre următoarele, două afirmații sunt false privind Pemfigusul


vulgar
A. Apare mai ales la vârstnici
B. Se caracterizează prin apariția de autoanticorpi în derm
C. Semnul Nicolski este negativ
D. Histopatologic,membrana bazală rămâne intactă
E. Dacă afecțiunea nu e tratată,mortalitatea este crescută
R: B, C (3, p 205)

12. Care sunt cele trei afirmații adevarăte privind Pemfigoidul bulos?
A. Este o afecțiune infecțioasă frecventă
B. Vârsta pacienților este peste 65 de ani
C. Bulele sunt situate predominant pe zonele flexoare și în zona
perineală
D. Semnul Nicolski este pozitiv
E. Imunofluorescența evidențiază anticorpi anti membrană bazală
R: B, C, E (3, p 205,206)

13. In porfiria cutanată tardivă sunt trei enunțuri adevărate

242
A. Un factor declanșator important este efortul fizic
B. Pe tegumentele fotoexpuse apar leziuni buloase cronice
C. Nivelul ALT și AST este scăzut
D. Nivelul uroporfirinogen-decarboxilazei este scăzut
E. Flebotomia periodică face parte din arsenalul terapeutic
R: B, D, E (3, p 208)

14. Dintre următoarele ,două enunțuri sunt false privind keratozele


actinice
A. Este o leziune precanceroasă
B. Clinic se prezintă ca o veziculo-bulă
C. Riscul de evoluție spre carcinomul spinocelular este de 21%
D. 5-Fluoro uracilul sistemic este util în tratamentul lor
E. Biopsia evidențiază epiteliu displazic
R: B, D (3, p 206)

15. Care sunt cele două enunțuri adevărate în legătură cu carcinomul


spinocelular (scuamocelular)
A. Are originea în celulele stratului granulos
B. Apare pe răni cronice
C. Apare pe zone fără fotoexpunere
D. Celulele anaplastice se pot extinde în derm
E. Tratamentul de elecție este radioterapia
R: B, D (3, p 206)

16. Carcinomul bazocelular are următoarele 3 caracteristici


A. Se dezvoltă din celulele stratului bazal epidermic
B. La examenul histopatologic celulele eozinofilice au dispoziție
palisadică
C. Clinic apar ca papule perlate cu telangiectazii
D. Tratamentul de elecție este chirurgical
E. Metastazează frecvent
R: A, C, D (3, p 207)

17. Patru tipuri clinice de melanom sunt enumerate mai jos


A. Extensiv in suprafață
B. Nodular
C. Perlat

D. Acral lentiginos

243
E. Lentigo malign
R: A, B, D, E (3 ,p 208)

18. Care sunt cele trei caracteristici adevărate în privința melanomului


A. Au margini neregulate
B. Păstrează aceleași dimensiuni de-a lungul timpului
C. Biopsia excizională evidențiază melanocite tipice
D. Excizia chirurgicală se face cu o margine peste 2 cm în cazul
tumorilor cu peste 2mm grosime
E. Metastazele apar mai frecvent în creier,plămân și tractul gastro -
intestinal
R: A, D, E (3 , p 208)

19. Care sunt cele două enunțuri adevărate în legătură cu eritemul


polimorf?
A. Leziunile sunt localizate preponderant pe torace
B. Leziunile au un aspect în țintă
C. Examenul de laborator arată număr normal de eozinofile
D. La examenul histopatologic se pot observa keratinocite necrotice
E. Corticoterapia nu este eficientă în tratamentul său
R: B, D (3, p 202)

20. Sindromul Stevens-Johnson are următoarele caracteristici, cu două


excepții
A. Este o forma severă de eritem polimorf
B. Nu afectează mucoasele
C. Suprafața corporală afectată este sub 10%
D. Suprafața corporală este peste 10%
E. Semnul Nikolsky este pozitiv
R: B, D (3, p 202)

244
CAPITOLUL 35
PEDIATRIE

1. Purpura Henoch-Schonlein:
A. Este o vasculită mediată prin complexe imune de tip IgA ce
afectează doar arteriolele;
B. Este cea mai comună vaculită la copii;
C. Complicațiile frecvente sunt cele gastrointestinale si renale;
D. Tratamentul cu corticosteroizi este utilizat pentru formele
gastrointestinale severe sau cu afectare renala;
E. Examenul de urina poate sa arate leucociturie si cilindrurie.
R: B, C, D (3, p. 215)

2. Boala Kawasaki:
A. Este una din cauzele de cardiopatii dobândite la copii;
B. Din punct de vedere imagistic angiografia poate detecta
neregularități ale vaselor mari;
C. Este a doua cea mai frecventă vasculită la copii;
D. In cadrul tratamentului în boala Kawasaki se numără ASA,
imunoglobuline administrate iv;
E. Este o boală inflamatorie necrozantă a vaselor mici.
R: A, C, D (3, p. 215)

3. Tetralogia Fallot:
A. Se caracterizează dpdv al ECG prin deviație axială stângă;
B. Factori de risc sunt sindromul Down, sindromul cri-du-chat și
trisomia 21;
C. Din punct de vedere radiologic aspectul cordului este în sabot;
D. Clinic se caracterizează prin cianoza precoce, astenie, dispnee;
E. Tratamentul este complex (prostaglandină, propranolol, morfină,
fluide iv, poziție genu-pectorala în timpul episoadelor de
cianoză, corecție chirurgicală).
R: C, D, E (3, p. 214)

4. Circulația fetală:
245
A. Hb fetală are o afinitate mai mare pentru CO2;
B. După naștere are loc o scădere a nivelului seric de
prostaglandină E2;
C. Arterele ombilicale duc sângele oxigenat la placentă;
D. În cadrul modificărilor după naștere a circulației fetale, rezistența
vasculară crescută induce închiderea canalului venos și
constricția arterei și venei ombilicale;
E. Schimbul de gaze are loc în circulația utero placentară.
R: B, D, E (3, p. 212)

5. Bronșiolita acută:
A. Infecția virală a bronhiolelor este cauzată de virusul
parainfluenza tip 3 cel mai frecvent și VSR cel mai rar;
B. Cel mai frecvent apare toamna și la copii mai mari de 2 ani;
C. Radiografia pulmonară nu este necesară pentru diagnostic;
D. Detresa respiratorie poate duce la insuficiență cardiacă;
E. Are risc crescut de a dezvolta astm bronșic.
R: C, E (3, p. 217)

6. Fibroza chistică:
A. Fibroza chistică de pancreas se transmite autozomal dominan;t
B. Se manifestă cel mai frecvent în decada 30-40 de viață,
C. Afectează un singur plămân și sistemul gastrointestinal;
D. Factor de risc - rasa albă;
E. Testul sudorii arată creșterea sodiului și a clorului peste
60mEq/L la copil.
R: D, E (3, p. 218-219)

7. Epiglotita:
A. Infecție rapid progresivă a epiglotei și a țesuturilor adiacente;
B. În trecut cel mai frecvent cauzată de stafilococul auriu;
C. Frecvent întâlnită infecția cu haemophylus influenzae tip a;
D. Din punct de vedere clinic se caracterizează prin disfagie,
hipersalivație, stridor discret si anxietate;
E. Infecția epiglotei nu poate cauza obstrucția cailor aeriene.
R: A, D (3, p. 216)

8. Pertussis:

246
A. Cauzată doar de Bordetella parapertussis;
B. Faza catarală durează între 1 și 3 săptămâni cu simptome
nespecifice;
C. Faza de convalescență durează între 1 și 2 luni cu o scădere
graduală a fecvenței și severității tusei;
D. Dpdv al rezultatelor de laborator -hemograma poate sa fie cu
leucocitoza marcata si limfocitoza;
E. Complicațiile cele mai frecvente sunt pneumonia și apneea la
sugari.
R: D, E (3, p. 217)

9. Sindromul de detresă respiratorie a NN:


A. Nou născuții prematuri între 24-37 săptămâni gestaționale, mai
ales înainte de 30 de săptămâni, au deficienta de surfactant
datorită imaturității pulmonare;
B. ASTRUP-ul arată creșterea CO2 și scăderea O2;
C. Imagistica arată atelectazie pulmonara stângă;
D. Se manifesta in primele 2 săptămâni după naștere;
E. Tratamentul consta în administrare a de corticosteroizi mamei
înainte de inițierea nașterii.
R: A, B, E (3, p. 217-218)

10. Enterocolita necrotică:


A. Este o necroza idiopatica a submucoasei;
B. ASTRUP-ul evidențiază acidoză respiratorie și scăderea
sodiului;
C. Radiografia abdominala evidențiază distensie intestinală, aer în
peretele intestinal și aer liber subdiafragmatic;
D. Tratamentul constă în nutriție orală, antibiotice –
aminoglicozide;
E. Factorii de risc – naștere prematură și greutate mică la naștere.
R: C, E (3, p. 220)

11. Boala Hirschsprung:


A. Se caracterizează prin absența inervației autonome intestinale;
B. Dpdv clinic sunt prezente vărsăturile, constipația severa,
distensia abdominală;
C. Biopsia intestinală evidențiază absența ganglionilor;
D. Tratamentul de bază este cu antiinflamatoare nespecifice,
antibiotice;

247
E. Radiografia pulmonară demonstrează prezența atelectaziei.
R: A, B, C (3, p. 220)

12. Icterul fiziologic:


A. Cauzele fiziologice ale hiperbilirubinemiei la nn se rezolvă în
aproximativ 2 luni;
B. Clinic se caracterizează prin icter, sclere icterice, convulsii,
apnee in caz de icter nuclear (Kern);
C. Dpdv biologic – creștere a bilirubinei indirecte;
D. Tratamentul – fototerapie utila pentru icterul fiziologic ce
durează câteva zile;
E. Suspiciunea cauzelor ne fiziologice trebuie elucidată si necesita
exanguinotransfuzie, imunoglobulina iv.
R: B, C, D, E (3, p. 221)

13. Deficit de creștere:


A. Copiii sub percentila a treia a greutatii pentru varsta;
B. Tratamentul corect se face cu o dieta normocalorică; educarea
părinților în vedere unei nutriții si alimentații adecvate;
C. Se poate datora unei patologii subiacente sau a neglijenței;
D. Dpdv biologic sunt necesare examenul de urina,
hemoleucograma, ionograma serică, testarea pentru fibroza
chistica;
E. Obligatoriu trebuie căutate indicii pentru cauze organice si de
verificat un eventual abuz asupra copilului.
R: A, C, D ,E (3, p. 221)

14. Tumora Wilms:


A. Factori de risc în tumora wilms pot fi: istoric familial,
neurofibromatoza, alte anomalii genitourinare
B. Este o tumoră benignă cu origine renală;
C. Biologic pentru a evalua funcția renala sunt necesare HLG,
ureea, creatinina;
D. CT-ul poate evidenția metastazele;
E. Tratamentul constă doar în radioterapie, chimioterapie.
R: A, C, D (3, p. 221-222)

15. Deschiderea anormala de uretră:


A. Poate fi deasupra sau în partea inferioară a penisului;

248
B. Defectul apare doar la examenul clinic;
C. Tratamentul constă în corecția chirurgicală (ideal la 2 ani);
D. Poate contribui la infertilitate;
E. Înainte de corecția chirurgicala se va efectua circumcizia.
R: A, D (3, p. 222)

16. Criptorhidia:
A. Tratamentul constă în administrarea exogena de hCG sau
orhidopexie înainte de vârsta de 5 ani;
B. Complicațiile criptorhidiei sunt infertilitatea, cancerul testicular;
C. În criptorhidie testiculele sunt localizate în cavitatea abdominală;
D. Testiculele sunt localizate in loja renală;
E. Tratamentul chirurgical se va face după vârsta de 5 ani.
R: A, B, C (3, p. 222)

17. Enurezis:
A. Înseamnă micțiune în pat în perioada 27 de zile – 1 an;
B. Tratamentul enurezisului constă în circumcizie la vârsta de 1 an;
C. Majoritatea cazurilor cu enurezis se rezolvă până la vârsta de 4
ani;
D. Cazurile patologice de enurezis sunt frecvente;
E. Elementele neobișnuite în istoric sau examinare trebuie să
ghideze investigațiile suplimentare.
R: C, E (3, p. 222)

18. Hipotiroidismul congenital:


A. Investigațiile specifice pentru diagnostic sunt T4 si TSH;
B. Dpdv imagistic radiografia evidențiază dezvoltate osoasa
corespunzătoare vârstei;
C. Tratamentul consta in levothyroxină după vârsta de 2 ani;
D. Este cauzat de deficienta severa de iod agenezia de tiroida sau
patologiei ereditare ale sintezei hormonilor tiroidieni;
E. Diagnosticul de certitudine (biologic) se pune atunci când T4
este crescut si TSH-ul scăzut.
R: A, D (3, p. 222)

19. Deficiența de glucozo-6-fosfataza:


A. Una din multiplele patologii de stocare a glicogenului;

249
B. Cunoscută ca glicogenoza de tip 2;
C. Apare în primele luni de viată (3-4luni);
D. Examenul clinic evidențiază splenomegalie, extremități subțiri;
E. Biologic hipoglicemie, ac lactic crescut, lipide crescute, ac uric
crescut.
R: A, C, E (3, p. 222-223)

20. Boala hemolitica a nou-născutului:


A. Dacă celulele Rh- fetale intra in circulația mamei Rh+, pot
apărea anticorpi anti-Rh;
B. Administrarea de imunoglobuline Rh0(D) (Rh0GAM) in primele
72 de ore de la naștere la feți Rh+, va preveni dezvoltarea Ac
anti Rh;
C. Factori de risc: mama Rh- cu orice istoric de hemoragie materno-
fetală;
D. Hemoliza nu va cauza cel mai probabil decesul fătului;
E. Transfuzia fetala intrauterina poate fi necesara daca patologia se
dezvolta intrauterin.
R: B, C, E (3, p. 223)

21. Anemia Fanconi:


A. Transmiterea genetică este autozomal dominanta;
B. Se asociază cu insuficienta circulatorie;
C. Alfa fetoproteina serica este crescuta;
D. Biopsia medulara evidențiază hipocelularitate;
E. Tratamentul consta in antibiotice, transfuzii, transplant medular
sau de celule stem hematopoetice, androgeni.
R: C, D, E (3, p. 223)

22. Neruroblastomul:
A. Tumora renala;
B. Factori de risc: neurofibromatoza, feocromocitomul, sd Turner;
C. Biologic – acidul vanil mandelic si homovanilic crescute in urina
pe 24 de ore;
D. Tratament – rezecție chirurgicala, chimioterapie si radioterapie;
E. Prognostic nefavorabil – metastazează în plămân și ficat.
R: B, C, D (3, p. 224)

23. Rabdomiosarcomul:
A. Tumora mușchilor striați la copil;

250
B. Biologic transaminazele stabilesc diagnosticul;
C. Tratament – rezecție chirurgicală, radioterapie, chimioterapie;
D. Tumorile mari nu cauzează frecvent efect de masă asupra
structurilor adiacente;
E. Creșterea transaminazelor și creatininei arată extinderea tumorii.
R: A, C (3, p. 224)

24. Convulsii febrile:


A. Convulsii ale copiilor între 6 luni și 5 ani, asociate cu febră;
B. Febră mai mare de 38 de grade cu creștere rapidă a temperaturii,
crize tonicoclonice <15min.;
C. EEG este necesar pentru diagnosticul convulsiilor febrile;
D. Riscul de epilepsie la pacienții care au prezentat convulsii febrile
recurente este foarte scăzut;
E. Convulsiile febrile complexe au o probabilitate de recurență
mare.
R: A, B, C, E (3, p. 224-225)

25. Hidrocefalia copilăriei:


A. Poate fi comunicantă sau necomunicantă;
B. Puncția lombară poate fi efectuată obligatoriu și de urgență;
C. Complicații – risc de 50% mortalitate înainte de vârsta de 1 an,
în lipsa tratamentului;
D. Imagistica – ecografia, ct, irm vor arăta ventriculi măriți în
dimensiuni;
E. Nu are risc crescut de infecție bacteriană de șunt.
R: A, D (3, p. 225)

26. Paralizia cerebrală:


A. Tulburări ale funcționării neuronului motor;
B. Factori de risc – prematuritatea, traumatismele la naștere,
convulsiile neonatale, hemoragia intracerebrală;
C. Necesită terapie farmacologică, fizioterapie , intervenție
chirurgicală;
D. Pacienții cu paralizie cerebrală spastică au tonus scăzut la nivelul
membrelor superioare;
E. Reflexele osteotendinoase profunde sunt scăzute.
R: A, B, C (3, p. 226)

27. Retinoblastomul:

251
A. Tumoră benignă a retinei la copil;
B. Anumite cazuri au afectare genetică ce crește riscul de tumoră la
ambii ochi;
C. Mutații ale genei RBI la testarea genetică;
D. Crioterapia și fotocoagularea laser sunt utilizate pentru tumorile
mici;
E. Nu există risc de pierderea vederii dacă tumora este adiacentă
corneei.
R: B, C, D (3, p. 226-227)

28. Displazia de dezvoltare a șoldului:


A. Factori de risc – sexul masculin mai frecvent decât sexul
feminin;
B. Copii născuți în poziție pelviană, oligohidramnios;
C. Manevrele Barlow și Ortolani – pozitive;
D. Pliuri tegumentare simetrice;
E. Dislocarea perinatală a capului femural din acetabul perturbă
dezvoltarea normală a articulației șoldului.
R: B, C, E (3, p. 227)

29. Boala Legg Calve Perthes:


A. Imagistica Rx arată șolduri asimetrice, capul femural afectat este
mic, cu os sclerotic și lărgirea spațiului articular;
B. 50% din cazurile netratate își revin complet;
C. Risc crescut de complicații ale șoldului în perioada de adult
inclusiv osteoartroza;
D. Mers normal;
E. Debutul este acut prin scăderea amplitudinii mișcării.
R: A, B, C (3, p. 228)

30. Boala Osgood Schlatter:


A. Inflamație de tip osteocondrită;
B. Tratament antiinflamatoare nesteroidiene și exerciții de
întindere;
C. Durere care nu este influențată de activitate;
D. Cel mai frecvent la fete în timpul puseului de creștere;
E. Tratament cu cortizon.
R: A, B (3, p. 228)

31. Fractura claviculară:

252
A. Cea mai frecventă fractură la copii;
B. Necesită tratament la nou născuți cu cortizon;
C. Bandaj în formă de 8;
D. Durere în axul clavicular;
E. Netratată poate determina osteoporoza la adulți.
R: A, C, D (3, p. 229)

32. Rahitismul:
A. Calcificarea vicioasă a osului cauzată de aportul , absorbția sau
metabolismul deficitar de vitamina D;
B. Investigații de laborator: FAL scăzută, fosfor crescut;
C. Radiografic- lărgirea cartilajelor de creștere;
D. Tratamentul: suplimentarea cu fosfor, vitamina D, în caz de
aport deficitar;
E. Cartilajul de creștere devine hipotrofic cu calcificare.
R: A, C, D (3, p. 229)

33. Artrita juvenilă idiopatică:


A. Artropatie migratorie;
B. Clasificată ca pauciarticulară, poliarticulară sau sistemică;
C. Laborator: variază cu subtipul;
D. Fără modificări la nivelul articulației implicate;
E. Imagistic: radiografia poate demonstra osteopenia și scleroza
subcondrală.
R: B, C, E (3, p. 229)

34. Distrofia musculară Duchenne:


A. Cea mai frecventă distrofie musculară letală;
B. Debutul bolii între 6 luni și 2 ani;
C. Laborator- CK scăzut;
D. Complicații – scolioză, cardiopatie progresivă;
E. Tratament: fizioterapie, corticosteroizi.
R: A, D, E (3, p. 229-230)

35. Sinovita tranzitorie a soldului:


A. Inflamația maligna a articulației;
B. Precedata de viroza, IACRS;
C. Tratament: managementul durerii cu AINS;
D. Prognostic sever;
E. Laborator: leucopenie, VSH crescut, PCR- normal.

253
R: B, C (3, p. 230)

36. Șoldul septic:


A. De obicei la copii cu vârstă între 10 și 14 ani;
B. Laborator: lichid sinovial < 50.000mm3;
C. Streptococul β hemolitic este cel mai frecvent microorganism
implicat;
D. Imagistică: colecție intraarticulară la ecografie;
E. Tratament: consult ortopedic de urgență, aspirație articulară,
antibiotice.
R: D, E (3, p. 231)

37. Creșterea fizică:


A. Greutatea la naștere se dublează până la 3 luni, se triplează până
la 18 luni;
B. Creșterea ponderală inadecvată poate rezultă din aport dietetic
deficitar;
C. Prevalența obezității copilului (IMC pentru vârstă și sex >
percentila 90);
D. Greutatea sub percentila 5 nu sugerează deficitul de creștere;
E. De la vârsta de 2 ani la 13 ani creșterea anuala a greutății este de
aproximativ 2 kg.
R: B, C, E (3, p. 232)

38. Circumferința capului:


A. Se măsoară în primii 1-5 ani de viață;
B. Macrocefalia se asociază cu boli metabolice cerebrale;
C. Trendul anomaliilor de creștere ajută la sugerarea anumitor
patologii;
D. Rata de creștere scăzută spre normal nu sugerează întârzierea
constituțională a creșterii;
E. Microcefalia nu poate fi asociată cu anomalii anatomice
craniene.
R: B, C (3, p. 231-232)

39. Adolescența:
A. Cuprinde intervalul 10-17 ani;
B. Pubertatea începe cu 12-24 de luni mai devreme la băieți;
C. Modificările fizice sunt clasificate conform stadiilor Tanner;

254
D. Adolescența târzie (17-21 de ani) este caracterizată prin
augmentarea autoconștientizării, o relație mai deschisă cu
părinții;
E. Adolescența timpurie (9-11 ani) se caracterizează prin gândire
corectă și comportament independent precoce.
R: C, D (3, p. 233-236)

40. Abuzul copilului:


A. Copii cu risc crescut includ nou-născuții și copii cu întârziere în
dezvoltare și dizabilități intelectuale;
B. Abuzul copilului nu se suspicionează atunci când anamneza nu
se corelează cu examenul obiectiv;
C. Poate fi fizic, sexual, psihologic, neglijare;
D. Semnele fizice care sugerează trauma neaccindentală includ și
hemoragiile retiniene;
E. Nu necesită spitalizare pentru evaluarea completă sau motive de
siguranță.
R: A, C, D (3, p. 236-237)

41. Deficitul de IgA se caracterizează prin:


A. Se datorează producției anormale de imunoglobuline de către
limfocitele B;
B. Pacienții prezintă incidență crescută a infecțiilor digestive și
respiratorii;
C. Diagnosticul se stabilește prin nivelul scăzut al tuturor
imunoglobulinelor, dar în special IgA;
D. Se asociază cu infecții retiniene și cerebrale;
E. Tratament: imunoglobuline în doză foarte mare.
R: A, B (3, p. 237)

42. Trisomia 18:


A. Incidență 1/6000 nașteri, crește cu vârsta mamei;
B. Retardul mental este ușor;
C. Asociază malfomații cardiace;
D. Anomaliile tractului gastro-intestinal sunt minore;
E. Cavitatea bucală este subdezvoltată.
R: A, C, E (3, p. 240)

43. Scarlatina:

255
A. Febră și rush asociate cu infecția streptococică;
B. Complicații: pneumonia post streptococică;
C. Se va trata cu aminoglicozide, AINS;
D. Laborator: test rapid pentru streptococ, cultură din secreția
faringiană;
E. Complicații: glomerulonefrită poststreptocicică.
R: A, D, E (3, p. 241)

44. Otita medie acută:


A. Infecția urechii externe;
B. Cel mai frecvent motiv de administrare a antibioticelor la copil;
C. Laborator: HLG, imunogramă;
D. Tratamentul: amoxicilină în doză mare;
E. Complicații: Pierderea auzului.
R: B, D, E (3, p. 241)

45. Rujeola:
A. Cauzată de virusul rujeolic;
B. Complicații: gastroenterită;
C. Tratament suportiv (vit A, izolare, notificarea controlului de boli
transmisibile);
D. Laborator: testare PCR;
E. Clinic: erupție veziculoasă, tuse, scaune diareice.
R: A, C, D (3, p. 241-242)

46. Rubeola:
A. Cunoscută și ca „pojarul de 3 zile”;
B. Laborator: testare PCR;
C. Tratament: antibiotice;
D. Începe printr-o pneumonie cu pleurezie;
E. Asociată cu adenopatii posterioare cervicale și occipitale.
R: A, B, E (3, p. 242)

47. Eritemul infecțios (a 5a boală a copilăriei):


A. Cauzat de adenovirusuri;
B. Tratament: antibiotic, imunoglobuline;
C. Compllicații: hidrops fetal;
D. Clinic – febră, aspect de obraji palmuiți;
E. Anemie aplastică este o complicație (bolnavii cu sideremie
scazută - boala celulelor în seceră).

256
R: C, D, E (3, p. 242)

48. Infecția de tract urinar:


A. Prima cea mai frecventă infecție bacteriană la copil;
B. Factori de risc: vârsta <12 luni, T>39grdC, febră>2 zile, sex
feminin;
C. Tratamentul – antibiotic, cefalosporine;
D. Proteus este cel mai comun microorganism implicat;
E. Complicații: pielonefrita, urosepsis.
R: B, C, E (3, p. 242)

49. Tulburarea de hiperactivitate și deficit de atenție se


caracterizează prin:
A. Tulburarea de inatenție și hiperactivitatea la copii de vârsta 6
luni- 2 ani;
B. Factori de risc: mai frecvent la sexul feminin;
C. Hiperactivitatea, impulsivitatea;
D. Tratamentul: antibiotice, antivirale;
E. Diagnosticul necesită prezența a 6 simptome de inatenție sau 6
simptome de hiperactivitate și impulsivitate.
R: C, E (3, p. 242-243)

50. Sindromul Tourette:


A. Patologia cronică a ticurilor;
B. Se asociază frecvent cu ADHD;
C. Tratament: imunoglobuline;
D. Diagnosticul: prezența ticurilor > 6 luni;
E. Ticurile motorii se asociază cu ticuri verbale.
R: A, B, E (3, p. 243)

257
CAPITOLUL 36
AFECȚIUNI GINECOLOGICE ȘI MAMARE

1. Următoarele sunt evenimente pubertare la femeie:


A. Adrenarha (producerea de androgeni de origine adrenală);
B. Gonadarha (activarea gonadelor de către FSH şi LH);
C. Telarha (apariţia ţesutului mamar);
D. Pubarha (apariţia părului pubian);
E. Telarha (activarea gonadelor de către FSH şi LH).
R: A, B, C, D (3, p. 273)

2. Cele mai frecvente cauze de pubertate precoce heterosexuală


la fetițe sunt:
A. Hiperplazia adrenală congenitală (HAC);
B. Expunerea la androgenii de origine exogenă;
C. Tumorile secretante de estrogeni;
D. Sindromul de ovare micropolichistice;
E. Insuficienţa ovariană.
R: A, B (3, p. 274)

3. Gonadotropina corionică umană (hCG):


A. Are acţiune LH-like după implantarea ovocitului fertilizat;
B. Menţine viabilitatea corpului galben şi secreţia de
progesteron;
C. Stimulează dezvoltarea glandelor endometriale;
D. Stimulează proliferarea endometrială;
E. Stimulează dezvoltarea foliculilor ovarieni.
R: A, B (3, p. 275)

4. Semnele și simptomele întâlnite la menopauză sunt:


A. Bufeuri (secundare disfuncţiei termoreglatorii);
B. Dispareunie;
C. Polakiurie, disurie;
D. Hipertrofie vaginală;
E. Tulburări de somn.
R: A, B, C, E (3, p. 277)
5. Următoarele sunt metode hormonale contraceptive:
258
A. Medroxiprogesteronacetat (Depo-Provera);
B. Inel intravaginal;
C. Patch-uri contraceptive transdermice;
D. Sterilet cu eliberare de progestativ;
E. Diafragme sau cupole cervicale.
R: A, B, C (3, p. 277-278)

6. Selectați variantele corecte referitoare la steriletul de cupru:


A. Reprezintă un obiect introdus în uter de către medic, cu
eliberare lentă de progestativ, pentru a preveni fecundaţia şi a
interfera cu transportul spermei;
B. Poate fi plasat ca şi contracepţie de urgenţă imediat după actul
sexual ;
C. Poate fi lăsat pe loc 5 ani;
D. Prezintă risc crescut de avort spontan şi perforaţie uterină;
E. Poate fi lăsat pe loc 10 ani.
R: B, E (3, p. 278)

7. Sindromul Asherman (sinechia uterină):


A. Este definit prin prezenţa de aderenţe intrauterine;
B. Este rezultatul unor proceduri chirurgicale sau al infecţiei
intrauterine;
C. Asociază dureri colicative în abdomenul inferior asociate cu
menstruaţia;
D. Reprezintă un tip de durere uşoară, unilaterală, recurentă, ce
apare la mijlocul ciclului, anterior ovulaţiei;
E. Reprezintă prezenţa de ţesut endometrial în afara cavităţii
uterine (ex. ovare, ligament larg).
R: A, B (3, p. 279)

8. În caz de amenoree, investigațiile de laborator cuprind:


A. Testul β-hCG pentru excluderea sarcinii;
B. Dozarea TSH, LH, FSH şi PRL;
C. Testul la progesteron;
D. Dozarea de FSH şi LH care poate sugera sindromul de ovar
micropolichistic (SOPC);
E. Testul la estro-progestativ.
R: A, B, C, E (3, p. 279)

9. Amenoreea secundară poate avea drept cauze:

259
A. Insuficienţa ovariană;
B. Afecţiuni hipotalamice sau pituitare;
C. Sindrom de ovare micropolichistice;
D. Anomalii cromozomiale cu disgenezii gonadice;
E. Anomalii anatomice (ex. absenţa uterului, sept vaginal).
R: A, B, C (3, p. 279)

10. Care dintre următoarele reprezintă semne și simptome


întâlnite în dismenoree?
A. Dureri colicative în abdomenul superior asociate cu
menstruaţia;
B. Sensibilitate moderată abdominală;
C. Cefalee;
D. Mastodinie;
E. Labilitate emoţională.
R: B, C (3, p. 280)

11. Cauzele cele mai plauzibile ale endometriozei sunt


reprezentate de:
A. Fluxul menstrual retrograd;
B. Diseminarea vasculară/limfatică din uter în cavitatea pelvină;
C. Diseminarea iatrogenă (ex. în timpul secţiunii cezariene);
D. Disfuncţii hipotalamo-hipofizare;
E. Sarcină molară.
R: A, B, C (3, p. 281)

12. Menstruaţiile sunt considerate anormale atunci când:


A. Apar la intervale >35 zile;
B. Durează >7 zile;
C. Pierderea de sânge este >80 mL;
D. Pierderea de sânge este >50 mL;
E. Apar la intervale de 30 de zile.
R: A, B, C (3, p. 281)

13. Sindromul de ovar micropolichistic (SOPC) se caracterizează


prin:
A. Anovulaţie;
B. Hiperandrogenism;
C. Oligoovulație;
D. Flux menstrual retrograd;

260
E. Anomalii cromozomiale cu disgenezii gonadice.
R: A, B, C (3, p. 282)

14. Tratamentul unui cancer endometrial va conţine:


A. Histerectomie parțială abdominală cu salpingo-ooforectomie;
B. Prelevarea de ganglioni limfatici;
C. Radioterapie adjuvantă;
D. Tratament hormonal;
E. Embolizarea arterei uterine.
R: B, C (3, p. 287)

15. Simptomatologia unui cancer de col uterin poate include:


A. Sângerare vaginală (postcoitală sau spontană);
B. Durere pelvină;
C. Secreţie cervicală;
D. Menstruații abundente;
E. Disurie.
R: A, B, C (3, p. 287)

16. Cancerul de col uterin poate fi detectat prin:


A. Frotiul Pap;
B. Biopsia prin „ciupire" a leziunilor vizibile;
C. Biopsia prin conizație;
D. Histeroscopie;
E. Biopsia endometrială.
R: A, B, C (3, p. 287)

17. Tabloul clinic al unei tumori ovariene benigne poate


cuprinde:
A. Dureri în etajul abdominal inferior (mai rar în cazul tumorilor
funcţionale sau torsiunii tumorale);
B. Masă ovariană palpabilă la examenul bimanual;
C. Sensibilitate abdominală;
D. Balonare (numai după creştere semnificativă);
E. Cicluri menstruale neregulate.
R: B, C,D (3, p. 288)

18. CA-125 poate fi crescut în următoarele situații:

261
A. Endometrioză;
B. Leiomiomatoză;
C. Lupus eritematos sistemic;
D. Poliartrită reumatoidă;
E. Granulom inghinal.
R: A, B, C (3, p. 288)

19. Lichenul scleros:


A. Este o afecţiune inflamatorie cronică a regiunii anogenitale la
femeile aflate în perioada pre-menstruală şi în
postmenopauză;
B. Este o leziune precursoare pentru carcinomul vulvar cu celule
scuamoase;
C. Pielea vulvară apare subţire şi ridată precum "hârtia de
ţigaretă";
D. Determină plăci albe poligonale care interesează zona vulvară
şi perianală, cu afectarea
vaginului;
E. Determină leziuni lucioase, reticulare, eritematoase ale
vulvei, cu ulceraţii.
R: A, B, C (3, p. 289)

20. Reprezintă modificări ecografice sugestive pentru


malignitate în cancerul ovarian:
A. Contur neregulat;
B. Septuri multiple;
C. Extensie pelvină;
D. Septuri puţine;
E. Contur regulat.
R: A, B, C (3, p. 289)

21. Vestibulodinia:
A. Determină o simptomatologie strict legată de vestibulul
vulvar;
B. Se manifestă prin durere severă la atingerea vestibulului sau
penetrare, sensibilitate şi eritem;
C. Se va trata cu antidepresive triciclice;
D. Necesită relaxarea muşchilor vaginali prin exerciții Kegel sau
terapie de desensibilizare;

262
E. Este o afecţiune inflamatorie cronică a regiunii anogenitale la
femeile aflate în perioada premenstruală şi în postmenopauză.
R: A, B, C (3, p. 289)

22. Semnele și simptomele întalnite în cadrul unui cancer mamar


sunt:
A. Masă tumorală mamară dureroasă;
B. Masă tumorală palpabilă, solidă, mobilă;
C. Aspect de "coajă de portocală";
D. Retracţie mamelonară;
E. Secreţie mamelonară.
R: C, D, E (3, p. 291)

23. Următoarele afirmații sunt corecte despre papilomul


intraductal:
A. Este o leziune benignă a ţesuturilor ductale cu potenţial
malign;
B. Duce la apariția unei mase tumorale retroareolare
nepalpabile;
C. Poate determina o secreţie mamelonară sanguinolentă;
D. Este cea mai frecventă tumoră mamară benignă (proces
proliferativ ce interesează un sigur duct);
E. Este o formațiune ce variază în dimensiuni pe parcursul
ciclului menstrual.
R: A, C (3, p. 291)

24. Caracteristicile carcinomului mucinos, formă a cancerului de


sân, sunt:
A. Tumoră bine delimitată;
B. Creştere lentă;
C. Creştere rapidă;
D. Prognostic mai bun decât în carcinomul invaziv;
E. Angioinvazie.
R: A, B, D (3, p. 291)

25. Următorii sunt factori de risc pentru apariția cancerului


mamar:
A. Prima sarcină târzie (35 ani);
B. Expunerea la dietilstilbestrol;
C. Nuliparitatea;

263
D. Expunere prelungită la androgeni;
E. Menarha tardivă.
R: A, B, C (3, p. 291)

26. Caracterele specifice unui fibroadenom mamar sunt:


A. Masă tumorală unică, fermă;
B. Margini bine delimitate;
C. Sensibilitate la estrogeni;
D. Masă tumorală palpabilă retroareolară;
E. Masă tumorală imobilă.
R: A, B, C (3, p. 291)

27. Selectați tratamentul corespunzător fiecărei etiologii în cazul


vaginitelor:
A. Ceftriaxonă (Trichomonasvaginalis);
B. Clindamicină (Gardnerellavaginalis);
C. Doxiciclină (Gardnerellavaginalis);
D. Metronidazol (Trichomonas);
E. Fluconazol (Candida albicans).
R: B, D, E (3, p. 283)

28. Sunt semne si simptome de vaginită:


A. Iritație vaginală;
B. Disurie;
C. Inflamația mucoasei vaginale;
D. Prurit;
E. Sângerare după contact sexual.
R: A, C, D (3, p. 282)

29. Următoarele contraceptive prezintă risc crescut de tromboză


venoasă profundă:
A. Patch-urile contraceptive transdermice;
B. Medroxiprogessteron acetat (Depo-Provera);
C. Inelul intravaginal;
D. Contraceptivele orale combinate;
E. Contraceptivele ce conțin doar progestativ.
R: A, C, D (3, p. 277-278)

264
30. Simptomatologia în cazul unei infecții cu HPV poate
cuprinde:
A. Veruci exofitice, conopidiforme pe regiunea genitală;
B. Multiple papule mici, roz, pe zona de contact;
C. Vezicule dureroase, multiple, mici, cu bază eritematoasă;
D. Papule mari în zonele de mucoasă ale corpului;
E. Erupție maculopapulară pe zona de contact.
R: A, B (3, p. 285)

31. Tratamentul în cazul unei infecții vaginale cu Candida


albicans cuprinde:
A. Metronidazol;
B. Fluconazol topic;
C. Clotrimazol topic;
D. Miconazol;
E. Nistatin.
R: C, D, E (3, p. 282)

32. În cazul unei sindrom de ovar micropolichistic (SOPC),


datele de laborator arată:
A. Creșterea LH-ului;
B. Raport LH/FSH < 2;
C. Creșterea DHEA;
D. Test negativ la administrarea de progesteron;
E. Creșterea testosteronului total.
R: A, C, E (3, p. 282)

33. Vaccinarea anti-HPV este posibilă pentru următoarele


tipuri:
A. 6;
B. 18;
C. 16;
D. 11;
E. 26.
R: A, B, C, D (3, p. 285)

34. Factorii de risc pentru endometrioză sunt:


A. Nuliparitatea;
B. Indicele scăzut de masă corporală;
C. Menarha<12 ani;

265
D. Vârstă tânără la primul contact sexual;
E. Infertilitatea.
R: A, B, E (3, p. 281)

35. Efectele secundare inserării inelului intravaginal sunt:


A. Risc crescut de infecții de tract urinar;
B. Creștere ponderală;
C. Cefalee;
D. Disconfort legat de dispozitiv;
E. Sângerare de privație.
R: C, D, E (3, p. 278)

36. Care dintre următoarele variante sunt considerate corecte


despre cancerul endometrial:
A. Markerul tumoral CA-125 este specific pentru cancerul
endometrial;
B. Este un adenocarcinom al endometrului, cel mai frecvent
legat de expunerea la niveluri mari de estrogeni;
C. Se întâlnește, cel mai frecvent, la femeile aflate in
postmenopauză;
D. La examenul obiectiv, uterul este, de obicei, dureros sau se
pot palpa uterul sau ovarele fixate;
E. Cancerul de endometru, care nu se datorează expunerii în
exces la estrogeni endogeni sau exogeni, are un prognostic
mai bun decât tumorile estrogen dependente.
R: B, C (3, p. 286-287)

37. Semnele si simptomele de cervicite includ, mai puțin:


A. Sensibilitate la mobilizarea colului uterin;
B. Dispareunie;
C. Dureri la nivelul abdomenului inferior;
D. Sângerare după contact sexual;
E. Secreție vaginală purulentă (mai ușoară în infecțiile cu
Neisseriagonorrhoeae).
R: A, C, E (3, p. 283)

266
38. Semnele și simptomele ce pot fi întâlnite în cazul unei tumori
ovariene benigne sunt, cu excepția:
A. Cicluri menstruale neregulate;
B. Scădere în greutate;
C. Balonare;
D. Vărsături;
E. Dureri în etajul abdominal inferior.
R: A, B (3, p. 288)

39. Următoarele modificări sunt cauzate de adenomioză:


A. Durere pelviană ciclică;
B. Creștere asimetrică a uterului;
C. Sindrom metabolic;
D. Sensibilitate uterină;
E. Menstruații abundente.
R: A, D, E (3, p. 281)

267
CAPITOLUL 37
OBSTETRICĂ

1. Referitor la vârstă gestațională, sunt adevărate următoarele


afirmații:
A. Aceasta se poate calcula în funcție de data ultimei menstruații
a mamei;
B. Vârsta gestațională este cu 2 săptămâni mai mare decât vârstă
embrionară;
C. Vârsta gestațională este cu 4 săptămâni mai mare decât vârstă
embrionară;
D. Regula lui Naegel poate fi folosită pentru a estima data
nașterii, pornind de la data ultimei menstruații;
E. Dezvoltarea embrionară și fetală durează aproximativ 42 de
săptămâni.
R: A, B, D (3, p. 295)

2. Următoarele afirmații sunt adevărate în legătură cu


pseudocieza:
A. Mai poate fi numită și sarcină falsă;
B. Au loc modificări ale axei hipotalamo-hipofizare-ovariene;
C. Se produce somatizarea stresului;
D. Persoanele cu această afecțiune pot fi psihotice;
E. Se caracterizează prin semne și simptome de sarcină la femeile
care nu sunt gravide.
R: A, B, C, E (3, p. 295)

3. Pe perioada sarcinii, sunt considerate modificări fiziologice ale


aparatului gastrointestinal următoarele, cu excepția:
A. Sialoree;
B. Motilitate gastrică scăzută;
C. Tranzit intestinal accelerat;
D. Diaree;
E. Disfagie.
R: C, D, E (3, p. 296)

268
4. Evaluarea vârstei gestaţionale în al doilea trimestru se va
realiza prin:
A. Circumferinţa abdominală fetală;
B. Diametrul biparietal;
C. Lungimea femurului;
D. Circumferinţa craniană;
E. Lungimimea cranio-caudală.
R: A, B, C, D (3, p. 296)

5. Efectele insuficienţei de calciu pe perioada sarcinii sunt:


A. Mineralizarea anormală a oaselor materne;
B. Hipertensiune;
C. Naștere prematură, greutate scăzută la naștere;
D. Afectarea creșterii fetale şi placentare;
E. Defect de tub neural.
R: A, B, C (3, p. 297)

6. Creșterea exagerată în greutate pe perioada sarcinii poate


determina următoarele:
A. Diabet gestațional;
B. Macrosomie fetală;
C. Creșterea frecvenței nașterii prin cezariană;
D. Restricție de creștere fetală;
E. Naștere prematură.
R: A, B, C (3, p. 297)

7. Următoarele rezultate sunt concludente pentru trisomia 21


(sindromul Down) în cadrul testului complet integrat:
A. PAPP-A scăzută;
B. PAPP-A crescută;
C. Translucența nucală crescută;
D. Translucența nucală scăzută;
E. α -fetoproteina crescută.
R: A, C (3, p. 299)

8. Complicațiile fetale ale diabetului zaharat gestațional sunt:


A. Naștere traumatică;
B. Dezvoltare neurologică întârziată;
C. Insuficiență utero-placentară:
D. Maturare pulmonară întârziată;

269
E. Polihidramnios.
R: C, D, E (3, p. 299)

9. Sindromul HELLP se caracterizează prin:


A. Hemoliză;
B. Enzime hepatice crescute;
C. Trombocitopenie;
D. Trombocitoză;
E. Coagulare intravasculară diseminată (CID).
R: A, B, C (3, p. 301)

10. Hyperemesis gravidarum:


A. Nu necesită spitalizare, fiind în mod obișnuit tratat în
ambulatoriu;
B. Constă în greață și vărsături severe, care afectează 50% dintre
gravide;
C. Poate fi complicat de dezechilibre hidro-electrolitice, scădere
în greutate și cetonurie;
D. Este tratat prin spitalizare, administrare de fluide și
antiemetice;
E. Poate determina apariția crizelor convulsive materne, putând fi
sever și fatal.
R: C, D (3, p. 302)

11. Despre tromboza venoasă profundă (TVP) maternă este


adevărat:
A. Riscul de tromboză venoasă profundă crește în timpul sarcinii
din cauza stazei venoase și a creșterii relative a factorilor de
coagulare circulanți;
B. Diagnosticul clinic este mai facil la pacientele gravide;
C. Terapia cu heparină se poate complica cu hemoragie sau
trombocitopenie;
D. Warfarina se poate utiliza în siguranță în perioada sarcinii;
E. Anticoagulantele trebuie continuate după naștere timp de 6
săptămâni.
R: A, C, E (3, p. 302)

270
12. Cauzele pentru care infecțiile tractului urinar sunt mai
frecvente în timpul sarcinii sunt:
A. Imunosupresia maternă;
B. Scăderea peristaltismului ureteral (secundar creșterii
progesteronului);
C. Creșterea peristaltismului ureteral;
D. Obstrucția căilor urinare;
E. Volumul crescut de sânge.
R: A, B, D (3, p. 302)

13. În cazul unei infecții materne de tract urinar se pot administra


următoarele antibiotice:
A. Ciprofloxacin;
B. Amoxicilină;
C. Nitrofurantoină;
D. Cefalexin;
E. Ceftriaxonă.
R: B, C, D (3, p. 302)

14. Tutunul determină următoarele riscuri asupra fătului:


A. Deces fetal intrauterin;
B. Dezvoltare intelectuală afectată;
C. Defecte cardiace congenitale;
D. Risc crescut de infecții respiratorii neonatale;
E. Palatoschizis.
R: A, B, D (3, p. 303)

15. Următoarele afirmații sunt adevărate despre rujeolă:


A. Determină risc crescut de anemie neonatală;
B. Diagnosticul clinic la mamă este confirmat prin anticorpii IgM
sau IgG după dezvoltarea erupției cutanate;
C. Mama trebuie imunizată înainte de a încerca să rămână
însărcinată;
D. Imunogobulinele serice pot fi administrate mamei în caz de
infecție în timpul sarcinii;
E. Vaccinul este indicat în timpul sarcinii.
R: B, C, D (3, p. 304)

271
16. Diagnosticul sifilisului matern se va face prin:
A. RPR sau VDRL prenatal precoce;
B. Confirmare prin FTA-ABS;
C. Screening pentru IgG realizat precoce prenatal;
D. Cultură cervicală și analize imunologice;
E. Analiză PCR din lichidul amniotic.
R: A, B (3, p. 304)

17. Cele mai frecvente cauze ale sângerării vaginale în sarcina


precoce sunt:
A. Amenințarea sau iminența de avort spontan;
B. Patologia cervico-vaginală;
C. Sângerarea fiziologică – legată de implantare;
D. Sarcina ectopică;
E. Boala inflamatorie pelvină.
R: A, C, D (3, p. 305)

18. Următoarele afirmații sunt corecte referitor la sarcina


ectopică:
A. Reprezintă implantarea zigotului în afara uterului;
B. În caz de ruptură, durerea abdominală devine severă şi poate fi
însoțită de hipertensiune, tahicardie și semne de iritație
peritoneală;
C. B-hCG care este crescut comparativ cu valoarea estimată
pentru vârsta de sarcină ar trebui să ridice suspiciunea de
sarcină ectopică;
D. Ultrasonografia poate arăta lichid intraabdominal liber dacă s-a
produs ruptura;
E. Oricărei femei de vârstă fertilă care prezintă dureri abdominale
trebuie să i se facă un test de sarcină.
R: A, D, E (3, p. 305)

19. În ceea ce priveşte sevrajul narcotic dat de consumul de


droguri recreaţionale şi riscurile materno-fetale asociate în
timpul sarcinii:
A. Este un risc dat de administrarea de opioide;
B. Poate fi fatal pentru făt;
C. Este un risc dat de administrarea de cocaină;

272
D. Poate fi fatal pentru mamă;
E. Este un risc dat de administrarea de halucinogene.
R: A, B (3, p. 303)

20. Scorul Bishop măsoară:


A. Dilataţia (cm);
B. Poziţia colului;
C. Consistenţa colului;
D. Scurtarea şi ştergerea colului (%);
E. Frecvenţa cardiacă fetală.
R: A, B, C, D (3, p. 313)

21. Ce cuprinde tratamentul pentru o sarcină oprită în evoluție?


A. Misoprostol;
B. Dilatație și chiuretaj;
C. Administrarea de imunoglobulină anti D;
D. Repaus la pat, limitarea activității;
E. Intervenție chirurgicală de urgenţă.
R: A, B, C (3, p. 306)

22. Următoarele afirmații sunt adevărate despre restricția de


creştere intrauterină simetrică:
A. Apare în 80% dintre cazuri;
B. Determina scădere globală în dimensiuni a fătului;
C. Determina reducerea dimensiunilor abdomenului, cu păstrarea
dimensiunilor craniului și membrelor;
D. Apare tardiv în cursul sarcinii;
E. Apare devreme în sarcină.
R: B, E (3, p. 306)

23. Cele mai frecvente cauze de restricția de creştere intrauterină


asimetrică sunt:
A. Sănătatea precară a mamei;
B. Anomaliile cromozomiale;
C. Consumul matern de droguri;
D. Sarcina multiplă;
E. Insuficienţa uteroplacentară.
R: A, D, E (3, p. 306)

273
24. Oligoamniosul se asociază cu:
A. Insuficienţă placentară;
B. Restricție de creştere intrauterină;
C. Anomalii cardiace ale fătului;
D. HTA;
E. Suferinţă fetală.
R: A, B, D, E (3, p. 307)

25. Despre hidramnios, se poate afirma:


A. Este un exces de lichid amniotic în sacul gestațional (indexul
de lichid amniotic >25 cm);
B. Poate apărea datorită scăderii diurezei fătului datorită
diabetului matern, sarcinilor multiple, anemiei fetală
sauanomaliilor cromozomiale;
C. Sarcinile >32 săptămâni pot fi supuse doar amnioreducției;
D. Sarcinile >32 săptămâni necesită administrarea de indometacin
(doze crescute treptat);
E. Înălțimea fundului uterin care poate fi crescută în raport cu
vârsta gestațională.
R: A, C, E (3, p. 307)

26. Complicațiile unui hidramnios pot fi:


A. Naşterea înainte de termen;
B. Ruptura prematură de membrane – PROM;
C. Avortul spontan;
D. Distociile de prezentație;
E. Alterarea respirației materne.
R: A, B, D, E (3, p. 307)

27. Scorul Bishop oferă 2 puncte pentru:


A. Dilatație: 5-6 cm;
B. Poziția colului: anterior;
C. Stația: +1, +2 cm;
D. Scurtarea și ștergerea colului: 60-70%;
E. Consistența colului este fermă.
R: B, D (3, p. 313)

274
28. Infecția maternă cu Parvovirusul B19 are următoarele efecte
asupra fătului/nou-născutului, cu excepția:
A. Cecitate;
B. Meningită;
C. Anemie hemolitică;
D. Creșterea producției eritrocitare;
E. Hidrops fetal.
R: A, B, D (3, p. 304)

29. În săptămâna 24 de sarcină :


A. SNC este matur;
B. Începe producția de surfactant;
C. Unghiile sunt prezente;
D. Fetusul ajunge la termen;
E. Se regăsesc primele șanse de supraviețuire în caz de naștere
prematură.
R: B, C, E (3, p. 295)

30. Alegeți afirmațiile adevărate referitoare la evaluarea vârstei


gestaționale după 20 de săptămâni:
A. Obezitatea maternă afectează precizia măsurătorii pentru
evaluarea vârstei gestaționale;
B. Fibroamele uterine afectează precizia măsurătorii pentru
evaluarea vârstei gestaționale;
C. Înălțimea fundului uterin este utilizată pentru evaluarea vârstei
gestaționale;
D. Diametrul biparietal este utilizat pentru estimarea vârstei
gestaționale;
E. Circumferința abdominală este utilizată pentru evaluarea
vârstei gestaționale.
R: A, B, C (3, p. 296)

31. Complicațiile perinatale sau postnatale ale diabetului zaharat


gestațional sunt:
A. Sindrom de detresă respiratorie fetală;
B. Insuficiență utero-placentară;
C. Hipercalcemie;
D. Hipoglicemie;
E. Dezvoltare neurologică întârziată.
R: A, D, E (3, p. 209)

275
32. Factorii de risc pentru ruptura prematură de membrane
(PROM) sunt:
A. Nutriția maternă precară;
B. Distociilede prezentație;
C. Infecția urinară;
D. Incontinența cervico-istmică;
E. Ruptura prematură de membrane în sarcinile anterioare.
R: A, D, E (3, p. 308)

33. Următoarele afirmații sunt adevărate despre citomegalovirus:


A. Ganciclovirul poate reduce efectele la nou-născuți;
B. Vaccinul este indicat în timpul sarcinii;
C. Igiena corespunzătoare reduce riscul de transmitere;
D. Mama trebuie imunizată înainte de a încerca să rămână
însărcinată;
E. Determină risc crescut de anemie neonatală.
R: A, C (3, p. 304)

34. Despre operația cezariană transversală joasă, este adevărat:


A. Există risc redus de ruptură uterină, de sângerare, de apariție a
aderențelor la intestin sau de infecție;
B. Este utilizată atunci când fătul este situat în prezentație
transversală;
C. Se practică o incizie transversală a peretelui muscular anterior
al uterului;
D. Este de preferat inciziei clasice și este efectuată mai rar decât
aceasta;
E. Se practică o incizie transversală la nivelul segmentului
inferior uterin.
R: A, E (3, p. 314)

35. Printre factorii de risc care determină un travaliu prematur se


numără:
A. Sarcini multiple;
B. Vârstă > 35 de ani;
C. Oligohidramnios;
D. Placenta previa;
E. Dezlipire prematură de placentă normal inserată.
R: A, D, E (3, p. 308)

276
36. În cadrul profilului biofizic, evaluarea ultrasonografică constă
în măsurarea:
A. Indicelui de lichid amniotic;
B. Frecvenței respiratorii fetale;
C. Flexia coloanei vertebrale;
D. Frecvenței cardiace fetale;
E. Mișcărilor fetale.
R: A, B, E (3, p. 311)

37. Selectați modificările materne ce au loc în perioada de lăuzie


fiziologică:
A. În cazul mamelor care alăptează, menstruația apare la 6-8
săptămâni postoperator;
B. Uterul scade în dimensiuni și consistența colului crește în
primele 3 săptămâni postpartum;
C. Pereții vaginului își recapătă progresiv fermitatea;
D. Secrețiile uterine (lohiile) sunt roșii în primele zile după
naștere, dar devin progresiv mai închise la culoare;
E. Rezistența vasculară periferică totală scade rapid datorită
eliminării circulației uteroplacentare.
R: B, C (3, p. 315)

38. Un scor Bishop crescut se va asocia cu:


A. Poziția cervicală anterioară și o stație fetală avansată;
B. Ștergerea și dilatația colului uterin, înmuierea acestuia;
C. O posibilitate crescută de naștere pe cale vaginală;
D. Suferință fetală acută;
E. O rată de efectuare a operației de cezariană de 30%.
R: A, B, C (3, p. 312)

277
39. Tratamentul hemoragiei postpartum cuprinde:
A. Dilatarea si chiuretajul uterin ( D&C) nu elimină cu succes
resturile placentare;
B. Dacă inversiunea uterină este cauza hemoragiei, este
recomandată repoziționarea manuală a uterului;
C. Masajul uterului si administrarea de oxitocină pentru a ajuta la
creșterea tonusului uterin și la scăderea intensității hemoragiei;
D. În caz de hipertensiune arterială, se poate administra
metilergometrină;
E. În cazurile severe sau refractare la tratament, poate fi necesară
histerectomia.
R: B, C, E (3, p. 315)

278
CAPITOLUL 38
TULBURĂRI PSIHICE

1. Sevrajul în cazul amfetaminelor determină:


A. Creșterea apetitului alimentar;
B. Depresie;
C. Convulsii;
D. Anxietate;
E. Oboseală.
R: A, B, D, E (3, p. 329)

2. Tulburarea de personalitate de tip paranoid presupune:


A. Acțiunile celorlați sunt consecvent interpretate ca vătămătoare sau
înșelătoare;
B. Frecvent are reacții de furie;
C. Frecvent interpretează greșit comentariile;
D. Neîncredere persistentă și suspiciune față de alții;
E. Tratament cu antipsihotice în doze mari.
R: A, B, C, D (3, p. 328)

3. Medicamentele antidepresive de tip modulatori ai serotoninei


sunt:
A. Trazodonă;
B. Bupropion;
C. Mirtazapină;
D. Nefazodonă;
E. Vilazodonă.
R: A, D, E (3, p. 322)

4. Medicamentele antidepresive de tip IMAO sunt:


A. Isocarboxazid;
B. Imipramină;
C. Selegilină;
D. Tranilcipromină;
E. Nortriptilină.
R: A, C, D (3, p. 322)

279
5. În clasa SSRI a medicamentelor antidepresive regăsim:
A. Duloxetină;
B. Desvenlafaxină;
C. Venlafaxină;
D. Paroxetină;
E. Sertralină.
R: D, E (3, p. 322)

6. Medicamentele antidepresive de tip ADT:


A. Sunt următoarele: citalopram, escitalopram, fluoxetină,
fluvoxamină, paroxetină, sertralină;
B. Inhibă recaptarea noradrenalinei;
C. Tratament de linia a2-a sau a3-a în depresie;
D. Supradozarea lor la de 5 ori doza terapeutică poate fi fatală;
E. Blochează receptorii alfa2 și receptorii serotoninei, crescând
neutrosmisia noradrenergică.
R: B, C, D (3, p. 322)

7. Referitor la schizofrenie putem afirma următoarele:


A. Psihoză severă care produce limitări semnificative în capacitatea
de funcționare;
B. Diagnosticul necesită prezența a două sau mai multe simptome
pentru cel puțin o lună de zile dintr-un interval de 6 luni și
afectarea funcționării sociale mai mult de 6 luni;
C. Neurolepticele cu potență ridicată au crescute efecte adverse
colinergice;
D. Neurolepticele cu potență redusă sunt frecvent medicație de primă
linie;
E. Antipsihpoticele reprezintă baza tratamentului.
R: A, B, E (3, p. 319)

8. În grupul tulburărilor anxioase regăsim:


A. Tulburarea psihotică scurtă;
B. Tulburarea de anxietate socială;
C. Fobia specifică;
D. Tulburarea dismorfică corporală;
E. Tulburarea de adaptare.
R: B, C (3, p. 324)

9. Printre tulburările anxioase se numără:

280
A. Tulburarea anxietății generalizate;
B. Anorexia nervoasă;
C. Tulburarea conversivă;
D. Tulburarea nasofobică;
E. Tulburarea de panică.
R: A, E (3, p.323)

10. Tulburările dispoziției afective sunt următoarele:


A. Tulburarea depresivă majoră;
B. Tulburarea afectivă bipolară;
C. Tulburarea ciclotimică;
D. Tulburarea depresivă persistentă;
E. Tulburarea de acumulare.
R: A, B, C, D (3, p. 321, 323)

11. Printre antipsihoticele atipice se enumeră:


A. Aripiprazol;
B. Olanzapina;
C. Risperidona;
D. Haloperidol;
E. Clorpromazina.
R: A, B, C (3, p. 319)

12. Sindromul neuroleptic malign se caracterizează prin:


A. Rigiditate de tip plastic;
B. Reflexe diminuate;
C. Pupile normale;
D. Tensiunea arteriala crescută;
E. Piele uscată.
R: A, B, D (3, p. 320)

13. Tulburarea delirantă se caracterizează prin:


A. Psihoză caracterizată prin prezența doar a unui episod de delir;
B. Psihoză caracterizată prin prezența unuia sau mai multor deliruri
cu o durată de cel puțin 6 luni;
C. Psihoză caracterizată prin prezența unuia sau mai multor deliruri
cu o durată de cel puțin o lună;

D. Nu întrunește criterile pentru schizofrenie;

281
E. Nu există o afectare a funcționalității, cu excepția consecințelor
delirului.
R: C, D, E (3, p. 320)

14. Tulburarea depresivă majoră se caracterizează prin:


A. Durează mai mult de 2 săptămâni și afectează funcționarea socială
și/sau ocupațională;
B. Nu poate fi atribuită unei substanțe psihoactive sau unei condiții
medicale;
C. După remisiune, epsioadele depresive au o recurență de doar 1%;
D. Din punct de vedere patologic, depresia este cauza unei activități
scăzute a serotoninei, noradrenalinei și dopaminei;
E. Diagnosticul presupune dispoziție depresivă pe o durată mai mică
de 2 săptămâni.
R: A, B, D (3, p. 321)

15. Caracteristicile tulburării depresive majore cu simptome atipice


sunt:
A. Reactivitate afectivă;
B. Hiperfagie;
C. Hipersomnie;
D. Dispoziție depresivă;
E. Hipersensibilitate la rejecție.
R: A, B, C, E (3, p. 321)

16. Episodul maniacal se definește ca:


A. Dispoziție afectivă elevată, expansivă sau iritabilă cu o durată de
cel puțin o săptămână;
B. Trei sau mai multe din următoarele simptome: grandomanie,
logoree, necesar redus de somn, fugă de idei, hiperactivitate cu un
scop anume;
C. Episoadele nu produc o afectare semnificativă a capacității de
funcționare;
D. Anhedonie;
E. Sentimentul de vinovăție/ lipsă de valoare.
R: A, B (3, p. 322)

282
17. Printre simptomele schizofreniei se numără:
A. Lipsa voinței;
B. Sărăcirea vorbirii;
C. Comportament organizat;
D. Deliruri (percepție senzorială în absența stimulilor externi);
E. Halucinații (convingeri false despre sine și alte persoane).
R: A, B (3, p. 319)

18. Despre sindromul serotoninergic puterm afirma:


A. Zgomotele intestinale sunt normale;
B. Pupilele sunt midriatice;
C. Are o rezoluție rapidă;
D. Reflexele sunt diminuate;
E. Pielea este uscată.
R: A, C (3, p. 320)

19. Substanțele psihoactive care pot produce simptome depresive


sunt, cu excepția:
A. Beta blocantele;
B. Antihistaminicele;
C. Alcool;
D. Nifedipina;
E. Interferon alfa.
R: A, D (3, p. 321)

20. Despre atacurile de panică nu este adevărat:


A. Pot asocia palpitații și transpirații;
B. Asociază frica de responsabilitate;
C. Durează maxim 30 de minute;
D. Pot apărea parestezii;
E. Nu se poate întâlni dispneea.
R: B, E (3, p. 323)

21. Printre efectele adverse anticolinergice ale medicației


antipsihotice se numără:
A. Distonie;
B. Confuzie;
C. Constipație;

D. Akinezie;

283
E. Hipotensiune.
R: B, C, E (3, p. 319)

22. Controlul de urgență al psihozei sau al agitației la pacientul


schizofrenic se face cu:
A. Flufenazină;
B. Clorpromazină;
C. Loxapină;
D. Tiotixen;
E. Haloperidol.
R: A, C, D, E (3, p. 319)

23. Despre medicația antipsihotică din schizofrenie putem afirma


următoarele:
A. Neurolepticele cu potență redusă sunt medicamente de primă
linie;
B. Antipsihoticele atipice sunt medicația de primă linie;
C. Medicamentele de primă linie blochează receptorii dopaminergici
D2;
D. Medicamentele de primă linie blochează receptorii dopaminei și
serotoninei;
E. Neurolepticele cu potență ridicată sunt medicamente de primă
linie.
R: B, D (3, p. 319)

24. Simptomele negative regăsite în diagnosticul schizofreniei sunt


următoarele:
A. Apatie;
B. Retracție socială;
C. Lipsa voinței;
D. Aplatizare afectivă;
E. Tulburări de somn.
R: A, B, C, D (3, p. 319)

25. Fobia socială se caracterizează prin:


A. Frica excesivă sau anxietate în legătură cu situațiile sociale;
B. Frica este proporțională cu amenințarea reală pe care situația
socială o reprezintă;
C. Situațiile sociale produc anxietate care poate fi ușoară sau severă;

284
D. Frica de un obiect, activitate sau situație anume ce apare din
copilărie;
E. Beta blocantele pot fi folosite pentru a preveni tahicardia.
R: A, C, E (3, p. 324)

26. Tulburarea obsesiv-convulsivă se caracterizează prin


următoarele, cu excepția:
A. Prezența obsesiilor și/sau compulsiilor, ce afectează
funcționalitatea și viața cotidiană;
B. Obsesiile sunt comportamente repetitive sau acte mentale, care au
ca scop diminuarea sau prevenirea anxietății;
C. Compulsiile sunt gânduri sau impsulsiuni recurente, persistente,
intruzive și nedorite, care produc anxietate sau disconfort;
D. Diagnosticul necesită prezența obsesiilor și compulsiilor, care
afectează semnificativ viața de zi cu zi;
E. Evenimentele stresante pot exacerba comportamentele
compulsive.
R: B, C (3, p. 324)

27. Tulburarea acută de stres prezintă următoarele caracteristici:


A. Este o tulburare asociată traumei sau factorilor de stres;
B. Debutează în prima lună după evenimentul declanșator;
C. Se remite în maxim o lună;
D. Debutează în primele 3 luni de la evenimentul declanșator;
E. Se remite în decurs de 6 luni.
R: A, B, C (3, p. 325)

28. Tulburarea de adaptare se definește prin următoarele afirmații,


cu excepția:
A. Simptome comportamentale și emoționale, ca răspuns la anumite
evenimente sau situații stresante;
B. Apare în primele 2 luni de la eveniment și produc o afectare
semnificativă în capacitatea de funcționare;
C. Disconfort excesiv în raport cu cel așteptat, după un eveniment
stresant;
D. Simptomele încetează la cel puțin un an de la eveniment;
E. Antidepresivele sau antipsihoticele pot fi folosite ca terapie
medicamentoasă.
R: B, D, E (3, p. 325)

285
29. Tulburarea cu simptome somatice prezintă:
A. Unul sau mai multe simptome somatice care pot să nu fie datoare
unei condiții medicale identificate;
B. Simptome dureroase;
C. Simptome neurologice;
D. Simptome gastrointestinale;
E. Pacienții refuză să consulte un medic pentru a primi un
diagnostic.
R: A, B, C, D (3, p. 326)

30. Alegeți din următoarele, medicația antipsihotică care utilizează


drept mecanism blocarea receptorilor dopaminei și serotoninei:
A. Tioridazina;
B. Clorpromazina;
C. Loxapina;
D. Ziprasidona;
E. Olanzapina.
R: D, E (3, p. 319)

31. Alegeți din următoarele, caracteristicile supradozării de


anticolinergice:
A. Pupilele sunt midriatice;
B. Tratmanentul farmacologic adjuvant cuprinde Fizostigmina;
C. Zgomotele intestinale sunt diminuate-ileus;
D. Reflexele sunt diminuate;
E. Debutul este progresiv.
R: A, B, C (3, p. 320)

32. Alegeți din următoarele, tratamentul adjuvant farmacologic în


hipertermia malignă:
A. Ciproheptadina;
B. Amantadina;
C. Dantrolen;
D. Fizostigmina;
E. Azumolene.
R: C, E (3, p. 320)

286
33. Rigiditatea de tip plastic ”țeavă de plumb” nu se întâlnește în:
A. Sindromul neuroleptic malign;
B. Supradoza de anticolinergice;
C. Sindromul serotoninergic;
D. Sindromul toxic anticolinergic;
E. Hipertermia malignă.
R: B, C, D, E (3, p. 320)

34. Alegeți afirmațiile adevărate cu privire la tulburarea depresivă


majoră:
A. Cuprinde TDM cu debut peripartum;
B. Cuprinde TDM cu ritmicitate sezonieră;
C. Cauzează frecvent psihoză;
D. Cuprinde TDM cu simptome atipice;
E. Pacientul nu este niciodată psihotic în afara episoadelor afective.
R: A, B, D, E (3, p. 321)

35. Alegeți afirmațiile adevărate cu privire la tulburarea depresivă


persistentă:
A. Poate include depresie ușoară cronică, ce întrunește criteriile de
tulburare depresivă majoră;
B. Poate include depresia majoră cronică;
C. Diagnosticul necesită prezența disforiei;
D. Cuprinde episoade ciclice de depresie și manie (sau hipomanie);
E. Cuprinde simptome depresive prezente în majoritatea zilelor pe o
perioadă mai mare de 2 ani.
R: B, C, E (3, p. 321)

36. Intoxicația cu halucinogene cuprinde următoarele simptome:


A. Halucinații;
B. Deliruri, anxietate;
C. Euforie;
D. Nistagmus;
E. Mioză.
R: A, B (3, p. 330)

37. Tratamentul în abuzul de nicotină cuprinde:


A. Naloxona;
B. Metadona;
C. Vareniciclina;

287
D. Nicotina transdermică sau masticabilă;
E. Hipnoza.
R: C, D, E (3, p. 330)

38. Sevrajul în urma abuzului de feniciclidină cuprinde următoarea


simptomatologie:
A. Crampe stomacale;
B. Nivel variabil de vigilitate;
C. Diaree;
D. Comportament violent;
E. Mialgii.
R: B, D (3, p. 330)

39. Sindromul de realimentare la pacienții cu anorexie severă care


își reiau alimentația cuprinde:
A. Hipofosfatemie;
B. Hipermagnezemie;
C. Hipercalcemie;
D. Hipocalcemie;
E. Hipomagnezemie.
R: A, D, E (3, p. 327)

40. Anorexia nervoasă se caracterizează prin:


A. Imagine corporală distorsionată;
B. Frica intensă de a se îngrășa;
C. Aport caloric redus;
D. Alimentare compulsivă;
E. Episoade de alimentație compulsivă și compensare ulterioară,
apar cel puțin o dată pe săptămână, pe o durată mai mare de trei
luni.
R: A, B, C (3, p.327)

41. Din grupul A de tulburări de personalitate fac parte:


A. Tipul paranoid;
B. Tipul schizoid;
C. Tipul bordeline;
D. Tipul depedendent;
E. Tipul schizotipal.
R: A, B, E (3, p. 328)

288
42. În cadrul tulburării de personalitate de tip schizoid se regăsesc:
A. Neîncredere persistentă și suspiciune față de alții;
B. Incapacitatea de a lega relații apropiate;
C. Detașare socială;
D. Relații instabile, sentiment de gol interior;
E. Răceală afectivă, tocire afectivă.
R: B, C, E (3, p. 328)

43. Tipul de personalitate bordeline se caracterizează prin:


A. Relații instabile, sentimentul de gol interior;
B. Frica de abandon;
C. Ideație suicidală;
D. Iritabilitate neadecvată;
E. Vorbire dezorganizată, convingeri bizare.
R: A, B, C, D (3, p. 328)

44. Tipul de personalitate schizotipal prezintă următoarele


caracteristici, cu excepția:
A. Comportament excentric, paranoia;
B. Anxietate socială;
C. Ideație suicidală;
D. Comportament agresiv față de oameni și animale;
E. Convingeri bizare.
R: C, D (3, p. 328)

45. Din grupul C al tulburărilor de personalitate fac parte:


A. Tipul Evitant;
B. Tipul Narcisist;
C. Tipul Depedent;
D. Tipul Schizotipal;
E. Tipul Obsesiv-compulsiv.
R: A, D, E (3, p. 328)

46. Tipul de personalitate histrionic este descris prin următoarele,


cu excepția:
A. Grandiozitate, fantezii despre succes;
B. Caută atenția, comportament teatral sau seducător;
C. Comportament agresiv față de oameni și animale;
D. Frică de abandon;
E. Lipsă de empatie, invidie față de ceilalți.

289
R:A, C, D, E (3, p. 328)

47. Tulburările de personalitate se definesc prin:


A. Model persistent de trăire interioară și comportament, care
deviază semnificativ de la normele culturale;
B. Este persistent;
C. Este flexibil;
D. Duce la afectarea capacității de funcționare;
E. În mod tipic, apare încă din copilărie.
R: A, B, D (3, p. 328)

48. Din grupul C de tulburări de personalitate nu fac parte:


A. Tipul schizoid;
B. Tipul dependent;
C. Tipul evitant;
D. Tipul narcisist;
E. Tipul paranoid.
R: D, E (3, p. 328)

49. Printre complicațiile utilizării cronice de marijuana se numără:


A. Psihoză;
B. Depresie;
C. Infertilitate;
D. Sindromul amotivațional;
E. Infecții parenterale.
R: A, B, C, D (3, p. 330)

50. Alegeți din următoarele, tratamentul în tulburarea de tip


schizoid:
A. Psihoterapia suportivă orientată pe realizarea de relații
confortabile cu ceilalți;
B. Doze mici de anxiolitice;
C. Psihoterapie centrată pe acceptarea deficiențelor;
D. Psihoterapie centrată pe dezvoltarea abilităților de relaționale
socială și a luării deciziilor;
E. Antipsihotice, inițial pentru a corecta comportamentul.
R: A, E (3, p. 328)

290
51. Alegeți afirmațiile adevărate referitoare la tulburările de
personalitate:
A. Deobicei apar la copii sub 5 ani;
B. Duc la afectarea capacității de funcționare;
C. Sunt un model persistent si inflexbil;
D. Reprezintă un model persistent de trăire interioară și
comportmanet, care nu deviază de la normele culturale;
E. Nu pot fi atribuite consumului de substanțe sau unei condiții
medicale.
R: B, C, E (3, p. 328)

52. Alegeți afirmațiile adevărate referitoare la bulimia nervoasă:


A. Reprezintă o tulburare de comportament alimentar, caracterizată
prin alimentare compulsivă;
B. Tratamentul cuprinde consilierea alimentară;
C. Medicația aprobată de FDA este reprezentată de Fluoxetină;
D. Terapia farmacologică nu și-a dovedit utilitatea;
E. Examinarea fizică poate decela eroziunea smalțului dentar.
R: A, B, C, E (3, p. 327)

53. Anorexia nervoasă este o tulburare de comportament alimentar,


caracterizată prin:
A. Alimentare compulsivă;
B. Imagine corporală distorsionată;
C. Refuzul de a menține o greutate corporală normală;
D. Frica intensă de a se îngrășa;
E. Aport caloric redus în raport cu necesarul.
R: B, C, D, E (3, p. 327)

54. Printre complicațiile sindromului de realimentare se numără:


A. Confuzie;
B. Rabdomioliză;
C. Trăsături accentuate de personalitate;
D. Colaps cardio-vascular;
E. Convulsii.
R: A, B, D, E (3, p. 327)

291
55. Alegeți răspunsul adevărat referitor la tulburările factice
(Sindromul Muchausen):
A. Reprezintă falsificarea semnelor fizice sau psihologice ale unei
boli în absența unei recompense evidente sau a unui beneficiu;
B. Preocuparea de a avea sau a contracta o boală gravă, în absența
simptomelor somatice evidente;
C. Dacă pacientul o acceptă, psihoterapia poate fi benefică;
D. Este denumită și tulburarea cu simptome neurologice funcționale;
E. Nu trebuie inițiate terapii inutile.
R: A, C, E (3, p. 326-327)

56. Alegeți din următoarele, metodele de tratament în anxietatea


socială( fobia socială):
A. SSRI sunt adesea eficace în reducerea anxietății, facilitând
interacțiunile sociale;
B. Beta blocantele sunt utilizate în cazurile severe;
C. Terapie cognitiv-comportamentală;
D. Benzodiazepinele sunt contraindicate;
E. Buspirona este consdierată mai sigură în tratamentul de lungă
durată.
R: A, C (3, p. 324)

57. Sindromul neuroleptic malign se caracterizează prin:


A. Enzime scăzute (CPK) ;
B. Hipotonie musculară;
C. Febră;
D. Semne vitale instabile;
E. Encefalopatie.
R: D, E (3, p. 321)

58. Printre efectele adverse ale Buspironei se enumeră:


A. Cefalee;
B. Greață;
C. Sedare;
D. Amețeală;
E. Confuzie.
R: A, B, D (3, p. 323)

292
59. În intoxicația cu alcool apar:
A. Scăderea inhibiției;
B. Vorbire neclară;
C. Afectarea coordonării;
D. Transpirații;
E. Delirium tremens.
R: A, B, C (3, p. 329)

60. Complicațiile utilizării cronice de benzodiazepine sunt


următoarele, cu excepția:
A. Depresie;
B. Oboseală;
C. Pierderi de memorie;
D. Aritmii;
E. Malnutriție.
R: A, B, D, E (3, p. 329)

61. În intoxicația cu amfetamine pot apărea:


A. Hiperactivitate;
B. Agitație psihomotorie;
C. Depresie;
D. Oboseală;
E. Midriază.
R: A, B, E (3, p. 329)

62. Complicațiile utilizării cronice de cocaină sunt:


A. Aritmii;
B. Moartea subită de cauză cardiacă;
C. Euforie și grandiozitate;
D. Sedare și depresie;
E. Ideație suicidală.
R: A, B, E (3, p. 329)

63. Dintre substanțele halucinogene fac parte următoarele:


A. Cofeina;
B. LSD;
C. Mescalina;
D. Ketamina;
E. Benzodiazepine.
R: B, C, D (3, p. 330)

293
64. Sunt efecte anticolinergice ale antipsihoticelor:
A. Confuzie;
B. Retenție urinară;
C. Distonie;
D. Akatizie;
E. Hipertensiune.
R: A, B (3, p. 319)

65. Caracterizează sindromul neuroleptic malign, cu excepția:


A. Reflexe crescute;
B. Pupile normale;
C. Piele uscată;
D. Evoluție prelungită;
E. Rigiditate de tip ” țeavă de plumb”.
R: A, B, C (3, p. 320)

66. Despre abuzul de opiozi este adevărat, cu excepția:


A. Intoxicația cu opiozi determină euforie, paranoia, gura uscată;
B. Sevrajul la opiozi se poate manifesta prin crampe stomacale,
grață, vărsături;
C. Se tratează cu metadonă;
D. Complicațiile utilizării cronice includ flashback-uri;
E. În timpul intoxicației cu opiozi scade nivelul de vigilitate.
R: A, D (3, p. 330)

67. Simptomele tulburării legate de consumul de substanțe pot fi:


A. Dorința persistentă de a opri sau a reduce consumul de substanță
(cravings);
B. Consumul unei cantități de substanță mai mare decât cea
intenționată;
C. Toleranță;
D. Stări de nevoie imperioasă de a consuma substanța;
E. Salvarea de energie pentru procurarea și consumarea substanței.
R: B, C, D (3, p. 329)

294
68. În tabloul clinic al anorexiei nervoase se pot identifica:
A. Surplus ponderal;
B. Fixație pe prevenția surplusului ponderal;
C. Lanugo;
D. Eroziuni ale smalțului dentar;
E. Amenoree.
R: B, C, E (3, p. 327)

69. Modelul de trăire interioară și comportament al tulburărilor de


personalitate se caracterizează prin:
A. Apare, în mod tipic, în copilărie;
B. Nu este atribuit consumului de substanțe;
C. Pacienții au conștiința faptului că necesită ajutor;
D. Este persistent și inflexibil;
E. Nu afectează capacitatea de funcționare.
R: B, D (3, p. 328)

70. În tabloul clinic al anorexiei nervoase se pot identifica:


A. Fixație pe prevenția surplusului ponderal;
B. Afectarea severă a imaginii corporale;
C. Amenoree;
D. Intoleranță la frig;
E. Tahicardie.
R: A, B, C, D (3, p. 327)

71. Sunt adevărate următoarele afirmații despre tulburarea


conversivă, cu excepția:
A. Este denumită și tulburarea cu simptome neurologice organice;
B. Debutul este acut;
C. Presupune apariția unor deficite senzoriale sau motorii, în absența
unei condiții medicale;
D. Pacienții par detașați emoțional de pierderea funcționalității sau
de simptomele neurologice;
E. Tratamentul de primă intenție include terapia cognitiv-
comportamentală.
R: A, E (3, p. 326)

295
72. Tulburarea de personalitate de tip bordeline poate include
următoarele, cu excepția:
A. Relații instabile, sentimente de gol interior, impulsivitate;
B. Caută atenție, grandiozitate, ideație suicidală;
C. Frica de critică și disconfort, iritabilitate neadecvată, invidie;
D. Stima de sine scăzută, instabilitatea dispoziției afective;
E. Îi vede pe ceilalți ca fiind integral buni, fie integral răi.
R: B, C (3, p. 328)

73. Următoarele afirmații despre neuroleptice sunt adevărate:


A. Neurolepticele cu potență ridicată blochează receptorii
dopaminergici D2 în vederea controlului de urgență al psihozei
sau al agitației;
B. Antipsihoticele atipice prezintă efecte anticolinergice, creștere
ponderală și mai puține efecte adverse comparativ cu
antipsihoticele tradiționale;
C. Neurolepticele cu potență redusă blochează receptorii dopaminei
și serotoninei, fiind medicamente de primă linie în tratamentul de
întreținere al tulburărilor psihotice;
D. Atipsihoticele cu potență crescută au mai multe efecte secundare
de tip anticolinergic și mai puține efecte de tip extrapiramidal;
E. Antipsihoticele cu potență redusă au mai puține efecte secundare
de tip extrapiramidal și mai multe efecte de tip anticolinergic.
R: A, B, E (3, p. 319)

74. Simptomele intruzive de retrăire a evenimentului traumatizant


din PTSD pot fi:
A. Amintiri intruzive;
B. Autoizolare;
C. Flashback-uri;
D. Vise neplăcute;
E. Detașare emoțională.
R: A, C, D (3, p. 325)

75. Fobia specifică:


A. Frică de un obiect, activitate sau situație anume;
B. Comportament evetiant;
C. Apare în perioada adultă;

296
D. Confrutarea cu obiectul fricii induce apariția atacului de panică;
E. Pacientul se confruntă cu obiectul.
R: A, B, D, E (3, p. 324)

76. Obsesiile se definesc ca:


A. Gânduri recurente;
B. Impulsiuni persistente;
C. Produce anxietate;
D. Produce euforie;
E. Produce disconfort.
R: A, B,C, E (3, p. 324)

77. Tratamentul specific în tulburarea dismorfică corporală


presupune:
A. Psihoterapie pentru tulburarea de percepție a corpului;
B. Antidepresivele pot ajuta în cazurile refractare la terapie;
C. De evitat efectuarea operațiilor inutile;
D. Se recomandă antipsihoticele;
E. Terapie cognitiv-comportamentală țintită pe acumulare.
R: A, B, C (3, p. 325)

78. Modulatorii serotoninei sunt următorii cu excepția:


A. Paroxetină;
B. Nefazodonă;
C. Doxepin;
D. Trazodonă;
E. Bupropion.
R: A, C, E (3, p. 322)

79. Episodul hipomaniacal poate fi descris ca:


A. Dispoziție afectivă elevată, expansivă sau iritabilă, ce durează cel
puțin 4 zile;
B. Episodul nu produce o afectare semnificativă a capacității de
funcționare;
C. Dispoziție afectivă elevată, cu o durată de cel puțion o săptămână;
D. Episoadele produc o afectare semnificativă a capacității de
funcționare;
E. Trei sau mai multe simptome de manie.
R: A, B, E (3, p. 322)

297
80. Sevrajul de alcool se caracterizeză prin următoarele, cu
excepția:
A. Sedare, lentoare psihomotrie;
B. Halucinații tactile;
C. Anxietate, depresie;
D. Delirium tremens;
E. Greață, vărsături.
R: A, C (3, p. 329)

81. Tratamentul în cazul consumului de amfetamine presupune:


A. Consiliere în scopul reabilitării;
B. Antipsihotice;
C. Benzodiazepine;
D. Anticonvulsionante;
E. Naltrexona.
R: A, B, C (3, p. 329)

82. Referitor la antipsihoticele atipice, sunt adevărate următoarele


afirmații:
A. Sunt medicamente de primă linie în controlul de urgență al
psihozei sau al agitației;
B. Prezintă mai puține efecte secundare comparativ cu
antipsihoticele tradiționale;
C. Clozapina este cel mai eficient antipsihotic, dar este rezervat
pentru psihoza refractară la tratament, din cauza riscului de
agranulocitoză;
D. Din această clasă fac parte haloperidolul, clorpromazina,
flufenzaina, olanzapina și loxapina;
E. Prezintă efecte adeverse extrapiramidale precum distonie,
parkinsonism, akinezie, akatizie.
R: B, C (3, p. 319)

83. Următoarele afirmații referitoare la tulburările de personalitate


sunt adevărate:
A. Sunt dificil de tratat;
B. Apar în adolescența târzie;
C. Nu pot fi atribuite unei condiții medicale;

298
D. Modelul de comportament al acestora este diferit, dar nu deviază
de la normele culturale;
E. Prezintă trăsături ce se referă la percepția altor persoane ,
afectivitate, relații interpersonale și controlul pulsiunilor.
R: A, B, C, E (3, p. 328)

84. Sunt antidepresive tetraciclice, cu excepția:


A. Venlafaxina;
B. Duloxetina;
C. Mirtazapina;
D. Imipramina;
E. Trazodona.
R: A, B, D, E (3, p. 322)

85. Nu sunt determinate de intoxicația cu opiozi următoarele:


A. Crampe stomacale;
B. Greață;
C. Sindrom amotivațional;
D. Euforie;
E. Mioză.
R: A, B, C (3, p. 330)

86. Referitor la pacienții cu tulburare de personalitate de tip


histrionic, este adevărat că:
A. Caută atenție;
B. Prezintă comportament excentric și inadecvat;
C. Vorbesc teatral;
D. Consideră relațiile mai profunde decât sunt în realitate;
E. Nu prezintă interes pentru relațiile sexuale.
R: A, C, D (3, p. 328)

87. Următoarele afirmații sunt adevărate referitor la abuzul de


fenciclidină:
A. În timpul sevrajului pacienții pot manifesta comportament violent
instalat brusc;
B. Tratamentul abuzului include benzodiazepine, antipsihotice, acid
ascorbic;
C. Printre complicațiile utilizării cronice se numără infecțiile
parenterale;

299
D. Tratamentul abuzului include metadona și natrexona;
E. În intoxicație, pacienții se pot manifesta impulsiv sau agresiv.
R: A, B, E (3, p. 330)

88. Complicațiile utilizării cronice de alcool pot fi:


A. Accident vascular cerebral;
B. Delirium tremens;
C. Malnutriție;
D. Ciroză;
E. Halucinații tactile.
R: C, D (3, p. 329)

89. Este adevărat despre tulburarea de personalitate paranoidă, cu


excepția:
A. Pacienții sunt rezervați în a se confesa;
B. Prezintă instabilitate în dispoziția afectivă;
C. Prezintă anhedonie;
D. Psihoterapia suportivă poate fi utilă în tratarea acesteia;
E. Pacienții interepretează greșit comentariile, în mod frecvent.
R: B, C (3, p. 328)

90. Sindromul neuroleptic malign se caracterizează prin:


A. Complicație frecventă a medicației antipsihotice;
B. Apare la câteva ore de la inițierea tratamentului;
C. Are o rata scăzută a mortalității;
D. Febră;
E. Encefalopatie.
R: D, E (3, p. 321)

91. Tulburarea depresivă majoră cu simptome psihotice este o


depresie asociată cu:
A. Deliruri;
B. Hipersomnie;
C. Halucinații;
D. Anhedonie;
E. Hiperfagie.
R: A, C (3, p. 321)

300
92. Tulburarea de personalitate de tip dependent se manifestă prin,
cu excepția:
A. Frică de responsabilitate;
B. Frică de intimitate;
C. Lipsa încrederii în judecata proprie;
D. Nevoia de a fi sprijinit de alții;
E. Retragere socială.
R: B, E (3, p. 328)

93. Printre efectele adverse ai modulatorilor ai serotoninei se


numără:
A. Aritmii;
B. Disfuncție sexuală;
C. Priapism;
D. Sedare;
E. Crize hipertensive.
R: C, D (3, p. 322)

94. Condițiile medicale care pot produce depresie includ:


A. Tumorile cerebrale;
B. Boala Parkinson;
C. Hipertiroidism;
D. Hiperparatiroidism;
E. Accident vascular cerebral.
R: A, B, D, E (3, p. 321)

95. Sunt efecte adverse ale tratamentului cu litiu:


A. Insuficiență renală;
B. Anomalia Ebstein;
C. Hipertiroidism;
D. Diabet zaharat tip II;
E. Scădere ponderală.
R: A, B (3, p. 323)

96. Printre caracteristicile tulburării de personalitate de tip obsesiv


convulsiv se numără, cu excepția:
A. Frica de intimitate;
B. Dificultate de a-și exprima dezacordul;

301
C. Dificultatea de a lucra cu alții;

D. Încăpățânarea;
E. Invidia față de alții.
R: A, B, E (3, p. 328)

97. Tulburare de personalitate de tip schizotipal se caracterizează


prin:
A. Convingeri bizare;
B. Detașare socială;
C. Comportament agresiv față de oameni;
D. Comportament excentric;
E. Comportament riscant.
R: A, D (3, p. 328)

98. Nu reprezintă substanțe psihoactive care pot produce simptome


depresive:
A. Neurolepticele conveționale;
B. Escitalopram;
C. Alcool;
D. Sertralina;
E. Antihistaminice.
R: B, D (3, p. 321)

99. Sunt tulburări psihotice:


A. Schizofrenia;
B. Tulburarea delirantă;
C. Tulburarea schizoafectivă;
D. Tulburarea depresivă majoră;
E. Tulburarea afectivă bipolară.
R: A, B, C (3, p. 319, 320, 321)

302
CAPITOLUL 39
EPIDEMIOLOGIE ȘI ETICĂ

1. Reprezintă cerințe ale unui studiu științific:


A. subiecții trebuie să fie reprezentativi pentru populația examinată
B. limitarea studiilor “dublu orb” datorită multiplelor erori de
investigații și observaționale
C. prezența unui număr suficient de subiecți
D. exprimarea consimțământului informat al subiecților
E. existența unor grupuri martor adecvate, în examinarea eficienței
unui tratament
R: A, C, D, E (3, pag. 333)

2. Despre studiul transversal, sunt adevărate afirmațiile:


A. pot fi consumatoare de resurse financiare și de timp
B. depinde de studiul original
C. cuprinde un număr mare de persoane la un moment dat, pentru a
evalua expunerea și prevalența bolii
D. reprezintă standardul de aur pentru testarea medicamentelor
E. nu poate fi folosit pentru testarea ipotezelor
R: C, E (3, pag. 333)

3. Meta-analiza:
A. reprezintă comparația retrospectivă a pacienților care au boala cu
martori sănătoși
B. depinde de studiul original
C. nu poate rezolva conflicte din literatura de specialitate
D. are o dimensiune mai mare
E. este incapabilă să elimine factorii limitativi din studiile originale
R: B, D, E (3, pag. 333)

4. Studiile de cohortă:
A. sunt capabile să examineze expunerile rare
B. facilitează studierea bolilor rare
C. examinează un grup de subiecți expuși la o situație sau factor dat

D. prezintă un risc relativ


303
E. depind de tipul studiul original
R: A, C, D (3, pag. 333)

5. Trialul clinic randomizat:


A. alocare randomizată a pacienților din grupurile de studiu da
B. poate fi controlat pentru un singur factor de confuzie
C. deseori consumator de resurse financiare și de timp
D. standard de aur pentru testarea medicamentelor
E. comparație retrospectivă a pacienților care au boala cu martori
sănătoși
R: A, C, D (3, pag. 333)

6. Reprezintă avantaje ale studiilor caz-control:


A. pot fi examinate grupuri de dimensiuni mici
B. standardul de aur pentru testarea medicamentelor
C. determină incidența bolii
D. pot fi studiate mai multe tipuri de expunere
E. se pot analiza bolile rare sau cele cronice într-o perioadă scurtă de
timp
R: A, D, E (3, pag. 333)

7. Nu pot fi folosite pentru testarea ipotezelor:


A. studiile caz-control
B. trialul clinic randomizat
C. seria de cazuri
D. studiul transversal
E. meta-analiza
R: C, D (3, pag. 333)

8. Reprezintă tipuri de erori în studiile clinice:


A. de memorie
B. observaționale
C. de selecție
D. de inducție
E. auto-selecția
R: A, B, C, E (3, pag. 334)

9. Reprezintă tipuri de erori în studiile clinice:

304
A. pre-selecția
B. perioada de latență
C. de investigație
D. de publicare
E. de memorie
R: B, C, D, E (3, pag. 334)

10. Despre eroarea de selecție, putem afirma:


A. rezultatele studiului nu se aplică populației generale
B. eficacitatea screeningului este exagerată
C. reprezintă repartizarea nerandomizată a subiecților în grupurile de
studiu
D. eficacitatea terapiei nu este descrisă cu precizie de grupul studiat
E. detectează bolile lent progresive și le omite pe cele rapid progresive
R: A, C (3, pag. 334)

11. Despre variabilele de confuzie, putem afirma:


A. afectează doar grupurile experimentale
B. interferează cu relația dintre grupurile experimentale și cele de
control
C. afectează doar grupurile de control
D. nu interferează cu relația dintre grupurile experimentale și cele de
control
E. afectează atât grupurile experimentale cât și grupurile de control
R: B, E (3, pag. 334)

12. Sunt consecințe ale erorii “perioada de latență”:


A. rezultatele studiului nu se aplică populației generale
B. eficacitatea screeningului este exagerată
C. perioada de la apariția bolii la rezultat depinde de screening
D. perioada de la diagnostic la rezultat dă falsa impresie a creșterii
duratei de supraviețuire
E. perioada de la apariția bolii la rezultat rămâne aceeași, indiferent de
screening
R: D, E (3, pag. 334)

13. Incidența reprezintă:

305
A.

B.
C. numărul de cazuri noi care apare într-o perioadă dată de timp într-o
populație
D. numărul de persoane cu o anumită afecțiune într-o perioadă dată de
timp
E.
R: C, E (3, pag. 334)

14. Rata de fatalitate a cazurilor, reprezintă:


A.
B. procentul de persoane cu o anumită boală care decedează într-o
anumită perioadă de timp
C. probabilitatea de a dezvolta boala la un grup expus la un factor risc
specific
D.
E. estimează riscul relative dacă prevalența este scăzută
R: B, D (3, pag. 334)

15. Prevalența reprezintă:


A. numărul de persoane cu o anumită afecțiune într-o perioadă dată de
timp
B.

C.
D. probabilitatea expunerii în rândul pacienților care au o boală
comparativ cu probabilitatea expunerii în rândul pacienților fără
acea afecțiune
E.
R: A, B (3, pag. 334)

16. Despre riscul relativ (RR), putem afirma:


306
A. reprezintă probabilitatea expunerii în rândul pacienților care au o
boală comparativ cu probabilitatea expunerii în rândul pacienților
fără acea afecțiune
B. valoarea 1 sugerează o relație între expunere și boală
C. reprezintă diferența privind ratele de îmbolnăvire între populația
expusă și cea neexpusă
D. valoarea 1 nu sugerează nicio relație între expunere și boală
E. reprezintă probabilitatea de a dezvolta boala la un grup expus la
un factor de risc specific comparativ cu probabilitatea la un grup
neexpus
R: D, E (3, pag. 335)

17. Valoarea riscului relativ (RR) este:


A. 1 – sugerează o relație între expunere și boală
B. <1 – sugerează o relație negativă între expunere și boală
C. <0 – sugerează o relație negativă între expunere și boală
D. >1 – sugerează o relație pozitivă între expunere și boală
E. 0 – sugerează o relație pozitivă între expunere și boală
R: B, D (3, pag. 335)

18. Despre riscul relativ (RR), putem afirma:


A. se determină prin studii de cohortă
B. RR=
C. valoarea 1 nu sugerează nicio relație între expunere și boală
D. se determină prin studii caz-control
E. reprezintă diferența privind rata bolii în urma tratamentului printr-o
intervenție specifică
R: A, B, C (3, pag. 335)

19. Despre raportul cotelor (OR, odds ratio), sunt adevărate afirmațiile:
A. valoarea 1 nu sugerează nicio relație între expunere și boală
B. se determină prin studii caz-control da
C. estimează riscul relativ dacă prevalența este ridicată
D. reprezintă probabilitatea expunerii în rândul pacienților care au o
boală comparativ cu probabilitatea expunerii în rândul pacienților
fără acea afecțiune da
E. reprezintă probabilitatea ca un test de screening să fie pozitiv la
pacienții care au boala
R: B, D (3, pag. 335)

307
20. Despre raportul cotelor (OR, odds ratio), putem afirma:
A. dă estimarea cea mai precisă a riscului relativ (RR) în cazurile de
boli rare
B. se determină prin studii de cohortă
C. estimează riscul relativ (RR) dacă prevalența este scăzută
D. valoarea 1 nu sugerează nicio relație între expunere și boală
E. estimează riscul relativ (RR) dacă incidența este scăzută
R: A, C (3, pag. 335)

21. Riscul atribuit (RA), reprezintă:


A. numărul de pacienți care trebuie tratați pentru a salva un pacient
B. diferența dintre rata bolii la populația expusă și rata bolii la
populația neexpusă
C. probabilitatea ca un test de screening să fie pozitiv la pacienții care
au boala
D. diferența privind ratele de îmbolnăvire între populația expusă și cea
neexpusă
E. diferența dintre rata bolii în grupul martor și rata bolii în grupul de
intervenție
R: B, D (3, pag. 335)

22. Reducerea absolută a riscului (RAR), este:


A. diferența privind rata bolii în urma tratamentului printr-o
intervenție specifică
B. numărul de pacienți care trebuie tratați pentru a salva un pacient
C. diferența dintre rata bolii în grupul martor și rata bolii în grupul de
intervenție
D. diferența dintre rata bolii la populația expusă și rata bolii la
populația neexpusă
E. direct proporțională cu numărul necesar pentru tratament (NNT)
R: A, C (3, pag. 335)

23. Numărul necesar pentru tratament (NNT), este:


A. numărul de pacienți care trebuie tratați pentru a salva un pacient
B. invers proporțional cu reducerea absolută a riscului (RAR)
C. diferența privind rata bolii în urma tratamentului printr-o
intervenție specifică

308
D. direct proporțional cu reducerea absolută a riscului (RAR)
E. probabilitatea ca un test de screening să fie negativ la pacienții care
nu au boala
R: A, B (3, pag. 335)

24. Despre sensibilitatea testelor de diagnostic, putem afirma:


A. reprezintă probabilitatea ca un test de screening să fie pozitiv la
pacienții care au boala
B. cele mai acceptabile teste de screening au de obicei sensibilitatea >
80%
C. reprezintă probabilitate ca un test de screening să fie negativ la
pacienții care nu au boala
D. cele mai acceptabile teste de screening au de obicei sensibilitatea >
70%
E. rezultate fals-negative apar la pacienții care au boala și cu test
negativ, aporoximate prin 1
R: A, B, E (3, pag. 335)

25. Despre specificitate, putem afirma:


A. reprezintă probabilitate ca un test de screening să fie negativ la
pacienții care nu au boala
B. cele mai acceptabile teste de confirmare au de obicei specificitatea
>85%
C. reprezintă probabilitatea ca un test de screening să fie pozitiv la
pacienții care au boala
D. rezultatele fals-negative apar la pacienții care nu au boala și cu test
pozitiv
E. cele mai acceptabile teste de confirmare au de obicei specificitatea
>75%
R: A, B (3, pag. 335)

26. Testele de screening:


A. sunt folosite pentru a valida faptul că un pacient cu test pozitiv are
într-adevăr boala
B. în mod ideal, au sensibilitate ridicată și specificitate moderată
C. dau estimarea cea mai precisă a riscului relativ în cazurile de boli
rare

D. caută detectarea fiabilă a unei boli la un pacient fără diagnosticarea


incorectă la persoane care nu au boala

309
E. în mod ideal, au sensibilitate și specificitate ridicată
R: D, E (3, pag. 335)

27. Despre „valoarea predictivă pozitivă” (VPP), sunt adevărate


afirmațiile:
A. reprezintă probabilitatea ca un pacient cu un test pozitiv să aibă o
boală
B. prevalența scăzută va fi asociată cu o valoarea predictivă pozitivă
ridicată a unui test de screening
C. reprezintă probabilitatea ca un pacient cu un test negativ să nu aibă
o boală
D. reprezintă performanța unui test de diagnostic luând în considerare
numai numărul de rezultate reale
E. prevalența ridicată va fi asociată cu o valoarea predictivă pozitivă
ridicată a unui test de screening
R: A, E (3, pag. 336)

28. Rata de probabilitate pozitivă (RPP):


A. este invers proporțională cu sensibilitatea
B. reprezintă probabilitatea ca o persoană care are boala să aibă un test
pozitiv comparativ cu probabilitatea ca o persoană fără boala
respectivă să aibă test pozitiv
C. este direct proporțională cu (1-specificitatea)
D. este direct proporțională cu sensibilitatea
E. reprezintă probabilitatea ca un pacient cu un test pozitiv să aibă o
boală
R: B, D (3, pag. 336)

29. Rata de probabilitate negativă (RPN):


A. reprezintă probabilitatea ca o persoană fără boală să aibă un test
negativ comparativ cu probabilitatea ca o persoană bolnavă să aibă
test negativ
B. este direct proporțională cu sensibilitatea
C. este invers proporțională cu specificitatea
D. este invers proporțională cu (1-sensibilitatea)
E. detectează diferența reală între grupuri
R: A, C (3, pag. 336)

30. Sunt tipuri de erori:

310
A. ipoteza alternativă
B. eroarea de tip III
C. eroarea de tip I
D. ipoteza nulă
E. eroarea de tip II
R: C, D, E (3, pag. 336)

31. Eroarea de tip I:


A. ipoteza alternativă este respinsă, chiar dacă este adevărată (fals-
negativ)
B. riscul scade o dată cu creșterea dimensiunii eșantionului
C. ipoteza nulă nu este respinsă, chiar dacă este falsă (fals-negativă)
D. afirmă că nu există nicio asociere între expunere și boală sau
tratament și răspuns
E. ipoteza nulă este respinsă, chiar dacă este adevărată (fals-pozitivă)
R: B, E (3, pag. 336)

32. Eroarea de tip II:


A. ipoteza nulă nu este respinsă, chiar dacă este falsă (fals-negativă)
B. riscul scade o dată cu creșterea dimensiunii eșantionului
C. în anumite condiții, este identică cu eroarea de tipul I
D. ipoteza nulă este respinsă, chiar dacă este adevărată (fals-pozitivă)
E. ipoteza alternativă nu este respinsă, chiar dacă este falsă (fals-
negativă)
R: A, B (3, pag. 336)

33. Puterea unui studiu:


A. arată diferența detectabilă statistic între grupuri
B. este egală cu valoarea probabilității (valoarea p)
C. reprezintă abilitatea unui studiu de a detecta o diferență reală între
două grupuri
D. cele cu putere insuficientă pot stabili că două grupuri sunt egale
când în realitate sunt statistic diferite
E. este mare, când raportul dintre eroare de tip I și eroarea de tip II
este mai mic decât 0
R: C, D (3, pag. 336)

34. Despre eroarea de investigație, putem afirma:

311
A. reprezintă repartizarea nerandomizată a subiecților în grupurile de
studiu
B. pacienții cu experiențe negative sunt mai predispuși să-și
amintească detalii negative
C. eficacitatea terapiei nu este descrisă cu precizie în grupul de studiu
D. interpretarea subiectivă a datelor de către investigator duce la
devierea către concluziile “dorite”
E. asemănarea rezultatelor studiului cu ipoteza propusă este incorectă
R: D, E (3, pag. 334)

35. Avantajele studiilor caz-control, sunt:


A. pot fi folosite pentru estimarea prevalenței bolii după expunere
B. se pot analiza bolile rare sau cele cronice într-o perioadă scurtă de
timp
C. pot fi studiate mai multe tipuri de expunere
D. pot fi examinate grupuri de dimensiuni mici
E. reprezintă standardul de aur pentru testarea medicamentelor
R: B, C, D (3, pag. 333)

36. Despre eroarea “auto-selecția”, sunt adevărate afirmațiile:


A. pacienții cu un anumit istoric medical sunt mai predispuși să
participe la un studiu legat de boala lor
B. eficacitatea terapiei nu este descrisă cu precizie de grupul de studiu
C. subiecții nu sunt reprezentativi pentru populația generală și induc
variabile de confuzie
D. rezultatele studiului nu se aplică populației generale
R: A, C (3, pag. 334)

37. Sunt drepturi ale pacientului:


A. competența
B. consimțământul informat
C. raportarea publică
D. dezvăluirea completă a informațiilor
E. confidențialitatea
R: B, C, D, E (3, pag. 337)

38. Pentru a fi considerat competent în a participa la un studiu, un


pacient trebuie:

312
A. să înțeleagă situația sa medicală
B. să nu fie diagnosticat în prezent cu psihoză sau intoxicație
C. să fie capabil să ia decizii care sunt în acord cu valorile sale
anterioare
D. să înțeleagă că nu se poate răzgândi ulterior, cu privire la
acceptarea terapiei
E. să aibă în mod obligatoriu peste 16 ani
R: A, B, C (3, pag. 337)

39. Sunt boli care, în cursul studiilor statistice, trebuiesc raportate:


A. neoplazii
B. boala Lyme
C. HIV
D. scleroza multiplă
E. hepatitele
R:B, C, E (3, pag. 337)

40. Sunt boli care, în cursul studiilor statistice, trebuiesc raportate:


A. tuberculoza
B. rabia
C. meningita
D. bolile cu transmitere sexuală
E. tulburarea bipolară
R: A, B, C, D (3, pag. 337)

41. În studiile statistice, confidențialitatea nu este obligatorie atunci


când pacientul:
A. permite medicului să împărtășească informațiile altor persoane
desemnate
B. are o boală care este raportabilă din punct de vedere legal (raportată
numai către instituțiile desemnate)
C. nu înțelege situația lui medicală
D. nu este capabil să ia decizii care sunt în acord cu valorile sale
anterioare
E. se consideră că ar avea tendințe suicidale sau criminale
R: A, B, E (3, pag. 337)

313
42. În studiile statistice, confidențialitatea nu este obligatorie atunci
când pacientul:
A. este un adolescent cu o afecțiune potențial dăunătoare pentru sine
sau pentru alții
B. a fost împușcat sau a suferit alt tip de rană penetrantă în uirma unui
atac
C. este diagnosticat în prezent cu psihoză sau intoxicație
D. are vârsta sub 16 ani
E. nu permite medicului să împărtășească informațiile altor persoane
desemnate
R: A, B (3, pag. 337)

43. Despre moarte, sunt adevărate afirmațiile:


A. pacienții hipotermici trebuiesc încălziți până la o temperatură
normală a corpului înainte ca decesul să poată fi declarat
B. pentru a defini moartea formală, sunt necesare atât moartea
cerebrală cât și moartea cardiacă
C. moartea cerebrală este inevitabil însoțită de moartea cardiacă, în
definiția formală
D. pentru a defini moartea formală, poate fi folosită fie moartea
cerebrală cât și moartea cardiacă (nu sunt necesare ambele)
E. absența activității pe EEG definește moartea cerebrală
R: A, D (3, pag. 338)

44. Despre moartea cerebrală putem afirma:


A. este definită ca absența ireversibilă a activității întregului creier la
un pacient, care durează > 6 ore
B. definiția ei în Statele Unite diferă în funcție de stat și instituție
C. absența activității pe EEG nu definește moartea cerebrală, dar poate
ajuta la analiza promptă
D. este definită ca absența ireversibilă a activității întregului creier la
un pacient, care durează > 8 ore
E. absența activității pe EEG definește moartea cerebrală
R: A, B, C (3, pag. 338)

45. Despre donarea de organe putem afirma:


A. pacienții nu se pot declara ca donatori de organe înainte de moarte
B. pacienții sau familiile pot defini exact ce organe pot fi donate
C. pacienții se pot declara ca donatori de organe înainte de moarte
D. pacienții sau familiile nu pot defini exact ce organe pot fi donate

314
E. nu poate fi realizată la pacienții > 60 ani
R: B, C (3, pag. 338)

46. Organele pot fi considerate inadecvate pentru transplant în caz de:


A. HIV
B. sepsis
C. neoplasm cu diseminare
D. tulburare bipolară
E. hipotermie
R: A, B, C, E (3, pag. 338)

47. Organele pot fi considerate inadecvate pentru transplant în caz de:


A. compromiterea funcției organului
B. vârsta > 60 ani
C. hemoglobinopatie
D. neoplasm nevindecat
E. hipertemie
R: A, C, D (3, pag. 338)

48. Organele pot fi considerate inadecvate pentru transplant în caz de:


A. vârsta > 80 ani
B. ischemie prelungită
C. neoplasm in situ
D. infecție/boală cu specificitate de organ
E. sepsis
R: A, B, D, E (3, pag. 338)

49. Sunt caracteristice ale morții:


A. absența reflexelor de trunchi cerebral (nervi cranieni)
B. prezența hipotermiei sau intoxicației
C. absența potențialelor evocate auditive de trunchi cerebral
D. absența circulației cerebrale la examenul radiologic
E. tahipneea când pacientul este decuplat de la ventilator pentru o
durată considerată suficientă pentru a produce o conducere normală
hipercarbică
R: A, C, D (3, pag. 338)

315
50. Sunt caracteristice ale morții:
A. absența hipotermiei sau intoxicației
B. absența liniei izoelectrice pe EEG
C. aspectul pacientului nu poate fi explicat de o condiție medicală care
mimează moartea
D. apneea când pacientul este decuplat de la ventilator pentru o durată
considerată suficientă pentru a produce o conducere normală
hipercarbică
E. prezența slabă a reflexelor de trunchi cerebral (nervi cranieni)
R: A, C, D (3, pag. 338)

316
TEMATICA ȘI BIBLIOGRAFIA PENTRU EXAMENUL DE
9
LICENȚĂ 9

DOMENIUL MEDICINĂ

1. Adam Feather, David Randall, Mona Waterhouse: Kumar şi Clark


Medicină Clinică. Leonard Azamfirei, Anca Dana Buzoianu, Dan Ionut
Gheonea - coordonatorii ediţiei în limba română, Ediţia a 10-a, Editura
Hipocrate, Bucureşti, 2021

CAP. 8 - SEPSISUL SI TRATAMENTUL INFECŢIILOR


BACTERIENE - p.151-168 CAP. 9 - ECHILIBRUL HIDRO-
ELECTROLITICSI ACIDO-BAZIC
1. Apa şi electroliţii - p.172-190
2. Dezechilibrele acido-bazice - p.195-202 CAP. 10. TERAPIE
INTENSIVĂ
1. Insuficienta respiratorie - p.224-232
2. Sindromul de detresă respiratorie acută - p.232-233 CAP. 16 -
HEMATOLOGIE
1. Anemia. Anemia microcitară. Anemia macrocitară - p.327-336
2. Anemia hemolitică dobândită - p.351-355 CAP. 18 -
REUMATOLOGIE
1. Artrita reumatoidă - p.437-447
2. Spondilartrita - p.447-454
3. Lupusul eritematos sistemic - p.457-459 (până la ’’Sindromul
antifosfolipidic”)
CAP. 21 - ENDOCRINOLOGIE
1. Hipotiroidismul. Hipertiroidismul - p.611-618 (până la ”Orbitopatia
Graves”)
2. Carcinomul tiroidian - p.621-622
CAP. 23 - DIABETUL ZAHARAT - p.699-736 (până la ’Implicaţiile
psihosociale ale diabetului zaharat”)
CAP. 26 - NEUROLOGIE
1. Coma şi alte tulburări ale conştientei - 832-836
2. Accidentul vascular cerebral - p.836-845
3. Hemoragia intracraniană - p.845-848
4. Infecţiile sistemului nervos - Meningita. Encefalita - p.869-872 (până la
”HTV-ul şi neurologia”)
317
CAP. 28 - PNEUMOLOGIE
1. Astmul - p.949-955
2. Bronhopneumopatia obstructivă cronică - p.955-961 (până la ”Apneea
în somn de tip obstructiv”)
3. Pneumonia - p.963-967
4. Tuberculoza - p.968-972 (până la ’’Infecţia cu micobacterii
netuberculoase”)
5. Neoplaziile tractului respirator - Tumorile maligne - p.975-982 (până la
’’Tumorile benigne”)
CAP. 29 - BOALA VENOASĂ TROMBOEMBOLICĂ - p.1001-1017 CAP.
30 - CARDIOLOGIE
1. Resuscitarea cardiacă - p.1045-1048 (până la „Cardioversia cu curent
direct”)
2. Aritmiile cardiace - p.1051-1069
3. Insuficienta cardiacă - p.1069-1079
4. Boala coronariană - Angina. Sindroamele coronariene acute - p.1079-
1091
5. Valvulopatii - p. 1091-1103
6. Endocardita infecţioasă - p. 1103-1107
7. Bolile miocardului - Miocardita. Cardiomiopatiile - p.1118-1124
8. Bolile pericardului - p.1125-1128
CAP. 31 - HIPERTENSIUNEA ARTERIALĂ - p.1133-1145
CAP. 32 - GASTROENTEROLOGIE
1. Boala de reflux gastroesofagian - p. 1162-1166 (până la „Tulburările de
motilitate”)
2. Infecţia cu Helicobacter Pylori - Boala ulceroasă peptică. Alte afecţiuni
asociate infecţiei cu H. pylori. Antiinflamatoarele nesteroidiene,
Helicobacter pylori şi boala ulceroasă - p. 1172-1176 (până la
„Gastropareza”)
3. Bolile inflamatorii intestinale - p.1198-1208
CAP. 34 - BOLILE HEPA TICE
1. Hepatita - Hepatita virală - p.1275-1284
2. Ciroza hepatică - p.1289-1303
CAP. 36 - TULBURĂRI RENALE SI ALE TRACTULUI URINAR
1. Glomerulul şi afectarea glomerulară - p.1351-1368
2. Nefritele tubulointerstiţiale (NTI) - p. 1385-1387
3. Boala cronică de rinichi - p.1392-1405

CAP. 37 - INFECŢII TRANSMISIBILE PE CALE SEXUALĂ SI


INFECŢIA CU VIRUSUL IMUNODEFICIENŢEI UMANE
Virusul imunodeficienţei umane şi sindromul imunodeficienţei dobândite -
318
p.1425- 1450
De asemenea, precizăm următoarele:
- conţinutul trimiterilor la alte capitole (pagini), cadrane, figuri care apar în
textul tratatului, nu face parte din tematica concursului de rezidentiat; în
consecinţă, din acele capitole (pagini) nu se vor formula întrebări de concurs.

2. Peter F. Lawrence - Chirurgie generală şi specialităţi chirurgicale.


Octavian Creţu, Viorel Jinga, Viorel Scripcariu - coordonatorii ediţiei în
limba română, Ediţia a 6-a, Editura Hipocrate, Bucureşti, 2021

CAP. 1 - EVALUAREA ŞI MANAGEMENTUL PERIOPERATOR AL


PACIENTULUI CHIRURGICAL - p.1-12
CAP. 4 - SÂNGERĂRILE CHIRURGICALE: TULBURĂRI ALE
HEMOSTAZEI, STĂRI
DE HIPERCOAGULABILITATE ŞI TERAPIA CU PRODUSE
SANGUINE LA
PACIENTUL CHIRURGICAL - p.39-45
CAP. 8 - INFECŢIILE CHIRURGICALE - p.75-84
CAP. 9 - TRAUMATOLOGIE - p.87-106
CAP. 10 - ARSURILE - p.108-123
CAP. 11 - HERNIILE PERETELUI ABDOMINAL - p.125-138
CAP. 12 - ESOFAGUL
1. Anatomia esofagului - pg. 140-142
2. Carcinomul esofagian - pg. 151-156
3. Perforaţia esofagiană - p. 156-157
4. Herniile hiatale - p.157-158
5. Tulburări ale motilităţii esofagiene - p. 158-161
6. Leziuni esofagiene benigne - p.161-162
7. Ingestia de corpi străini - p.162
8. Ingestia substanţelor caustice - p.162-164
CAP. 13 - STOMACUL ŞI DUODENUL
1. Anatomie - pg.167-168
2. Afecţiuni gastrice maligne - pg.172-174
3. Afecţiuni duodenale maligne - 179-180
4. Complicaţii postgastrectomie - 180-182
5. Tratamentul chirurgical al obezităţii - p. 182-191
CAP. 14 - INTESTINUL SUBŢIRE ŞI APENDICELE
1. Bolile intestinului subţire - Ocluzia intestinului subţire - p.193-199
(până la „Boala Crohn a intestinului subţire”)
319
2. Bolile intestinului subţire - Ischemia mezenterică acută - p.204-206
3. Bolile intestinului subţire - Tumorile intestinului subţire - p.206-209
4. Bolile intestinului subţire - Anomalii congenitale - p.209-2011
5. Bolile apendicelui - pg.211-214
CAP. 15 - COLON, RECT ŞI ANUS
1. Anatomie - p.216-219
2. Afecţiuni benigne colonice - p.222-225
3. Ocluzia intestinului gros - p.228-229
4. Volvulus de colon - p.229-230
5. Pseudo-ocluzia acută a colonului - p.230
6. Polipii şi cancerul colo-rectal - p.230-235
7. Canalul anal şi rectul - p.235-239
8. Infecţiile cu transmitere sexuală - p.239-240
CAP. 16 - CĂILE BILIARE - p.242-255
CAP. 17 - PANCREASUL - p.257-275
CAP. 20 - FICATUL ŞI SPLINA
1. Anatomia ficatului. Fiziologia ficatului - p.317
2. Leziunile hepatice traumatice - p.317-319
3. Tumorile hepatice, chisturile şi abcesele hepatice - p.319-324
4. Anatomia şi fiziologia splinei - p.333-335
5. Anomaliile splinei - consideraţii diagnostice generale - p.335-337
6. Afecţiunile chirurgicale ale splinei - p.337-341
7. Consecinţe şi complicaţii după splenectomie - p.341-342
CAP. 26 - BOLILE SISTEMULUI VASCULAR
1. Boala arterială periferică - p.520-527
2. Bolile venelor - p.534-541
CAP. 27 - OTORINOLARINGOLOGIA: BOLILE CAPULUI ŞI
GÂTULUI - p.548-583 CAP. 28 - CHIRURGIE ORTOPEDICĂ: BOLILE
SISTEMULUI MUSCULOSCHELETAL - p.585-633
CAP. 29 - UROLOGIE: AFECŢIUNILE APARATULUI URO-GENITAL
- p.635-674

320
3. Latha Ganti, David Lebowitz, Javier Rosario, Ariel Vera: Sinopsis de
medicină. Cristina Oana Mărginean, Cătălina Poiană - coordonatorii ediţiei
în limba română, Ediţia a 5-a, Editura Hipocrate, Bucureşti, 2021

CAP. 9 - DERMATOLOGIE - p.196-209


CAP. 10 - PEDIATRIE - p.212-243
CAP. 12 - AFECŢIUNI GINECOLOGICE ŞI MAMARE- p.273-292
CAP. 13 - OBSTETRICĂ - p.295-316
CAP. 14 - TULBURĂRILE PSIHICE - p.319-330
CAP. 15 - EPIDEMIOLOGIE ŞI ETICĂ - p.333-338

321
322

S-ar putea să vă placă și